100% found this document useful (6 votes)
4K views233 pages

Crash Course PDF

Uploaded by

Zainab Jawad
Copyright
© © All Rights Reserved
We take content rights seriously. If you suspect this is your content, claim it here.
Available Formats
Download as PDF, TXT or read online on Scribd
100% found this document useful (6 votes)
4K views233 pages

Crash Course PDF

Uploaded by

Zainab Jawad
Copyright
© © All Rights Reserved
We take content rights seriously. If you suspect this is your content, claim it here.
Available Formats
Download as PDF, TXT or read online on Scribd
You are on page 1/ 233

1000 SBAs and EMQs for

Medical Finals
First edition authors
Neel Sharma
2 nd Edition

CRASH COURSE
SERIES EDITORS
Philip Xiu
BA, MA, MB BChir, MRCP
GP Registrar, Yorkshire Deanery
Shreelata Datta
MD, MRCOG, LLM, BSc (Hons), MBBS
Honorary Senior Lecturer
Imperial College, London,
Consultant Obstetrician and Gynaecologist
King’s College Hospital
London, UK
FACULTY ADVISORS
Juhu Joseph
FRCS (Tr & Orth)
Consultant Orthopaedic & Trauma Surgeon, Northumbria Healthcare NHS
Foundation Trust
Vidhya Nair
MBBS, FRCP
Consultant in Elderly Medicine
Leeds Teaching Hospitals NHS Trust

1000 SBAs and EMQs


for Medical Finals
Philip Xiu
BA, MA, MB BChir, MRCP
GP Registrar, Yorkshire Deanery
Senior Content Strategist: Jeremy Bowes
Senior Content Development Specialist: Alex Mortimer
Project Manager: Andrew Riley
Design: Christian Bilbow

© 2019 Elsevier Limited. All rights reserved.

This book and the individual contributions contained in it are protected under copyright by the Publisher (other than as may be
noted herein).

First edition 1998


Second edition 2019

ISBN: 978-0-7020-7384-7
eISBN: 978-0-7020-7352-6

The right of Philip Xiu to be identified as author of this work has been asserted by him in accordance with the Copyright,
Designs and Patents Act 1988.

No part of this publication may be reproduced or transmitted in any form or by any means, electronic or mechanical, including
photocopying, recording, or any information storage and retrieval system, without permission in writing from the publisher.
Details on how to seek permission, further information about the Publisher’s permissions policies and our arrangements with
organizations such as the Copyright Clearance Center and the Copyright Licensing Agency, can be found at our website: www.
elsevier.com/permissions.

This book and the individual contributions contained in it are protected under copyright by the Publisher (other than as may be
noted herein).

Notices
Practitioners and researchers must always rely on their own experience and knowledge in evaluating and using any information,
methods, compounds or experiments described herein. Because of rapid advances in the medical sciences, in particular,
independent verification of diagnoses and drug dosages should be made. To the fullest extent of the law, no responsibility
is assumed by Elsevier, authors, editors or contributors for any injury and/or damage to persons or property as a matter
of products liability, negligence or otherwise, or from any use or operation of any methods, products, instructions, or ideas
contained in the material herein.

Printed in China
Series Editors’ foreword

The Crash Course series was conceived by Dr Dan Horton-Szar who as series editor
presided over it for more than 15 years – from publication of the first edition in 1997,
until publication of the fourth edition in 2011. His inspiration, knowledge and wisdom
lives on in the pages of this book. As the new series editors, we are delighted to be
able to continue developing each book for the twenty-first century undergraduate
curriculum.

The flame of medicine never stands still, and keeping this all-new fifth series relevant
for today’s students is an ongoing process. Each title within this new fifth edition
has been re-written to integrate basic medical science and clinical practice, after
extensive deliberation and debate. We aim to build on the success of the previous
titles by keeping the series up-to-date with current guidelines for best practice, and
recent developments in medical research and pharmacology.

We always listen to feedback from our readers, through focus groups and student
reviews of the Crash Course titles. For the fifth editions we have reviewed and
re-written our self-assessment material to reflect today’s ‘single-best answer’ and
‘extended matching question’ formats. The artwork and layout of the titles has also
been largely re-worked and are now in colour, to make it easier on the eye during
long sessions of revision. The new on-line materials supplement the learning process.

Despite fully revising the books with each edition, we hold fast to the principles
on which we first developed the series. Crash Course will always bring you all the
information you need to revise in compact, manageable volumes that still maintain
the balance between clarity and conciseness, and provide sufficient depth for those
aiming at distinction. The authors are junior doctors who have recent experience
of the exams you are now facing, and the accuracy of the material is checked by a
team of faculty editors from across the UK.

We wish you all the best for your future careers!

Shreelata Datta and Philip Xiu

v 
Prefaces

Author
This book provides a practical, no-nonsense approach to tackling the medical
finals exam, in a format that is written specifically for the exam. Each chapter
address one speciality in medicine or surgery and begins with easier questions
before moving onto harder Ebel classified questions. The detailed answer
explanations provides you with a good grounding of the most common topics
covered, as well as harder topics to distinguish the distinction level canditates.

I have been at the coalface of exams and teaching and as such have recent
knowledge of the stresses that exams can cause. The solution is to cover as
much of the most common areas that are tested by the examiners, and to see
as many possible permutations of common exam clinical scenarios as possible,
which this book covers with its 1000 questions in both SBAs and EMQs. As in
clinical practice, I have toiled over this book and gained great satisfaction from
producing it, and then gratefully and humbly watch it be markedly improved and
corrected by the Faculty Editors Dr Nair and Mr Joseph. Both of which have
carefully screened over each word, and without whom the book would not have
been possible.

I am indebted to those who have taught me, those who have worked with me
and most importantly, those I have been priviliged enough to care for.

All that remains is to wish you the very best of luck.

Philip Xiu

Faculty Advisors
The purpose of this book is to cover medical and surgical cases which may
feature in final exams.

We hope you find it a useful revision aid which will stimulate your thirst for
learning and help you get through finals!

We wish you the best of luck!

Juhu Joseph and Vidhya Nair

vi 
Series Editors’ acknowledgements

We would like to thank the support of our colleagues who have helped in the
preparation of this edition, namely the junior doctor contributors who helped
write the manuscript as well as the faculty editors who check the veracity of the
information.

We are extremely grateful for the support of our publisher, Elsevier, whose staffs’
insight and persistence has maintained the quality that Dr Horton-Szar has set-
out since the first edition. Jeremy Bowes, our commissioning editor, has been a
constant support. Alex Mortimer and Barbara Simmons our development editors
have managed the day-to-day work on this edition with extreme patience and
unflaggable determination to meet the ever looming deadlines, and we are ever
grateful for Kim Benson’s contribution to the online editions and additional online
supplementary materials.

vii 
Contents

Series Editors’ foreword . . . . . . . . . . . . . . . . . . . . . . . . . . . v 15. Urology. . . . . . . . . . . . . . . . . . . . . . . . . . . . . . . . . . . 119


Prefaces . . . . . . . . . . . . . . . . . . . . . . . . . . . . . . . . . . . . . . . vi 16. Breast surgery. . . . . . . . . . . . . . . . . . . . . . . . . . . . . 125
Series Editors’ acknowledgements. . . . . . . . . . . . . . . . . . vii
Normal values. . . . . . . . . . . . . . . . . . . . . . . . . . . . . . . . . . . ix Answers
Section 3 Medicine answers . . . . . . . . . 129
Introduction. . . . . . . . . . . . . . . . . . . . . . . . . . . . . . . . . . . . 1  1. Cardiology. . . . . . . . . . . . . . . . . . . . . . . . . . . . . . . . 131
 2. Gastroenterology . . . . . . . . . . . . . . . . . . . . . . . . . . 139
Questions  3. Respiratory . . . . . . . . . . . . . . . . . . . . . . . . . . . . . . . 147
 4. Renal . . . . . . . . . . . . . . . . . . . . . . . . . . . . . . . . . . . . 153
Section 1 Medicine questions . . . . . . . . . . 5
 5. Neurology. . . . . . . . . . . . . . . . . . . . . . . . . . . . . . . . . 157
 1. Cardiology. . . . . . . . . . . . . . . . . . . . . . . . . . . . . . . . . . 7
 6. Rheumatology. . . . . . . . . . . . . . . . . . . . . . . . . . . . . 165
 2. Gastroenterology . . . . . . . . . . . . . . . . . . . . . . . . . . . 19
  7. Endocrine and diabetes. . . . . . . . . . . . . . . . . . . . . 169
 3. Respiratory . . . . . . . . . . . . . . . . . . . . . . . . . . . . . . . . 31
 8. Haematology. . . . . . . . . . . . . . . . . . . . . . . . . . . . . . 177
 4. Renal . . . . . . . . . . . . . . . . . . . . . . . . . . . . . . . . . . . . . 41
 5. Neurology. . . . . . . . . . . . . . . . . . . . . . . . . . . . . . . . . . 47 Section 4 Surgery answers. . . . . . . . . . . 181
 6. Rheumatology. . . . . . . . . . . . . . . . . . . . . . . . . . . . . . 59   9. Trauma and surgical emergencies . . . . . . . . . . . . 183
  7. Endocrine and diabetes. . . . . . . . . . . . . . . . . . . . . . 65 10. Perioperative care. . . . . . . . . . . . . . . . . . . . . . . . . . 187
 8. Haematology. . . . . . . . . . . . . . . . . . . . . . . . . . . . . . . 77 11. Gastrointestinal surgery. . . . . . . . . . . . . . . . . . . . . 189
12. Cardiothoracics and vascular surgery . . . . . . . . . 197
Section 2 Surgery questions . . . . . . . . . . 85
13. Orthopaedics. . . . . . . . . . . . . . . . . . . . . . . . . . . . . . 199
  9. Trauma and surgical emergencies . . . . . . . . . . . . . 87
14. Ear, nose and throat . . . . . . . . . . . . . . . . . . . . . . . . 203
10. Perioperative care. . . . . . . . . . . . . . . . . . . . . . . . . . . 93
15. Urology. . . . . . . . . . . . . . . . . . . . . . . . . . . . . . . . . . . 207
11. Gastrointestinal surgery. . . . . . . . . . . . . . . . . . . . . . 95
16. Breast surgery. . . . . . . . . . . . . . . . . . . . . . . . . . . . . 211
12. Cardiothoracics and vascular surgery . . . . . . . . . 105
13. Orthopaedics. . . . . . . . . . . . . . . . . . . . . . . . . . . . . . 109
14. Ear, nose and throat. . . . . . . . . . . . . . . . . . . . . . . . 115 Index. . . . . . . . . . . . . . . . . . . . . . . . . . . . . . . . . . . . . . . . 213

viii
Normal values

HDL cholesterol
HAEMATOLOGY  Male 0.8–1.8 mmol/L
Haemoglobin  Female 1.0–2.3 mmol/L
 Male 135–177 g/L LDL cholesterol <4.0 mmol/L
 Female 115–165 g/L Triglycerides
Mean corpuscular 27–32 pg  [H+] 35–45 nmol/L
haemoglobin (MCH)  pH 7.35–7.45
Mean corpuscular 32–36 g/dL
haemoglobin concentration BIOCHEMISTRY
(MCHC) (SERUM/PLASMA)
Mean corpuscular volume 80–96 fL
(MCV) Alanine aminotransferase (ALT) 5–40 U/L
Packed cell volume (PCV) Albumin 35–50 g/L
 Male 0.40–0.54 L/L Alkaline phosphatase 39–117 U/L
 Female 0.37–0.47 L/L Amylase 25–125 U/L
White blood count (WBC) 4–11 × 109/L Aspartate aminotransferase 12–40 U/L
  Basophil granulocytes <0.01–0.1 × 109/L (AST)
  Eosinophil granulocytes 0.04–0.4 × 109/L Bicarbonate 22–30 mmol/L
 Lymphocytes 1.5–4.0 × 109/L Bilirubin <17 μmol/L (0.3–1.5
 Monocytes 0.2–0.8 × 109/L mg/dL)
  Neutrophil granulocytes 2.0–7.5 × 109/L Calcium 2.20–2.67 mmol/L
Platelet count 150–400 × 109/L (8.5–10.5 mg/dL)
Serum B12 160–925 ng/L Chloride 98–106 mmol/L
(150–675 pmol/L) C-reactive protein <10 mg/L
Serum folate 2.9–18 μg/L Creatinine 79–118 μmol /L
(3.6–63 nmol/L) (0.6–1.5 mg/dL)
Red cell folate 149–640 μg/L Creatine kinase (CPK)
Red cell mass  Female 24–170 U/L
 Male 25–35 mL/kg  Male 24–195 U/L
 Female 20–30 mL/kg   CK–MB fraction <25 U/L (<60% of total
Reticulocyte count 0.5–2.5% of red cells activity)
(50–100 × 109/L) Ferritin
Erythrocyte sedimentation rate <20 mm in 1 hour  Female 6–100 μg/L
(ESR)  Male 20–260 μg/L
 Postmenopausal 12–230 μg/L
COAGULATION α-Fetoprotein <10 kU/L
Glucose (fasting) 4.5–5.6 mmol/L
Bleeding time 3–9 min (70–110 mg/dL)
(Ivy method) γ–Glutamyl transpeptidase
Activated partial 23–31s (γ–GT)
thromboplastin time (APTT)  Male 11–58 U/L
Prothrombin time 12–16s  Female 7–32 U/L
  International Normalized 1.0–1.3 Glycosylated (glycated) 3.7–5.1%
Ratio (INR) haemoglobin (HbA1c)
D-dimer <500 ng/mL Iron 13–32 μmol /L (50–150
μg/dL)
LIPIDS AND LIPOPROTEINS Iron–binding capacity 42–80 μmol /L
(total) (TIBC) (250–410 μg/dL)
Cholesterol 3.5–6.5 mmol/L Magnesium 0.7–1.1 mmol/L
(ideal <5.2 mmol/L) Osmolality 275–295 mOsm/kg

ix 
Normal
 values

Phosphate 0.8–1.5 mmol/L BLOOD GASES (ARTERIAL)


Potassium 3.5–5.0 mmol/L
Prostate–specific ≤4.0 μg/L PaCO2 4.8–6.1 kPa (36–46
antigen (PSA) mmHg)
Protein (total) 62–77 g/L PaO2 10–13.3 kPa
Sodium 135–146 mmol/L (75–100mmHg)
Urate 0.18–0.42 mmol/L  Male 0.70–2.1 mmol/L
(3.0–7.0 mg/dL)  Female 0.50–1.70 mmol/L
Urea 2.5–6.7 mmol/L (8–25 Bicarbonate 22–26 mmol/L
mg/dL)

x
Introduction

Crash Course: 1000 SBAs and EMQs for Medical Finals, that students at different universities may not be directly
is intended to provide medical students, with a convenient comparable. The questions can be drawn from a local
tool for assessing and improving their knowledge of bank of questions and a percentage from the Medical
medicine. Schools Council Assessment Alliance (MSCAA) question
There are actually more than 1000 questions in the bank (questions contributed by all UK Medical Schools
book; which are similar in both complexity and format to and used across the UK).
the medical finals level questions you might expect (i.e.,
suitable for use in a Final MB qualifying examination). Reliability and validity of finals
We have graded the questions in the rough ratio of
difficulties that a student might face in finals; this allows Assessment of competence and knowledge should thus
the student to choose which ones to tackle. be based across all of Miller’s triangle, to get an accurate
The explanations for each of the questions not only assessment of a student’s performance. This sampling
goes into detail as to why the answer option is correct, method crosses methodologies (in content, in time,
but also expands the subject matter to suggest why other across examiners and patient feedback), which leads to
answers are wrong. higher validity and reliability of a student’s performance.
This book will focus on the written part of medical
finals, namely that of SBA and EMQs.
PURPOSE OF MEDICAL FINALS
EXAMINATIONS SINGLE BEST ANSWER (SBA)
The aim of medical finals is to measure the clinical SBA style questions are the most popular means
competence (i.e., professional knowledge, skills and of written assessment in both undergraduate and
attitudes) against a defined criteria and standards. postgraduate medical examinations.
To pass finals, students should demonstrate safety
and competence at the level of a day 1 Foundation Anatomy of an SBA
Programme Year 1 (FY1) doctor.
Contrary to many student’s fears, ranking of They have a stem or vignette, describing the problem.
candidates is only a secondary objective. They present five options all of which are plausible, but
one of which is clearly correct. The options should be
homogeneous (i.e., all the options should be diagnoses,
Elements of competence investigations or mechanisms) and not a mix.
Competence can be divided up into three distinct categories: Stem: They have a stem or vignette, describing the
Knowledge – demonstrating factual regurgitation and problem.
applied clinical reasoning Lead-in: The lead-in asks the question.
Skills – clinical skills and communication skills Answer options: There are 5 answer options for each
Attitudes – and professional behaviour question item.
This can be summarized in Miller’s triangle: Example:
Cognition is tested at the base of the triangle: ‘Knows’ and Stem: A term baby is born with a vitelline duct still
‘Knows how’ and this is tested primarily by written/computer- present. Her baby check was normal and she was
based assessments, whereas Behaviour is tested under the discharged 2 days after birth. She remains well until she
apex of the triangle ‘Shows how’ and ‘Does’ and these are attends university at 19 years of age, when she develops
tested under performance or hands on assessment. colicky abdominal pain.
This book will address the extended matching questions Lead-in: What is the likely cause?
(EMQs) and single best answer (SBA) component. Answer options:
a. Appendicitis
b. Meckel Diverticulum
How do medical schools compare? c. Exomphalos
Currently, medical finals for each medical school are d. Gastroschisis
different. Each school has their own curriculum syllabus e. Atresia
and uses different methods of examination, which means Answer: B

1 
Hints and tips for the test-smart students

concentration and serum alkaline phosphatase


EXTENDED-MATCHING activity are within normal limits. Answer: E
QUESTIONS (EMQs) 2. A 15-year-old girl has a two-week history of
fatigue and back pain. She has widespread
EMQs are a popular method of assessment in bruising, pallor, and tenderness over the vertebrae
undergraduate and postgraduate medical examinations. and both femurs. Full blood count shows
This self-assessment book will provide you with a mixed haemoglobin concentration of 70 g/dL, leukocyte
set of EMQs divided by themes that most commonly count of 2.0 × 109/L, and platelet count of 15 ×
appear in your exams. 109/L. Answer: A
The most important skill in answering EMQs
successfully is in choosing the correct option out of a list
of very similar symptoms or signs.
HINTS AND TIPS FOR THE
TEST-SMART STUDENTS
Anatomy of an EMQ Students who are knowledgeable about exam technique
can pick up on cues in the question or options that lead
Themes: EMQs are divided into themes, which defines
them to the correct answer without having to use the full
the subject of the question.
depth of their medical knowledge. The most important
Answer options: There can be between 8 to 15 answer
tricks that lead to this are listed below.
options for each question item, and some answers can be
used more than once.
Lead-in: The lead-in defines the task. Grammatical cues
Stems: They have a stem or vignette describing the One or more options do not follow grammatically from
problem. For each item question, select one lettered the stem and lead-in. This occurs when question writers
option that is most closely associated with the question. pay more attention to the correct answer than to the
Example: distractors. Usually, the correct answer makes grammatical
Theme: sense but some or all of the distractors do not. This gives
Fatigue test-smart students clues on how to answer.
Options:
A. Acute leukaemia
B. Anaemia of chronic disease Logical cues
C. Congestive heart failure Occur when a subset of the options is collectively
D. Depression exhaustive. For example:
E. Epstein–Barr virus infection Crime is
F. Folate deficiency A Spread evenly across all social classes
G. Glucose 6-phosphate dehydrogenase deficiency B Over-represented among the poor
H. Hereditary spherocytosis C Over-represented among the middle-classes
I. Hypothyroidism and rich
J. Iron deficiency D An indication of psychosexual maladjustment
K. Lyme disease E Reaching a plateau of tolerability for the nation.
L. Microangiopathic haemolytic anaemia
M. Miliary tuberculosis Options A, B and C include all possibilities, so the test-
N. Vitamin B12 (cyanocobalamin) deficiency smart student could eliminate D and E.
Lead-in:
For each patient with fatigue, select the most likely
diagnosis.
Absolute terms
Stems: Sometimes there are absolute terms such as ‘always’ or
1. A 19-year-old woman has had fatigue, fever, ‘never’ in the options. These options are less likely to be
and sore throat for the past week. She has a true and can be eliminated by the test-smart student – in
temperature of 38.3°C, cervical lymphadenopathy the real-world, things are rarely so cut and dried.
and splenomegaly. Initial laboratory results show
a white blood cell count of 18 × 109/L (80%
Long correct answers
lymphocytes, with many exhibiting atypical
features). Serum aspartate aminotransferase The correct answer option is longer, more specific or more
(AST, GOT) activity is 200 U/L. Serum bilirubin complete than the others.

2
Introduction 

Word repeats Acceptable knowledge


The correct answer repeats a word or phrase used on These are harder questions which stretch the
the stem. most able of students. These comprise around
10%–20% of any chapter, and are designed to
Convergence strategy be able to distinguish the average student from
the distinction level student. Some more esoteric
This one is tricky. Basically, this error occurs when the diagnosis and management of rarer disease are
correct option is given the most elements in common with covered here.
the other answers, i.e., it is not out at the extremes. A
simple example of this occurs when the options are: HOW DO SCHOOLS SET UP A HIGH
A Pencil and pen
B Pencil and highlighter
QUALITY WRITTEN EXAM?
C Pencil and crayon
A team of experienced question writers generate a first
D Pen and marker
draft of questions. These can be from consultants or
registrars who are involved in teaching. The questions are
Then by simply counting, Pencil appears 3 times, Pen
then reviewed and refined at a meeting of the examiners.
twice and the rest once each. So, the correct answer is
More senior examiners conduct tertiary level review
more likely to be A as it contains the two most common
and select questions for inclusion in the test paper (using
words. Sounds ridiculous? Perhaps, but it does happen
a blueprint syllabus) from the national MSCAA question
frequently so you have to take care to avoid it in your
bank.
questions.
The questions undergo refinement and review by
speciality senior examiners who may request clarification
DIFFICULTY OF QUESTIONS or changes. This set of questions are then screened by
The questions are divided up into different levels of a team of external examiners who are part of the quality
difficulties. assessment.
This book uses a modified Ebel method to divide the There may be final reviews before the questions are
questions and rate them in difficulty. We used subject put into use for the exam.
matter experts to rate questions according to their
importance, categorising them as essential, important or HOW TO MAKE THE BEST USE OF
acceptable. THIS BOOK
Medical finals examiners will often have this sort of grid
to define the percentage of questions that a borderline Some students like to use the book to simulate the time
student should be able to answer correctly. restrictions imposed by medical finals; you should be
For example, we would expect that a borderline ‘pass’ aiming to allocate about one minute per question. You
student should answer 90% of all essential questions can attempt the questions in chunks of 10 or more. After
correctly, and 70% of all important questions, as opposed answering chunks of questions, you should spend some
to 40%–50% of all acceptable questions which are there time in reviewing the explanations at the end of the chapter.
to stretch the most able of students. Even if you got the question right, the explanations will
help with surrounding topics and can help the student
Essential knowledge gain further marks in other questions.
Some students find it useful to check the explanations
Consists of essential knowledge, that all medical
after doing each question, without time restrictions.
students must know before qualification. These consist
This technique is good for the student that wants
of emergencies, and common management themes, to
to use the book to ‘learn’ the most high-yield
ensure that a student is safe to practice.
exam materials, in conjunction with their
own notes.
Important knowledge
Constitutes the bulk majority of questions, and these ONLINE INTERACTIVE
are important core topics which students should know ASSESSMENT – EBOOK
in order to pass finals. Students should be getting the
majority of these questions correct and these will form the Finally, do not forget to access/download your
bulk of any examination. accompanying Student Consult eBook (which comes

3 
Conclusion

with the printed version of this book – instructions on the


inside front cover). You can then access all the questions
CONCLUSION
anywhere from your laptop or mobile device and choose Whatever you choose to do, if you manage to finish the
either to read in the regular eBook format or to utilise the book it will put you in good stead for the real medical
‘Interactive Assessment’ tool, to help test yourself and finals.
monitor your progress.’ Good luck with your exams!

4
MEDICINE
QUESTIONS
Chapter 1

Cardiology . . . . . . . . . . . . . . . . . . . . . . . . 7
Chapter 2

Gastroenterology . . . . . . . . . . . . . . . . . . . 19
Chapter 3

Respiratory . . . . . . . . . . . . . . . . . . . . . . 31
Chapter 4

Renal . . . . . . . . . . . . . . . . . . . . . . . . . 41
Chapter 5

Neurology . . . . . . . . . . . . . . . . . . . . . . . 47
Chapter 6

Rheumatology . . . . . . . . . . . . . . . . . . . . . 59
Chapter 7

Endocrine and diabetes . . . . . . . . . . . . . . . . 65


Chapter 8

Haematology . . . . . . . . . . . . . . . . . . . . . 77
This page intentionally left blank
Cardiology 1
a. Sick sinus syndrome
SINGLE BEST ANSWER (SBA) b. First-degree AV block
QUESTIONS c. Mobitz type 1 block
d. Mobitz type 2 block
1. A 19-year-old university rower presents for the pre- e. Complete heart block
Oxford–Cambridge boat race medical evaluation.
He is healthy and has no significant medical history. 5. A 28-year-old man with no past medical history
However, his brother died suddenly during football and not on medications presents to the emergency
practice at age 15. Which one of the following is the department with palpitations for several hours and
most likely cause of the brother’s death? was found to have supraventricular tachycardia.
a. Aortic stenosis Carotid massage was attempted without success.
b. Congenital long QT syndrome What is the treatment of choice to stop the attack?
c. Congenital short QT syndrome a. Intravenous (IV) lignocaine
d. Hypertrophic cardiomyopathy (HCM) b. IV digoxin
e. Wolff–Parkinson–White syndrome c. IV amiodarone
d. IV adenosine
2. A 65-year-old man presents to the heart failure e. IV quinidine
outpatient clinic with increased shortness of breath
and swollen ankles. On examination his pulse was
6. A 75-year-old cigarette smoker with known ischaemic
100 beats/min, blood pressure 100/60 mmHg
heart disease and a history of cardiac failure presents
and jugular venous pressure (JVP) +10 cm water.
to the emergency department with a 6-hour history of
The patient currently takes furosemide 40 mg BD,
increasing dyspnoea. His ECG shows a narrow complex
spironolactone 12.5 mg, bisoprolol 2.5 mg OD and
regular tachycardia with a rate of 160 beats/min. What is
ramipril 2.5 mg BD. Which of the following is true?
the most appropriate initial step in patient care?
a. Diuretics reduce the degree of neurohormonal
a. His heart rate should be slowed using IV atenolol
activation in heart failure
to aid in the diagnosis of the rhythm
b. Diuretics alone rarely provide rapid symptomatic relief
b. He should be given a single IV dose of lignocaine
c. Beta-blockers are contraindicated
(50 mg) followed by an infusion at 4 mg/min
d. Treatment with spironolactone reduces mortality
c. He should be given IV adenosine to aid in the
e. Angiotensin-converting enzyme (ACE) inhibitors
diagnosis of the rhythm
are unlikely to induce hypotension if an adequate
d. His rhythm may represent a ventricular tachycardia
diuresis has been achieved with diuretics
and he should be immediately cardioverted
3. A 73-year-old woman complains of sudden- e. He should not be given high flow oxygen
onset chest pain. On examination you note her
pulse is regular at 53 beats/min. You request an 7. A 57-year-old man comes to his general practitioner
electrocardiogram (ECG) which demonstrates a time (GP) concerned about his general health. He is
interval of 3 seconds between each consecutive P particularly worried as there is a strong family history
wave. What is the most likely diagnosis? of heart disease. The GP performs an ECG which
a. Sick sinus syndrome shows a prolonged PR interval of 0.3 seconds. What
b. First-degree atrioventricular (AV) block is the most likely diagnosis?
c. Mobitz type 1 block a. Sick sinus syndrome
d. Mobitz type 2 block b. First-degree AV block
e. Complete heart block c. Mobitz type 1 block
d. Mobitz type 2 block
4. A 63-year-old man complains of gradual-onset e. Complete heart block
chest pain. On examination you note his pulse is
regular at 60 beats/min. You request an ECG which 8. A 75-year-old man is referred by the GP to the
demonstrates an increasing PR interval which cardiology outpatient clinic. The patient has atrial
eventually culminates in an absent QRS complex after fibrillation (AF) and hypertension. Examination reveals
the P wave. What is the most likely diagnosis? a blood pressure of 124/80 mmHg and his pulse

7 
Single best answer (SBA) questions

varies between 90 and 130 beats/min. His last a. Sodium bicarbonate 50-mg IV injection
echocardiogram (done 1 month ago) revealed left b. Defibrillation with 200 J
atrial chamber dimensions that are increased above c. One ampoule injection of IV calcium chloride
normal. What is the most effective management? d. One ampoule injection of IV adrenaline
a. Adenosine e. Carotid artery compression
b. No medication
c. Digoxin alone 13. You are a medical student attending a teaching
d. Cardioversion session on ECG interpretation. The consultant tells
e. Digoxin, with warfarin you that a patient was admitted to the coronary
care unit with an atrial rate of 300 beats/min and a
9. You are a house officer on call when a nurse bleeps ventricular rate of 150 beats/min. What is the most
you to review an ECG. The ECG demonstrates an likely diagnosis?
intermittent absence of the QRS complex but no a. Atrial flutter
evidence of progressive PR interval increase. What is b. First-degree AV block
the most likely diagnosis? c. Mobitz type 1 block
a. Sick sinus syndrome d. Mobitz type 2 block
b. First-degree AV block e. Complete heart block
c. Mobitz type 1 block
d. Mobitz type 2 block 14. A 50-year-old lawyer presents to the cardiology
e. Complete heart block outpatient clinic with recurrent episodes of dull
central chest pain radiating up to her jaw and down
10. A 48-year-old woman attends her GP for a her left shoulder, associated with sweating and
routine health check. She was found to have high nausea following exertion and alleviated by rest.
cholesterol (at 5.8 mmol/L) and high triglyceride She is a heavy smoker and her father died of a
level (at 2.7 mmol/L). She is a nonsmoker heart attack in his 40s. What is the most definitive
and consumes 10 units of alcohol per week. diagnostic test in this case?
Her body mass index is 29 kg/m2 and she is a. Echocardiography
otherwise well with no personal or family history b. Stress testing with echocardiography
of illnesses. Clinical examination and vital signs c. Stress testing with myocardial perfusion
were unremarkable. Her calculated QRISK is scintigraphy
12%. Which one of the following in her initial d. Coronary angiography
management is LEAST likely needed? e. Electrocardiography during an attack
a. Low carbohydrate diet
b. High consumption of monounsaturated fats 15. You are a house officer on call when you are asked
c. Regular aerobic exercise to review a patient in the coronary care unit. You
d. Start medical therapy with statins note that the patient has an atrial rate of 300 beats/
e. Decrease body mass index to <25 min and a ventricular rate of 150 beats/min. The
nurse says this is of new onset. Which management
11. You are a house officer on call when you are bleeped plan would you instigate first?
to see a patient complaining of dizziness. On further a. Sotalol
questioning the patient comments that he has b. Radiofrequency catheter ablation
blacked out on several occasions in the past. You c. Procainamide
perform an ECG which shows regular P and QRS d. Lignocaine
complexes occurring independently of one another. e. Electrical cardioversion
What is the most likely diagnosis?
a. Sick sinus syndrome 16. A previously fit and well 36-year-old woman
b. First-degree AV block presents to her GP with a 2-month history of
c. Mobitz type 1 block palpitations and sweats. These attacks come
d. Mobitz type 2 block on suddenly and are unrelated to food intake,
e. Complete heart block supine posture or exercise. She is a nonsmoker
and does not drink alcohol. Which of the following
12. You are called to see a 48-year-old man who is investigations is the LEAST useful to evaluate this
found to be unresponsive by the nursing staff. You patient?
cannot obtain a palpable carotid pulse and the a. Thyroid function tests
cardiac monitor shows ventricular fibrillation. What is b. Urinary collection for catecholamines
the first appropriate measure? c. Holter monitor

8
Cardiology 1

d. Echocardiography complexes on the cardiac monitor but no evidence


e. Upper gastrointestinal study of a pulse. What is the next most appropriate step in
management?
17. A 57-year-old Bangladeshi patient is admitted to a. Administration of colloid
accident and emergency (A&E) with palpitations. b. DC shock 200 J
He speaks very little English but tells you he has c. Amiodarone 300 mg IV
a ‘heart problem’. On examination you note his d. Adrenaline 1 mg IV
pulse is irregular at 145 beats/min. An ECG is e. Atropine 1 mg IV
performed which demonstrates no P waves. Which
management plan would you instigate first? 21. You are asked to review an 89-year-old arteriopath
a. Bisoprolol who complains of severe pain at rest in both feet,
b. Flecainide the toes of which are cold and purple. He has had a
c. Radiofrequency catheter ablation recent course of heparin and aspirin for suspected
d. Procainamide unstable angina. The major arterial pulses are
e. Lignocaine present in the limbs, but there are areas of lace-
like purplish discolouration on the skin over both
18. A 55-year-old man presents to a local district general knees. What is the most likely explanation for these
hospital A&E with acute central chest pain, radiating findings?
to his neck and left shoulder, and associated a. Allergic reaction to the heparin
with one episode of vomiting. His medical history b. Cholesterol embolism
includes hypertension and hyperlipidaemia. ECG c. Haemorrhage from the anticoagulants
and troponin show evidence of an acute myocardial d. Heparin-induced thrombocytopaenia
infarction (MI). Primary percutaneous coronary e. Sepsis
intervention is unavailable within close distance;
therefore a decision is made to perform thrombolytic 22. A 65-year-old man has NYHA class III chronic
therapy. This therapy is most beneficial when done heart failure. Despite conventional therapy with
within which of the following time limits from the appropriate dosages of a diuretic, an ACE inhibitor
onset of pain? and a beta-adrenergic blocker, his left ventricular
a. Within 2 days ejection fraction hovers around 35%, and he
b. Within 18 hours continues to have shortness of breath on exertion.
c. Within 12 hours His latest urea and electrolytes test results were
d. Within 24 hours normal and you consider adding digoxin to his
e. Within 36 hours treatment regimen. Which one of the following is true
regarding digoxin therapy in this situation?
19. A 37-year-old woman is complaining of palpitations a. A reasonable dosage is 0.50 mg/day orally
following recent surgery. On examination her pulse b. Serial drug levels are generally not necessary
rate is regular at 175 beats/min and blood pressure c. A loading dose will be necessary
is stable at 125/70 mmHg. She tells you she was d. It is not likely to improve the ejection fractions
seen the night before by the on-call cardiologist for e. It is the treatment of choice if the patient’s ECG
the same problem. Her past medical history includes shows AV block
asthma. You request an ECG which shows normal-
shaped QRS complexes but no P waves. What is 23. A 42-year-old man presents with shortness of
the next most appropriate step in management? breath and haemoptysis. On examination you note
a. Adenosine a loud first heart sound (HS) and a rumbling mid-
b. Verapamil diastolic murmur at the apex. What is the most likely
c. Carotid sinus massage diagnosis?
d. Direct current (DC) cardioversion a. Mitral stenosis
e. Flecainide b. Mitral regurgitation
c. Aortic stenosis
20. While on call you are bleeped to see a patient d. Aortic regurgitation
who has had a massive bleed PR. On examination e. Tricuspid stenosis
you note he is drowsy with a blood pressure of
65/40 mmHg and pulse rate of 130 beats/min. While 24. A 16-year-old boy athlete is brought to A&E after
you obtain IV access, he arrests. The nurse calls the he collapsed during football training. On arrival, he
crash team and you commence cardiopulmonary appears alert and orientated. His vital signs and
resuscitation (CPR). After 2 minutes, you note QRS physical examination are normal. He remembers

9 
Single best answer (SBA) questions

feeling dizzy prior to collapsing but cannot recall radiating to his neck. What is the most likely
much else. He is normally healthy and not on any diagnosis?
regular medications. Blood tests have been sent. a. Mitral stenosis
What is the most appropriate next step for this b. Mitral regurgitation
patient? c. Aortic stenosis
a. Admit him for cardiac monitoring d. Aortic regurgitation
b. Request a neurology review e. Tricuspid stenosis
c. Order an ECG
d. Order electroencephalography (EEG) 29. A 50-year-old man has persistently elevated
e. Schedule a tilt-table test blood pressure of about 185/110 mmHg. He
has been complaining of headaches for a few
25. A 55-year-old woman presents with shortness of weeks but otherwise well. ECG reveals abnormal
breath on exertion and fatigue. On examination you voltage changes and ST segment depression
note a soft first HS and a pansystolic murmur at the in the left ventricular leads. He was also found
apex radiating to her axilla. What is the most likely to have evidence of arteriovenous nipping on
diagnosis? fundoscopy. What is the most appropriate
a. Mitral stenosis management?
b. Mitral regurgitation a. Advise weight loss and see again in 2 months
c. Aortic stenosis b. Arrange an exercise stress test
d. Aortic regurgitation c. Hospitalize and give urgent IV antihypertensive
e. Tricuspid stenosis medication
d. Begin oral antihypertensives and see in
26. A man has been diagnosed with essential 2 months
hypertension by his GP and needs to commence e. Begin oral antihypertensives and see in 3 days
on the appropriate treatment. In the treatment
of essential hypertension, which of the following 30. A 55-year-old woman presents with dyspnoea and
statements is true? orthopnoea. On examination you note a blowing
a. Alpha-blockers such as prazosin are unlikely to early diastolic murmur at the left sternal edge. What
cause postural hypotension is the most likely diagnosis?
b. Patients of Afro-Caribbean descent respond well a. Mitral stenosis
to ACE inhibitors b. Mitral regurgitation
c. ACE inhibitors are indicated if there is unilateral c. Aortic stenosis
renal artery stenosis d. Aortic regurgitation
d. Treatment with bendrofluazide may result in e. Tricuspid stenosis
hypokalaemia
e. ACE inhibitors may cause ankle swelling as a 31. A 42-year-old man is reviewed in the outpatient
side effect cardiology department following a recent
echocardiogram (echo). The echo demonstrates
27. An otherwise healthy 50-year-old male presents right ventricular dysfunction. On examination you
to the GP with palpitations and is noted to note an elevated JVP in addition to a pansystolic
have an irregular heartbeat. He is otherwise fit murmur at the lower left sternal edge. What is the
and healthy. This resolves without treatment. most likely diagnosis?
Total duration was less than 2 hours. Full blood a. Mitral stenosis
count, metabolic profile, thyroid studies, ECG b. Mitral regurgitation
and echocardiogram were all normal. Which one c. Tricuspid regurgitation
of the following would be the most appropriate d. Aortic stenosis
treatment? e. Aortic regurgitation
a. Aspirin
b. Clopidogrel 32. A 50-year-old Caucasian man sees his GP
c. Do nothing for a blood pressure check as he has recently
d. Dipyridamole been diagnosed with essential hypertension.
e. Warfarin His only other medical history is type II diabetes
mellitus with no end-organ damage, which is
28. A 62-year-old man presents with angina and well controlled on oral hypoglycaemic agents.
dyspnoea. On examination you note an ejection His blood pressure has been 145/90 mmHg
systolic murmur at the upper right sternal border persistently despite diet and exercise for 3 months.

10
Cardiology 1

Which of the following is the first-line medication heart failure. Which investigation is most likely to
for blood pressure control in this patient? demonstrate the aetiology?
a. Start on losartan a. CXR
b. Start on bisoprolol b. Echocardiogram
c. Start on thiazide diuretic c. ECG
d. Start on nifedipine d. Coronary angiography
e. Start on enalapril e. 24-hour ECG

33. A 62-year-old man presents with angina and 37. A 46-year-old woman presents with shortness of
dyspnoea. On examination you note a murmur breath. She comments that she feels particularly
at the right upper sternal border radiating to his breathless when lying down and uses five pillows
neck. Which investigation is most likely to lead to a to sleep at night. On examination her pulse rate is
diagnosis? regular at 122 beats/min and her blood pressure is
a. Chest X-ray (CXR) stable at 122/65 mmHg. You note bibasal crackles
b. Echocardiogram when listening to her chest. You suspect heart failure
c. ECG and order a CXR and echocardiogram. The following
d. Coronary angiography may all be features on her CXR EXCEPT?
e. 24-hour ECG a. Kerley B lines
b. Hilar haziness
34. A 42-year-old man presents with shortness of breath c. Fluid in the left horizontal interlobar fissure
and haemoptysis. On examination you note a loud d. Upper lobe venous engorgement
first HS and a rumbling mid-diastolic murmur at e. Cardiomegaly
the apex. You request an echocardiogram and a
CXR. The following are all features of the CXR in this 38. A 55-year-old banker is admitted to A&E
condition EXCEPT? complaining of sudden-onset shortness of breath.
a. Large right atrium Following an echocardiogram, he is diagnosed with
b. Large left atrium heart failure. What is the most likely aetiological
c. Kerley B lines cause for cardiac failure in the Western world?
d. Pulmonary venous hypertension a. Ischaemic heart disease
e. Narrowed carina b. Hypertension
c. Valvular dysfunction
35. A 50-year-old man presents to the emergency d. Cardiomyopathy
department with a mild headache. He is known e. Arrhythmia
to have poorly controlled hypertension and is
on multiple antihypertensive medications. On 39. A 66-year-old man with heart failure secondary to
examination his blood pressure is 205/110 mmHg previous MIs is on regular digoxin and furosemide
and there is no evidence of end-organ involvement. without much effect. A recent echocardiogram
His headache is still present but there is no focal shows global dysfunction. His renal function is
neurology identified. Which of the following is the normal. Which medication should be added to his
least appropriate in the management of this patient’s management?
blood pressure? a. ACE inhibitors
a. Rapidly lowering blood pressure in the b. Verapamil
emergency department c. Disopyramide
b. Adjust treatment and follow-up within 24 to 48 d. Phosphodiesterase inhibitors
hours of presentation in the community e. Propranolol
c. Initiate a maintenance dose of an oral medication
before discharge 40. A 65-year-old man presents with shortness of
d. Consider a short observation period before breath at rest. Physical examination reveals
discharge evidence of pitting oedema over the sacrum and
e. Discharge the patient, emphasizing the leg oedema to the midthigh. An echocardiogram is
importance of close follow-up and compliance requested which demonstrates an ejection fraction
with medications of 0.4. What is the next most appropriate step in
management?
36. A 55-year-old man presents to A&E with shortness a. ACE inhibitor
of breath and fatigue. On examination you note b. Angiotensin II receptor antagonist
pitting oedema in both his lower limbs. You suspect c. Beta-blocker

11 
Single best answer (SBA) questions

d. Calcium channel antagonist A&E department with chest pain made worse on
e. Diuretic breathing. On examination he is febrile, and has a
‘scratchy’ sound heard on auscultation in systole
41. A 72-year-old man with known cardiac failure and diastole. ECG reveals widespread saddle-
comes to see his GP. He is currently on a diuretic. shaped ST segment elevation. What is the most
His GP decides to commence an ACE inhibitor. likely diagnosis?
The following are all side effects of ACE inhibitors a. Unstable angina
EXCEPT? b. Hyperventilation syndrome
a. Renal failure c. Pulmonary emboli
b. Hypokalaemia d. Evolving cardiac infarction
c. Rash e. Acute pericarditis
d. Angioedema
e. Cough 46. A 56-year-old woman presents to A&E complaining
of sudden-onset central chest pain. She comments
42. A 64-year-old man with known cardiac failure that it came on at rest. She is referred for an
presents with acute shortness of breath and a cough exercise test and coronary angiogram. The exercise
productive of frothy pink sputum. He comments test is stopped early due to chest pain and ST
that he feels nauseous. On examination you note depression in the lateral leads. However, she has
wheezes and crackles throughout his chest. What is normal coronary arteries on the angiogram. What is
the next most appropriate step in management? the most likely diagnosis?
a. Glyceryl trinitrate (GTN) infusion a. Decubitus angina
b. Aminophylline b. Prinzmetal angina
c. Metoclopramide c. Cardiac syndrome X
d. Furosemide d. Unstable angina
e. Mechanical ventilation e. Stable angina

43. A 44-year-old female comes to your office 47. A 55-year-old man has been recently diagnosed with
with chest pain of several days’ duration. She hypertension and commenced on treatment. He
describes the pain as sharp and stabbing, and attends his GP to find out more about the condition.
indicates that it is located at the left sternal Which of the following is the commonest cause of
border; it is increased by coughing and palpation. hypertension?
There is no family history of heart disease, nor is a. Polycystic kidney disease
there a personal history of diabetes, hypertension, b. Renin-producing tumour
smoking or hyperlipidaemia. A physical c. Undetermined cause
examination, an ECG and chest radiographs d. Oral contraceptive pills
are all normal. Further diagnostic studies should e. Coarctation of aorta
include which of the following?
a. A treadmill exercise test with ECG and blood 48. You are on call in A&E when a patient with sudden-
pressure monitoring onset chest pain is admitted. You request an ECG
b. A stress echocardiogram which shows evidence of T-wave inversion in
c. Referral for cardiac catheterization leads V2–V6 but no evidence of ST elevation. You
d. Stress myocardial perfusion imaging prescribe aspirin, enoxaparin and clopidogrel. What
e. No additional tests is the most appropriate dose of enoxaparin?
a. 40 mg once a day
44. A 53-year-old man comes to see his GP complaining b. 20 mg once a day
of chest pain. The GP performs an ECG which c. 1 mg/kg once a day
demonstrates ischaemic changes. The following are d. 1.5 mg/kg twice a day
all risk factors for ischaemic heart disease EXCEPT? e. 1 mg/kg twice a day
a. Hyperlipidaemia
b. Smoking 49. A previously fit and well 30-year-old gym instructor
c. Hypertension is admitted to hospital following a stab wound to
d. High homocysteine levels the left side of his chest. He has a blood pressure
e. High dietary folic acid intake of 80/45 mmHg and heart rate of 125 beats/min.
Breath sounds are present throughout but his HSs
45. A 23-year-old man who has recently recovered from are muffled. His JVP appears raised. What additional
an upper respiratory tract infection presents to the feature is most likely to be present in this patient?

12
Cardiology 1

a. Diastolic murmur over the praecordium radiating 54. A 42-year-old lawyer presents with sudden-onset
towards the left axilla chest pain. You suspect a myocardial infarct and
b. S3 explain to the patient that you need to measure a
c. Wide-split S2 protein from his heart to confirm your suspicion.
d. An 18-mmHg blood pressure drop during Which serum investigation is most likely to lead to a
inspiration diagnosis?
e. An 18-mmHg blood pressure drop during a. Troponin
expiration b. Creatine kinase
c. Myoglobin
50. You are a house officer on call when you are asked d. Aspartate aminotransferase
to see a patient with chest pain. The patient tells e. Lactic dehydrogenase
you that he suffers from angina and this pain is
similar to previous episodes of chest pain. An ECG 55. Which ECG change would lead you to initiate
is performed which rules out an MI. What is the next thrombolytic therapy?
most appropriate step in management? a. Right bundle-branch block
a. Atenolol b. ST segment elevation in left ventricular leads
b. Glyceryl trinitrate (GTN) c. Widespread ST segment depression
c. Diltiazem d. Q waves in septal leads
d. Nicorandil e. Prolongation of the QT interval
e. Aspirin
56. A 32-year-old obese woman presents to the GP with
51. An ECG is performed on a 45-year-old man as part sudden-onset crushing chest pain and nausea. The
of a routine medical check-up. His past medical GP is concerned about a myocardial infarct and calls
history includes a previous MI some years ago. an ambulance. What is the next most appropriate
The ECG shows presence of Q waves. Q waves in step in management?
inferior MI appear in which ECG leads? a. Metoprolol
a. I and II b. Morphine
b. II, III and aVF c. Metoclopramide
c. V1, V2 and V3 d. Aspirin
d. I, aVF and V6 e. Streptokinase
e. V4, V5 and V6
57. A 45-year-old man attends the emergency
52. A 56-year-old man presents with central chest department with worsening breathlessness and
pain at rest. He comments that he feels short fevers. The medical team suspects a diagnosis of
of breath and nauseous. An ECG demonstrates infective endocarditis. Which of the following is the
tall R waves, ST segment depression and tall T most suggestive feature of infective endocarditis?
waves in leads V1 and V2. What is the most likely a. Appearing and changing murmurs
diagnosis? b. Erythema marginatum
a. Anterior myocardial infarct c. Juxta-articular nodes
b. Lateral myocardial infarct d. Atrial fibrillation
c. Inferior myocardial infarct e. Lymphadenopathy
d. Anterio-lateral myocardial infarct
e. Posterior myocardial infarct 58. A 43-year-old man presents with central chest pain.
He is diagnosed with an acute MI and referred for
53. A 21-year-old fit university student with no past immediate angioplasty. The following are all late
medical history has sudden loss of consciousness complications of a myocardial infarct EXCEPT?
1 hour into a gym session. CPR is administered by a. Mitral valve regurgitation
bystanders. On arrival of the paramedics, he has b. Thromboembolism
regained consciousness. What is the most effective c. Ventricular aneurysm
means of preventing sudden death in high-risk d. Pericarditis
patients with asymptomatic HCM? e. Ventricular septal rupture
a. Amiodarone
b. An implantable cardioverter–defibrillator (ICD) 59. An elderly woman complains of abdominal
c. Chronic dual-chamber pacing distension, due to ascites, upper abdominal pain
d. Metoprolol from an enlarged tender liver and flatulence. She is
e. Verapamil mildly jaundiced and has marked pitting of the legs.

13 
Single best answer (SBA) questions

Her arterial pulse is rapid and irregular, and she is d. Osler nodes
sitting propped up in bed where it is evident that her e. Splinter haemorrhages
face is cyanosed, and her jugular veins are engorged
and pulsating in systole. She is passing little urine 64. A 35-year-old man complains of fatigue, night sweats
and the urine contains 2+ of protein. What is the and weight loss. On examination you note a systolic
most likely diagnosis? murmur and evidence of splenomegaly. Which
a. Cirrhosis of the liver investigation is most likely to lead to a diagnosis?
b. Congestive heart failure a. Serology testing
c. Malignant ascites b. CXR
d. Nephrotic syndrome c. ECG
e. Superior mediastinal obstruction d. Full blood count
e. Blood cultures
60. A 35-year-old IV drug abuser presents with night
sweats and weight loss. On examination he is pyrexial 65. A 31-year-old man presents with several months of
with a temperature of 38.5°C. You suspect a diagnosis shortness of breath and chest discomfort. An ECG
of infective endocarditis. Which of the following shows evidence of T-wave flattening. You request
organisms is most likely to account for his condition? an echocardiogram which demonstrates dilated
a. Streptococcus mutans ventricles with global hypokinesis. What is the most
b. Streptococcus sanguis likely diagnosis?
c. Enterococcus faecalis a. Dilated cardiomyopathy
d. Staphylococcus aureus b. Hypertrophic cardiomyopathy
e. Coxiella burnetii c. Restrictive cardiomyopathy
d. Arrhythmogenic right ventricular
61. A 31-year-old woman presents with night sweats. cardiomyopathy
On examination you detect a systolic murmur. She e. Myocarditis
has severe gum and tooth decay and has recently
had two teeth taken out. Which of the following 66. A 20-year-old student sees his GP for blurred vision.
organisms is most likely to account for her condition? On further questioning, he reports he had a ‘lens
a. Streptococcus mutans problem’ a year ago. On examination he also has
b. Streptococcus sanguis pectus excavatum, a high arched palate and signs of
c. Enterococcus faecalis hypermobility. Which of the following cardiac disorders
d. Staphylococcus aureus is often associated with this patient’s condition?
e. Coxiella burnetii a. Aortic valve stenosis
b. Coarctation of the aorta
62. A 40-year-old man with an underlying cardiac c. Mitral valve prolapse
condition is about to undergo a root canal treatment d. Ventricular septal defect
with a new crown to be fitted in the dentist’s office. e. Ebstein anomaly
Which one of the following conditions strongly
warrants antibiotics prophylaxis? 67. A 27-year-old footballer presents with laboured
a. Atrial septal defect breathing and chest pain. An ECG shows evidence
b. Ventricular septal defect of left ventricular hypertrophy. What is the most likely
c. Previous coronary artery bypass grafting involving diagnosis?
the use of the internal mammary arteries a. Dilated cardiomyopathy
d. Presence of a dual chamber pacemaker b. Hypertrophic cardiomyopathy
e. None of the above c. Restrictive cardiomyopathy
d. Arrhythmogenic right ventricular cardiomyopathy
63. You are attending a seminar on infective e. Myocarditis
endocarditis. The lecturer comments that some
clinical features of the condition result from 68. A 42-year-old woman presents with chest pain.
immune complex deposition. The following She comments that the pain is worse on inspiration
features are all the result of immune complex but is relieved by leaning forward. Her past medical
deposition EXCEPT? history includes breast carcinoma. What is the most
a. Splenic abscess likely diagnosis?
b. Janeway lesions a. Acute pericarditis
c. Roth spots b. Pericardial effusion

14
Cardiology 1

c. Pericardial tamponade d. Serological evidence of active infection with an


d. Constrictive pericarditis organism consistent with infective endocarditis (IE)
e. Myocarditis e. Definite diagnosis can be made with three minor
criteria
69. A 65-year-old man sees his GP with worsening
central chest pain and breathlessness following 73. A 65-year-old hypertensive patient is reviewed in the
exertion for 3 months. These symptoms usually outpatient clinic complaining of visual disturbances.
resolve with rest. Lately he has also been feeling On examination you note flame-shaped
dizzy. On examination a systolic murmur is heard. haemorrhages. What is the most likely diagnosis?
ECG shows large R waves in leads I, aVL and V4–6, a. Grade 1 retinal disease
and large R waves in leads V1–3. What is the most b. Grade 2 retinal disease
likely diagnosis? c. Grade 3 retinal disease
a. Myocardial infarction d. Grade 4 retinal disease
b. Unstable angina e. Grade 5 retinal disease
c. Stable angina
d. Aortic stenosis 74. A 45-year-old teacher with known mitral valve
e. Hypertrophic cardiomyopathy prolapse presents to his GP with a 2-week history
of fevers, rigors, chest pain and breathlessness. He
70. A 43-year-old obese woman with known breast recently underwent a tooth extraction. On examination
cancer complains of shortness of breath. On he is febrile with multiple splinter haemorrhages.
examination you note soft HSs and obscuration of A systolic murmur is heard and appears to have
the apex beat. Which investigation is most likely to increased in intensity since his last GP visit. What is
lead to a diagnosis? the most likely causative agent in this case?
a. CXR a. Candida albicans
b. Echocardiogram b. Staphylococcus aureus
c. ECG c. Pseudomonas aeruginosa
d. Arterial blood gas d. Streptococcus viridans
e. Coronary angiography e. Enterococcus

71. A 63-year-old retired doctor presents with 75. A 64-year-old female comes to your surgery with
nausea and headaches. His past medical history a past medical history of hypertension. She is on
includes type II diabetes mellitus. On examination an ACE inhibitor as well as on a range of other
you note his blood pressure is 165/76 mmHg. medications. Which one of the following would be
What is the next most appropriate step in most likely to blunt the antihypertensive effects of an
management? ACE inhibitor?
a. Beta-blockers a. Chlorpromazine
b. Calcium channel antagonists b. Allopurinol
c. ACE inhibitors c. Ibuprofen
d. Diuretics d. Spironolactone
e. Alpha-blockers e. Paracetamol

72. A previously fit and well 45-year-old man presents


to A&E with fevers and malaise. On examination
EXTENDED-MATCHING
he has multiple splinter haemorrhages and
a systolic murmur. A diagnosis of infective QUESTIONS (EMQs)
endocarditis is suspected. Dukes criteria is used
to help with the diagnosis of the condition. Which Cardiac murmurs and added sounds
of the following statements is INCORRECT in Each answer can be used once, more than once or not
Dukes criteria? at all.
a. Presence of Osler nodes is a minor a. Aortic regurgitation
criterion b. Aortic stenosis
b. Positive blood for rheumatoid factor is a minor c. Bicuspid aortic valve
criterion d. Mitral regurgitation
c. Echocardiographic evidence of prosthetic valve e. Mitral stenosis
dehiscence is a major criterion f. Patent ductus arteriosus

15 
Extended-matching questions (EMQs)

g. Transposition of the great vessels Drugs


h. Tricuspid regurgitation Each answer can be used once, more than once or not
i. Ventricular septal defect at all.
a. Propranolol
For each scenario below, choose the most likely
b. Nicorandil
corresponding option from the list given above.
c. Aspirin
1. A 60-year-old man presents with heart failure. d. Nifedipine
On examination he has a collapsing pulse, an e. Captopril
early diastolic murmur and a displaced apex f. Digoxin
beat. g. Bisoprolol
2. A 56-year-old man gives a history of rheumatic h. Bendrofluazide
fever. He has flushed cheeks, an irregularly i. Clopidogrel
irregular pulse and a mid-diastolic murmur. j. Simvastatin
3. A 75-year-old woman presents to A&E
following a drop attack. Her ECG shows left For each scenario below, choose the most likely
ventricular hypertrophy. On examination, a corresponding option from the list given above.
harsh systolic murmur is heard over both
1. Used in the treatment of cardiac failure. Known to
carotid arteries.
cause a persistent cough.
4. Three days following his MI, Mr Thompson has a
2. Contraindicated in renal artery stenosis.
sudden deterioration, developing left ventricular
3. Associated with bradycardia, headaches and fluid
failure. He has a new pansystolic murmur radiating
retention.
from the apex to the axilla.
4. First-line management in acute coronary
5. A baby who is cyanotic at birth with a loud long
syndrome.
systolic murmur in whom tetralogy of Fallot is
5. Specifically contraindicated in asthmatics and
suspected.
individuals with peripheral vascular disease.

Chest pain Clinical features of cardiac disease


Each answer can be used once, more than once or not Each answer can be used once, more than once or not
at all. at all.
a. Angina pectoris a. Aortic stenosis
b. Pericarditis b. Aortic regurgitation
c. Myocardial infarction c. Mitral stenosis
d. Aortic dissection d. Mitral regurgitation
e. Reflux oesophagitis e. Infective endocarditis
f. Pulmonary infarct f. Rheumatic fever
g. Pneumonia g. Tricuspid stenosis
h. Costochondritis h. Tricuspid regurgitation
i. Pneumothorax i. Pulmonary hypertension
j. Pulmonary embolus j. Pulmonary embolism

For each scenario below, choose the most likely For each scenario below, choose the most likely
corresponding option from the list given above. corresponding option from the list given above.
1. A middle-aged man presenting with a 1. A pansystolic murmur auscultated at the apex and
central crushing chest pain on exercise but radiating into the axilla.
relieved by rest. 2. Associated with erythematous macules
2. Associated with central crushing chest pain on the palms and haemorrhages under
which occurs at rest and may radiate to the jaw the nails.
or arms. 3. Associated with pink-coloured rings on the trunk
3. Severe tearing chest pain which may radiate to and subcutaneous nodules over the joints.
the back. 4. Associated with a slow rising carotid pulse and
4. Central chest discomfort which is sharp in nature ejection systolic murmur at the right upper sternal
with a tender area on palpation. border.
5. Sharp chest pain aggravated by movement, 5. Known to cause a right parasternal heave and a
respiration and changes in posture. loud pulmonary second sound.

16
Cardiology 1

Murmurs 2. A patient complains of waking in the middle of


Each answer can be used once, more than once or not the night and racing out of bed to the window,
at all. gasping for breath.
3. A patient sleeps on four pillows at night so he can
a. Aortic regurgitation
catch his breath.
b. Aortic stenosis
4. A patient with a diagnosis of heart failure
c. Austin–Flint murmur
complains of this symptom several weeks after
d. Flow murmur
starting an ACE inhibitor.
e. Graham Steell murmur
5. Congestion due to right heart failure can cause a
f. Mitral regurgitation
nutmeg appearance and this symptom.
g. Mitral stenosis
h. Tricuspid regurgitation
i. Tricuspid stenosis Electrocardiograph findings
j. Ventricular septal defect Each answer can be used once, more than once or not
at all.
For each scenario below, choose the most likely a. Anterior myocardial infarction
corresponding option from the list given above. b. Atrial fibrillation
c. Atrial flutter with 2:1 block
1. A 60-year-old Egyptian man with a pansystolic
d. Digitalis effect
murmur loudest at the right sternal edge, a raised
e. Inferior myocardial infarction
JVP and a pulsatile liver.
f. Pericarditis
2. A 70-year-old woman in atrial fibrillation, an apex
g. Posterior myocardial infarction
beat palpable in the mid-axillary line and a loud
h. Sick sinus syndrome
pansystolic murmur which is heard all over the
i. Third-degree heart block
heart.
j. Ventricular tachycardia
3. A 14-year-old girl with a flat occiput, single
palmar crease and a pansystolic murmur.
For each scenario below, choose the most likely
She has a chromosomal abnormality affecting
corresponding option from the list given above.
chromosome 21.
4. An elderly gentleman with a collapsing pulse and 1. ST depression in V1–3 and tall R wave in V1 and V2.
an early diastolic murmur which is loudest when 2. ST elevation that is present in all leads and is
sitting forward. saddle shaped.
5. A diastolic murmur which is early and continues 3. ST elevation of 3 mm in leads II, III and aVF.
past mid-diastole. 4. Broad complex QRS with a rate of 45 beats/min;
P waves are present.
5. No P waves, narrow complex QRS rate of 150
Symptoms of heart failure
beats/min with sawtooth baseline.
Each answer can be used once, more than once or not
at all.
a. Ankle swelling Heart disease
b. Cough Each answer can be used once, more than once or not
c. Hepatomegaly at all.
d. Orthopnoea a. Acute myocardial infarction
e. Paroxysmal nocturnal dyspnoea b. Aortic stenosis
f. Pink frothy sputum c. Atrial myxoma
g. Pulmonary fibrosis d. Complete heart block
h. Pulmonary oedema e. Cor pulmonale
i. Shortness of breath f. Dilated cardiomyopathy
g. Fibrinous pericarditis
For each scenario below, choose the most likely h. Infective endocarditis
corresponding option from the list given above. i. Mitral valve stenosis
1. A patient has been on a cardiology ward for j. Tricuspid incompetence
3 weeks with a diagnosis of pulmonary oedema. He
is receiving large doses of furosemide. His systolic For each scenario below, choose the most likely
blood pressure is 74 mmHg and increases on being corresponding option from the list given above.
placed head down in bed. On auscultation he has 1. A 58-year-old woman with known asymptomatic
fine crackles throughout both lung fields. aortic stenosis presents with fever and feeling

17 
Extended-matching questions (EMQs)

generally unwell. She has just had three fillings uncle died in their 60s from ‘heart attacks’.
performed by her dentist. She smokes 30 cigarettes a day. ECG: sinus
2. A 49-year-old man presents to his GP with tachycardia and right bundle-branch block.
shortness of breath on walking up hills. He also D-dimer 0.71 mg/L, troponin I 0.01 mg/L. CXR: no
experiences central chest pain which is relieved by active disease (NAD).
resting, as well as occasional dizziness. He has a 2. A 58-year-old insulin-dependent diabetic is
loud systolic murmur. admitted at the request of his GP. He has been
3. On the second day post-MI, a 71-year-old man complaining of intermittent central chest pain
develops pleuritic sounding chest pain. This is for the past month. He is a poor historian but
relieved by ibuprofen. The ECG demonstrates claims he has not noticed any relationship to the
saddle-shaped ST changes. pain. He has suffered from AF and hypertension
4. A 63-year-old man presents to A&E with for 6 years. He currently takes digoxin, atenolol,
shortness of breath. He has a loud pansystolic diclofenac and simvastatin. O/E: body mass
murmur and a displaced apex beat. CXR index 31 kg/m2, no chest wall tenderness, HS I
demonstrates a straight left heart border and + II + ejection systolic murmur in aortic area. No
cardiothoracic ratio of >50%. He admits to clinical evidence of deep vein thrombosis. HbA1C
previous excess alcohol intake. 10.5%, random glucose 13.2 mmol/L, digoxin
5. A 44-year-old woman attends her respiratory level 1.3 mmol/L. ECG: 1-mm depression in leads
clinic appointment. She has known chronic V1–V6. CXR: NAD.
obstructive pulmonary disease and is a 3. A 45-year-old Moroccan man (working in the
previous heavy smoker. Her most recent local chicken factory) attended A&E with central
symptoms are ankle swelling and an exercise chest discomfort. His English is broken, but you
tolerance of 15 yards. establish that this has been a problem for the past
week and that it is sharp in nature (‘like a knife,
Chest pain doctor’). He has no recent trauma. Examination
Each answer can be used once, more than once or not is unremarkable. CXR: scarring of right apex and
at all. calcified hilar nodes. You return to the patient
and he confirms previously being treated for
a. Myocardial infarct
tuberculosis back in Morocco.
b. Pulmonary embolus
4. A 36-year-old man is admitted via A&E
c. Pericarditis
with right-sided chest pain associated with
d. Left lower lobe pneumonia
shortness of breath. There is no fever. O/E: Thin
e. Rib fracture
gentleman, smells of tobacco and alcohol. He is
f. Oesophageal reflux disease
uncooperative to examination. Reduced breath
g. Aortic dissection
sounds on the right side. Tender over the anterior
h. Costochondritis
chest wall. CXR: peripheral area of decreased lung
i. Angina pectoris
markings on the right side.
j. Herpes zoster
5. A 56-year-old insulin-dependent diabetic is
admitted at the request of a concerned relative
For each scenario below, choose the most likely
(a local doctor). He has been complaining of
corresponding option from the list given above.
niggling chest pain for the past 2 days, with
1. A 48-year-old housewife is admitted with an associated feeling of shortness of breath.
left-sided chest discomfort, sharp in nature, which O/E: apyrexial, no chest wall tenderness. Nil
is worse on coughing. She has no sputum. She adventitious sounds. HS I + II + 0. CXR: NAD.
takes the oral contraceptive pill and propranolol ECG: ST elevation 3-mm leads V4–V6 with
hydrochloride (Half-Inderal LA). Her father and T-wave inversion. Troponin 6.5 mg/L.

18
Gastroenterology 2
d. Vitamin B12 deficiency
SINGLE BEST ANSWER (SBA) e. Haemochromatosis
QUESTIONS
5. A 57-year-old alcohol-dependant lady presented for the
1. A 56-year-old man presents to the general practitioner first time to the accident and emergency department
(GP) complaining of weight loss of 2 kg over 3 months with abdominal pain and a distended abdomen.
and lethargy. On further questioning he reveals that Examination of the abdomen reveals the presence of
his stools are paler than normal, and his urine dark in spider naevi and shifting dullness evident. She is treated
colour. He denies being in any pain. On examination his for decompensated alcoholic liver disease. What vitamin
sclerae look yellow. What is the most likely diagnosis? should she be given intravenously?
a. Gallstones a. Vitamin A
b. Acute pancreatitis b. Vitamin B1
c. Autoimmune hepatitis c. Vitamin B12
d. Pancreatic cancer d. Vitamin C
e. Hepatocellular carcinoma e. Vitamin D
2. A 40-year-old gentleman has been suffering from
6. A 32-year-old lady with type 1 diabetes went to see
reflux symptoms for 1 year. He drinks approximately
her GP complaining of a 10-day history of pain when
40 units of alcohol a week and has recently managed
swallowing both solids and liquids. She had recently
to give up smoking. His GP sends him for an
completed a course of antibiotics for a urinary tract
upper gastrointestinal (GI) endoscopy for further
infection. She has been referred for an upper GI
investigation. This looked suspicious for Barrett
endoscopy. What is this most likely to show?
oesophagus, so a biopsy was taken. What would you
a. Scleroderma
expect the histology to show?
b. Oesophageal carcinoma
a. Columnar cells
c. Achalasia
b. Squamous cells
d. Oesophageal candidiasis
c. Glandular cells
e. Benign oesophageal stricture
d. Transitional cells
e. Connective tissue cells
7. A 50-year-old male attended accident and emergency
3. A 52-year-old female attended her local walk-in with a large-volume haematemesis. On examination
centre complaining of tiredness. On further he has gynaecomastia and spider naevi. Rectal (PR)
questioning she had pruritus. A full panel of blood examination revealed black stool. What is the most
tests were sent off, and she was informed later likely cause of his upper GI bleed?
in the week that one of the tests had come back a. Oesophageal varices
positive for mitochondrial autoantibodies. What is b. Oesophagitis
the diagnosis? c. Mallory–Weiss tear
a. Primary biliary cirrhosis (PBC) d. Duodenal ulcer
b. Secondary biliary cirrhosis e. Gastric carcinoma
c. Primary sclerosing cholangitis
d. Autoimmune hepatitis 8. A patient is brought into hospital after having a
e. Wilson disease witnessed seizure lasting 2 minutes. He is known to
have hepatic cirrhosis and has become confused and
4. A 58-year-old male went to the GP complaining of disorientated over the past few days. On examination
feeling more tired than usual. He also felt that he had he has a coarse flap with outstretched hands. What is
become more ‘tanned’ throughout the winter months. the most likely diagnosis?
On examination he had hepatomegaly. What is the a. Hyponatraemia
diagnosis? b. Hepatic encephalopathy
a. Wilson disease c. Carbon dioxide retention
b. Secondary biliary cirrhosis d. Epilepsy
c. Iron-deficient anaemia e. Illicit drug use

19 
Single best answer (SBA) questions

9. A 21-year-old male, who had been feeling unwell d. Colorectal cancer


with flu-like symptoms for the past week, went e. Pancreatic cancer
to see the GP as his eyes looked yellow. Routine
bloods showed a bilirubin of 68 μmol/L; however, 14. A 21-year-old female presented to the accident and
the rest of his liver function tests (LFTs) were normal. emergency department complaining of vomiting
What is the most likely diagnosis? blood. She had been out the previous night and
a. Viral hepatitis drank copious amounts of alcohol, and had
b. Rotor syndrome been vomiting several times throughout the day.
c. Gilbert syndrome Examination was unremarkable, and there was no
d. Dubin–Johnson syndrome evidence of black stools. A nurse witnessed a vomit
e. Gallstones and reported that there were just small amounts of
red streaks in it. What is the most likely diagnosis?
10. A 23-year-old medical student organizes her elective a. Alcoholic hepatitis
in India. Whilst there, she develops fever, diarrhoea b. Acute gastritis
and abdominal cramps. The symptoms persist for c. Gastric ulcer
a week before she visits a doctor for some advice. d. Oesophageal varices
What is the most likely cause? e. Mallory–Weiss tear
a. Giardiasis
b. Hepatitis A 15. A 23-year-old university student presented with general
c. Salmonellosis flu-like symptoms for 2 weeks including a sore throat.
d. Yersiniosis On examination she has enlarged cervical lymph nodes.
e. Rotavirus Routine blood tests show the following measurements:
bilirubin, 27 μmol/L; alanine transaminase (ALT), 494
11. A 51-year-old male was diagnosed last year with IU/L; gamma-glutamyl transpeptidase, 62 IU/L; white
chronic respiratory disease, which was fairly severe cell count (WCC), 13.3; lymphocytes, 6.0. What is the
and affected his mobility. This had come as a shock likely cause of the raised liver enzymes?
as he had never previously smoked. His LFTs had a. Gilbert syndrome
previously been mildly deranged. He went to see his b. Gallstone
GP as he looked more yellow than normal. What is c. Viral illness
the most likely cause? d. Autoimmune hepatitis
a. Gilbert syndrome e. Acute pancreatitis
b. Mirizzi syndrome
c. Haemochromatosis 16. A 50-year-old male was being treated on a surgical
d. α1-Antitrypsin deficiency assessment ward for acute pancreatitis. His initial
e. Alcoholic hepatitis GLASGOW score was 3. On ward round, 7 days
after admission, the consultant requested an
12. A 67-year-old lady presents with weight loss of 3 kg investigation to look for any evidence of pancreatic
over 4 months and intermittent abdominal pain. necrosis. What is the imaging used normally?
Examination is unremarkable apart from an enlarged a. Magnetic resonance imaging (MRI) abdomen
supraclavicular lymph node on the left side of the b. Ultrasound scan (USS) abdomen
body. What is the most likely diagnosis? c. Abdominal X-ray (AXR)
a. Cholangiocarcinoma d. Computed tomography (CT) abdomen
b. Pancreatic cancer e. Endoscopic ultrasound scan (EUS)
c. Sigmoid cancer
d. Hepatocellular carcinoma 17. A 47-year-old female presents with severe
e. Gastric cancer intermittent abdominal pain over the last 2 months.
She has had multiple previous admissions with
13. A 71-year-old male presents to the GP with weight acute pancreatitis. Her most recent CT scan showed
loss, lethargy and decreased appetite over the past calcified deposits in the pancreas. She denies any
6 months. The GP organizes for blood samples alcohol intake for the past 6 months. What is the
to be taken and requests tumour markers. most likely diagnosis?
Alpha-fetoprotein (AFP) comes back elevated at 58 a. Acute pancreatitis
g/L. What is this suggestive of? b. Chronic pancreatitis
a. Hepatocellular carcinoma c. Mesenteric ischaemia
b. Cholangiocarcinoma d. Pancreatic cancer
c. Ampullary adenoma e. Gastric ulcer

20
Gastroenterology 2

18. A 24-year-old male was referred for a colonoscopy d. Doxycycline


to investigate his symptoms of weight loss and e. Ciprofloxacin
diarrhoea. The endoscopist thought that they could
view skip lesions in the terminal ileum and took 23. A 32-year-old male of Asian origin presents with
several biopsies. What is the histology most likely weight loss, diarrhoea and stools that are difficult
to show? to flush away. He had a colonoscopy, and a biopsy
a. Ulcerative colitis later confirmed villous atrophy. He was started on
b. Coeliac disease a gluten-free diet; however, his symptoms failed to
c. Irritable bowel syndrome improve. What is the most likely diagnosis?
d. Crohn disease a. Tropical sprue
e. Bacterial enterocolitis b. Coeliac disease
c. Chronic pancreatitis
19. A 16-year-old female attends the GP surgery to d. Crohn disease
ask for some advice. She suffers from severe travel e. Irritable bowel syndrome
sickness. The GP prescribes an antiemetic. Which
one is likely to be the most effective? 24. A 70-year-old male has pancreatic cancer. He has
a. Levomepromazine recently been started on strong opioids to control
b. Ondansetron his pain. He is visited by the palliative care nurses
c. Metoclopramide and complains of difficulty in opening his bowels,
d. Domperidone stating that his stools are much harder than they
e. Cyclizine used to be. What is the most appropriate laxative to
put him on?
20. A 32-year-old male presented with abdominal pain a. Lactulose
and vomiting. He described the pain as a sudden b. Sodium docusate
onset, severe pain which radiates through to his c. Fybogel
back. He had an ultrasound earlier this year which d. MOVICOL
showed gallstones. You suspect acute pancreatitis. e. Senna
What investigation is used to confirm this?
a. Arterial blood gases 25. A 42-year-old alcoholic presents with heartburn. He
b. Urea and electrolytes test results (U&Es) comments that it is aggravated by bending and lying
c. Serum amylase down but relieved by drinking milk. What is the next
d. AXR most appropriate initial investigation?
e. CT abdomen a. 24-hour oesophageal pH monitoring
b. Barium swallow
21. A 38-year-old male visits his GP complaining of c. Upper GI endoscopy
intermittent abdominal pain. This is epigastric in d. Chest X-ray
nature and radiates through to the back. It is more e. AXR
severe before meal times and relieved by eating. A
14
C urea breath test is arranged which comes back 26. A 40-year-old obese city worker presents with
positive. What is the most likely diagnosis? heartburn. He comments that he tried ‘Gaviscon’,
a. Duodenal ulcer which barely relieved the pain. Which management
b. Gastro-oesophageal reflux disease (GORD) plan would you instigate first?
c. Barrett oesophagus a. H2 receptor antagonists
d. Achalasia b. Domperidone
e. Gastric ulcer c. Nissen fundoplication
d. Proton-pump inhibitors
22. A patient had been in hospital for several weeks e. Metoclopramide
following a car crash. Whilst there, he developed
offensive-smelling diarrhoea, passing at least six 27. A 35-year-old woman complains of difficulty in
motions a day. He had a stool sample sent off which swallowing and weight loss. She comments that
confirmed Clostridium difficile. Which of the following she is unable to swallow both liquid and solid food.
antibiotics is considered high risk for developing A barium swallow demonstrates a tapered lower
pseudomembranous colitis? end of her oesophagus. What is the most likely
a. Cefotaxime diagnosis?
b. Vancomycin a. Achalasia
c. Flucloxacillin b. Systemic sclerosis

21 
Single best answer (SBA) questions

c. Diffuse oesophageal spasm his motion looks like tar. What is the most likely
d. Hiatus hernia aetiological cause for his symptoms?
e. Oesophageal carcinoma a. Mallory–Weiss tear
b. Reflux oesophagitis
28. A 65-year-old Chinese man presents complaining c. Gastric varices
of swallowing difficulties and weight loss of d. Oesophageal varices
approximately 2 stone over a 1-month period. e. Peptic ulcer
An upper GI endoscopy is arranged which
demonstrates evidence of a tumour mass. The 33. A 21-year-old rugby player presents to the
following are all risk factors for squamous cell accident and emergency with a 1-day history of
oesophageal carcinoma EXCEPT? haematemesis. He had recently been on a drinking
a. Pickled vegetables binge following his team winning the local final. He
b. Salted fish comments that the bleeding began after an episode
c. Smoking of severe vomiting. An endoscopy is performed.
d. Alcohol What is the most likely diagnosis?
e. GORD a. Mallory–Weiss tear
b. Reflux oesophagitis
29. A middle-aged man presents complaining of c. Gastric varices
swallowing difficulties and chest pain. A barium d. Oesophageal varices
swallow is arranged which demonstrates a e. Peptic ulcer
‘corkscrew’-like appearance. What is the most likely
diagnosis? 34. A 75-year-old woman presents with a 1-day history
a. Achalasia of haematemesis. She comments she has been
b. Systemic sclerosis suffering from back pain for the past month for
c. Diffuse oesophageal spasm which she has been taking diclofenac as prescribed
d. Hiatus hernia by her GP. What is the most likely aetiological cause
e. Oesophageal carcinoma for her symptoms?
a. Mallory–Weiss tear
30. A 42-year-old man presents with pain in his b. Reflux oesophagitis
stomach after meals. An endoscopy is performed c. Peptic ulcer
which shows evidence of gastritis and infection d. Gastric varices
with Helicobacter pylori. What is the next most e. Oesophageal varices
appropriate step in management?
a. Metoclopramide, omeprazole and 35. While on call you are asked to see a patient who has
amoxicillin just passed melaena. On examination you note he
b. Omeprazole, metronidazole and cimetidine is drowsy with a pulse of 124 beats/min and blood
c. Omeprazole, metronidazole and pressure of 75/44 mmHg. Which management plan
clarithromycin would you instigate first?
d. Omeprazole, ranitidine and amoxicillin a. Upper GI endoscopy
e. Domperidone, omeprazole and amoxicillin b. Colonoscopy
c. Blood samples for group and save and
31. A 56-year-old man presents with epigastric pain, cross-match
vomiting and weight loss. He is a heavy drinker and d. Intravenous (IV) access and fluid resuscitation
admits to consuming half a bottle of vodka each day e. Laparotomy
over a 10-year period. On examination you note a
purple-coloured peri-orbital skin rash. What is the 36. A 45-year-old man presents with shortness of
next most appropriate initial investigation? breath and fatigue. Routine blood investigations
a. Abdominal CT scan reveal a haemoglobin of 102 g/L, a mean
b. Abdominal MRI scan corpuscular volume (MCV) of 75 fL and a
c. AXR serum ferritin of 10 μg/L. What is the next most
d. Upper GI endoscopy appropriate initial investigation?
e. EUS a. Upper GI endoscopy
b. Colonoscopy
32. A 42-year-old man presents with a 1-day history c. Upper GI endoscopy and colonoscopy
of haematemesis. On closer questioning he d. Barium meal and follow through
commented that when he opens his bowels e. Mesenteric angiography

22
Gastroenterology 2

37. A 14-year-old girl presents with abdominal pain 42. A 55-year-old woman presents with abdominal pain
and diarrhoea. She is found to be iron and folate and diarrhoea. She notices fresh red blood in the
deficient on initial investigation. What is the most toilet pan when she opens her bowels. She has
likely diagnosis? smoked 25 cigarettes a day since the age of 30
a. Crohn disease and drinks 8 units of alcohol a week. Her mother
b. Ulcerative colitis was diagnosed with bowel cancer at 40 years of
c. Whipple disease age. She enjoys eating steak but admits she does
d. Irritable bowel syndrome not have a taste for fruit or vegetables. You arrange
e. Coeliac disease a barium study which confirms the presence of a
colonic tumour. What is the most likely aetiological
38. A 19-year-old woman presents with abdominal pain cause for her tumour?
and diarrhoea. She comments that there is a strong a. Poor diet
family history of coeliac disease. Which investigation b. Smoking
is most likely to lead to a diagnosis? c. Alcohol
a. Upper GI endoscopy and distal duodenal d. Family history
biopsy e. Age
b. Full blood count
c. Small-bowel barium follow-through 43. A 32-year-old woman with severe Crohn disease
d. Immunoglobulin A (IgA) and tissue underwent a subtotal colectomy with ileostomy
transglutaminase (tTG) serology formation. She comes to the GP complaining of
e. Antireticulin antibodies increased stoma output. She comments that she is
emptying her bag up to six times per day and that
39. A 55-year-old man has recently undergone the contents are loose and watery. What is the next
complicated small bowel surgery. He complains of most appropriate initial investigation?
producing loose stools which he describes as being a. Ileoscopy
offensive and difficult to flush. Which investigation is b. Barium meal and follow-through
most likely to lead to a diagnosis? c. Hydrogen breath test
a. Upper GI endoscopy d. Abdominal CT scan
b. Rectal examination e. Stool culture
c. Hydrogen breath test
d. Barium meal 44. A 65-year-old woman is on long-term total parenteral
e. Abdominal CT scan nutrition (TPN) via a Hickman line following a small
bowel resection. While on call you are asked to review
40. A middle-aged man with ulcerative colitis is referred the patient as she has a temperature of 38.5°C. What
to accident and emergency by his GP with an is the next most appropriate step in management?
acute flare, opening his bowels eight times per day. a. Peripheral blood cultures and continue TPN
Routine blood investigations reveal a platelet count b. Hickman line blood cultures and continue TPN
of 576 × 109/L and C-reactive protein (CRP) of 156 c. Peripheral and Hickman line blood cultures and
mg/L. What is the next most appropriate step in continue TPN
management? d. Peripheral and Hickman line blood cultures and
a. Oral steroids stop TPN
b. IV steroids e. Paracetamol as required and recheck
c. Ciclosporin temperature in 30 minutes.
d. Mesalazine
e. Azathioprine 45. A 65-year-old man presents with right-sided
abdominal pain. On examination you note a
41. A patient with Crohn disease comes to see his moderately enlarged liver and a yellow discoloration
GP. He is currently on mesalazine but has been of his skin and eyes. What is the next most
experiencing an increase in bowel frequency and appropriate initial investigation?
gradual onset of abdominal pain. What is the next a. Abdominal USS
most appropriate step in management? b. Endoscopic retrograde
a. Steroids cholangiopancreatography (ERCP)
b. Ciclosporin c. Viral markers
c. Azathioprine d. Liver biopsy
d. Metronidazole e. Magnetic resonance cholangiopancreatography
e. Methotrexate (MRCP)

23 
Single best answer (SBA) questions

46. A 23-year-old South American man presents d. Hepatitis B testing


with abdominal pain and diarrhoea over a 3-day e. IgM levels
period followed by jaundice. He comments that
his urine appears darker than usual but says he is 51. A 43-year-old man with a long-standing history of
drinking plenty of water. He has recently travelled alcohol abuse presents to accident and emergency
to Lima to see his family. What is the most likely with haematemesis. On examination you note a liver
diagnosis? flap and evidence of moderate hepatomegaly. You
a. Hepatitis D suspect a diagnosis of cirrhosis. Which investigation
b. Hepatitis A is most likely to assess the patient’s liver function?
c. Hepatitis B a. Liver biochemistry
d. Hepatitis C b. Serum electrolytes
e. Hepatitis E c. Serum AFP
d. Prothrombin time
47. A 56-year-old Egyptian woman presents with e. Hepatitis serology
fatigue and nausea. She complains that her skin
feels itchy and that she has noticed blood when 52. A 25-year-old city worker presents with a 2-day
she passes urine. On examination you note a history of haematemesis following a weekend of
yellow tinge to her skin and sclera. What is the binge drinking. Which management plan would you
most likely diagnosis? instigate first?
a. Hepatitis A a. Upper GI endoscopy
b. Hepatitis D b. IV terlipressin
c. Hepatitis B c. Laparotomy
d. Hepatitis E d. Transjugular intrahepatic portosystemic shunting
e. Hepatitis C (TIPS)
e. Sucralfate
48. A 32-year-old man presents with abdominal pain,
diarrhoea and jaundice. He has recently been to 53. A middle-aged man is diagnosed with liver cirrhosis
Dubai with his wife. He comments that the best secondary to chronic alcohol abuse. On examination
thing about Dubai is the seafood. What is the next you note dullness in his flanks. Which management
most appropriate step in management? plan would you instigate first?
a. Alpha-interferon a. Paracentesis
b. Lamivudine b. Sodium restriction alone
c. Ribavirin c. Sodium restriction and diuretics
d. Watchful waiting d. Transjugular intrahepatic portosystemic
e. Pegylated alpha-interferon shunting (TIPS)
e. Albumin infusion
49. A 32-year-old man presents to accident and
emergency following a paracetamol overdose. 54. A 35-year-old man with alcoholic cirrhosis
Routine blood investigations reveal an ALT complains of abdominal discomfort. On examination
of 1800 U/L and a prothrombin time of 42 you note the presence of ascites and notable
seconds. On examination he appears drowsy peritonism. Routine blood investigations reveal a
and is not communicative. What is his grade of WCC of 23 × 109/L and CRP of 107 mg/L. What is
encephalopathy? the most likely aetiological cause for his symptoms?
a. Grade 1 encephalopathy a. Staphylococcus aureus
b. Grade 2 encephalopathy b. Escherichia coli
c. Grade 3 encephalopathy c. Enterococcus faecalis
d. Grade 4 encephalopathy d. Proteus mirabilis
e. Grade 5 encephalopathy e. Staphylococcus epidermidis

50. A middle-aged woman with autoimmune thyroiditis 55. A middle-aged man presents to the accident and
presents with nausea and fatigue. On examination emergency with confusion. His wife comments that
you note jaundice and spider naevi. Which he does not appear to be his usual self and that he
investigation is most likely to lead to a diagnosis? has been drinking excessively for many years. On
a. Abdominal USS examination you note a liver flap and a sweet smell
b. Antinuclear antibodies on his breath. How would you usually make the
c. Serum ferritin levels diagnosis leading to his confusion?

24
Gastroenterology 2

a. Electroencephalography (EEG) difficult to flush. What is the next most appropriate


b. Head CT scan initial investigation?
c. Abdominal CT scan a. Erect chest X-ray
d. Abdominal USS b. AXR
e. Clinical assessment c. ERCP
d. Abdominal CT scan
56. A 47-year-old woman complains that her skin is e. MRCP
feeling itchy and her stools are difficult to flush. On
examination she has moderate hepatomegaly and 61. An 85-year-old frail woman presents with abdominal
investigations demonstrate a raised serum alkaline pain and weight loss of over 2 stone in a 1-month
phosphatase. Which investigation is most likely to period. An abdominal USS is performed which
lead to a diagnosis? confirms the diagnosis as pancreatic cancer with
a. Serum IgM liver metastases. What is the next most appropriate
b. Serum aspartate aminotransferase (AST) step in management?
c. Serum bilirubin a. Surgery
d. Serum antimitochondrial antibodies (AMAs) b. Palliative therapy
e. Antinuclear factor c. Radiotherapy
d. Chemotherapy
57. A middle-aged man complains of malaise and e. Surgery and chemotherapy
arthropathy. He is a newly diagnosed diabetic and
on examination has evidence of hepatomegaly.
Which investigation is most likely to lead to a EXTENDED-MATCHING
diagnosis?
QUESTIONS (EMQs)
a. Serum AST
b. Serum bilirubin
c. Serum albumin Liver problems
d. INR Each answer can be used once, more than once or not
e. Serum ferritin at all.
a. Alcoholic liver disease
58. A middle-aged woman with suspected alcoholic b. Biliary atresia
liver disease is referred for a liver biopsy. The c. Gallstones
histology report states that there is necrosis of d. Hepatitis A
the liver cells and infiltration of leucocytes with the e. Hepatitis B
presence of Mallory bodies within the hepatocytes. f. Hepatitis C
What is the next most appropriate step in g. Metastatic liver disease
management? h. Physiological jaundice
a. Liver transplant i. PBC
b. Steroids j. Wilson disease
c. Stop alcohol completely
d. Continue alcohol in moderation For each scenario below, choose the most likely
e. Multivitamins and nutritional support corresponding option from the list given above.
1. An obese 60-year-old man presents to
59. A 65-year-old man presents with weight loss and the medical admissions unit following
abdominal pain. On examination you note evidence haematemesis with unsteadiness, a flapping
of ascites. Routine blood investigations reveal a tremor and erythematous palms.
markedly raised AFP level and deranged LFTs. What 2. Two years after major abdominal surgery, a
is the next most appropriate initial investigation? 75-year-old woman presents to her GP with
a. AXR weight loss and jaundice. On examination she
b. Abdominal MRI scan has an enlarged knobbly liver.
c. Liver biopsy 3. A 4-week-old baby girl is taken to see the GP by
d. USS a concerned breastfeeding mother. The baby is
e. Hepatic angiogram deeply jaundiced.
4. A 20-year-old soldier returning from the Gulf has
60. A middle-aged woman with chronic alcohol abdominal pain, vomiting and pale stools with
dependency presents with abdominal pain and jaundice. His liver edge is just palpable, smooth
weight loss. She comments that her stools are and slightly tender.

25 
Extended-matching questions (EMQs)

5. An overweight 45-year-old woman presents 2. An 18-year-old university student presenting with


with right upper quadrant pain associated with haematemesis following a period of vigorous
meals. On examination she is slightly tender in vomiting.
the right upper quadrant on deep inspiration. 3. A cause of iron-deficiency anaemia affecting the
colon, caecum or terminal ileum in individuals with
Hepatobiliary disorders aortic stenosis.
Each answer can be used once, more than once or not 4. A 58-year-old bank manager comes to you with
at all. symptoms of heartburn and odynophagia.
a. Wilson disease 5. A 67-year-old male has symptoms associated
b. α1-Antitrypsin deficiency with epigastric discomfort which is typically
c. Hepatitis A relieved by food. The diagnosis was confirmed by
d. Cirrhosis a gram-negative bacterium seen on a biopsy slide.
e. Hepatitis E
f. Primary sclerosing cholangitis
g. Haemochromatosis Drugs
h. Hepatocellular carcinoma Each answer can be used once, more than once or not
i. Hepatitis C at all.
j. PBC a. Omeprazole
b. Aspirin
For each scenario below, choose the most likely c. Penicillamine
corresponding option from the list given above. d. Acamprosate
e. Mesalazine
1. An autosomal recessive disorder associated with f. Steroids
the C282Y mutation on chromosome 6. g. Nifedipine
2. An autosomal recessive disorder associated with h. Cisapride
cirrhotic changes and the presence of periodic i. Chlordiazepoxide
acid–Schiff-positive staining globules within j. Alpha interferon
hepatocytes.
3. A condition known to primarily affect middle-aged For each scenario below, choose the most likely
women resulting in pruritus, jaundice, xanthomas corresponding option from the list given above.
and bone disease.
4. An autosomal recessive disorder known to cause 1. A 45-year-old male chronic alcohol user who has
basal ganglia damage and renal failure best had three recent admissions to hospital following
managed by trientene. an alcoholic ‘binge’. He is demanding medication
5. A condition associated with abdominal pain, to help prevent him drink again.
jaundice and cirrhosis seen in individuals with 2. A 34-year-old patient with known ulcerative colitis
inflammatory bowel disease (IBD). admitted to accident and emergency following an
acute flare.
3. A 65-year-old man with a history of stroke
Gastrointestinal bleeding presenting with an episode of haematemesis.
Each answer can be used once, more than once or not 4. A middle-aged obese man presenting with
at all. indigestion following consumption of spicy foods.
a. Mallory–Weiss tear 5. A 23-year-old female presenting with abdominal
b. Angiodysplasia discomfort and diarrhoea. A colonoscopy and
c. Haemorrhoids subsequent biopsies demonstrate evidence
d. Colon cancer of transmural inflammation with noncaseating
e. Duodenal ulcer granulomas.
f. Oesophageal varices
g. Reflux oesophagitis
h. Gastritis Dysphagia
i. Gastric erosions Each answer can be used once, more than once or not
j. Anal fissure at all.
a. Achalasia
For each scenario below, choose the most likely b. Oesophageal carcinoma
corresponding option from the list given above. c. Benign oesophageal stricture
1. A 30-year-old female presenting with spots of bright d. Oesophagitis
red blood on the toilet paper after passing stool. e. Pharyngeal pouch

26
Gastroenterology 2

f. Oesophageal pouch Pancreatitis


g. Oesophageal web Each answer can be used once, more than once or
h. Foreign body not at all.
i. Candida a. Alcohol
j. Cytomegalovirus b. ERCP
c. Gallstones
For each scenario below, choose the most likely d. Grey-Turner sign
corresponding option from the list given above. e. Hyperlipidaemia
1. A patient presenting with swallowing difficulties f. Pancreatic divisum
and oral discomfort following a recent renal g. Pancreatic necrosectomy
transplant. h. Saponification
2. Associated with swallowing difficulties, persistent i. Scorpion venom
coughing and iron-deficiency anaemia. j. Trauma
3. Associated with regurgitation of food as a result of
abnormal motility of the cricopharyngeus muscle For each scenario below, choose the most likely
and inferior constrictor. corresponding option from the list given above.
4. A middle-aged man presenting with swallowing 1. A 45-year-old care home assistant presents
difficulties of both liquid and solid food following with acute onset of upper abdominal pain. His
recent sclerotherapy of oesophageal varices. abdomen is guarded, with bowel sounds. He
5. Associated with intermittent dysphagia of both liquid denies any alcohol consumption in the past
and solid food. A chest radiograph demonstrates 4 years. His serum amylase is 800 mmol/L; his
evidence of a fluid level behind the heart. serum sodium is 118 mmol/L. The laboratory
technician calls to inform you of the low sodium,
and you notice a milky flashback from the cannula
in the man’s hand.
Gastroenteritis 2. A 23-year-old woman is involved in a traffic
Each answer can be used once, more than once or not
collision. Her motorcycle is hit from the side by
at all.
a careless driver and she is found unconscious,
a. Clostridium perfringens
lying on her back, by the ambulance crew. On
b. Campylobacter
admission, during the secondary survey, you
c. Bacillus cereus
notice she has bruising over her flanks.
d. Vibrio cholerae
3. A 47-year-old man presents with recurrent
e. Escherichia coli
pancreatitis. He claims that he is abstinent from
f. Yersinia enterocolitica
alcohol. At ERCP, the sphincter of Oddi cannot
g. Salmonella
be cannulated, and the diagnosis is confirmed by
h. Staphylococcus aureus
MRCP.
i. Vibrio parahaemolyticus
4. A 25-year-old man presents to hospital with severe
j. Norwalk virus
abdominal pain having returned from holiday in
Gran Canaria (Spain). His serum amylase and
For each scenario below, choose the most likely
lipase are significantly elevated. He denies having
corresponding option from the list given above.
had a drink since his return 2 days ago.
1. Known to cause nausea, vomiting and diarrhoea 5. A 40-year-old woman is admitted to hospital for
approximately 1 to 6 hours following ingestion of treatment of acute pancreatitis. She describes a
contaminated dairy products or cold meat. recent history of sharp right upper quadrant pain
2. Associated with vomiting and later diarrhoea that would last for 2 hours at a time.
following consumption of reheated rice.
3. Associated with abdominal pain, diarrhoea
and rectal bleeding following consumption of Diarrhoea
contaminated poultry. Each answer can be used once, more than once or not
4. Known to cause bloody diarrhoea within at all.
1 to 2 days of consuming contaminated a. Angiotensin-converting enzyme inhibitor
meat. The use of antibiotics is specifically b. Amoebic dysentery
contraindicated. c. Beta-blocker
5. Associated with diarrhoea, abdominal pain and d. Cancer of the colon
bloody stools following contact with infected e. Diverticular disease
animals or ice cream. f. Giardiasis

27 
Extended-matching questions (EMQs)

g. Irritable bowel syndrome habit in the run up to her final examinations. Her
h. Metformin weight and appetite are stable, but she admits to
i. Ulcerative colitis intermittent crampy abdominal pains. Her periods
j. Zollinger–Ellison syndrome are regular. O/E: Unremarkable. CRP, WCC and
alpha-1-glycoprotein are normal. Barium enema
For each scenario below, choose the most likely was normal at outpatients.
corresponding option from the list given above. 3. A 26-year-old accountant attends surgical
1. A 23-year-old man with type A personality outpatients complaining of ongoing diarrhoea
presents with a change in bowel habit. He has with the occasional bloody stool. He has weight
alternating constipation with pellet-like stool and loss of 6 kg and a reduced appetite over the
diarrhoea. He occasionally passes mucus but past 2 months. He is currently on ibuprofen
never any blood. for joint pains but is otherwise well. In the past
2. A 55-year-old man with newly diagnosed diabetes 3 months he has taken 9 days of sick leave. O/E:
complains of diarrhoea after his last consultation His lips are swollen. The left iliac fossa (LIF) is
with you. mildly tender. No masses palpable. Proctoscopy:
3. A 68-year-old man presents with diarrhoea normal. Alpha-1-glycoprotein 3.6 kU/L. He is
alternating with constipation and passage of blood currently awaiting a barium enema and small
per rectum. He has no weight loss or tenesmus. bowel series.
Colonoscopy demonstrates no masses but makes 4. A 45-year-old postman attends outpatients with
the diagnosis. a 3-month history of altered bowel habit, reduced
4. A 25-year-old woman presents with profuse appetite and occasional PR bleeding. He drinks
diarrhoea with passage of blood and mucus. She 38 units of alcohol a week. His younger brother
describes cramping abdominal pains. Examination in Australia attends the hospital for a ‘bowel
reveals left iliac fossa tenderness. She has also problem’. O/E: Tender LIF with a fullness in the
lost weight. same area. 2-cm Hepar. Flexible sigmoidoscopy
5. A 45-year-old man presents with diarrhoea in reveals an erythematous rectosigmoid. Barium
association with acid reflux. Gastroscopy reveals enema: no focal lesion seen.
multiple duodenal ulcers, and serum gastrin levels 5. A 59-year-old man is admitted from the clinic
are 10 times the upper limit of normal. with a 2-month history of tenesmus and a 4-kg
weight loss over the past month. Admission
blood test results: Hb 96 g/L, WCC 9.1 × 109/L,
Altered bowel habit MCV 69.2 fL, K 3.2 mmol/L, Na 137 mmol/L, urea
Each answer can be used once, more than once or not 6.0 mmol/L and creatinine 79 mmol/L. O/E PR:
at all. tender, blood on glove and impacted stool (poor
a. Ulcerative colitis cooperation from the patient). CEA 7.6 ng/mL.
b. Crohn disease Barium enema: ‘apple core stricture’.
c. Colorectal carcinoma
d. Irritable bowel syndrome
e. Large bowel obstruction Jaundice
f. Gastroenteritis Each answer can be used once, more than once or not
g. Diverticulitis at all.
h. Scleroderma a. Cholangiocarcinoma
i. Ischaemic colitis b. Hepatic metastases
c. Hepatocellular carcinoma
For each scenario below, choose the most likely d. Pancreatic carcinoma
corresponding option from the list given above. e. Gallbladder carcinoma
1. A 65-year-old retired policeman consults his GP f. Biliary colic
with a 3-month history of diarrhoea associated g. Alcoholic hepatitis
with bloating sensations. His weight and appetite h. Hepatic cirrhosis
are stable. He admits to a variable diet over the i. Chronic pancreatitis
years. He has been stressed lately following the j. Liver abscess
death of his wife. O/E: Unremarkable. PR: normal.
Proctoscopy: normal. Hb 96 g/dL, WCC 13.5 × For each scenario below, choose the most likely
109/L, carcinoembryonic antigen (CEA) 3.4 ng/mL. corresponding option from the list given above.
2. A 23-year-old university student attends the 1. A 64-year-old former miner is admitted with
University Health Service with altered bowel painless jaundice and a distended abdomen.

28
Gastroenterology 2

He admits to 10-kg weight loss over the past g. Hiatus hernia


2 months. His admission LFTs showed an h. Oesophageal varices
obstructive picture. CA 19-9: 686 U/mL. He drinks i. Retrosternal thyroid
20 units of alcohol per week and is an ex-smoker j. Oesophageal web
of 3 years.
2. A 56-year-old lady is admitted with jaundice but For each scenario below, choose the most likely
is otherwise well. She has a history of hepatitis corresponding option from the list given above.
C acquired whilst a missionary in Africa 30 years 1. A 68-year-old man attends outpatients with a
ago. O/E: 2-cm firm hepar. No ascites or 4-month history of increasing dysphagia and
splenomegaly. USS: multiple focal lesions within halitosis. His weight is stable as is his appetite,
the liver, confirmed on CT. although he complains of bringing up undigested
3. A 48-year-old salesman is admitted with severe food at times. He also complains of the feeling of
epigastric pain of 2 days’ duration. He admits food getting stuck in his throat. He is a teetotaller
to smoking 30 cigarettes a day and being a and nonsmoker. He is otherwise well. Examination
‘moderate drinker’. He is not aware of any is unremarkable.
change in the colour of his stools or urine. 2. A 52-year-old lady consults her GP with a
He has no past medical history and claims to 2-month history of increasing difficulty in
have never been in hospital before. O/E: Mildly swallowing. Her appetite is stable, although she
icteric with central abdominal discomfort. His is tired, but this has been ongoing for years.
amylase is 560 U/L. Your colleague informs you She has a history of microcytic anaemia and is
that she had problems controlling this patient’s on ferrous fumarate. She is a nonsmoker and
analgesic demands overnight. AXR: spiculated teetotaller. She has never taken proton-pump
calcification on the left side extending over the inhibitors or H2 receptor antagonists.
midline. 3. A 71-year-old former train driver is admitted to
4. A 71-year-old woman is transferred from hospital with 2 stone in weight loss of unknown
a peripheral hospital following a 10-day origin. On systematic questioning he admits to
admission with jaundice. An ERCP was increasing difficulty with swallowing his favourite
performed which demonstrated mild Sunday lunch and for the past 3 weeks has eaten
extrahepatic duct dilatation. Her LFTs were only soup. He was previously a heavy drinker
of a mixed picture. You note that previous with 100 pack-years of smoking. O/E: 4 cm
imaging investigations were performed 7 years hard, irregular hepatomegaly. Barium swallow
ago whilst an inpatient in your hospital. An old demonstrated a stricture in the lower third of the
AXR report commented: ‘No obstruction or oesophagus.
extraluminal gas. Porcelain gallbladder’. 4. A 65-year-old nun attends outpatients
5. A 49-year-old woman is admitted on the acute following a complaint to her GP of increasing
surgical ‘take-in’ with a 3-day history of jaundice, difficulty swallowing. While awaiting this
fever and lethargy. O/E: Overweight, icteric with appointment, she tells you that she was
right upper quadrant tenderness. No abdominal admitted to hospital with a ‘really bad’
masses are palpable. Her chart shows a pyrexia pneumonia on the right side of her chest
of above 38°C with temperature spikes. She for which she was given IV antibiotics. She
admits to avoiding fatty foods since her last is a teetotaller and nonsmoker. She has no
admission 4 months ago when she underwent medical history and on systemic review reveals
‘tests’. Old notes reveal these to be USS no respiratory or neurological findings. O/E:
abdomen and ERCP. WCC 18.7 × 109/L, CRP no active disease (NAD). CXR: widening of
176 109 mg/L. the mediastinum with an air fluid level and a
resolving right basal pneumonia.
5. A 36-year-old medical secretary visits surgical
Dysphagia outpatients with a 2-month history of increasing
Each answer can be used once, more than once or not difficulty swallowing and 1 stone in weight
at all. loss. She admits to poor sleep and occasional
a. Achalasia diarrhoea of late. O/E: Systolic flow murmur,
b. Motor neurone disease weakness of the girdle muscles. No superficial
c. Oesophageal carcinoma neck swelling. CXR: superior mediastinal mass
d. Pharyngeal pouch with mild tracheal shift. You are awaiting the
e. Corrosive stricture results of full blood count/U&E/thyroid function
f. Barrett oesophagus tests and acetylcholine receptor antibodies.

29 
Extended-matching questions (EMQs)

Biliary disease 2. A 57-year-old woman complains of a 6-month


Each answer can be used once, more than once or not history of epigastric and right upper quadrant
at all. pain, which occurs once or twice a week and lasts
a. Acute cholecystitis for 1–5 hours. There are no significant abnormal
b. Acute pancreatitis findings on physical examination.
c. Biliary colic 3. A 42-year-old woman with a history of biliary
d. Biliary peritonitis colic and intermittent jaundice is admitted
e. Cholangiocarcinoma as an emergency with a 2-day history of
f. Cholangitis more severe abdominal pain radiating into
g. Empyema of the gallbladder her back, associated with profuse vomiting.
h. Gallbladder carcinoma On examination, she is morbidly obese, is
i. Gallstone in common bile duct dehydrated, has a tachycardia and generalized
j. Mucocoele of the gallbladder vague abdominal tenderness.
4. A 65-year-old man is admitted as an emergency
For each scenario below, choose the most likely with a 6-week history of intermittent abdominal
corresponding option from the list given above. pain. In the last few days, the pain has become
more severe and he has felt ‘fluish’. He has a
1. A 70-year-old diabetic man with known ischaemic
swinging fever, is not clinically jaundiced, but has
heart disease and gallstones presents with a
a right upper quadrant mass.
history of right hypochondrial pain for 24 hours,
5. An 80-year-old man is admitted as an emergency
which suddenly becomes severe and generalized.
with a 3-week history of progressive, painless
On examination, he is sweaty, but afebrile,
jaundice. On examination, he is confused, deeply
has a tachycardia and generalized abdominal
jaundiced, but afebrile. Abdominal examination is
tenderness, especially on the right side, with
normal.
absent bowel sounds.

30
Respiratory 3
up blood. On examination he has notable finger
SINGLE BEST ANSWER (SBA) clubbing with coarse crackles in both lung bases
QUESTIONS on auscultation. What is the most likely cause of his
symptoms?
1. A 65-year-old lifelong smoker presents with a wheeze,
a. Cystic fibrosis
breathlessness and cough with sputum production.
b. Inhaled foreign body
On examination you note reduced chest expansion
c. Measles
bilaterally. He has lived in central London since the
d. Klebsiella
age of 10. Which of the following is most likely to be
e. Whooping cough
causing his symptoms?
a. Interstitial lung disease
6. You suspect bronchiectasis in a young patient. Which
b. Asthma
of the following investigations is most likely to confirm
c. α1-antitrypsin deficiency
the diagnosis?
d. Chronic obstructive pulmonary disease (COPD)
a. Sputum culture
e. None of the above
b. Sweat test
2. A 60-year-old man with severe COPD is admitted with c. Bronchoscopy
an acute exacerbation. What is the most appropriate d. Chest X-ray (CXR)
initial investigation? e. High-resolution CT
a. Computed tomography (CT) chest scan
b. Spirometry 7. A child recently diagnosed with cystic fibrosis is admitted
c. Arterial blood gas following an exacerbation of his symptoms. What is the
d. Peak flow next most appropriate step in management?
e. Pulmonary function tests (PFTs) a. Prednisolone
b. N-acetylcysteine
3. A 68-year-old woman is admitted under your care c. Sodium reabsorption blockers
with increasing breathlessness, wheeze and sputum d. Antibiotics
production. She has COPD. What is the next most e. Aminophylline
appropriate step in management?
a. 60% oxygen 8. A teenager presents to the GP with symptoms
b. 24% oxygen indicative of cystic fibrosis. A sweat test is performed.
c. Oral steroids Which of the following is diagnostic?
d. Noninvasive positive-pressure ventilation a. A chloride value between 40 and 60 mmol/L
e. Antibiotics b. A chloride value less than 40 mmol/L
c. A sodium value of less than 60 mmol/L
4. An overweight middle-aged man presents to the d. A sodium value above 60 mmol/L
general practitioner (GP) after his wife complains e. A sodium value between 40 and 60 mmol/L
about his loud snoring. He tends to drink large
amounts of alcohol in the evenings before going to 9. A 35-year-old farmer presents with acute wheeze,
bed. Which of the following features is he most likely cough and shortness of breath (SOB). He has smoked
to complain of? 10 cigarettes a day for 6 months. He likes to keep fit
a. Morning headaches by running marathons. His grandfather had asthma
b. Nocturnal choking as a child. He has lived in central London for the past
c. Reduced libido year. Which of the following is the most likely cause of
d. Daytime sleepiness his condition?
e. Personality changes a. Asthma
b. COPD
5. A child is admitted with a cough and sputum c. Occupation
production, which is thick, foul smelling and green in d. Exercise
colour. The mother notes episodes of him coughing e. Air pollution

31 
Single best answer (SBA) questions

10. A known asthmatic presents to the GP 15. A 42-year-old man presents to the GP with
complaining of gradually worsening of her symptoms suggestive of pneumonia. He also
symptoms. She is already taking a regular β2 complains of generalized muscular pains. He has
agonist inhaler. What is the next most appropriate many pets at home and mentions he recently bought
step in management? two parrots. Which of the following organisms would
a. Long-acting β2 agonist inhaler account for his condition?
b. Low-dose inhaled steroid a. Chlamydia psittaci
c. High-dose inhaled steroid b. Haemophilus influenzae
d. Oral prednisolone c. Staphylococcus aureus
e. Intravenous hydrocortisone d. Mycoplasma pneumoniae
e. Legionella pneumophila
11. A woman is admitted to hospital with acute severe
asthma. Her respiratory rate is greater than 16. A 23-year-old known intravenous drug abuser
25 breaths/min and peak flow is less than 50% is admitted with a fever, productive cough and
of her predicted value. What is the next most dyspnoea. Which of the following organisms is most
appropriate step in management? likely to account for his condition?
a. Noninvasive ventilation a. Chlamydia pneumoniae
b. Antibiotic therapy b. Haemophilus influenzae
c. Intravenous magnesium c. Staphylococcus aureus
d. Intravenous aminophylline d. Mycoplasma pneumoniae
e. Nebulized salbutamol e. Legionella pneumophila

12. A middle-aged woman is admitted to hospital with 17. A 32-year-old man presents to the GP with a fever,
a cough, sputum production, fever and dyspnoea. productive cough and chest pain. He has recently
A diagnosis of pneumonia is made. Which of the been abroad to Dubai and stayed at a newly built
following organisms is most likely to account for her hotel. He mentioned that he had bouts of diarrhoea
symptoms? in the last few days of his trip, which he put down to
a. Streptococcus pneumoniae the foreign food. Which of the following organisms is
b. Haemophilus influenzae most likely to account for his condition?
c. Legionella pneumophila a. Chlamydia pneumoniae
d. Staphylococcus aureus b. Haemophilus influenzae
e. Aspergillus fumigatus c. Staphylococcus aureus
d. Mycoplasma pneumoniae
13. An elderly woman is admitted to hospital with e. Legionella pneumophila
pneumonia. The following are all risk factors used
in predicting mortality in community-acquired 18. A 40-year-old man is diagnosed with a mild
pneumonia EXCEPT? pneumonia. He has no known drug allergies. What is
a. Urea greater than 7 mmol/L the most appropriate treatment?
b. Age of 65 years a. Amoxicillin
c. Respiratory rate of 20 breaths/min or more b. Metronidazole
d. Systolic blood pressure less than c. Azithromycin
90 mmHg d. Cefuroxime
e. Confusion e. Flucloxacillin

14. A middle-aged man presents with a cough, 19. You suspect pneumonia in a 35-year-old woman.
sputum production, fever and dyspnoea. Red- Which of the following investigations is most likely to
coloured papules are noted on the back of his confirm the diagnosis?
hands. Routine blood investigations demonstrate a. Blood culture
that he is anaemic. Which of the following b. Sputum culture
organisms is most likely to account for his c. White cell count
condition? d. CXR
a. Chlamydia pneumoniae e. Arterial blood gas
b. Haemophilus influenzae
c. Staphylococcus aureus 20. An Asian man presents with a 2-month history
d. Mycoplasma pneumoniae of fever and a cough productive of blood-tinged
e. Legionella pneumophila sputum. He has recently moved to the United

32
Respiratory 3

Kingdom from abroad. What is the most likely a. Heart


causative organism? b. Central nervous system
a. Mycoplasma pneumoniae c. Bones and joints
b. Streptococcus pneumoniae d. Liver
c. Mycobacterium avium-intracellulare e. Eyes
d. Mycobacterium tuberculosis
e. Pneumocystis jiroveci 26. You suspect sarcoidosis in an Afro-Caribbean
man who presents with a cough and evidence
21. A West Indian immigrant has recently been of lymphadenopathy. Which of the following
diagnosed with tuberculosis (TB). Which of the investigations is likely to be most useful in confirming
following cells are solely responsible for engulfing a diagnosis?
bacilli and forming granulomatous lesions? a. CXR
a. Neutrophils b. Lung function tests
b. Macrophages c. CT chest
c. Langhans giant cells d. Serum angiotensin-converting enzyme (ACE)
d. Epithelioid cells e. Lymph node biopsy
e. T lymphocytes
27. A middle-aged man presents with exertional
22. An Asian immigrant presents with a fever and a dyspnoea and a nonproductive cough. On
cough productive of blood-tinged sputum. On examination he has finger clubbing and fine
examination you note an enlarged liver and enlarged inspiratory crackles are noted bibasally on
cervical lymph nodes. A CXR shows patchy auscultation. What is the most likely diagnosis?
shadowing in the upper zones. You suspect TB. a. Silicosis
What is the next most appropriate investigation you b. Pneumoconiosis
would order to confirm your suspicion? c. TB
a. Bronchoscopy d. Cryptogenic fibrosing alveolitis
b. Sputum smear e. Allergic bronchopulmonary aspergillosis
c. Mantoux test
d. Lymph node biopsy 28. Which of the following is the most common cause of
e. CT chest scan extrinsic allergic alveolitis?
a. Forking mouldy hay
23. A middle-aged man is diagnosed with TB. He has b. Handling pigeons
no known drug allergies. Which is the least likely c. Turning germinating barley
drug to be included in the initial treatment? d. Turning mushroom compost
a. Ethambutol e. Contaminated humidifier systems in air
b. Pyrazinamide conditioners
c. Streptomycin
d. Rifampicin 29. A shipbuilding yard worker presents with
e. Isoniazid breathlessness. A CT chest scan is ordered, and
a diagnosis of asbestos-induced lung damage is
24. A 55-year-old man has recently been diagnosed made. He is distraught with the diagnosis and wants
with TB. He has been started on quadruple to know if he is eligible for industrial injury benefits.
therapy and complains of a colour change in his Which of the following findings would allow him to
urine. What drug is most likely to be responsible pursue a claim?
for this? a. Pleural plaques
a. Rifampicin b. Asbestosis
b. Isoniazid c. Mesothelioma
c. Ethambutol d. Bilateral diffuse pleural thickening
d. Pyrazinamide e. All of the above
e. Streptomycin
30. A 65-year-old woman presents with breathlessness
25. An Afro-Caribbean man presents with increasing and haemoptysis. She is diagnosed with squamous
dyspnoea. A CXR shows bilateral hilar cell carcinoma (SCC) of the lung. She has been a
lymphadenopathy. Routine blood investigations smoker for as long as she can remember. She has
demonstrate a raised serum calcium. Which of the lived in central London for almost 30 years now
following is also likely to be affected? and she worked in a shipbuilding yard for most of

33 
Single best answer (SBA) questions

her working life. Both her father and mother died of d. Pancreatitis
lung cancer. Which of the following is the most likely e. Connective tissue disease
cause of her cancer?
a. Smoking 36. A 45-year-old woman presents with SOB and chest
b. Gender pain. She comments that she has recently travelled
c. Family history to Australia. An electrocardiogram demonstrates
d. Occupation evidence of right heart strain. You suspect a
e. City living pulmonary embolus. Which investigation is most
likely to lead to a diagnosis?
31. A middle-aged man is diagnosed with lung cancer. a. CT pulmonary angiogram (CTPA)
He worked in a shipbuilding yard for most of his life. b. D dimers
He is distraught with the diagnosis as he says he c. Ventilation perfusion scan
has never smoked in his life. Which of the following d. CXR
types of lung cancer is he most likely to suffer from? e. Arterial blood gas
a. Squamous
b. Large cell 37. You are called to review a 70-year-old man admitted
c. Adenocarcinoma for treatment of pneumococcal pneumonia, who
d. Alveolar cell has become more unwell with swinging fevers,
e. Small cell worsening breathlessness and pleuritic chest pain.
His inflammatory markers are rising and a repeat CXR
32. A 53-year-old woman is diagnosed with lung cancer. shows a new right-sided pleural effusion. What is the
Her main symptoms on presentation included next most appropriate diagnostic investigation?
breathlessness and haemoptysis. She complained a. CT scan
of stretch marks on her abdomen and that her skin b. Bronchoscopy
bruised easily. On examination you note purple c. Ultrasound-guided diagnostic pleural tap
abdominal striae. Which of the following types of d. PFTs
lung cancer is she most likely to suffer from? e. Recheck the sputum for resistant organisms
a. Squamous
b. Large cell 38. A 76-year-old man with COPD was admitted to
c. Adenocarcinoma accident and emergency department with acute
d. Alveolar cell on chronic SOB. He has a chest drain inserted for
e. Small cell a pneumothorax. Which of the following is true of
chest drain management?
33. A 63-year-old man is diagnosed with lung a. He should have his drain clamped if he has to
cancer. Which of the following extrapulmonary travel to the radiology department without a
manifestations is he most likely to suffer from? medical escort.
a. Clubbing b. The drain should only be removed when it has
b. Dermatomyositis stopped bubbling.
c. Lambert–Eaton myasthenic syndrome (LEMS) c. The drain may be clamped if it only bubbles on
d. Hypercalcaemia coughing.
e. Thrombophlebitis migrans d. The drain should only be removed when it has
stopped swinging.
34. With regard to pleural fluid, which of the following is e. Pain relief is usually not necessary in the first few
most important in determining whether an effusion is days after insertion.
a transudate or exudate?
a. Bacterial count 39. A 76-year-old man is admitted to intensive care unit
b. Amylase levels (ICU) for potassium replacement. A central venous line
c. Malignant cells is inserted via the subclavian vein. During the procedure
d. Glucose levels he becomes acutely breathless with decreasing oxygen
e. Protein levels saturation. What is the most likely diagnosis?
a. The wire has threaded into the left ventricle
35. Which of the following typically causes a b. His chronic obstructive airways disease has
transudative pleural effusion? been exacerbated
a. Heart failure c. A pneumothorax has been created
b. Pneumonia d. A myocardial infarction has been induced
c. Bronchial carcinoma e. Air embolism has occurred

34
Respiratory 3

40. A 58-year-old man with longstanding emphysema 44. A 50-year-old heavy smoker of 30 pack years
presents to the emergency department with sudden presents to his GP with weight loss of 8 kg over
onset of breathlessness and left-sided pleuritic 4 months and a chronic cough with two episodes of
chest pain. On arrival, he is hypoxic, tachypnoeic, haemoptysis. He denies any history of fever. Chest
hypotensive and tachycardic. His trachea is deviated radiograph shows a rounded lesion measuring 5 cm
to the right and breath sounds are markedly reduced in diameter in the right upper lobe. What is the most
on the left side of his chest. What is the next step in likely diagnosis?
the management of this patient? a. Cytomegalovirus infections
a. CXR b. Granulomatous inflammation
b. Insert large bore cannula into the left second c. Pneumocystis carinii (jirovecii)
intercostal space pneumonia
c. Insert chest drain with negative suction on the d. Bronchial carcinoma
left side e. Mesothelioma
d. 1-L saline bolus
e. Oxygen via nasal cannulae 45. An outbreak of pneumonia involving over 20 cases,
some fatal, has occurred in one town and most of
41. A 40-year-old nonsmoker presents to the emergency the sufferers have stayed in the same hotel. Features
department with sudden-onset breathlessness of the illness were high fever, rigors, confusion,
and left-sided chest pain. On examination, he is myalgia, abdominal pain, vomiting and diarrhoea.
unable to complete sentences and has low oxygen Chest radiograph showed bilateral shadowing. The
saturation levels. He has diminished breath sounds clinical diagnosis was confirmed by a fluorescent
on the left side with tracheal deviation towards the antibody titre, which showed a significant rise in
right. Heart sounds are normal. What is the most paired sera taken a month apart. What is the most
likely explanation for these findings? likely pathogen?
a. Pneumonia a. Haemophilus influenzae
b. Asthma b. Legionella pneumophilia
c. Congestive heart failure c. Mycoplasma pneumoniae
d. Pneumothorax d. Staphylococcus aureus
e. Pulmonary embolism e. Streptococcal pneumoniae

42. A 66-year-old woman with history of weight loss 46. You see a 55-year-old female for the first time. She
presents to the respiratory clinic with an 8-week has a 2-year history of chronic daily cough; thick,
history of nonproductive cough and chronic clearing malodorous sputum and occasional haemoptysis.
of throat. She has a 52-pack year smoking history She has been treated with antibiotics for recurrent
and quit 25 years ago. She was subsequently respiratory infections, but is frustrated with her
diagnosed with carcinoma of the lung. Which of the continued symptoms. She has never smoked.
following is NOT a complication of carcinoma of the Her forced expiratory volume in 1 second:forced
lung? vital capacity (FEV1:FVC) ratio is 60% and a CT
a. Adrenal metastases scan shows bronchial wall thickening and luminal
b. Hypertrophic pulmonary osteoarthropathy dilatation. Which of the following is the most likely
c. Hypocalcaemia diagnosis?
d. Hyponatraemia a. Asthma
e. Ptosis b. Bronchiectasis
c. COPD
43. A 65-year-old factory worker presents with a d. Bronchiolitis
dry cough, right-sided chest pain and finger e. Emphysema
clubbing. He has lost 4 kg in weight in the last
2 weeks. He has a 30-year smoking history. On 47. A 67-year-old obese man presents with loud chronic
examination the right lung is dull to percussion snoring and gasping episodes during sleep. His
with reduced chest expansion. What is the most wife has witnessed episodes where it appears as if
likely diagnosis? he has stopped breathing. He reports unrefreshing
a. Asbestosis sleep, multiple awakenings from sleep and morning
b. Bronchioalveolar lung cancer headaches. He has excessive daytime sleepiness,
c. Mesothelioma which is interfering with his daily activities. He has
d. Coal worker’s pneumoconiosis been treated for hypertension, gastro-oesophageal
e. Small cell lung cancer (SCLC) reflux and type 2 diabetes. For adults with

35 
Extended-matching questions (EMQs)

obstructive sleep apnoea, which one of the following year. On examination, there are bibasal fine
is the most effective treatment? end-inspiratory crackles and evidence of finger
a. Continuous positive airway pressure (CPAP) clubbing. A subsequent plain chest radiograph
b. Mandible-positioning oral appliances shows evidence of lower lobe fibrotic changes.
c. Modafinil What is the most likely cause in this patient’s
d. Uvulopalatal surgery condition?
e. Weight reduction to achieve a body mass index a. Smoking
<30 kg/m2 b. Silicosis
c. Asbestosis
48. A 35-year-old man who has recently been treated d. Chronic bronchitis
for Hodgkin lymphoma presents to you with a e. Hypersensitivity pneumonitis
nonproductive cough. His CXR shows shadowing
in one of the upper zones. His oxygen saturation is 52. A 30-year-old woman known to have asthma
95% on air. What is the most likely diagnosis? presents to the emergency department with a
a. Bleomycin toxicity 2-day history of worsening breathlessness and
b. Pneumocystis carinii pneumonia chest tightness despite regular salbutamol inhalers.
c. Pulmonary lymphoma She has been suffering from an upper respiratory
d. Radiation pneumonitis tract infection for the past few days. Which of the
e. TB following findings on clinical examination would be
an ominous sign for an impending respiratory failure
49. A 56-year-old female is seen in the respiratory clinic in this patient?
with a 1-year history of breathlessness. Physical a. Tachycardia
examination shows tachypnoea, widespread b. Tachypnoea
crackles and finger clubbing. Spirometry shows a c. Silent chest
low vital capacity and total lung capacity associated d. Increased wheezing
with a relatively well-preserved peak flow rate. e. All of the above
Arterial blood gases reveal hypoxaemia. What is the
most likely diagnosis? 53. A 36-year-old woman with a history of moderate
a. Emphysema persistent asthma presents to the emergency
b. Bronchial asthma department with progressive worsening of SOB,
c. Pulmonary fibrosis wheezing and cough over 3 days. She has been
d. Bilateral pleural effusions compliant with her maintenance asthma regimen,
e. Bronchitis which consists of an inhaled corticosteroid and a
leukotriene receptor antagonist for maintenance
50. A 65-year-old retired rock miner sees his GP with therapy and salbutamol as rescue therapy. Which
worsening breathlessness, decreased exercise one of the following is most consistent with a
tolerance, dry cough and weight loss. He reports diagnosis of asthma?
no past medical history. On examination he appears a. Reduced FEV1 and reduced FEV1:FVC ratio
slightly tachypnoeic at rest. Fine inspiratory crackles b. Reduced FEV1 and normal FEV1:FVC ratio
are heard in the upper zones. Subsequent chest c. Reduced FEV1 and increased FEV1:FVC ratio
radiograph reveals multiple round opacities in the d. Reduced FVC1 and normal FEV1:FVC ratio
upper lobes with calcified hilar lymphadenopathy. e. Reduced FVC1 and increased FEV1:FVC ratio
What is the most likely diagnosis?
a. Asbestosis
b. Silicosis
EXTENDED-MATCHING
c. Cystic fibrosis
d. Aspergillosis QUESTIONS (EMQs)
e. TB
Pneumonia
51. A 60-year-old nonsmoker presents to his GP with Each answer can be used once, more than once or not
worsening breathlessness for several months, at all.
mainly on exertion. He has also developed a a. Mycoplasma pneumoniae
nonproductive cough and nonpainful deformities b. Chlamydia psittaci
of his fingernails. He worked in the building c. Staphylococcus aureus
industry for 30 years before his retirement this d. Legionella pneumophila

36
Respiratory 3

e. Haemophilus influenzae b. Dermatomyositis


f. Aspiration pneumonia c. Herpes zoster
g. Pseudomonas aeruginosa d. Acanthosis nigricans
h. Pneumocystis jiroveci e. Hypertrophic pulmonary osteoarthropathy
i. Streptococcus pneumoniae f. Horner syndrome
j. Mycobacterium avium intracellulare g. Pancoast tumour
h. Cushing syndrome
For each scenario below, choose the most likely i. Thrombophlebitis migrans
corresponding option from the list given above. j. Clubbing
1. A 32-year-old man presenting with SOB and
feeling generally unwell. He has a known history of For each scenario below, choose the most
cystic fibrosis. likely corresponding option from the list given above.
2. A common cause of pneumonia in patients with 1. A 75-year-old man recently diagnosed with lung
COPD. cancer complaining of painful wrists and ankles.
3. A middle-aged man presenting with a cough and 2. A middle-aged man recently presenting with
generalized muscular pains. He is known to be a haemoptysis and shoulder pain.
keen collector of rare birds. 3. A 65-year-old woman presenting with
4. An intravenous drug abuser presenting with SOB breathlessness, haemoptysis and pigmented skin
and a fever. in her axillae.
5. A middle-aged woman having recently returned 4. A middle-aged woman with known lung cancer
from Las Vegas presenting with a nonproductive presenting with dropping of her upper eyelid,
cough, fever and diarrhoea. pupillary constriction and a sunken eye on the
right side.
5. A patient recently diagnosed with lung cancer
Respiratory drugs presenting with lower limb weakness and difficulty
Each answer can be used once, more than once or not in climbing stairs.
at all.
a. Ethambutol
b. Pyrazinamide Diseases of the respiratory tract
c. Isoniazid Each answer can be used once, more than once or not
d. Rifampicin at all.
e. Metronidazole a. Bronchiectasis
f. Erythromycin b. Sinusitis
g. Clarithromycin c. Rhinitis
h. Flucloxacillin d. Pharyngitis
i. Tobramycin e. Laryngotracheobronchitis
j. Ticarcillin f. Influenza
g. Emphysema
For each scenario below, choose the most likely h. Acute bronchitis
corresponding option from the list given above. i. Asthma
1. Associated with induction of liver enzymes. j. Obstructive sleep apnoea
2 Associated with blurred vision and colour blindness.
3. An intravenous drug abuser diagnosed with For each scenario below, choose the most likely
pneumonia associated with patchy areas of corresponding option from the list given above.
consolidation on his CXR. 1. Dilatation and destruction of lung tissue distal to
4. Known to cause liver inflammation and gout. terminal bronchioles.
5. A middle-aged woman recently diagnosed with a 2. A 23-year-old man presenting with sudden-onset
stroke aspirates when given some water. cough, fever and muscular aches.
3. Associated with a hoarse voice and barking cough.
4. An 18-year-old woman with cystic fibrosis
Lung cancer presenting with a persistent cough productive of
Each answer can be used once, more than once or not at all. yellow/green-coloured sputum.
a. Lambert–Eaton myasthenic 5. A middle-aged man presenting with cough, chest
syndrome (LEMS) tightness and wheeze.

37 
Extended-matching questions (EMQs)

Investigative findings a respiratory rate of 30 breaths/min and has absent


Each answer can be used once, more than once or not breath sounds on the right side of his chest.
at all. 3. A 60-year-old man who has been a lifelong
smoker and has been losing weight presents with
a. COPD
acute SOB, absent breath sounds and dullness to
b. Obstructive sleep apnoea
percussion on the left side.
c. Sarcoidosis
4. A 58-year-old woman who has painful, swollen
d. Wegener granulomatosis
hands, a hump at the top of her spine and fine
e. Bronchiectasis
bibasal inspiratory crackles.
f. Cystic fibrosis
5. A 70-year-old man who worked in the steel
g. Extrinsic allergic alveolitis
industry, presenting with swollen ankles, a raised
h. Asthma
jugular venous pressure and fine inspiratory
i. Cryptogenic fibrosing alveolitis
bibasal lung crackles.
j. Pneumonia

For each scenario below, choose the most likely


corresponding option from the list given above. Shortness of breath
Each answer can be used once, more than once or not
1. A middle-aged woman with SOB and a ground-
at all.
glass appearance on her CXR.
2. A 55-year-old farmer presenting with a cough a. Pulmonary embolus
and SOB. His CXR demonstrates fluffy nodular b. Aortic stenosis
shadowing. c. Congestive cardiac failure
3. A lifelong smoker with an FEV1:FVC ratio of 62% d. Asthma
with an increase in FEV1 of 12% following use of e. Cryptogenic fibrosing alveolitis
salbutamol. f. Pleural effusion
4. A 23-year-old woman presenting with a fever, g. Bronchial carcinoma
cough and SOB. Routine blood investigations h. COPD
reveal a white cell count of 23 × 109/L. A CXR i. Right middle lobe pneumonia
demonstrates evidence of patchy consolidation. j. Pneumothorax
5. A 45-year-old man presenting with haemoptysis.
A CXR reveals evidence of cavitation and routine For each scenario below, choose the most likely
blood investigations demonstrate increased levels corresponding option from the list given above.
of antineutrophil cytoplasmic antibodies. 1. A 55-year-old baker is admitted with increasing
SOB over the past 6 weeks along with chest
tightness and two syncopal episodes whilst at
Lung diseases work. He suffered from asthma as a child. He
Each answer can be used once, more than once or not admits to 20 pack years of smoking. Admission
at all. blood test results, including D-dimer, are
a. Asbestosis unremarkable. CXR: No active disease (NAD).
b. Asthma Peak expiratory flow rate 450 L/min.
c. Bird fancier’s lung 2. A 32-year-old woman of childbearing age recently
d. COPD returned from a 2-month sabbatical in Thailand.
e. Cor pulmonale She has been short of breath for the past week.
f. Pleural effusion O/E: Reduced air entry at the right middle and
g. Pneumoconiosis lower zones. No calf swelling. Oxygen saturations
h. Pulmonary embolus 92% (right atrial (RA)). D-dimer 0.45 mg/L, white
i. Rheumatoid lung cell count (WCC) 13.6 × 109 /L. CXR: indistinct
j. Pneumothorax right heart border.
3. A 61-year-old retired plumber is admitted with
For each scenario below, choose the most likely increasing SOB over the past 2 months. He
corresponding option from the list given above. smokes 20 cigarettes a day. SOB is associated
1. A 65-year-old woman with tar-stained with mild pleuritic chest pain. O/E: Reduced
fingers, central cyanosis and a widespread breath sounds at the left lower zone and
wheeze. decreased vocal resonance. CXR: loss of the left
2. A 30-year-old man who was involved in a road traffic hemidiaphragm silhouette. Hb 12.6 g/dL, WCC
accident has an area of bruising over his chest, has 8.4 × 109/L.

38
Respiratory 3

4. A 24-year-old male master’s student is admitted d. Escherichia coli


to a district general hospital acutely SOB for e. Legionella pneumophila
the past day. He recalls a similar incident during f. Streptococcus pneumoniae
his first year at university. He returned from a g. Candida albicans
summer tour of Australasia 1 week ago. O/E: h. Mycobacterium tuberculosis
Oxygen saturations 93% (RA). Haemodynamically i. Coxiella burnetii
stable. Reduced breath sounds on left side and j. Mycoplasma pneumoniae
asymmetrical chest expansion. A medical student
informs you ‘the lungs look different’ on the CXR. For each scenario below, choose the most likely
5. A 73-year-old former mill worker is admitted corresponding option from the list given above.
dyspnoeic with mild haemoptysis mixed with her 1. The patient has been ventilated on the intensive
sputum. She has had three previous admissions therapy unit (ITU) for a week following a
in the last 5 months with similar presentations. complicated laparotomy and now has lung
O/E: Thin, grade 2 finger clubbing. Increased infiltrates and mucopurulent secretions coming up
anteroposterior chest diameter with scattered the endotracheal tube.
wheeze and a few bibasal crepitations, right more 2. A young, previously fit man presents acutely
than left. Arterial blood gas (on 28% oxygen): unwell with pneumonia and his chest radiograph
partial pressure of oxygen (PO2) 13.6 kPa, partial shows numerous small abscesses.
pressure of carbon dioxide (PCO2) 5.6 kPa. 3. A middle-aged man is admitted to ITU with a
CXR: borderline cardiomegaly, blunted right severe, acute pneumonia. He is the second
costophrenic angle. Chronic inflammatory ITU admission and the third case of pneumonia
changes throughout and a bulky left hilum. occurring in workers at the same factory in town.
4. A 71-year-old woman develops a chest infection
3 days after an uncomplicated operation to
Respiratory infections remove a Duke’s A colonic carcinoma.
Each answer can be used once, more than once or not 5. A 36-year-old woman has a 3-day history of fever,
at all. cough and pleuritic pain. She has coughed up a
a. Cytomegalovirus bit of blood in some purulent sputum. She has
b. Staphylococcus aureus a pleural rub and her chest radiograph shows
c. Pseudomonas aeruginosa segmental consolidation in the left upper lobe.

39 
This page intentionally left blank
Renal 4
5. A 56-year-old man is diagnosed with AKI. The cause
SINGLE BEST ANSWER (SBA) of this was attributed to interstitial nephritis. The
QUESTIONS following drugs all cause interstitial nephritis EXCEPT?
a. Nonsteroid antiinflammatory drugs (NSAIDs)
1. A 45-year-old man complains of breathlessness and b. Angiotensin-converting enzyme (ACE) inhibitors
feeling tired for the past week. He also remarks that c. Allopurinol
he has not been passing as much urine as usual. On d. Penicillin
clinical examination he is not dehydrated. He has a e. Cimetidine
serum creatinine of 500 μmol/L. Urine analysis reveals
a urine osmolality of 650 mOsm/kg and a urine sodium 6. A 65-year-old man complains of waking up in the
of 15 mmol/L. You diagnose acute kidney injury (AKI). night to pass urine for the past 3 weeks. He feels
What is the least likely cause of his renal failure? generally quite tired and complains of muscle
a. Decreased cardiac output weakness. Routine blood tests reveal a serum calcium
b. Severe liver failure of 1.76 mmol/L, a serum phosphate of 3.4 mmol/L, a
c. Hypovolaemia low vitamin D level and a raised parathyroid hormone
d. Renal artery obstruction level. You diagnose renal osteodystrophy. Which of
e. Acute tubular necrosis the following radiological signs is least likely?
a. Bamboo spine
2. A middle-aged woman presents with breathlessness for b. Subperiosteal erosions
the last week. She also comments that she has been c. Salt and pepper skull
feeling more tired than usual and that she has been d. Brown tumours
passing urine infrequently. On examination you note that e. Pseudofractures
her skin is pale and pigmented with several small bruises.
What is the most appropriate initial investigation? 7. A 67-year-old diabetic patient complains of
a. Urea and electrolytes bone and muscle pain for the last 4 weeks. She
b. Urine Stix testing and microscopy remarks that she feels more tired than usual and
c. Serum calcium, phosphate and uric acid is constantly itching. Routine blood investigations
d. Full blood count reveal an estimated glomerular filtration rate 28 mL/
e. Renal ultrasound min per 1.73 m2 and a haemoglobin of 8.7 g/dL. In
addition, her vitamin D level is low. What is the most
3. A 51-year-old man is diagnosed with AKI. He has appropriate step in her management?
recently been suffering from diarrhoea and vomiting a. Review of diabetic medication
and has lost his appetite. He takes captopril for his b. Reduction of protein intake
high blood pressure. He is oliguric with a urine output c. Erythropoietin injections
of less than 300 mL/day. Routine blood tests reveal d. Referral for dialysis
a potassium of 6.5 mmol/L. What is the first most e. Vitamin D analogues
appropriate step in his management?
a. Treat his high potassium 8. A 68-year-old man with chronic renal failure is
b. Fluid replacement referred for peritoneal dialysis. What is the most likely
c. Diuretics complication of this form of dialysis?
d. Withdraw captopril a. Cardiovascular disease
e. Nutrition replacement b. Peritonitis
c. Amyloidosis
4. A patient with AKI is referred for dialysis. The following d. Shoulder pain
are all indications for dialysis EXCEPT? e. Carpal tunnel syndrome
a. Severe metabolic acidosis
b. Uncontrollable hyperkalaemia 9. A 51-year-old man complains of pain in his loins and
c. Pulmonary oedema passing blood in his urine for the past 2 weeks. There
d. Pericarditis is a strong family history of polycystic kidney disease
e. Anaemia (PKD). On examination his blood pressure is recorded

41 
Single best answer (SBA) questions

as 160/95 mmHg. What investigation is most likely 14. A 31-year-old man complains of a painless swelling
to confirm the diagnosis of PKD? in his left testicle. Routine blood tests reveal a beta-
a. 24 Hour ambulatory blood pressure monitoring hCG level that is higher than normal. In addition, his
b. Urea and electrolytes AFP level is markedly raised. What is the most likely
c. Urine Stix testing and microscopy diagnosis?
d. Renal ultrasound a. Seminoma
e. Excretion urography b. Teratoma
c. Sertoli cell adenoma
10. A 67-year-old man complains of loin pain and d. Leydig cell adenoma
passing blood in his urine for the past week. On e. Testicular torsion
examination you note the presence of large irregular
kidneys. You are concerned about the possibility of 15. A 29-year-old man presents with a painless swelling
PKD. What is the most common mode of inheritance in his left testicle. He complains of a cough and
of this condition? shortness of breath. Ultrasound scanning reveals the
a. Autosomal recessive presence of a testicular tumour. Surgical exploration
b. X-linked recessive and subsequent histological examination reveal no
c. Autosomal dominant cystic spaces. In addition to an orchidectomy, what
d. X-linked dominant is the next most appropriate step in management?
e. Y linked a. Chemotherapy
b. Radiotherapy
11. A 57-year-old man presents with haematuria, loin c. Watchful waiting
pain and a solid mass in his right flank. Excretion d. Goserelin
urography demonstrates the presence of a e. Cyproterone acetate
space-occupying lesion. What is the most
appropriate step in his management? 16. A 35-year-old woman complains of accidentally
a. Medroxyprogesterone acetate passing urine when she coughs or laughs. She
b. Surgery has given birth to three children. Urine Stix testing
c. Interleukin 2 reveals a trace of protein. What is the most likely
d. Alpha-interferon diagnosis?
e. Radiotherapy a. Functional incontinence
b. Urge incontinence
12. A 70-year-old man complains of waking up in the c. Stress incontinence
night to pass urine, nonspecific bone pain and d. Overflow incontinence
weight loss. He mentions that there is often a delay e. Urinary tract infection (UTI)
in initiating urination and a sense of incomplete
voiding. What is the most appropriate initial 17. A 61-year-old man complains of hesitancy, a
investigation after a digital rectal examination? poor stream and terminal dribbling when passing
a. Urea and electrolytes urine. On examination you note the presence of a
b. Serum prostate-specific antigen (PSA) distended bladder palpable suprapubically. Which
c. Renal ultrasound investigation is most likely to lead to a diagnosis?
d. Transrectal ultrasound a. Radionuclide studies
e. Excretion urography b. Urodynamic study
c. Abdominal ultrasound
13. A 30-year-old man complains of a painless swelling d. Bladder scan
in his right testicle. Routine blood tests reveal a e. Cystoscopy
beta-human chorionic gonadotropin (beta-hCG) level
that is higher than normal. Surgical exploration and 18. A 43-year-old man with gout complains of severe
subsequent histological examination of the testis intermittent loin pain and microscopic haematuria
reveal the presence of cystic spaces. What is the which has lasted for several hours. What is the initial
most likely diagnosis? most appropriate step in management?
a. Seminoma a. Fluid replacement
b. Teratoma b. Allopurinol
c. Sertoli cell adenoma c. Pain relief
d. Leydig cell adenoma d. Shock wave lithotripsy
e. Testicular torsion e. Nephrolithotomy

42
Renal 4

19. A 23-year-old marathon runner complains of d. Urea and electrolytes


severe intermittent loin pain and vomiting soon e. Renal ultrasound
after a training session for her next marathon. On
examination she is severely dehydrated. Which of 24. A 6-year-old boy is diagnosed with nephrotic
the following investigations is most likely to lead to a syndrome. What is the most likely underlying cause
diagnosis? of this condition?
a. Serum calcium levels a. Amyloidosis
b. Abdominal X-ray of kidneys, ureters and b. Membranous nephropathy
bladder (KUB) c. Minimal change disease
c. Urea and electrolytes d. Focal segmental glomerulosclerosis
d. Midstream specimen of urine for culture and e. Systemic lupus erythematosus
sensitivity
e. 24-Hour urine collection for calcium 25. A patient presents to accident and emergency (A&E)
with bilateral pitting oedema, ascites and severe
20. A 24-year-old woman complains of urinary frequency loin pain. His urine dipstick shows proteinuria, liver
and dysuria. What organism is the most likely cause function tests show hypoalbuminaemia, and U&Es
of her symptoms? show an AKI. On examination his left kidney is
a. Escherichia coli palpable. Which of the following medications should
b. Proteus mirabilis be prescribed?
c. Klebsiella aerogenes a. Co-amoxiclav
d. Enterococci b. Dalteparin
e. Staphylococcus saprophyticus c. Diclofenac
d. Prednisolone
21. A 30-year-old man complains of urinary frequency, e. Warfarin
dysuria and suprapubic tenderness for the
past week. What is the most appropriate initial 26. A patient known to have HIV presents to A&E with
investigation? pitting oedema. He also describes his urine as being
a. Urine microscopy and culture frothy. A urine dip on admission shows proteinuria,
b. Bladder scan but no haematuria. A renal biopsy is taken and
c. Abdominal X-ray shows deposition of immunoglobulin M (IgM) and
d. Urodynamic studies C3 in some of the glomeruli. What is the most likely
e. Cystoscopy underlying diagnosis?
a. Berger disease
22. An elderly man is catheterized having failed to pass b. Focal segmental glomerulosclerosis
urine for the past 2 days. Soon after, he becomes c. Goodpasture syndrome
significantly confused and complains of generalized d. Mesangiocapillary glomerulonephritis type II
loin pain. On examination he is pyrexial with a e. Proliferative glomerulonephritis
temperature of 39.5°C. What is the most appropriate
step in his management? 27. A 67-year-old man with stage 5 CKD presents to
a. High fluid intake A&E with sharp central chest pain. The pain is made
b. Trimethoprim worse on inspiration and when he lies down. The
c. Nitrofurantoin pain can be relieved slightly by sitting forwards.
d. Amoxicillin Which of the following treatment regimens should be
e. Gentamicin started to treat this patient’s underlying condition?
a. Allopurinol
23. A 4-year-old boy presents with swelling of his ankles, b. Colchicine
genitals and abdomen. On examination you also c. Diclofenac
note slight swelling of his arms. His jugular venous d. Haemodialysis
pressure is not raised. His mother mentions that his e. Primary coronary angioplasty
urine appears frothy. Which investigation is most
likely to lead to a diagnosis? 28. A patient with PKD has reached end-stage renal
a. 24-Hour urinary protein and serum albumin failure, and is about to undergo his first dialysis
measurement session. Routine blood tests are taken. Which of the
b. Urine microscopy and culture following is most likely to be within the normal range
c. Plasma lipids measurement in this patient?

43 
Single best answer (SBA) questions

a. Haemoglobin d. Congestive cardiac failure


b. Parathyroid hormone e. Pulmonary fibrosis
c. Serum creatinine
d. Serum phosphate 33. A 69-year-old female is referred to the renal clinic for
e. Serum urate worsening kidney function. Her GP noticed a gradual
increase in creatinine over the past few years. On
29. A patient with ongoing membranous nephropathy examination there were two large ballotable masses
has developed hypoalbuminaemia as a result of on both flanks, as well as a nontender suprapubic
proteinuria. Which of the following drugs would mass, which is dulled to percussion. Which of
reduce the loss of albumin via the kidneys? following is the most likely cause of the decline in
a. Amiloride renal function?
b. Bumetanide a. Amyloidosis
c. Ramipril b. Bilateral renal artery stenosis
d. Indapamide c. Hydronephrosis
e. Human albumin solution d. PKD
e. Metastatic lung cancer
30. A 32-year-old male is brought into hospital after
passing dark brown urine following an accident 34. A 60-year-old woman with end-stage renal
at work where he was pinned against a wall by a failure on haemodialysis is found unconscious at
loader. Which of the following treatments should be home by her husband. Her husband reports
initiated immediately? that she has missed two dialysis sessions this past
a. IV 0.9% saline week. Electrocardiogram (ECG) shows evidence
b. IV sodium bicarbonate of hyperkalaemia. Which of the following
c. IV insulin and 50% dextrose ECG findings is associated with
d. IV dexamethasone hyperkalaemia?
e. Haemodialysis a. Short PR interval
b. ST-segment depression
31. A 43-year-old female presents to A&E with c. T-wave inversion
shortness of breath, cough and haemoptysis. d. U waves
She also reports haematuria. Initial blood tests e. Tall T waves
show a raised white cell count (eosinophilia),
raised urea, erythrocyte sedimentation rate, 35. A 38-year-old man was referred to the
C-reactive protein and perinuclear antineutrophil nephrology department by his GP following
cytoplasmic antibodies (P-ANCA). Which of abnormal findings on urinalysis. Which of the
the following is the most likely underlying following is NOT true in patients with nephrotic
diagnosis? syndrome?
a. Churg–Strauss syndrome a. Massive proteinuria
b. Goodpasture syndrome b. Hypoalbuminemia
c. Kawasaki disease c. Periorbital oedema
d. Microscopic polyangiitis d. Hyperlipidemia
e. Wegener granulomatosis e. Prolonged bleeding time

32. A 62-year-old male was started on ramipril by his 36. A 65-year-old man known to have end-stage
general practitioner (GP) to treat his high blood renal failure secondary to diabetes mellitus,
pressure. Three days later, he presented to A&E with who has been on haemodialysis for 8 years,
a cough, dyspnoea and wheeze, which is made presents to his GP with a 2-month history of
worse by lying down. On examination he was found numbness and tingling of his left hand.
to have bilateral fine inspiratory crackles throughout Clinical examination shows atrophy of thenar
both lungs. His arterial blood gas showed a type 1 prominence. What is his most likely
respiratory failure. Which of the following conditions diagnosis?
is this patient likely to have, which has resulted in his a. Carpal tunnel syndrome
symptoms? b. Cervical root impingement
a. Acquired angioedema c. Thoracic outlet syndrome
b. Asthma d. Myocardial infarction
c. Bilateral renal artery stenosis e. Pronator teres syndrome

44
Renal 4

2. Associated with the deposition of insoluble fibrillar


EXTENDED-MATCHING protein and diagnosed by Congo Red staining of
QUESTIONS (EMQs) tissues.
3. A multifactorial pathogenesis associated with
Renal disease depressed bone marrow activity and decreased
Each answer can be used once, more than once or not erythropoietin formation.
at all. 4. A 65-year-old man with known renal failure
a. Acute nephritic syndrome presenting with postural hypotension, nausea and
b. Nephrotic syndrome vomiting.
c. UTI 5. A condition characterized by incomplete
d. Tubulointerstitial nephritis mineralization of osteoid tissue.
e. Renal stone disease
f. Urinary tract obstruction
g. PKD
Clinical features of renal disease
Each answer can be used once, more than once or not
h. Medullary sponge kidney
at all.
i. Renal carcinoma
j. Renal hypertension a. Acute nephritic syndrome
b. Nephrotic syndrome
For each scenario below, choose the most likely c. Acute renal failure
corresponding option from the list given above. d. Tubulointerstitial nephritis
e. Renal stone disease
1. An uncommon condition associated with
f. Urinary tract obstruction
dilatation of the collecting ducts in the papillae and
g. Chronic renal failure
cyst formation.
h. Medullary sponge kidney
2. An autosomal dominant condition associated
i. Renal carcinoma
with cyst formation and mutations in the
j. Renal hypertension
PKD1 gene.
3. A 7-year-old boy presenting with haematuria and
For each scenario below, choose the most likely
proteinuria following a sore throat.
corresponding option from the list given above.
4. A condition associated with proteinuria, oedema
and hypoalbuminaemia. 1. Typical features include swelling of the ankles,
5. A middle-aged man presenting with genitals and abdomen.
polyuria and nocturia. He is known to 2. Associated with shortness of breath, generalized
suffer from rheumatoid arthritis for which he takes lethargy and bone pain.
NSAIDs. 3. Typical features include haematuria, loin pain and
a mass in the flank.
4. May be associated with pain and swelling of the
Complications of renal failure big toe.
Each answer can be used once, more than once or not 5. Typical features include haematuria, proteinuria
at all. and oedema in children following a sore throat.
a. Anaemia
b. Osteomalacia
c. Osteoporosis Investigations
d. Osteosclerosis Each answer can be used once, more than once or not
e. Amyloidosis at all.
f. Autonomic neuropathy a. Urea and electrolytes
g. Cardiomyopathy b. PSA
h. Peptic ulceration c. Urine microscopy and culture
i. Acute pancreatitis d. Magnetic resonance angiography
j. Gout e. X-ray of the kidneys, ureters and bladder
f. Renal ultrasound
For each scenario below, choose the most likely g. Renal biopsy
corresponding option from the list given above. h. Serum albumin
1. Associated with an increase in trabeculae typically i. Cystoscopy
in cancellous bone. j. Urodynamics

45 
Extended-matching questions (EMQs)

For each scenario below, choose the most likely Diseases of the tubules and interstitium
corresponding option from the list given above. Each answer can be used once, more than once or not
1. A patient with known ischaemic heart disease at all.
taking ACE inhibitors who subsequently develops a. Toxic acute tubular necrosis
renal failure. b. Amyloidosis
2. A 24-year-old long-distance runner presenting c. PKD
with severe right-sided groin pain and vomiting. d. Urate nephropathy
3. A 65-year-old man presenting with a poor urinary e. UTI
stream, terminal dribbling and hesitancy. f. Sickle-cell disease nephropathy
4. A 75-year-old woman presenting with confusion, g. Chronic pyelonephritis
pain on passing urine and fever. h. Drug-induced tubulointerstitial nephritis
5. A 35-year-old woman presenting with urinary i. Goodpasture syndrome
incontinence following coughing or standing. j. Ischaemic acute tubular necrosis
k. Acute pyelonephritis
Management of renal disease
Each answer can be used once, more than once or not For each scenario below, choose the most likely
at all. corresponding option from the list given above.
a. Haemodialysis 1. A 26-year-old woman who is 26 weeks
b. Peritoneal dialysis pregnant presents with a 2-day history of
c. Surgery dysuria and increased frequency of micturition.
d. Alpha-blockers On inspection of the urine sample, you notice it
e. Amoxicillin is cloudy.
f. Analgesia 2. A 63-year-old man who has vesicoureteric reflux
g. Prednisolone presents to the GP’s surgery with a 3-day history
h. Haemofiltration of fever, general malaise and loin pain. He says he
i. Transplantation had also noticed that he suddenly needs to go to
j. Antihypertensives the toilet without warning.
3. A 28-year-old male dies from a motorcycle
For each scenario below, choose the most likely accident in which he lost a lot of blood following
corresponding option from the list given above. trauma to his abdomen. Histological examination
1. A 60-year-old gentleman presenting with of the kidneys reveals infiltration of inflammatory
frequency, nocturia and postvoid dribbling. cells and tubular cells, flattened and vacuolated
2. A 22-year-old marathon runner presenting with tubular cells and interstitial oedema.
severe left-sided loin pain. 4. A 33-year-old woman undergoing chemotherapy
3. An elderly gentleman presenting with fever, for leukaemia goes into AKI.
confusion and dysuria. 5. A 30-year-old woman is found to have severe
4. A condition associated with proteinuria and hypertension. Her mother reports that she had
hypoalbuminaemia. bouts of ‘undiagnosed’ fever as a young child
5. The mainstay form of treatment in individuals and wet the bed until age 12 following which she
presenting with haematuria, loin pain and a mass has always had nocturia. Urinalysis reveals the
in the flank. presence of protein.

46
Neurology 5
conduction studies demonstrate slowing of motor
SINGLE BEST ANSWER (SBA) conduction with segmental demyelination. You decide
QUESTIONS to perform a lumbar puncture (LP). What is the most
likely finding on cerebrospinal fluid (CSF) analysis?
1. A 35-year-old woman complains of generalized a. Normal protein, normal glucose, normal cell count
facial weakness along with difficulties in chewing. On b. Low protein, raised glucose, raised cell count
examination you note ptosis and a slow downward c. Low protein, low glucose, low cell count
drift of her outstretched arms. In addition, you observe d. Raised protein, normal glucose, raised cell count
that her speech appears slower than normal. What is e. Raised protein, normal glucose, normal cell count
the most appropriate initial investigation?
a. Acetylcholine receptor antibodies 6. A 54-year-old woman with diabetes presents with
b. Electromyography (EMG) pain and tingling in her right hand. On examination
c. Muscle biopsy you note wasting of her thenar muscles. Which nerve
d. Creatine kinase levels is most likely to be affected?
e. Head computed tomography (CT) scan a. Radial
b. Ulnar
2. A 45-year-old lawyer complains of difficulty in
c. Median
releasing his grasp after shaking peoples’ hands.
d. Brachial
What term most appropriately describes this?
e. Musculocutaneous
a. Myositis
b. Muscular dystrophy
7. A 65-year-old man presents with weakness in his
c. Myasthenia
hands and arms. On examination you note significant
d. Myopathy
wasting and fasciculation. You suspect the possibility
e. Myotonia
of motor neurone disease. Which investigation is most
3. A 7-year-old boy presents with difficulty in running and likely to lead to a diagnosis?
in rising to an erect position from the floor. His mother a. EMG
mentions that to become upright he often uses his b. Muscle biopsy
hands to climb up his legs. What is the most likely c. Creatine kinase levels
diagnosis? d. Head CT scan
a. Limb girdle dystrophy e. None of the above
b. Facioscapulohumeral dystrophy
c. Duchenne muscular dystrophy 8. A 32-year-old man presents with ataxia, dysarthria
d. Becker muscular dystrophy and visual disturbance. On examination you note
e. Myotonia congenita absent lower limb tendon reflexes and an upgoing
plantar response. You also note absence of joint
4. A 32-year-old alcoholic complains of numbness position and vibration sense in his lower limbs. A
and tingling sensation in his hands. On general visual examination demonstrates nystagmus and
examination you note nystagmus. What is the most pale-coloured optic discs. What is the most likely
likely diagnosis? diagnosis?
a. Vitamin B6 deficiency a. Cauda equina lesion
b. Vitamin B1 deficiency b. Friedreich ataxia
c. Vitamin B12 deficiency c. Syringomyelia
d. Vitamin A deficiency d. Syringobulbia
e. Vitamin C deficiency e. Spinal cord compression

5. A 45-year-old man presents with a 1-week history 9. A 40-year-old woman presents with difficulty in
of progressive weakness and numbness in his walking. Neurological examination reveals significant
distal limbs. On examination you note areflexia and weakness in her legs, as well as notable spasticity. In
muscle weakness but with normal sensation. Nerve addition, you note a loss of sensation to pain and

47 
Single best answer (SBA) questions

temperature in her upper limbs. Which investigation 14. A 61-year-old woman complains of a headache and
is most likely to lead to a diagnosis? scalp tenderness when she combs her hair. She
a. Spinal magnetic resonance imaging (MRI) scan presents as an emergency due to sudden loss of
b. Spinal X-ray vision in her right eye. Routine blood investigations
c. Spinal CT scan reveal an erythrocyte sedimentation rate (ESR)
d. Muscle biopsy of 65 mm/h. You perform a routine fundoscopy
e. EMG assessment. What is most likely to be seen on
fundoscopy?
10. A 54-year-old woman presents with back pain a. Cherry-red spot
and weakness in her legs. On examination you b. Flame-shaped haemorrhages
note significant spasticity in her legs. Appropriate c. Cotton wool spots
imaging demonstrates the presence of a vertebral d. Silver wiring
lesion. What is the most appropriate step in e. Papilloedema
management?
a. Dexamethasone 15. A 28-year-old woman complains of a throbbing
b. Surgical decompression headache accompanied by nausea, vomiting
c. Physiotherapy and difficulty looking at bright lights. Prior to her
d. Neck brace headache, she recalls seeing flashes and jagged
e. Baclofen lines. What is the most likely diagnosis?
a. Trigeminal neuralgia
11. A 38-year-old man complains of headaches that b. Giant-cell arteritis
typically last for 4 weeks followed by a period of c. Cluster headache
remission. He describes the headache as being d. Migraine
quite severe and located primarily around his right e. Tension headache
eye. During such episodes he suffers from watering
of his eye, as well as significant redness. What is the 16. A 30-year-old man complains of a throbbing
most likely diagnosis? headache and visual loss. He mentions that the
a. Trigeminal neuralgia headache comes on after eating chocolate biscuits
b. Giant-cell arteritis which he is very fond of. What is the most likely
c. Cluster headache aetiological cause for his headache?
d. Migraine a. Serotonin
e. Tension headache b. Adrenaline
c. Noradrenaline
12. A 65-year-old woman complains of a headache and d. Dopamine
scalp tenderness when she combs her hair. She also e. Acetylcholine
suffers from jaw pain while eating and has noticed
that her vision is intermittently poor. What is the most 17. A 35-year-old man complains of a 2-day history
likely diagnosis? of headaches. During the consultation, he begins
a. Trigeminal neuralgia to experience a headache which he describes
b. Giant-cell arteritis as throbbing in nature. He also mentions he feels
c. Cluster headache nauseous and his vision is deteriorating. His
d. Migraine past medical history includes ischaemic heart
e. Tension headache disease. What is the next most appropriate step in
management?
13. A 60-year-old woman complains of a a. Sumatriptan
headache and scalp tenderness when b. Ergotamine
she combs her hair. On examination you note her c. Methysergide
temporal arteries are tender, firm and pulseless. d. Paracetamol and metoclopramide
What is the next most appropriate step in e. Pizotifen
management?
a. Paracetamol 18. A 63-year-old man accompanied by his wife
b. Triptans presents with headaches and vomiting. His wife
c. Ergotamine is keen to mention that he has become generally
d. Pizotifen slow and that his personality is not what it used
e. Steroids to be. She is worried as he is beginning to show

48
Neurology 5

little interest in his usual hobbies. He is referred d. Head CT scan


for a CT head scan which reveals the presence e. Syphilis serology
of a tumour. What is the most likely site of his
tumour? 23. A 23-year-old university student is diagnosed with
a. Temporal lobe meningitis of bacterial aetiology. The cause has been
b. Parietal lobe assigned as bacterial. What is the most likely finding
c. Frontal lobe on CSF analysis?
d. Occipital lobe a. Lymphocytes, increased protein, normal
e. Cerebellum glucose
b. Lymphocytes, decreased protein, increased
19. A 55-year-old man presents with headache, vomiting glucose
and papilloedema. He describes the headache as c. Lymphocytes, normal protein, normal glucose
getting worse when he coughs but decreasing when d. Polymorphs, normal protein, normal glucose
he stands up. A CT head scan reveals the presence e. Polymorphs, increased protein, decreased
of a tumour. What additional findings may there be glucose
on neurological examination?
a. Cranial nerve (CN) II nerve palsy 24. A 35-year-old man presents with blurred vision and
b. CN III nerve palsy right-sided eye pain. In addition, he mentions he
c. CN IV nerve palsy experiences a ‘pins and needles’-like sensation on
d. CN V nerve palsy occasions. The following are all sites which may be
e. CN VII nerve palsy affected in his condition EXCEPT?
a. Optic nerve
20. A 55-year-old woman complains of a sudden-onset b. Periventricular white matter
severe headache and an intolerance of bright lights. c. Spinal cord
On examination you note neck stiffness. Routine d. Cerebellum
observations reveal a temperature of 38.5°C. You e. Peripheral nerves
suspect meningitis. What is the most likely organism
responsible for her symptoms? 25. A 32-year-old man presents with blurred vision and
a. Neisseria meningitidis right-sided eye pain. In addition, he complains of
b. Staphylococcus aureus numbness and weakness in his legs. What is the
c. Group B streptococci most appropriate initial investigation?
d. Listeria monocytogenes a. MRI brain scan
e. Enterovirus b. Electrophysiology
c. LP
21. A 10-year-old boy presents with a headache d. Brain CT scan
and difficulty when looking at bright lights. On e. Cerebral angiography
examination you note neck stiffness and a
nonblanching rash on his abdomen. He is pyrexial 26. A 31-year-old woman is diagnosed with MS
with a temperature of 39°C. There is no known following investigations which include an LP. What is
history of drug allergies. What is the next most the most likely finding on CSF analysis?
appropriate step in management? a. Decreased polymorphs
a. Paracetamol b. Monoclonal bands
b. Cefotaxime c. Increased glucose
c. Ampicillin d. Decreased protein
d. Benzylpenicillin e. Oligoclonal bands
e. Chloramphenicol
27. A 32-year-old man complains of a tremor
22. A 19-year-old university student complains of a when he holds a glass or spoon. He notices it
headache and a stiff neck. He also mentions that he improves when he drinks alcohol. What is
finds it difficult to look at bright lights. Observations the most likely diagnosis?
reveal a temperature of 39°C. What is the most a. Chorea
appropriate initial investigation? b. Hemiballismus
a. Blood culture c. Myoclonus
b. LP d. Benign essential tremor
c. Viral serology e. Tic

49 
Single best answer (SBA) questions

28. A 45-year-old man with Parkinson disease an inability to move his eyes vertically. What is the
complains of a jerky movement which spreads from most likely diagnosis?
one part of his body to the other ever since he was a. Drug-induced parkinsonism
started on treatment for his condition. What is the b. Idiopathic parkinsonism
most likely diagnosis? c. Parkinsonism plus
a. Chorea d. Methylphenyltetrahydropyridine-induced
b. Hemiballismus parkinsonism
c. Myoclonus e. Postencephalitic parkinsonism
d. Benign essential tremor
e. Tic 34. A 28-year-old man presents to accident and
emergency (A&E) following a fitting episode. His
29. A 35-year-old woman with epilepsy was witnessed girlfriend witnessed the event and described it as a
as having a sudden jerking movement of her right jerking action of his entire body. She mentioned that
arm while she fell asleep. What is the most likely the event did not last long. After the episode, the
diagnosis? gentleman remembers feeling drowsy and that his
a. Chorea tongue was sore. What is the most likely diagnosis?
b. Hemiballismus a. Tonic-clonic seizure
c. Myoclonus b. Absence seizure
d. Benign essential tremor c. Simple partial seizure
e. Tic d. Complex partial seizure
e. Myoclonic seizure
30. A 30-year-old man complains of prolonged
spasms of muscle contraction. He is now receiving 35. A 7-year-old girl was witnessed by her father as
botulinum toxin which has improved his symptoms having episodes where she suddenly stopped what
dramatically. What is the most likely diagnosis? she was doing, stared and then regained normal
a. Chorea activity. What is the most likely diagnosis?
b. Hemiballismus a. Tonic-clonic seizure
c. Myoclonus b. Absence seizure
d. Dystonia c. Simple partial seizure
e. Tic d. Complex partial seizure
e. Myoclonic seizure
31. A 65-year-old woman presents with a hand tremor.
She has slowed considerably in day to day activities 36. A 45-year-old man presents following a fitting
and now requires support for activities of daily living. On episode. He described it as a jerking-like movement
examination you note an expressionless face, slowness which began in his right hand and spread to the
of movements and marked rigidity in her upper limbs. left side of his body. Following the episode, he
What is the initial most appropriate step in management? mentioned his upper limbs felt weak. What is the
a. Bromocriptine most likely diagnosis?
b. Selegiline a. Tonic-clonic seizure
c. Amantadine b. Absence seizure
d. Benzhexol c. Focal motor seizure
e. Levodopa d. Complex partial seizure
e. Myoclonic seizure
32. A 62-year-old man presents with a shuffling gait
and reduced arm swing. On examination you note 37. A 39-year-old woman presents following a fitting
a resting tremor. Which neurotransmitter is he most episode. She recalls experiencing the smell of
likely to be lacking? burning rubber during the event. What is the most
a. Dopamine likely diagnosis?
b. Serotonin a. Tonic-clonic seizure
c. Noradrenaline b. Absence seizure
d. Adrenaline c. Simple partial seizure
e. Acetylcholine d. Complex partial seizure
e. Myoclonic seizure
33. A 53-year-old schizophrenic presents with a resting
hand tremor. On examination you note slowness of 38. A 32-year-old man is diagnosed with grand
movements, marked rigidity in his upper limbs and mal epilepsy. He is started on appropriate

50
Neurology 5

treatment. After 4 weeks, he complains of hair d. Blood glucose


loss. Routine blood investigations reveal an e. Blood cultures
alanine aminotransferase of 55 U/L, an aspartate
aminotransferase of 55 U/L, an alkaline phosphatase 43. A 55-year-old man presents with a sudden-onset
of 140 U/L and a bilirubin of 3 mg/dL. Which drug is severe headache. On examination you note neck
most likely to be responsible for these findings? stiffness and that his right eye is abducted and looking
a. Phenytoin down. Fundoscopy assessment reveals papilloedema.
b. Carbamazepine What is the most appropriate initial investigation?
c. Ethosuximide a. Head CT scan
d. Sodium valproate b. Head MRI scan
e. Vigabatrin c. LP
d. Cerebral angiography
39. A 36-year-old man is diagnosed with grand mal e. Visual field assessment by perimetry
epilepsy. He is started on appropriate treatment.
During a routine review with his general practitioner 44. A 54-year-old man presents with a sudden-onset
(GP), blood investigations reveal a folate of 1.8 μg/L severe headache. On examination you note neck
and a low vitamin D level. Which drug is most likely stiffness. An LP is performed which reveals a
to be responsible for these findings? yellow-coloured supernatant after centrifugation
a. Phenytoin of the CSF. What is the most likely diagnosis?
b. Carbamazepine a. Intracerebral haemorrhage
c. Ethosuximide b. Subarachnoid haemorrhage
d. Sodium valproate c. Subdural haematoma
e. Vigabatrin d. Extradural haematoma
e. None of the above
40. A 34-year-old man is diagnosed with epilepsy. He is
started on appropriate treatment. He goes to see his 45. A 38-year-old alcoholic presents following a serious
GP complaining of a generalized skin rash and ‘night fall. He complains of a severe headache. On
terrors’. Which drug is most likely to be responsible examination he appears drowsy and confused. What
for this? is the most likely diagnosis?
a. Phenytoin a. Intracerebral haemorrhage
b. Carbamazepine b. Subarachnoid haemorrhage
c. Ethosuximide c. Subdural haematoma
d. Sodium valproate d. Extradural haematoma
e. Vigabatrin e. None of the above

41. A known epileptic is admitted to A&E following two 46. A 35-year-old man was found unconscious following
consecutive seizures. Since the episode, he has a boxing match. He regains consciousness while
failed to regain consciousness. He has a history of in A&E. During neurological assessment, he rapidly
poor compliance with medication. As the house deteriorates and becomes less aware of his
officer on call you attempt to obtain intravenous surroundings. Which artery is most likely to have
access but fail. What is the next most appropriate been ruptured?
step in management? a. Anterior cerebral
a. Phenytoin b. Middle cerebral
b. Enobarbital c. Posterior cerebral
c. Clonazepam d. Vertebral
d. Lorazepam e. Middle meningeal
e. Diazepam
47. A 62-year-old man presents with weakness on his
42. A known epileptic is admitted to A&E following right side. His symptoms, however, dramatically
two consecutive seizures. Since the episode he improve within 24 hours. What is the most likely
has failed to regain consciousness. He is known diagnosis?
to drink heavily. What is the most appropriate initial a. Stroke
investigation? b. Stroke in evolution
a. Serum electrolytes c. Minor stroke
b. Head CT scan d. Transient ischaemic attack (TIA)
c. LP e. Completed stroke

51 
Single best answer (SBA) questions

48. A 60-year-old woman is diagnosed as having had a 53. A 32-year-old alcoholic presents to A&E. On
TIA arising from emboli in her internal carotid artery. examination his speech is notably confused. His
She is likely to display all of the following features eyes open spontaneously and he withdraws to pain.
EXCEPT? What is his Glasgow Coma Scale (GCS) Score?
a. Amaurosis fugax a. 9
b. Aphasia b. 10
c. Hemiparesis c. 11
d. Hemisensory loss d. 12
e. Transient global amnesia e. 13

49. A 45-year-old obese man presents with right-sided 54. A 35-year-old man presents with visual disturbance.
weakness, loss of sensation and visual disturbances. He is later diagnosed with MS. What is the most
On examination you note increased tone, brisk likely type of his visual loss?
reflexes and an upgoing plantar response. Which a. Scotoma
artery is most likely to be occluded? b. Bitemporal hemianopia
a. Anterior cerebral c. Homonymous hemianopia
b. Middle cerebral d. Homonymous quadrantanopia
c. Posterior cerebral e. None of the above
d. Vertebral
e. Middle meningeal 55. A 32-year-old man is found unconscious at his
home. He is immediately transferred to A&E. On
50. A 53-year-old man presents with sudden-onset vomiting examination he is noted to have bilateral pupillary
and vertigo. On examination you note left-sided facial constriction. The following are all causes of pupil
numbness and a diminished gag reflex. There is also constriction EXCEPT?
reduced sensitivity to pain and temperature on the right a. CN III nerve palsy
side and evidence of a broad-based ataxic gait. Which b. Horner syndrome
artery is most likely to be occluded? c. Argyll Robertson pupil
a. Anterior cerebral d. Opiate addiction
b. Middle cerebral e. Lesion at the pons
c. Posterior cerebral
d. Posterior inferior cerebellar artery (PICA) 56. A 31-year-old woman presents with ptosis of her
e. Middle meningeal right eye. On examination you note that her right
eye is abducted, pointing down and fixed to light.
51. A diabetic lifelong smoker presents with weakness Which artery is most likely to be affected in this
on his right side. On examination you note an presentation?
audible bruit in his internal carotid artery. What is the a. Anterior cerebral
most appropriate initial investigation? b. Middle cerebral
a. Head CT scan c. Posterior cerebral
b. Plasma glucose d. Posterior communicating
c. Carotid Doppler e. Middle meningeal
d. Urea and electrolytes
e. Magnetic resonance angiography 57. A 56-year-old woman presents with left-sided
facial weakness. On examination you note
52. An obese lifelong smoker presents with a 12-hour dribbling from the corner of her mouth and that
history of right-sided weakness, loss of sensation she is unable to close her eyes. She complains of
and visual disturbances. On examination you note hearing disturbance and you observe the presence
increased tone, brisk reflexes and an upgoing plantar of vesicles in her left ear. What is the most likely
response. His blood pressure is 145/95 mmHg. A diagnosis?
head CT scan reveals the presence of a cerebral a. Otitis media
infarct. What is the next most appropriate step in b. Bell palsy
management? c. Ramsay Hunt syndrome
a. Antihypertensives d. Guillain–Barré syndrome
b. Thrombolysis e. Otitis externa
c. Heparin
d. Warfarin 58. A middle-aged man complains of a sensation
e. Aspirin whereby his surroundings are revolving around him.

52
Neurology 5

He was recently prescribed gentamicin following husband. She initially complained of epigastric
a severe episode of pneumonia. What structure is fullness, which was followed by a period of
most likely to be affected in his condition? unresponsiveness where she looked ‘blank’.
a. Labyrinth Shortly afterwards, she started to smack her
b. CN VIII lips. She appeared confused and disorientated
c. Brainstem for a short period afterwards. She could not
d. Cerebellum remember the events. Clinical examination is
e. Basal ganglia unremarkable. What is the most likely
diagnosis?
59. A middle-aged man is diagnosed with a a. Simple partial seizure
pseudobulbar palsy. The following are all findings of b. Cocaine overdose
such a condition EXCEPT? c. Complex partial seizure
a. Dysarthria d. Tonic-clonic seizure
b. Dysphagia e. Absence seizure
c. Nasal regurgitation
d. Exaggerated jaw jerk 64. A 24-year old man presents to his GP with a change
e. Tongue fasciculations in personality. His girlfriend reports that he has
become aggressive, irrational and labile in mood in
60. A middle-aged man with MS presents with an ataxic the last 2 months, which is ‘not his usual self at all’.
broad-based gait. He is likely to demonstrate the Subsequent brain CT shows evidence of a brain
following signs EXCEPT? tumour. Which part of his brain is most likely to be
a. Intention tremor affected?
b. Vertical nystagmus a. Temporal lobe
c. Dysarthria b. Parietal lobe
d. Pendular reflexes c. Pituitary gland
e. Decreased tone d. Occipital lobe
e. Frontal lobe
61. A 40-year old man suffered a femoral shaft fracture
following a road traffic accident. Forty-eight hours 65. A 62-year old man presents to his GP with severe
later, he became increasingly drowsy with a rash progressive memory loss, initially noticed by his
on his chest and abdomen and died soon after. A wife. She has also noticed speech disturbance,
postmortem examination shows multiple petechial unsteadiness on walking, uncontrollable
haemorrhages in the corpus callosum and centrum trembling and tendency to fall asleep during the
semiovale of the brain region. What is the most likely day. On examination he has an ataxic gait. The
diagnosis? patient reports no family history of psychiatric
a. Diffuse axonal injury or neurological diseases. What is the most likely
b. Fat embolism diagnosis?
c. Watershed infarction a. Creutzfeldt–Jakob disease (CJD)
d. Septic embolism b. Fatal familial insomnia
e. Pulmonary embolism c. Kuru
d. Gerstmann–Straussler–Scheinker syndrome
62. A 55-year old woman presents to her GP with a e. Normal old age phenomenon
3-month history of progressive cognitive impairment,
urinary incontinence and unsteady gait leading to 66. An 80-year old man is accompanied to the GP
falls noticed by her son. Other than suffering from surgery by his daughter. She expresses concerns
subarachnoid haemorrhage in her youth, she has that her father may be suffering from Alzheimer
no other medical history. What is the most likely disease (AD) and would like a formal assessment.
diagnosis? Which of the following is not characteristic of AD?
a. Hereditary spinocerebellar ataxia a. Aphasia
b. Wilson disease b. Apraxia
c. Normal pressure hydrocephalus (NPH) c. Agnosia
d. Huntington disease d. Sudden cognitive decline
e. Pseudotumour cerebri e. Repetitive statements

63. A previously fit and well 55-year old woman is 67. A 30-year-old teacher presents to her GP with
brought to the emergency department by her a 2-year history of recurrent severe headaches.

53 
Single best answer (SBA) questions

The headaches occur two to three times per d. Musculocutaneous nerve


year, with each episode lasting 3–4 hours. e. Dorsal scapular nerve
They are unilateral and usually associated with
vomiting and photophobia. She has tried various 71. A 20-year old man presents to the emergency
analgesics but with minimal effect. Neurological department with weakness in his left wrist,
examination is unremarkable. What is the most following a party the night before where he drank
likely diagnosis? heavily and slept uncomfortably on a
a. Migraine chair overnight. On examination there is no
b. Tension headache obvious injury, but he has a left-sided wrist
c. Cluster headache drop. What is the most likely causative agent in
d. Temporal arteritis this case?
e. Analgesia-related headache a. Injury to median nerve
b. Injury to ulnar nerve
68. A 35-year old woman presents to the emergency c. Injury to radial nerve
department with severe right-sided headaches. d. Fracture of the distal radius
She tells you that these ‘attacks’ happen every e. Carpal tunnel syndrome
few months, and are associated with watering of
her right eye and nasal congestion. Each episode 72. A secondary school rugby player reports a history
can last for up to 2 hours. She is completely well of progressive inability to participate in the sport
in between these episodes. Which of the following over the last month. Initially, he had double vision
management is most likely to provide relief for this following a practice session. He was better the
patient? following morning, but his symptoms recurred
a. Oxygen inhalation the day after. Now he has double vision most of
b. Corticosteroids the time, except right after sleeping. He says his
c. Nonsteroidal antiinflammatory drugs strength and agility are declining, and his coach
d. Amitriptyline thinks he is playing too poorly to remain on the
e. Verapamil team. On examination he has mild ptosis, which
increases if he tries to maintain an upward gaze. His
69. A 34-year-old woman presents to her GP with arm strength is initially 5/5, but he tires very rapidly.
sudden-onset severe dizziness and vomiting After a few minutes of isotonic exertion, he cannot
but no other symptoms. On examination she lift his arms against gravity. His deep tendon reflexes
has a horizontal nystagmus with the fast phase to are normal. Which of the following is the most likely
the right. What is the most likely explanation for these diagnosis?
findings? a. Guillain–Barré syndrome
a. Acoustic neuroma on the right side b. Psychophysiologic weakness
b. Horizontal nystagmus probably indicates a c. Postconcussion syndrome
central cause d. Myasthenia gravis (MG)
c. Tertiary syphilis e. Myotonic dystrophy
d. Otitis externa
e. Viral labyrinthitis 73. A 40-year-old woman presents to her GP with a
1-month history of fatigue and weakness. She
70. A 25-year old man sees his GP for muscle reports that her eyes feel droopy as the day
weakness. He underwent a left-sided lymph node progresses. She also has difficulty in swallowing
biopsy recently to investigate a painless lump found and food tends to be left in her mouth after an
in the posterior triangle of his neck. Since then, he attempt to swallow. She is otherwise well with
has been complaining of difficulty shrugging his no known medical history. Clinical examination
left shoulder and tilting his head to the left side. On is unremarkable except for the presence of mild
examination his left scapula is more prominent than bilateral ptosis. Which of the following radiological
the right, especially on left shoulder abduction, and finding is most likely to be associated with this
his left shoulder appears more drooped than the patient’s condition?
right. Which one of the following nerves is most likely a. A widened mediastinum on chest X-ray
to be injured? b. Air–fluid level on chest X-ray
a. Long thoracic nerve c. Dilated bowel loops on abdominal plain X-ray
b. Suprascapular nerve d. Multiple lesions in liver on abdominal CT
c. Accessory nerve e. Solitary pulmonary nodule on chest X-ray

54
Neurology 5

h. VIII
EXTENDED-MATCHING i. IX
QUESTIONS (EMQs) j. X

Delirium For each scenario below, choose the most likely


Each answer can be used once, more than once or not corresponding option from the list given above.
at all. 1. Associated with unilateral ptosis and a ‘down
a. Concussion and out’ pupil.
b. Diabetic ketoacidosis 2. Known to cause loss of taste on the anterior
c. Encephalitis two-thirds of the tongue.
d. Hepatic encephalopathy 3. Associated with diplopia when looking down and
e. Hypercalcaemia away from the affected side.
f. Hypercapnia 4. Associated with unilateral visual loss in the
g. Hyperosmolar, nonketotic state form of an area of depressed vision within
h. Hyponatraemia the visual field.
i. Opiate analgesia 5. Associated with unilateral sensory loss on the face
j. Subarachnoid haemorrhage and tongue.
k. Subdural haematoma
Diseases of the peripheral nerves
For each scenario below, choose the most likely Each answer can be used once, more than once or not
corresponding option from the list given above. at all.
1. A previously fit and well 13-year-old boy presents a. Vitamin B6 deficiency
to A&E with a 24-hour history of increasing b. Vitamin B1 deficiency
confusion. On examination he is unwell, smells of c. Vitamin B12 deficiency
ketones and is dehydrated. d. Charcot–Marie–Tooth disease
2. A 70-year-old lifelong smoker who is attending the e. Guillain–Barré syndrome
oncologist presents to the medical admission unit f. Carpal tunnel syndrome
with a 48-hour history of increasing confusion and g. Mononeuritis multiplex
vomiting. His blood gases are normal, but he is h. Amyloidosis
dehydrated. i. HIV associated neuropathy
3. A 60-year-old smoker with a 75-pack year history j. Autonomic neuropathy
presents with a productive cough and feeling
short of breath. He is drowsy but rousable. He has For each scenario below, choose the most likely
a bounding pulse and a flapping tremor. corresponding option from the list given above.
4. An 82-year-old lady with a history of dementia and 1. A diabetic patient presenting with pain and
falls presents with increasing confusion and more numbness in her hands, in addition to wasting of
frequent falls. She is drowsy with a mental test her thenar muscles.
score of 2/10. She has a right-sided weakness 2. Associated with distal limb wasting and
and upgoing plantars. weakness. In advanced cases, the legs may
5. A previously fit 25-year-old man presents to A&E resemble inverted champagne bottles.
with a severe headache and increasing confusion. 3. Associated with weakness and complaints of
While you are examining him, he starts to fit. While numbness in the distal limbs. Additional features
applying the oxygen mask, you notice a cold sore. include respiratory muscle weakness and postural
hypotension.
Cranial nerve lesions 4. Seen commonly in individuals with a long-standing
Each answer can be used once, more than once or not history of alcohol abuse.
at all. 5. A sensory neuropathy seen occasionally in
a. I individuals on isoniazid therapy.
b. II
c. III Diseases of the muscle
d. IV Each answer can be used once, more than once or not
e. V at all.
f. VI a. Limb girdle dystrophy
g. VII b. Facioscapulohumeral dystrophy

55 
Extended-matching questions (EMQs)

c. Myasthenia gravis Extrapyramidal diseases


d. Dystrophia myotonica Each answer can be used once, more than once or not at all.
e. Myotonia congenita
a. Idiopathic Parkinson disease
f. Cushing syndrome
b. Multiple sclerosis
g. Hypothyroidism
c. Drug-induced Parkinson disease
h. Hyperthyroidism
d. Parkinsonism plus
i. Osteomalacia
e. Benign essential tremor
j. Duchenne muscular dystrophy
f. Chorea
g. Huntington disease
For each scenario below, choose the most likely
h. Hemiballismus
corresponding option from the list given above.
i. Myoclonus
1. Presents in early childhood with weakness in the j. Tics
proximal leg muscles.
2. Fatigability of proximal limb muscles in association For each scenario below, choose the most likely
with immunoglobulin G (IgG) acetylcholine corresponding option from the list given above.
receptor antibodies.
1. A condition associated with signs of
3. An autosomal dominant condition
parkinsonism and an inability to move the eyes
characterized by distal muscle weakness and
vertically or laterally.
myotonia.
2. Associated with violent swinging movement of one
4. An autosomal dominant condition characterized
side of the body.
by myotonia in childhood.
3. An autosomal dominant condition associated with
5. An autosomal recessive disorder associated with
a defective gene on chromosome 4.
abnormal muscle function of the shoulder and
4. Brief repeated stereotypical movements involving
pelvic girdle.
the face and shoulders.
5. A condition typically presenting in adult life associated
Epilepsy with perivenular plaques of demyelination.
Each answer can be used once, more than once or not
at all.
a. Grand mal Headache 1
b. Petit mal Each answer can be used once, more than once or not
c. Myoclonic seizure at all.
d. Jacksonian motor seizure a. Cervical spondylitis
e. Temporal lobe epilepsy b. Chronic sinusitis
f. Status epilepticus c. Giant-cell arteritis
g. West syndrome d. Malignancy
h. Juvenile myoclonic epilepsy e. Meningitis
i. Frontal lobe epilepsy f. Ménière disease
j. Lennox–Gastaut syndrome g. Migraine
h. Subarachnoid haemorrhage
For each scenario below, choose the most likely i. Subdural haematoma
corresponding option from the list given above. j. Tension headache
1. A patient presenting with muscle jerking and
incontinence of urine. For each scenario below, choose the most likely
2. Commonly seen in childhood whereby the corresponding option from the list given above.
individual ceases activity and stares for a few 1. A fit and well 30-year-old man presents with
seconds. a sudden-onset severe headache, confusion,
3. Associated with jerking movements photophobia and neck stiffness.
beginning in the corner of the mouth 2. A 45-year-old woman presents with recurrent
and spreading to involve the limbs on the headaches for the last year. She has no focal
opposite side. neurological deficit and the headaches have not
4. Phenytoin is the first-line treatment in this form of been worsening.
epilepsy. 3. A 79-year-old with hypertension presents with
5. Associated with feelings of unreality or undue worsening headaches over the sides of his head,
familiarity. and jaw ache on chewing.

56
Neurology 5

4. An 18-year-old student who has just started e. Cluster headache


university presents with a worsening headache f. Giant-cell arteritis
and marked photophobia. Neck stiffness is not g. Subarachnoid haemorrhage
detectable. h. Subdural haemorrhage
i. Temporomandibular joint disease
Nerves of the upper limb j. Tension headache
Each answer can be used once, more than once or not
at all. For each scenario below, choose the most likely
a. Axillary nerve corresponding option from the list given above.
b. Long thoracic nerve of Bell 1. A 22-year-old woman presents to her GP
c. Lower brachial plexus with frontal headaches, worse in the morning.
d. Medial cutaneous nerve of forearm She has no other symptoms. On examination
e. Median nerve she has a body mass index of 32 kg/m2 with
f. Musculocutaneous nerve nothing else to note. CT of the head and LP
g. Nerve to coracobrachialis are normal. Her condition settles with dietary
h. Radial nerve advice.
i. Ulnar nerve 2. A 60-year-old man presents to his GP with severe
j. Upper brachial plexus headaches over the frontal region and shoulders.
The examination is unremarkable except for a
For each scenario below, choose the most likely thick band palpable over the right temple. The
corresponding option from the list given above. ESR is 97 mm/h.
1. A 14-year-old boy presents with an inability to 3. A 54-year-old woman presents with
grip in his right arm after falling from a tree and severe headaches; she is completely
catching himself on a low-lying branch. incapacitated by the pain which is
2. A 30-year-old pregnant woman presents with present over her right eye. She often
numbness and reduced grip in her left hand, sees zig-zag lines before the headache and
symptoms which are worse at night. On they are so severe that she presses over her
examination the numbness appears limited to eye to try and relieve the pain. These attacks
the thumb, index and middle fingers. The thumb last several hours and disappear as quickly as
is particularly weak in flexion, opposition and they start.
abduction. 4. A 27-year-old man complains of headaches
3. A 25-year-old man presents to A&E with a broken to his GP. They are temporal and are relieved
arm following an arm wrestling contest. X-ray by sleeping. He is anxious as his company is
demonstrates a fracture of the proximal third of merging and he risks losing his job.
the humerus. He has wrist drop and has a loss of 5. A 19-year-old medical student presents
sensation over the web space between the thumb with headache and photophobia of 3 hours’
and index finger. duration. She is unable to touch her chest
4. The same gentleman as above, on further with her chin. There are no signs of raised
examination, has anaesthesia over the regimental intracranial pressure. The CSF shows
patch and is unable to abduct his arm. reduced glucose and numerous
5. A 58-year-old woman presents having had a polymorphs.
mastectomy and axillary node clearance. On
asking her to press her hands against the wall in Strokes
front of her, you notice that the scapula on the Each answer can be used once, more than once or not
affected side is more prominent, with the medial at all.
border standing proud.
a. Lacunar infarct
b. Partial anterior circulation infarct (PACI)
Headache 2 c. Partial anterior circulation syndrome (PACS)
Each answer can be used once, more than once or d. Partial occipital circulation syndrome (POCS)
not at all. e. Pontine stroke
a. Bacterial meningitis f. PICA occlusion
b. Benign intracranial hypertension g. Total anterior circulation syndrome (TACS)
c. Cerebral abscess h. TIA
d. Cerebral metastasis i. Vertebrobasilar insufficiency

57 
Extended-matching questions (EMQs)

For each scenario below, choose the most likely reaction to light. Work-up imaging included CT
corresponding option from the list given above. brain which revealed an elliptical, roughly
1. A 60-year-old hypertensive man with ischaemic 3 × 4 cm2 area of high attenuation peripherally.
heart disease presents to A&E with a ‘funny turn’. 2. A 26-year-old postgraduate university student
On examination he has right hemiparesis, right was admitted with headache and a 2-day history
inattention and a right homonymous hemianopia. of recent alteration in her sense of smell. A mild
2. A 72-year-old diabetic woman with renal disease pyrexia was noted by nursing staff. LP revealed a
presents to A&E with a right hemiparesis affecting normal protein content, slightly raised leucocytes
her right arm and leg; sensation is normal on the (lymphocytes predominant) and a normal
right and left. plasma:CSF glucose ratio. CT brain was reported
3. A 55-year-old obese man is admitted to the ward as normal. MRI brain (T2 weighted) revealed a
following a stroke. On admission he was found to slightly asymmetrical distribution of high signal
have loss of pain and temperature sensation on within the temporal lobes bilaterally, more so on
the left side of his body and the right side of his the right side.
face, nystagmus to the right and a right Horner 3. A 28-year-old mother of two is seen at outpatient
syndrome. clinic with a short history of headache, which was
4. A 90-year-old woman attends her GP following an worse in the mornings and on stooping to attend
episode of weakness that affected her right side: to her young children. She has no past medical
she was unable to stand or use her arm. She is history, although she is attending a dietitian-led
quite distressed, and worries that the symptoms weight loss clinic. Her only medication is the oral
may return. On questioning, the episode seems to contraceptive pill as she does not wish for further
have lasted around 4 hours. children at present. O/E: Bilateral papilloedema.
5. A 78-year-old man is admitted with a CT brain: normal.
stroke. CT demonstrates an infarct in the 4. A 73-year-old lady who lives alone was found by
anterior circulation. On examination he has her neighbour to be behaving oddly of late and
a right hemiparesis and a right homonymous more unsteady on her feet over the past couple
hemianopia. of weeks. She has a medical history that includes
hypertension, atrial fibrillation, previous total hip
replacement (THR) and Parkinson disease. O/E:
Headache 3
She is disorientated in time and place. Her mood
Each answer can be used once, more than once or not
was fluctuant. CT brain: low attenuation crescenteric
at all.
region adjacent to the right temporal bone.
a. Subarachnoid haemorrhage 5. A 28-year-old working mother reattended
b. Migraine outpatient clinic complaining of a 9-month history
c. Skull fracture of headache. They are now occurring around
d. Tension headache three times a week, mostly on weekdays, and
e. Subdural haematoma were partially responsive to codeine-based
f. Extradural haematoma analgesics. The headache is felt across the whole
g. Intracerebral haemorrhage head, with no visual disturbance or nausea.
h. Cerebral infarct Complete neurological examination was normal.
i. Cerebral sinus thrombosis Brain CT had been done at her second visit,
j. Idiopathic intracranial hypertension largely on patient request for peace of mind and
k. Viral encephalitis was reported as normal.
6. A 76-year-old lady was admitted to A&E following
For each scenario below, choose the most likely a collapse at home. She lives alone and was
corresponding option from the list given above. found by her daughter on the bedroom floor.
1. An 18-year-old man was riding pillion on a Her past medical history included angina and
motorbike without a helmet. At 50 mph (80.5 km/h) peripheral vascular disease. Skull X-ray taken
he was seen to fall from the bike onto the road. initially was normal. O/E: Right-sided weakness.
At the scene, he was found to have sustained Speech was normal. CT brain (at 24 hours):
multiple traumatic injuries to the thorax and limbs. low attenuation area in the region of left internal
His GCS score was 8. His left pupil had a sluggish capsule with adjacent oedema.

58
Rheumatology 6
c. Pericarditis
SINGLE BEST ANSWER (SBA) d. Felty syndrome
QUESTIONS e. Scleromalacia perforans

1. A 65-year-old man complains of pain and stiffness in his


6. A 30-year-old man is diagnosed with rheumatoid
hands. On examination you note swelling of the distal
arthritis. He mentions that the pain is difficult to cope
interphalangeal (DIP) joints and first carpometacarpal
with and has been ongoing for the last 4 months. What
joint. What is the most likely diagnosis?
is the next most appropriate step in management?
a. Rheumatoid arthritis
a. Nonsteroidal antiinflammatory drugs (NSAIDs)
b. Osteoarthritis
b. Cyclooxygenase-2 (COX-2) inhibitors
c. Psoriatic arthritis
c. Surgery
d. Gout
d. Tumour necrosis factor-α blockers
e. Pseudogout
e. Methotrexate
2. A 70-year-old woman presents with pain, swelling and
stiffness in her knees. The latter is for a short period 7. A 23-year-old man presents with increasing pain,
(<30 minutes), worse early in the morning. Which swelling and stiffness in his lower back. On examination
investigation is most likely to lead to a diagnosis? you note a reduction in chest expansion and obvious
a. Full blood count kyphosis. What is the most likely diagnosis?
b. Erythrocyte sedimentation rate (ESR) a. Ankylosing spondylitis
c. Rheumatoid factor (RF) b. Psoriatic arthritis
d. Knee X-ray c. Osteoarthritis
e. C-reactive protein (CRP) d. Reactive arthritis
e. Enteropathic arthritis
3. A 50-year-old man presents to his general practitioner
(GP) with pain in his hands. The GP notes small 8. A 20-year-old man returns from a trip to Amsterdam.
swellings on his distal and proximal interphalangeal He complains of pain in his knees as well as pain on
(PIP) joints. What is the next most appropriate step in passing urine. What is the most likely diagnosis?
management? a. Ankylosing spondylitis
a. Surgery b. Psoriatic arthritis
b. Heat therapy c. Osteoarthritis
c. Hydrotherapy d. Reactive arthritis
d. Steroids e. Enteropathic arthritis
e. Paracetamol

4. A 35-year-old woman presents with pain, swelling 9. A 52-year-old man presents to his GP with pain in
and stiffness in her hands. On examination you note his hands. The GP notes swelling of his DIP joints
swelling of the PIP joints as well as small nodules just as well as pitting of his nails. What is the most likely
below her elbows. Which investigation is most likely to diagnosis?
lead to a diagnosis? a. Ankylosing spondylitis
a. Platelet count b. Psoriatic arthritis
b. Haemoglobin levels c. Osteoarthritis
c. RF d. Reactive arthritis
d. Antinuclear factor e. Enteropathic arthritis
e. ESR
10. A 30-year-old woman is admitted with a painful right
5. A 45-year-old woman presents to the GP with marked knee. On examination the knee is warm to touch.
ulnar deviation of her hands. You note an obvious Joint aspiration confirms the presence of a gram-
swan neck deformity of her fingers. What extra positive coccus. What is the most appropriate step
manifestation is she most likely to suffer from? in management?
a. Anaemia a. NSAIDs
b. Pleural effusion b. COX-2 inhibitors

59 
Single best answer (SBA) questions

c. Bed rest d. Anticardiolipin antibodies


d. Joint mobilization e. Lupus anticoagulant
e. Flucloxacillin and fusidic acid
16. A child is taken to the GP by his mother. On
11. A 25-year-old woman presents with a rash on her examination the GP notes a rash on his legs and
nose and cheeks, which she mentions gets worse in buttocks. The mother mentions that he has difficulty
the sunlight. She also complains of joint pains in her walking and is particularly worried as she has noticed
hands. Which investigation is most likely to provide a blood in his urine. The child was recently treated for a
diagnosis? chest infection. What is the most likely diagnosis?
a. Full blood count a. Churg–Strauss syndrome
b. Antinuclear antibodies (ANAs) b. Henoch–Schönlein purpura
c. Anticardiolipin antibodies c. Microscopic polyangiitis
d. Complement levels d. Polyarteritis nodosa
e. RF e. Juvenile arthritis

12. A 45-year-old woman presents with pain in her 17. A 55-year-old man presents with severe pain and
hands. On examination you note thickening of her swelling of his right big toe. He is obese and a
skin and tapering of her fingers. She also mentions heavy drinker. What is the most appropriate step in
that she has difficulty opening her mouth. Which management?
investigation is most likely to support the diagnosis? a. Allopurinol
a. Full blood count b. Probenecid
b. Hand X-ray c. Colchicine
c. Anticentromere antibodies d. NSAIDs
d. Anti-Jo 1 antibodies e. Steroids
e. Anti-Ro antibodies
18. An elderly woman who was diagnosed recently with
13. A 55-year-old man presents with weakness in his haemochromatosis presents with pain and stiffness
shoulders. He mentions that he has difficulty raising in her knees resembling osteoarthritis. Which of the
his hands above his head. On examination you note following investigations is most likely to provide a
a purple-coloured rash on his face. What is the most diagnosis?
likely diagnosis? a. White cell count (WCC)
a. Dermatomyositis b. Knee X-ray
b. Polymyositis c. Serum calcium
c. Polymyalgia d. Joint fluid microscopy
d. Overlap syndrome e. Uric acid levels
e. Systemic lupus erythematosus (SLE)
19. An Asian woman presents to the GP complaining of
14. A 60-year-old woman presents with weakness in her generalized muscle pains. She mentions that she is
shoulders and lower limbs. She mentions that she struggling to walk and has difficulty rising from her
has difficulty walking upstairs and getting up from chair. What is the most likely abnormality on serum
sitting. You suspect a diagnosis of polymyositis. investigation?
Which investigation is most likely to confirm the a. High potassium
diagnosis? b. High calcium
a. Anti-Jo 1 antibodies c. Low sodium
b. ESR d. Low vitamin D
c. Electromyography (EMG) e. Low magnesium
d. Magnetic resonance imaging (MRI) of the spine
e. Muscle biopsy 20. A 70-year-old woman has been diagnosed with
osteoporosis. She is a heavy smoker and drinker.
15. A middle-aged woman presents with dry eyes and Her medical history includes a fractured distal radius.
a dry mouth. She also mentions that her fingers What is the most appropriate step in management?
feel colder than usual. Which antibody is usually a. Stop smoking
present? b. Stop alcohol
a. Anti-Ro and Anti-La antibodies c. Hormone replacement therapy
b. Anti-Jo 1 antibodies d. Bisphosphonates
c. Anticentromere antibodies e. Raloxifene

60
Rheumatology 6

21. A 75-year-old man presents with significant pain Finger clubbing


in his legs. On examination you note bowing of his Each answer can be used once, more than once or not
tibia. You suspect a diagnosis of Paget disease. at all.
Which investigation is most likely to confirm your
a. Alcoholic liver disease
suspicion?
b. Crohn disease
a. Serum magnesium levels
c. Emphysema
b. Serum alkaline phosphatase (ALP) levels
d. Familial clubbing
c. Vitamin B12 levels
e. Fibrosing alveolitis
d. Serum folate levels
f. Infective endocarditis
e. Serum sodium levels
g. Pulmonary abscess
h. Scleroderma
i. Squamous cell carcinoma
EXTENDED-MATCHING
For each scenario below, choose the most likely
QUESTIONS (EMQs) corresponding option from the list given above.
1. A 40-year-old man with right-sided chest pain with
Limp
weight loss of 3 months’ duration and a grumbling
Each answer can be used once, more than once or not
pyrexia.
at all.
2. A 20-year-old man with intermittent lower
a. Chondromalacia patellae abdominal pain on defecation associated with
b. Compartment syndrome blood and mucus.
c. Congenital dislocation of the hip 3. A 48-year-old woman presents with dyspnoea. On
d. Fractured neck of femur examination she has bilateral, fine, late inspiratory
e. Gout crackles.
f. Osteoarthritis of hip 4. An 18-year-old intravenous drug abuser who
g. Osteoarthritis of knee presents with fever and weight loss and who is
h. Perthes disease short of breath.
i. Slipped upper femoral epiphysis 5. A 15-year-old boy who presents to his GP surgery
j. Trochanteric bursitis with asthma.

For each scenario below, choose the most likely


corresponding option from the list given above. The painful joint
Each answer can be used once, more than once or not
1. On a GP visit to a nursing home, you are at all.
asked to see an 85-year-old woman with
dementia, recurrent falls and pain on a. Ankylosing spondylitis
mobilizing. She can now only walk with b. Fibromyalgia
assistance. On examination, when lying c. Gout
supine, she has a shortened externally rotated d. Osteoarthritis
left leg. e. Pseudogout
2. A 65-year-old man with high blood pressure and f. Psoriatic arthritis
ischaemic heart disease presents with a hot, g. Reiter syndrome
swollen and erythematous knee joint. His regular h. Rheumatoid arthritis
medications include a thiazide diuretic. i. Septic arthritis
3. A 13-year-old boy presents with niggly pain in j. SLE
the hip that is much worse after being tackled at
football. For each scenario below, choose the most likely
4. A 25-year-old man presents in accident corresponding option from the list given above.
and emergency (A&E) with a limp and 1. A 27-year-old man with low back pain and
excruciating pain in the lateral aspect of his morning stiffness. He had an episode of iritis a
lower leg following training for a year previously.
marathon. 2. A 40-year-old woman with symmetrically
5. A 58-year-old man presents to his GP with pain painful fingers and wrists. There is little
in his left knee of several months’ duration. evidence of synovitis. Her ESR is raised and
On examination he has a restricted range of she has an erythematous rash over her nose
movement in his left hip. and cheeks.

61 
Extended-matching questions (EMQs)

3. A 37-year-old man with gonococcal urethritis 2. A woman presenting with a scaly facial rash made
presents with pain in the hip and a limp. worse by sunlight.
4. An 82-year-old woman presents with a painful 3. A 25-year-old woman with a history of three
right hip and knee, worse on exercise. On consecutive miscarriages.
examination she has a positive Trendelenburg test 4. A 55-year-old man with a purple-coloured rash
and walks with a limp. around his eyes.
5. A 75-year-old man with a painful great toe, 1 week 5. A 65-year-old woman presenting with difficulty
after starting therapy for mild heart failure. in opening her mouth and palpable swellings in
her fingers.
Arthritis
Each answer can be used once, more than once or not Vasculitis
at all. Each answer can be used once, more than once or not
at all.
a. Enteropathic arthritis
b. Juvenile idiopathic arthritis a. Polymyalgia rheumatica
c. Tuberculous arthritis b. Temporal arteritis
d. Septic arthritis c. Kawasaki disease
e. Reactive arthritis d. Wegener granulomatosis
f. Osteoarthritis e. Churg–Strauss syndrome
g. Psoriatic arthritis f. Henoch–Schönlein purpura
h. Rheumatoid arthritis g. Essential cryoglobulinaemia
i. Gonococcal arthritis h. Polyarteritis nodosa
j. Salmonella arthritis i. Behçet disease
j. Microscopic polyangiitis
For each scenario below, choose the most likely
corresponding option from the list given above. For each scenario below, choose the most likely
corresponding option from the list given above.
1. Involvement of the DIP joints and first
carpometacarpal joint of the hands. 1. A 75-year-old man complaining of a history of
2. Involvement of the metacarpophalangeal joints stiffness and intense pain in his shoulder and
(MCPs) and PIP joints of the hands. back.
3. Involvement of the MCPs and PIP joints of the 2. A 55-year-old woman complaining of a sudden-
hands with pitting of the nails. onset headache associated with a loss of
4. Involvement of the knee joint with visual pulsation in her temporal artery.
discomfort and pain on passing urine. 3. A middle-aged man with abdominal pain and
5. Involvement of the lower limb joints in a patient deranged renal function. A renal biopsy confirms
with ulcerative colitis. inflammation of medium-sized arteries.
4. A middle-aged man presenting with generalized
abdominal pain and haematuria. Urine dipstick
Connective tissue disease reveals the presence of blood and protein. An
Each answer can be used once, more than once or not autoantibody screen reveals the presence of
at all. perinuclear antineutrophil cytoplasmic antibodies.
a. Antiphospholipid syndrome 5. A 43-year-old woman presenting with mouth
b. Systemic sclerosis ulcers and red-coloured painful lumps on her
c. SLE thighs.
d. Discoid lupus
e. Polymyositis Drugs
f. Dermatomyositis Each answer can be used once, more than once or not
g. Overlap syndrome at all.
h. Sjögren syndrome a. Methotrexate
i. Polymyalgia b. Penicillamine
j. Eczema c. Azathioprine
d. Sulfasalazine
For each scenario below, choose the most likely e. Hydroxychloroquine
corresponding option from the list given above. f. Auranofin
1. A woman presenting with a butterfly rash and joint g. Sodium aurothiomalate
discomfort. h. Diclofenac

62
Rheumatology 6

i. Infliximab e. Pseudogout
j. Leflunomide f. Gout
g. Septic joint
For each scenario below, choose the most likely h. Bone infarct
corresponding option from the list given above. i. Pathological fracture
1. Associated with mouth ulcers, proteinuria and j. SLE
thrombocytopenia.
2. Associated with headaches and visual For each scenario below, choose the most likely
disturbances. corresponding option from the list given above.
3. A therapeutic agent associated with a blue- 1. A 65-year-old lady presented to her GP with an
coloured skin discoloration in sunlight. acutely swollen left knee. She had no chronic
4. Associated with male infertility and hepatitis. illnesses and is on no medication. On examination
5. A therapeutic agent administered intramuscularly of her knee joint, there was evidence of an effusion.
known to cause mouth ulcers and pulmonary It was painful on palpation. All other joints were
fibrosis. unremarkable. Her old notes reveal a similar
problem in the past (an X-ray was taken previously).
It reported chondrocalcinosis of the left knee.
Back pain
2. A 9-year-old child presented to A&E with a
Each answer can be used once, more than once or not
16-hour history of a red, hot, tender, right knee.
at all.
On examination the right knee was warm to
a. Prolapsed intervertebral disc the touch and exquisitely painful. The knee was
b. Spondylolisthesis aspirated but no crystals were noted. WCC
c. Spinal stenosis was elevated. Ultrasound demonstrated a large
d. Ankylosing spondylitis effusion. No bony erosion seen.
e. Bacterial osteomyelitis 3. A 45-year-lady was seen at the outpatient clinic
f. Myeloma with a year-long history of pain in the small joints
g. Cauda equina syndrome of hands, feet and the left wrist. She complained
h. Tuberculosis osteomyelitis of stiffness on waking in the mornings. Her GP
i. Osteoporosis had undertaken a number of blood tests: ESR
j. Paget disease and CRP were elevated, ANA and RF were
reported as within normal range. O/E: warmth
For each scenario below, choose the most likely and swelling over multiple MCP and PIP joints
corresponding option from the list given above. bilaterally in keeping with active synovitis. No
1. A middle-aged woman presenting with low back rash or nodules noted. Radiographs of hands
pain, groin numbness and urinary retention. and feet: periarticular erosions at right third
2. Associated with an aching pain in the lower back and fourth and left third MCP joints. Soft tissue
when walking with radiation into the lower limbs. swelling noted adjacent to multiple MCP and PIP
The pain is eased by rest. joints in both hands.
3. A 23-year-old man presenting with sudden-onset 4. A 39-year-old gentleman was referred by his
back pain after weightlifting, which is exacerbated GP with lower back and joint pains. He had not
by movement and coughing. sustained any injury and gives an 8-month history
4. A 54-year-old man complaining of back stiffness of lower back pain and more recently discomfort
and shooting pains down his legs that worsens of the wrists and several fingers in the left hand.
throughout the day. On examination you note O/E: swelling and tenderness of the second, third
obvious lordosis and a waddling-like gait. and fourth DIP joints in the left hand. Both wrists
5. Pain and stiffness in the lower back known to tender and reduced range of active movement.
affect late teens. Inflammatory markers were raised. ANA and RF
negative. Radiographs of hands reveal normal
bone density. Erosions at the DIP joints correlate
Joint pain
with clinical findings.
Each answer can be used once, more than once or not
5. A 35-year-old lady is admitted to hospital with
at all.
a number of symptoms, including painful joints,
a. Rheumatoid arthritis tiredness and a rash. This has caused her to be
b. Psoriatic arthritis off work for the last 6 weeks. O/E: pale. Apyrexic.
c. Ankylosing spondylitis A number of small joints of both hands are
d. Reactive arthritis tender to touch. Admission blood tests reveal

63 
Extended-matching questions (EMQs)

the following: ESR, 67 mm/h; CRP, 8 ng/L; Hb, right elbow. A cannula is sited in his left forearm
10.2 g/dL; WCC, 2.4 × 109/L. Radiographs and for intravenous medication administration. X-ray
ultrasound of the joints were normal. reveals soft tissue swelling.
6. A 49-year-old man with a history of hypertrophic 7. A 47-year-old man is currently an inpatient in
obstructive cardiomyopathy and grade 3 heart the hepatology unit, which he has attended on
failure is admitted to the cardiology unit with acute a number of occasions in the past 5 years. He
pulmonary oedema. ECHO confirms preexisting has been a diabetic for the past 4 years. He
disease and an estimated ejection fraction of develops an acutely sore right knee. It is hot, red
24%. He receives treatment in addition to his and swollen, both subjectively and objectively.
regular cardiac medications. On day 4 of his Ultrasound of the knee confirms an effusion. It is
admission, he complains of a red, hot, swollen aspirated, the results of which shows no growth.

64
Endocrine and diabetes 7
c. Growth hormone
SINGLE BEST ANSWER (SBA) d. LH
QUESTIONS e. FSH

1. A middle-aged man is diagnosed with a pituitary


6. A 29-year-old man complains of headaches and visual
tumour following a magnetic resonance imaging
disturbances. He also remarks that he can express
(MRI) head scan. One of his complaints includes
milk from his breasts. What is the most appropriate
gradual loss of vision. With regard to his visual loss,
initial investigation?
which of the following features is he most likely to
a. MRI pituitary scan
demonstrate?
b. Visual field assessment by perimetry
a. Paracentral scotoma
c. Thyroid function tests
b. Mononuclear field loss
d. Serum prolactin level
c. Bitemporal hemianopia
e. Serum testosterone level
d. Homonymous hemianopia
e. Homonymous quadrantanopia
7. A 42-year-old man complains of visual disturbances
2. A 53-year-old woman is diagnosed with and headaches. He remarks his wedding ring feels
hypopituitarism. Which of the following hormones is tighter than usual. On examination you note a
most likely to be affected first? protruding jaw, interdental separation and spade like
a. Follicle-stimulating hormone (FSH) and luteinizing hands. Which investigation is most likely to confirm
hormone (LH) the diagnosis?
b. Thyroid-stimulating hormone (TSH) a. Serum growth hormone
c. Adrenocorticotropic hormone (ACTH) b. Serum insulin-like growth factor 1 (IGF-1)
d. Prolactin c. Serum prolactin
e. Growth hormone d. Fasting glucose
e. Visual field assessment by perimetry
3. A 65-year-old man is diagnosed with an underactive
pituitary gland. Which of the following is the most 8. A 40-year-old Caucasian woman presents to her
common cause of hypopituitarism? GP complaining of tiredness, weight gain and
a. Infection feeling unusually cold. On general examination
b. Vascular the GP notes a slow relaxing ankle jerk. The GP
c. Immunological diagnoses hypothyroidism. The patient enquires
d. Trauma about the cause of her condition. Which of
e. Neoplasm the following is the most likely cause of her
hypothyroidism?
4. A 41-year-old woman visits the general practitioner
a. Idiopathic
(GP) complaining of a sore mouth. On examination
b. Iodine deficiency
her tongue is very tender, and she feels that it has
c. Dyshormonogenesis
changed colour slightly. She was recently started on
d. Pituitary insufficiency
vitamin B12 injections for pernicious anaemia. What is
e. Hashimoto thyroiditis
the likely cause?
a. Candidiasis
9. A 35-year-old man complains of weight gain and
b. Herpes simplex virus
feeling cold. Thyroid function tests demonstrate a low
c. Glossitis
T4 and a raised TSH. He is also notably anaemic with
d. Sialadenitis
a haemoglobin of 10.2 g/dL. What is the most likely
e. Carcinoma of the tongue
type of anaemia he will demonstrate?
5. A 25-year-old woman is diagnosed with a. Aplastic anaemia
hypopituitarism. Which of the following hormones is b. Macrocytic
essential to replace first when managing her condition? c. Microcytic
a. Thyroid hormone d. Autoimmune haemolytic anaemia
b. Oestrogen e. Nonautoimmune haemolytic anaemia

65 
Single best answer (SBA) questions

10. A 55-year-old man with treated hypothyroidism 15. A middle-aged woman is diagnosed with
presents with confusion. His hypothyroidism is hyperthyroidism. Her GP commences carbimazole.
due to an underactive pituitary gland. Blood tests Which of the following is the most likely side effect of
reveal a sodium of 128 mmol/L and a glucose this medication?
of 2.8 mmol/L. Routine observations reveal a a. Rash
temperature of 35°C. What is the most appropriate b. Nausea and vomiting
step in management? c. Arthralgia
a. Sodium replacement d. Jaundice
b. Glucose replacement e. Agranulocytosis
c. Intravenous (IV) T3 replacement
d. IV hydrocortisone 16. A 29-year-old woman is diagnosed with
e. Warmed IV fluids hyperthyroidism. She is currently 28 weeks’
pregnant. The following are all suitable treatments for
11. A 35-year-old woman is diagnosed with an hyperthyroidism EXCEPT?
overactive thyroid gland. What is the most likely a. Carbimazole
aetiological cause of her thyrotoxicosis? b. Propranolol
a. Toxic multinodular goitre c. Radioactive iodine
b. Solitary toxic nodule d. Surgery
c. de Quervain thyroiditis e. Propylthiouracil
d. Graves disease
e. Thyroiditis factitia 17. A 35-year-old woman has recently undergone
radioactive iodine therapy for an overactive thyroid.
12. A 40-year-old woman complains of weight Her condition deteriorates and she becomes notably
loss and heat intolerance. Routine blood restless and delirious. On examination she has a
investigations confirm a diagnosis of Graves pulse rate of 120 beats/min and a temperature of
disease. Which of the following features are 40°C. She is severely dehydrated. What is the first
specific to this disease? most appropriate step in her management?
a. Lid lag a. Dexamethasone
b. Tremor b. Potassium iodide
c. Proximal myopathy c. Propranolol
d. Atrial fibrillation d. Carbimazole
e. Ophthalmoplegia e. IV fluids

13. A 35-year-old man complains of weight 18. A patient presents with exophthalmos and
loss and heat intolerance. On examination ophthalmoplegia. She has a known history of
you note an obvious goitre and red-coloured hyperthyroidism. Which of the following is the first
lesions on his shins. You suspect Graves disease. most appropriate step in management?
Which investigation is most likely to lead to a a. Steroids
diagnosis? b. Orbit irradiation
a. TSH c. Watchful waiting
b. T4 and T3 levels d. Lateral tarsorrhaphy
c. Thyroid ultrasound e. Normalization of thyroid status
d. TSH receptor antibodies
e. Thyroglobulin antibodies 19. A 36-year-old woman presents with a significantly
enlarged goitre. She complains of a discomfort
14. A 32-year-old woman complains of weight loss and in her neck as well as breathing difficulties. On
heat intolerance. She mentions that her heart is examination you note a solitary nodule. She is
always racing. A quick examination reveals marked worried about the possibility of cancer. Which of the
exophthalmos. What is the next most appropriate following investigations is most likely to reveal such a
step in management? diagnosis?
a. Propranolol a. TSH
b. Carbimazole b. T4 and T3 levels
c. Radioactive iodine c. Chest radiography
d. Surgery d. Fine-needle aspiration for cytology
e. Propylthiouracil e. Thyroid scan (125I)

66
Endocrine and diabetes 7

20. A 40-year-old woman presents with a goitre. On 25. A middle-aged woman is admitted to the medical
examination you note a nodule that feels hard admissions unit with severe abdominal pain and
and irregular in size. You also note the presence weakness. On examination she has a pulse rate of
of enlarged cervical lymph nodes. Fine-needle 120 beats/min, a blood pressure of 95/65 mmHg
aspiration confirms the presence of malignant cells. and a urine output of less than 30 mL/h. Routine
Routine blood investigations reveal a calcium level of blood tests reveal a random plasma glucose of
3.67 mmol/L. Which one of the following is the most 3.0 mmol/L. You note deep pigmentation of her
likely type of her thyroid cancer? buccal mucosa and skin creases. Her GP referral
a. Papillary letter comments that she has been feeling tired,
b. Follicular low in mood and suffering weight loss for the past
c. Anaplastic 1 month. What is the first most appropriate step in
d. Medullary cell her management?
e. Lymphoma a. Intramuscular hydrocortisone
b. IV glucose
21. A 35-year-old woman is diagnosed with thyroid c. IV hydrocortisone
cancer. Histological assessment confirms the d. IV dextrose
diagnosis as anaplastic with pulmonary metastases. e. IV fludrocortisone
What is the next most appropriate step in
management? 26. A 45-year-old gentleman is diagnosed with
a. Palliative impaired adrenal function. He has been taking
b. Radioactive iodine steroids on and off for the past 2 years for
c. Subtotal thyroidectomy treatment of severe Crohn disease. Which of the
d. Total thyroidectomy following investigations is most likely to reveal the
e. Chemotherapy cause of his adrenal failure?
a. Urea and electrolytes
22. A 43-year-old man is diagnosed with Cushing b. Adrenal antibodies
syndrome. Which one of the following is most likely c. Abdominal X-ray
to be decreased or inhibited in this condition? d. Tetracosactide (synacthen) short test
a. Protein catabolism e. Tetracosactide (synacthen) long test
b. Free water clearance
c. Circulating neutrophils 27. A 53-year-old woman complains that she has put
d. Uric acid production on a considerable amount of weight over the past
e. Circulating lymphocytes 2 months. She also mentions that her skin feels
thin and bruises easily. On examination you note
23. A 35-year-old woman complains of lethargy, low a plethoric complexion and purple striae on her
mood and loss of weight. On examination you abdomen. A 24-hour urinary cortisol is three times
note increased pigmentation of her buccal mucosa the upper limit of normal. What is the most likely
and skin creases. Her blood pressure drops to cause of her condition?
85/60 mmHg from 122/63 mmHg on standing. a. Adrenal adenoma
Which of the following investigations is most likely to b. Adrenal carcinoma
confirm her diagnosis? c. Adrenal hyperplasia
a. Serum urea and electrolytes d. Pituitary adenoma
b. Adrenal antibodies e. Carcinoid tumour
c. Abdominal X-ray
d. Tetracosactide (synacthen) short test 28. A 45-year-old woman is diagnosed with
e. Tetracosactide (synacthen) long test Cushing syndrome. Routine blood tests reveal
a significantly raised ACTH level. High-dose
24. A middle-aged man is diagnosed with adrenal gland dexamethasone suppresses plasma cortisol after
insufficiency. What is the most likely aetiological 48 hours. What is the most likely cause of her
cause for this condition? condition?
a. Autoimmune a. Adrenal adenoma
b. Adrenal gland tuberculosis b. Adrenal carcinoma
c. Haemorrhage c. Adrenal hyperplasia
d. Pituitary disease d. Pituitary adenoma
e. Adrenal gland tumour e. Carcinoid tumour

67 
Single best answer (SBA) questions

29. A 50-year-old man complains of feeling low in Her corrected serum calcium level is 2.01 mmol/L.
mood. On examination his blood pressure is What is the most likely diagnosis?
140/85 mmHg. His body mass index (BMI) is 31 kg/ a. Primary hyperparathyroidism
m2. He has purple streaks on his abdomen as well b. Secondary hyperparathyroidism
as pigmentation of skin creases. Investigations c. Tertiary hyperparathyroidism
confirm Cushing syndrome secondary to a pituitary d. DiGeorge syndrome
adenoma. What is the next most appropriate step in e. Pseudohypoparathyroidism
management?
a. External beam irradiation 34. A 42-year-old woman with breast cancer complains
b. Aminoglutethimide of nonspecific bone pain, abdominal discomfort
c. Ketoconazole and not opening her bowels for the past 1 week.
d. Metyrapone Her GP decides to admit her. Upon arrival, you note
e. Surgical excision of tumour she is severely dehydrated and confused. Routine
blood tests reveal a corrected serum calcium level of
30. A 30-year-old woman is admitted to accident 4.0 mmol/L. What is the first most appropriate step
and emergency (A&E) complaining of nausea and in her management?
headaches. She appears mildly confused and is a. Prednisolone
unaware of her surroundings. On examination her b. IV fluid replacement
blood pressure is 125/80 mmHg with no postural c. Bisphosphonates
drop. Investigations show a serum sodium of d. Calcitonin
122 mmol/L, a low plasma osmolality, a urine e. Phosphate
osmolality, which is much higher than plasma,
and a urinary sodium level, which is greater than 35. A 35-year-old man complains of numbness around
30 mmol/L. What is the most likely cause of her his mouth. On examination you note opposition of
symptoms? his thumb, extension of his interphalangeal joints
a. Renal tubular acidosis and flexion of his metacarpophalangeal joints. His
b. Glucocorticoid deficiency corrected serum calcium is 1.70 mmol/L. What is the
c. Syndrome of inappropriate antidiuretic hormone most likely cause of his symptoms?
(SIADH) a. Osteomalacia
d. Renal failure b. Pseudohypoparathyroidism
e. Salt-wasting nephropathy c. DiGeorge syndrome
d. Idiopathic hypoparathyroidism
31. What is the most appropriate management for the e. Renal failure
patient described above?
a. Furosemide 36. You suspect primary hyperaldosteronism in a
b. Hypertonic saline 65-year-old man with hypertension. Which of the
c. Hypotonic saline following investigations is most important in helping
d. Dimethylchlorotetracycline to confirm such a diagnosis?
e. Fluid restriction a. Plasma aldosterone-to-renin ratio
b. Serum aldosterone
32. A 32-year-old man with a recent diagnosis of mania c. Adrenal computed tomography (CT) scan
is treated with lithium. Three weeks later he returns d. Adrenal MRI scan
complaining of feeling thirsty and going to pass e. Serum renin
urine up to eight times a day. Which of the following
investigations is most likely to confirm the diagnosis? 37. A 45-year-old man complains of headaches,
a. Urine volume measurement sweating and nausea. On examination his pulse
b. Plasma sodium concentration rate is 135 beats/min and his blood pressure is
c. Urine osmolality 138/95 mmHg. Routine blood tests reveal a random
d. Plasma osmolality plasma glucose of 10.2 mmol/L. You suspect the
e. Water deprivation test possibility of a catecholamine-producing tumour.
Which of the following investigations is most likely to
33. A 25-year-old woman complains of abdominal pain confirm the diagnosis?
and nonspecific bone pain for the past 2 weeks. She a. Clonidine suppression and glucagon
also mentions that she has been feeling generally stimulation test
low in herself. Routine blood tests reveal a raised b. Abdominal CT scan
parathyroid hormone level and a low vitamin D level. c. Abdominal MRI scan

68
Endocrine and diabetes 7

d. 24-Hour urinary collection of metanephrines 43. A 32-year-old diabetic man on insulin is admitted to
e. Metaiodobenzylguanidine CT scintigraphy hospital having been found unconscious at home.
He quickly regains consciousness on arrival. On
38. A 57-year-old man presents to the GP complaining examination his eyes are sunken and his tongue
of feeling thirsty, weight loss and passing urine up appears dry. His breathing is deep at a rate of
to nine times a day. His BMI is 32 kg/m2. Routine 25 breaths/min. His blood pressure is 110/65 mmHg.
blood tests reveal a random blood glucose of Routine blood tests reveal a plasma glucose of
12.2 mmol/L. What is the next most appropriate step 22 mmol/L. You perform an arterial blood gas, which
in management? demonstrates a pH of 7.10, a partial pressure of
a. Diet therapy carbon dioxide (PCO2) of 3.5 kPa and a bicarbonate
b. Glibenclamide of 19 mmol/L. What is the next most appropriate
c. Insulin step in his management?
d. Acarbose a. IV saline
e. Rosiglitazone b. IV insulin
c. Sodium bicarbonate infusion
39. A 55-year-old man has been recently diagnosed with d. IV colloids
type II diabetes. His BMI is 35.5 kg/m2. What is the e. Intramuscular glucagon
most appropriate medical management?
a. Glibenclamide 44. A 65-year-old diabetic man who is a poor clinic
b. Metformin attendee presents to his GP complaining of blurred
c. Acarbose vision. Fundoscopy reveals the presence of yellow-
d. Repaglinide coloured exudates as well as cotton wool spots.
e. Pioglitazone What is the most likely diagnosis?
a. Background retinopathy
40. A newly diagnosed diabetic is commenced on b. Maculopathy
metformin. What is the most common side effect of c. Preproliferative retinopathy
this drug? d. Proliferative retinopathy
a. Lactic acidosis e. Cataracts
b. Hypoglycaemia
c. Pruritis 45. A 55-year-old diabetic woman complains of
d. Rash numbness, tingling and pain in her toes and hands.
e. Diarrhoea She reports that she feels like she is ‘walking
on cotton’. Examination reveals bilateral loss of
41. A 17-year-old boy complains of feeling tired, sensation on the palms and soles. What is the most
thirsty and going to pass urine more frequently likely diagnosis?
than normal. He has also developed abdominal a. Autonomic neuropathy
pain and vomiting in the last 2 hours. His serum b. Acute painful neuropathy
glucose level is 22 mmol/L and urinalysis shows c. Diabetic mononeuropathy
ketonuria. What is the next most appropriate step d. Diabetic amyotrophy
in management? e. Symmetrical sensory neuropathy
a. Acarbose
b. Lifestyle modification alone 46. A 29-year-old diabetic woman is trying for a baby.
c. Insulin She visits her GP to ask about whether her diabetes
d. Metformin may affect her pregnancy. Diabetes may result in the
e. Pioglitazone following complications during pregnancy EXCEPT?
a. Macrosomia
42. A diabetic patient on insulin is found unconscious b. Polyhydramnios
at home. He is immediately transferred to hospital. c. Respiratory distress syndrome
Routine blood tests reveal a glucose level of d. Preeclampsia
1.9 mmol/L. What is the next most appropriate step e. Neonatal hyperglycaemia
in management?
a. Intramuscular glucagon 47. A 31-year-old man complains of sweating and
b. Oral carbohydrate palpitations. He remarks that he often sees double.
c. IV dextrose Routine blood tests reveal a fasting plasma glucose
d. Diazoxide of 2.2 mmol/L. He undergoes an abdominal CT
e. Octreotide scan which reveals the presence of a pancreatic

69 
Single best answer (SBA) questions

tumour. What is the next most appropriate step in she has not lost any weight, she goes to the toilet
management? approximately once every half an hour and passes
a. Surgical excision of tumour large amounts of dilute urine. She has a past
b. Diazoxide medical history of a brain injury that occurred 1 year
c. Octreotide ago when she was in a road traffic accident. What is
d. Chemotherapy the most likely diagnosis?
e. Radiotherapy a. Type 1 diabetes
b. Type 2 diabetes
48. A 51-year-old man presents to A&E with nausea and c. Nephrogenic diabetes insipidus
vomiting. His wife, who attends with him, remarks that d. Neurogenic diabetes insipidus
earlier in the day he had a terrible headache, which e. Pituitary adenoma
felt as if he had ‘been hit on the head with a baseball
bat’. She told him to get some rest and on his way 52. A 40-year-old man who has been hospitalized with a
to the door he walked into a coffee table. He has a subarachnoid haemorrhage becomes progressively
medical history of hypertension. His current blood more confused and agitated. His urinary output is
pressure is 82/46 mmHg. On inspection the patient low. Blood tests show hyponatraemia, low plasma
appears confused, weak and sweaty. On examination osmolality and high urine osmolality. His blood
he has a third-nerve palsy and bitemporal pressure is 130/86 mmHg and he is euvolaemic.
hemianopia. What is the most likely diagnosis? What is the most likely diagnosis?
a. Sheehan syndrome a. Undiagnosed diabetes
b. Ruptured craniopharyngioma b. Undiagnosed hypothyroidism
c. Pituitary macroadenoma c. SIADH
d. Pituitary microadenoma d. Addison disease
e. Pituitary apoplexy e. Acute renal failure

49. A 35-year-old woman was referred to the hospital by 53. A 67-year-old woman has a bone mineral density
her GP with polyuria, polydipsia and altered sensations (BMD) of >2.5 standard deviations below the mean
in her limbs that has been progressively worsening BMD. She has been advised to stop smoking and
over the last 3 weeks. The hospital rules out type 2 given calcium and vitamin D supplements. She has
diabetes but investigations reveal she is hypertensive no independent clinical risk factors for fracture.
(154/80 mmHg), as well as hypokalaemic and What is the most appropriate pharmacological
alkalotic. The doctor diagnoses Conn syndrome. What intervention?
is the cause of Conn syndrome? a. Bisphosphonates
a. Cortisol and aldosterone deficiency b. Teriparatide
b. ACTH-secreting tumour c. Calcitonin
c. Adrenaline-secreting tumour d. Strontium ranelate and raloxifene
d. Aldosterone-secreting tumour e. No pharmacological intervention is required
e. Chronic excessive cortisol secretion
54. A 45-year-old man was admitted to the emergency
50. A 42-year-old male was admitted to the department after he was found confused by his
endocrinology ward with increasing weight, central family at home. His notes show that he has been
obesity and depression. He also has been having complaining of headaches and blurred vision for the
erectile dysfunction for the last month. Cushing last 4 months, as well as polyuria. He is overweight
disease is the suspected cause and an overnight and you suspect that he has undiagnosed diabetes
dexamethasone suppression test is ordered to mellitus type 2. What would be the most appropriate
confirm. What is the underlying cause of Cushing test to perform to confirm hyperglycaemia in this
disease? patient?
a. Adrenal adenoma a. 2-Hour plasma glucose
b. Pituitary adenoma b. Fasting glucose
c. Iatrogenic (i.e., steroid treatment) c. Glycosylated haemoglobin (HbA1c)
d. Ectopic ACTH d. Random blood glucose
e. Primary hyperaldosteronism e. Urine dipstick for glucose

51. A 25-year-old woman presents to her GP with a 55. A 29-year-old man is referred to the endocrinology
2-month history of polydipsia, polyuria and malaise. clinic with a 5-week history of unexplained flushing
On further questioning, the patient reveals that in his face and upper chest. The man has also been

70
Endocrine and diabetes 7

experiencing diarrhoea and vomiting that have d. 150/70 mm/Hg


worsened over time. The man has no family history e. As low a blood pressure as the patient
of any relevant conditions. The doctor examines the tolerates
patient and notes several murmurs on auscultation
that were not previously noted by the man’s GP. 59. A 56-year old patient presented with a month’s
The doctor suspects cardiac damage secondary history of nocturia and feeling dehydrated all the
to carcinoid syndrome. If liver metastases are the time. He was diagnosed with type 2 diabetes
source of the hormones, damage to which two mellitus and was started on sulphonylureas
valves in the heart is associated with carcinoid (gliclazide). Which one of the followings is NOT true
syndrome? regarding this medication?
a. Pulmonary and aortic a. Evidence of long-term prevention of
b. Pulmonary and tricuspid macrovascular complications of type 2
c. Aortic and mitral diabetes.
d. Tricuspid and mitral b. Hypoglycaemic effect of sulphonylureas is due to
e. Pulmonary and mitral enhancement of insulin secretion by pancreatic
beta cells.
56. A 19-year-old woman presents to her GP c. Preferably used in nonobese diabetic
complaining that she has been unable to get patients.
pregnant despite trying for 2 years. The woman has d. Sulphonylureas reduce fasting plasma glucose
also been having irregular periods since she started by an average of 2–4 mmol/L and HbA1c by 1%
puberty. The woman has hirsutism and poorly to 2%.
controlled acne, and her hair has also been getting e. The most common and most serious side effect
progressively thinner during the last few weeks. Her of sulphonylureas is hypoglycaemia.
family history reveals that her mother is obese and
suffers from type 2 diabetes. The patient is 135 cm 60. A 75-year-old man was brought to the GP by
tall and weighs 62 kg. The GP suspects polycystic a neighbour who was worried that he was not
ovary syndrome (PCOS) and refers the girl for tests eating and drinking properly since his wife had
at the hospital. Which of these hormones is raised in died. The examination was normal apart from what
PCOS? appeared clinically to be a folliculitis over his trunk
a. LH and buttocks. He was treated with antibiotics. He
b. FSH returned 4 months later with marked bruising in the
c. Oestradiol same distribution and now with gingival hypertrophy
d. Human chorionic gonadotrophin and bleeding aphthous ulcers. The full blood count
e. 17-Hydroxyprogesterone and clotting screen were normal. What is the most
likely diagnosis?
57. A 75-year-old man with type 2 diabetes mellitus a. Chronic lymphocytic leukaemia
who normally takes long-acting insulin once daily b. Gingivitis
is admitted to hospital with a urinary tract infection. c. Chronic myeloid leukaemia
He is hyperglycaemic, with ketones and glucose on d. Scurvy
urinalysis. He is orientated, alert and not vomiting. e. Vitamin K deficiency
What is the most appropriate hypoglycaemic agent
during his acute illness? 61. A previously fit and well 68-year-old retired nurse
a. Increase in long-acting insulin presents to her GP for routine health check. She
b. Soluble human insulin does not drink or smoke, and is not taking any
c. Metformin regular medications. Initial blood tests show Na
d. An oral sulphonylurea 136 mmol/L, K 3.1 mmol/L, bicarbonate 34 mmol/L
e. A combination of metformin and a sulphonylurea and chloride 94 mmol/L. A subsequent CT scan is
performed due to an elevated aldosterone-to-renin
58. A 55-year-old man with type 2 diabetes mellitus ratio, which shows bilateral adrenal enlargement.
complicated by diabetic retinopathy sees his GP Which of the following is the most appropriate
regularly for blood pressure monitoring. What is the treatment for this patient?
optimal blood pressure for patients with diabetes a. Amiloride
mellitus? b. Amlodipine
a. 130/90 mm/Hg c. Clonidine
b. 130/80 mm/Hg d. Bendroflumethiazide
c. 140/80 mm/Hg e. Spironolactone

71 
Single best answer (SBA) questions

62. A 50-year-old woman presents to the emergency a. ADH


department with a 3-day history of worsening b. Dehydroepiandrosterone (DHEA)
nausea, lethargy, abdominal pain and bony pains. c. Cortisol and ACTH levels
She is a chronic heavy smoker and has been d. IGF-1
recently diagnosed with lung cancer, and is awaiting e. Abdominal CT scan
treatment. Clinically she appears cachectic and
dehydrated. Blood results show a serum calcium of 66. A 65-year-old woman is brought in by her family
2.90 mmol/L (2.25–2.5 mmol/L). What is the most for progressive fatigue and confusion. Past medical
appropriate initial step in her management? history is notable for ovarian cancer. Physical
a. Calcitonin examination reveals dry mucous membranes.
b. Pamidronate Blood tests show an elevated adjusted serum
c. Thiazide diuretic calcium of 3.2 mmol/L, a low-normal albumin
d. IV saline level, a low-normal phosphorus level and elevated
e. Prednisolone alkaline phosphatase. What is the initial step in the
management of hypercalcemia of malignancy?
63. A 23-year-old woman with cystic fibrosis a. Fluid restriction
complicated by chronic cholestasis presents to her b. Bisphosphonates intravenously
GP with a 1-week history of muscle weakness and c. Phosphate depletion
tremor in her hands. She admits that she has been d. Aggressive rehydration
noncompliant with her medications. Neurological e. Diuresis with furosemide
examination reveals diminished tendon reflexes
throughout. What is the most likely diagnosis? 67. A previously fit and well 30-year-old woman presents
a. Hypoglycaemia to her GP with a 3-month history of amenorrhoea.
b. Vitamin K deficiency Clinical examination is unremarkable and urine
c. Vitamin E deficiency beta human chorionic gonadotropin is negative.
d. Vitamin B deficiency Subsequent blood tests show fasting morning
e. Vitamin D deficiency prolactin levels of 350 mg/L. What is the most
appropriate next step for this patient?
64. A 50-year-old woman is brought to the emergency a. Pituitary MRI
department by ambulance after she was found b. Head CT
on the floor unresponsive by her husband. c. Bromocriptine
She is severely hypoglycaemic on arrival, but d. Surgical intervention
regains consciousness following IV glucose. She e. Radiation therapy
reports no medical conditions and is not on any
medications, but tells you that in the last 2 months 68. A 35-year-old woman who is 8 months postpartum
she has had intermittent episodes of dizziness, presents to her GP with palpitations, sweats,
sweating and tremors, which seem to improve agitation and weight loss. On examination she
after eating a chocolate bar or biscuit. Subsequent has evidence of lid lag and tachycardia. There is a
blood tests show raised insulin, C-peptide, diffusely enlarged painless goitre with an audible
proinsulin and glucagon levels. What is the most bruit. Subsequent blood tests show normal
likely diagnosis? erythrocyte sedimentation rate level but ‘abnormal’
a. Insulinoma thyroid function tests. What is the most likely
b. Glucagonoma diagnosis?
c. Somatostatinoma a. Graves disease
d. Pancreatic adenocarcinoma b. Postpartum thyroiditis
e. Insulin administration c. Hashimoto thyroiditis
d. de Quervain thyroiditis
65. A 10-year-old boy presents to the emergency e. Thyroid tumour
department with worsening fatigue, dizziness,
muscle weakness, vomiting and diarrhoea. On arrival 69. A 45-year-old woman sees her GP after noticing
he appears dehydrated and hypotensive. Blood a lump in her neck. Following a series of
tests show sodium of 125 mmol/L, K 5.2 mmol/L investigations, a diagnosis of thyroid malignancy is
and serum glucose of 2.1 mmol/L. Despite IV suspected. Which of the following is a characteristic
fluids, his electrolytes have not showed much of thyroid malignancy?
improvement. Which of the following investigations is a. Solid cold nodules
most likely to give you the diagnosis? b. Cystic cold nodules

72
Endocrine and diabetes 7

c. Increased thyroxin level 5. A middle-aged man presenting with enlarged


d. Increased TSH level hands and feet. On further questioning he
e. None of the above complains of visual disturbances and headaches.

70. A 65-year-old man presents to the emergency


Thyroid disease
department with acute confusional state
Each answer can be used once, more than once or not
preceded by a 4-day history of fevers, cough and
at all.
breathlessness. Antibiotics were commenced for
treatment of community-acquired pneumonia. a. Hashimoto thyroiditis
Results from confusion screen showed low TSH, b. Lymphocytic thyroiditis
T4 and T3 levels. He is not known to have any c. Dyshormonogenesis
preexisting thyroid diseases. Which of the following d. Myxoedema coma
statements is incorrect? e. Graves disease
a. The abnormal findings on thyroid function tests f. Plummer disease
are not related to thyroidal illness. g. de Quervain thyroiditis
b. The most prominent alterations are low h. Thyroid cancer
serum T3. i. Thyroid crisis
c. The highest incidence of low TSH values occurs j. Toxic multinodular goitre
in the most severely ill group.
d. Most patients who are critically ill have reduced For each scenario below, choose the most likely
T4 levels. corresponding option from the list given above.
e. Thyroid hormone replacement is used to correct 1. Overactivity of the thyroid gland associated with
the low serum levels. fever and neck pain.
2. A common cause of hyperthyroidism associated
with immunoglobulin G (IgG) antibodies.
EXTENDED-MATCHING 3. A common cause of hypothyroidism associated
QUESTIONS (EMQs) with autoantibody formation and goitre
formation.
4. A middle-aged woman presenting with
Pituitary disease enlargement of the thyroid gland and a markedly
Each answer can be used once, more than once or not
raised serum calcium.
at all.
5. A woman with known hyperthyroidism presenting
a. Pituitary apoplexy with fever, tachycardia and restlessness following
b. Sheehan syndrome radioactive iodine therapy.
c. Empty sella syndrome
d. Kallmann syndrome
e. Pituitary tumour Diabetic complications
f. Syphilis Each answer can be used once, more than once or not
g. Haemochromatosis at all.
h. Sarcoidosis a. Background retinopathy
i. Lymphocytic hypophysitis b. Maculopathy
j. Anorexia c. Preproliferative retinopathy
d. Proliferative retinopathy
For each scenario below, choose the most likely e. Symmetrical sensory neuropathy
corresponding option from the list given above. f. Autonomic neuropathy
1. Associated with loss of smell and colour g. Acute painful neuropathy
blindness. h. Diabetic mononeuropathy
2. Associated with severe blood loss following i. Diabetic amyotrophy
childbirth. j. Mononeuritis multiplex
3. A middle-aged woman presenting with
sudden-onset headaches, nausea and visual For each scenario below, choose the most likely
disturbances. corresponding option from the list given above.
4. A 37-year-old obese woman with known 1. A common form of neuropathy affecting the toes
hypertension presenting with sudden-onset and soles of the feet.
headaches. A head MRI scan reveals an unusually 2. Associated with cotton wool spots and venous
small pituitary gland. beading.

73 
Extended-matching questions (EMQs)

3. Associated with blot haemorrhages and yellow- 2. A 34-year-old obese gentleman presenting with a
coloured exudates. plethoric complexion and purple-coloured striae
4. A diabetic patient complaining of wasting of his on his abdomen.
quadriceps muscles. 3. Associated with sudden-onset headaches,
5. A diabetic patient presenting with a burning palpitations and sweating.
sensation in his anterior lower limbs that is 4. A middle-aged woman presenting with weight loss
generally worse at night. and lethargy. Examination reveals hyperpigmentation
of oral mucosa and skin creases.
Drugs 5. A 45-year-old man presenting with increased
Each answer can be used once, more than once or not urinary frequency and thirst. Blood glucose levels
at all. are normal.
a. Glibenclamide
b. Tolbutamide Diabetes
c. Metformin Each answer can be used once, more than once or not
d. Acarbose at all.
e. Rosiglitazone a. Acanthosis nigricans
f. Repaglinide b. Diabetes insipidus
g. Protamine insulin c. Diabetic ketoacidosis (DKA)
h. Insulin lispro d. Fasting serum glucose of >7.1 mmol/L
i. Insulin aspart e. Glycosylated haemoglobin
j. Pioglitazone f. Hyperosmolar nonketotic coma
g. Pyoderma gangrenosum
For each scenario below, choose the most likely h. Random serum glucose >7.1 mmol/L but below
corresponding option from the list given above. 12 mmol/L
1. Associated with diarrhoea and abdominal i. Type 1 diabetes mellitus
bloating. This agent may also result in elevated j. Type 2 diabetes mellitus
serum transaminase levels.
2. Associated with diarrhoea and lactic acidosis. For each scenario below, choose the most likely
3. First-line agent in the treatment of type II diabetes corresponding option from the list given above.
in obese individuals. 1. A patient presents with polydipsia and polyuria,
4. A drug that works via closure of K+ adenosine often drinking 15–20 L of fluid per day. Fasting serum
triphosphate (ATP) channels but is best avoided in glucose is 5.2 mmol/L; serum osmolarity is reduced.
elderly people. 2. A 56-year-old man presents with a 1-week history
5. An insulin-releasing agent developed from of polyuria and polydipsia; he has been drinking
glibenclamide. copious amounts of cola to rehydrate himself and
feels very thirsty. Serum glucose is 37 mmol/L.
Investigations 3. A 14-year-old boy presents with a 2-day history of
Each answer can be used once, more than once or not lethargy, shortness of breath and polydipsia. His GP
at all. started amoxicillin for a chest infection 3 days ago.
On examination he is very dehydrated, smells of nail
a. Thyroid function tests
varnish remover and is making deep sighing breaths.
b. Oral glucose tolerance test
4. A patient is diagnosed with impaired glucose
c. Plasma aldosterone levels
tolerance.
d. Serum urea and electrolytes
5. This diagnosis is associated with a total lack of
e. Short synacthen test
insulin production and presents in the young.
f. Long synacthen test
g. Dexamethasone suppression test
h. Urine and plasma osmolality
i. Fasting blood glucose The pancreas and diabetes
j. 24-Hour urinary collection of metanephrines Each answer can be used once, more than once or not
at all.
For each scenario below, choose the most likely a. Type 2 diabetes
corresponding option from the list given above. b. Gestational diabetes mellitus
1. A middle-aged man presenting with enlargement c. Hyperosmolar hyperglycaemic state (HHS)
of his hands and feet. d. DKA

74
Endocrine and diabetes 7

e. The metabolic syndrome d. Cushing syndrome


f. Maturity-onset diabetes of the young (MODY) e. Congenital adrenal hyperplasia
g. Hypoglycaemia f. Addison disease
g. Bartter syndrome
For each scenario below, choose the most likely
corresponding option from the list given above. For each scenario below, choose the most likely
1. A 60-year-old woman whose recent-onset corresponding option from the list given above.
diabetes is entirely controlled by diet, metformin 1. A 45-year-old woman with buccal
and sulphonylureas. hyperpigmentation, weakness, abdominal pain,
2. A 15-year-old girl with diabetes who is known to hyperkalaemia and hypernatraemia.
suffer from a mutation in the glucokinase gene. 2. A 36-year-old man with a hypochloremic
3. A 50-year-old man suffering from mild fasting metabolic alkalosis, low serum renin and a mass
hyperglycaemia, hypertriglyceridaemia and central in the adrenal glands on MRI.
adiposity who has deranged function tests. 3. A 30-year-old female who presents with
4. A 10-year-old boy who is found in his room tachycardia, excessive sweating, pallor
drowsy, vomiting, severely dehydrated, suffering and weight loss. She was diagnosed with
from acidotic breathing (Kussmaul breathing). hypertension but it has been difficult to control
5. A 42-year-old pregnant woman with despite medications.
hyperglycaemia who has a family history of 4. A rheumatology patient on long-term
noninsulin-dependent diabetes and has previously steroids who presents with weight increase,
given birth to a large baby. moon face, menstrual irregularity and purple
striae.
The adrenal glands 5. A young girl presenting with virilism and
Each answer can be used once, more than once or not hirsutism.
at all.
a. Phaeochromocytoma
b. Conn syndrome
c. Renal artery stenosis

75 
This page intentionally left blank
Haematology 8
c. Myelofibrosis
SINGLE BEST ANSWER (SBA) d. Polycythaemia
QUESTIONS e. Chronic lymphocytic leukaemia (CLL)

1. A 70-year-old man attends his routine anticoagulation


5. A 5-year-old girl is brought by her mother to the
clinic appointment for international normalized ratio
emergency department after noticing multiple
(INR) check and warfarin dosing. He has been on
bruises on her arms and legs, and two episodes of
long-term warfarin for permanent atrial fibrillation for
nosebleeds. She has recently recovered from an upper
10 years. His INR is 7.2 but he is well in himself with
respiratory tract infection. She has no other symptoms
normal vital signs and no recent bleeding. What is the
and there is no relevant family history. What is the most
most appropriate management for this patient?
important investigation to establish the diagnosis?
a. Discontinue use of warfarin for 3 days and
a. Prothrombin time (PT)
recheck INR
b. Partial thromboplastin time (PTT)
b. Oral administration of 1  mg of vitamin K
c. Bleeding time
c. Intravenous administration of 10 mg of vitamin K
d. Peripheral blood film
d. Intravenous administration of 1 mg of vitamin K
e. INR
e. Reduce dose of warfarin

2. A 75-year-old man was referred to the haematology 6. A 65-year-old woman receives a blood transfusion
department by his general practitioner (GP) following a following a recent episode of haematemesis. Within
recent blood test which showed a platelet count of 90 × minutes, her blood pressure falls to 105/65 mmHg. She
109/L. Which of the following conditions is NOT a known begins to itch and complain of difficulty in breathing. On
cause for the development of thrombocytopaenia? examination you note swelling of her lips and face. After
a. Myelodysplastic syndromes stopping the blood transfusion, what is the next most
b. Chemotherapy appropriate step in management?
c. Haemolysis, elevated liver enzymes and low a. Adrenaline
platelet count (HELLP) syndrome b. Hydrocortisone
d. Proinflammatory cytokines c. Antihistamines
e. Liver cirrhosis d. Colloids
e. Crystalloids
3. A middle-aged obese man presents with a headache,
dizziness, tinnitus and visual disturbance. On
7. A 66-year-old woman has problems at work due to
examination you note a plethoric complexion and
lateness and reduced performance. You suspect that
evidence of hepatosplenomegaly. Routine blood
her problems might arise from excessive alcohol.
tests reveal a haemoglobin (Hb) of 202 g/L, a white
Which haematological test is likely to be most helpful
cell count of 22 × 109/L and a platelet count of
in supporting the diagnosis?
405 × 109/L. Further investigations reveal a raised red
a. Blood film
cell volume and uric acid level. What is the next most
b. Erythrocyte sedimentation rate
appropriate step in management?
c. Mean corpuscular volume (MCV)
a. Chemotherapy
d. Platelet count
b. Radioactive phosphorus
e. White cell count
c. Allopurinol
d. Venesection
e. Lipid-lowering agents 8. A 31-year-old woman presents to accident and
emergency having cut her hand while preparing
4. A 56-year-old man presents with weight loss and dinner. At the time she mentioned that it was bleeding
lethargy. Routine blood tests reveal a Hb of 101 g/L, a and so she bandaged her hand herself before coming
white cell count of 14 × 109/L and a platelet count of to hospital. You remove the bandage and notice the
410 × 109/L. Blood film examination reveals ‘tear drop’- bleeding has stopped. What factor is most likely to
shaped red cells. What is the most likely diagnosis? have initiated the clotting process?
a. Myelodysplasia a. Factor X
b. Chronic myeloid leukaemia b. Factor XIII
77 
Single best answer (SBA) questions

c. Factor V 13. An overweight 56-year-old woman returns from a


d. Factor VII trip to Hong Kong. She complains of a swollen left
e. Factor VIII leg. On examination you note calf tenderness and
iliofemoral venous distension. What is the most
9. A 15-year-old girl presents with bruising, epistaxis appropriate initial investigation?
and heavy periods. She has recently recovered a. Doppler ultrasound
from a viral infection. Physical examination is b. Venography
unremarkable. What is the most appropriate initial c. Ventilation perfusion scan
investigation? d. Platelet count
a. Platelet count e. Coagulation screen
b. Antiplatelet antibodies
c. Megakaryocyte count 14. A 13-year-old African boy who recently arrived in
d. Hb level the UK to join his parents presents to the GP with a
e. Bleeding time facial distortion for the past 10 months from a lesion
involving his jaw. On examination he has an enlarged
10. A 75-year-old man presents to his GP with a history right mandibular mass. Biopsy of one of the lymph
of easy bruising and lower back pain. Examination nodes reveals many lymphocytes of similar size and
reveals no obvious abnormality. Chest radiograph morphology, with some macrophages. Infection with
reveals multiple lytic lesions. Serum protein which of the following organisms is most likely to be
electrophoresis reveals a paraprotein band at the associated with development of the boy’s facial lesion?
gamma zone. Bone marrow biopsy shows the a. Adenovirus
presence of plasma cell infiltrate >10%. Peripheral b. Cytomegalovirus
blood film reveals numerous rouleaux formations and c. Epstein–Barr virus
plasma cells. Which of the following is likely to be d. Human immunodeficiency virus (HIV)
diagnosed? e. Human T-lymphotropic virus type 1 (HTLV-1)
a. CLL
b. Monoclonal gammopathy of uncertain 15. A middle-aged man is started on unfractionated
significance (MGUS) heparin for a suspected deep vein thrombosis.
c. Multiple myeloma What is the most appropriate method for monitoring
d. Solitary plasmacytoma this drug?
e. Waldenstrom macroglobulinaemia a. PT
b. INR
11. A middle-aged man has been diagnosed with a c. Activated partial thromboplastin time (APTT)
bleeding disorder. He is unsure of the name of the d. Platelet count
condition. However, he remembers the consultant e. Thrombin time
informing him that there are two main bleeding
disorders which affect males and that he suffers 16. A middle-aged man comes into hospital for routine
from the most common one. What is the next most surgery. A preoperative full blood count shows mild
appropriate step in management? anaemia and a raised white blood count of 30 × 109/L,
a. Factor X injection 90% of which are mature lymphocytes. On examination
b. Factor IX injection he has small clusters of mobile, firm lymph nodes the
c. von Willebrand factor injection size of peas in the posterior cervical triangle and an
d. Factor VIII injection easily felt spleen. What is the most likely diagnosis?
e. Factor VII injection a. CLL
b. Hodgkin disease
12. A 35-year-old man presents with multiple bruises on c. Infectious mononucleosis
his arms. He is deeply jaundiced and has recently d. Myelofibrosis
been found to have gallstones and a dilated biliary e. Viral throat infection
tree on ultrasound examination. Abnormalities of the
following clotting factors are likely to be responsible 17. A 35-year-old woman presents with slight bruising
for his bruising EXCEPT? on her arms. She is currently taking digoxin and
a. Factor II warfarin having been diagnosed with atrial fibrillation.
b. Factor VII Her INR is 7.6. What is the next most appropriate
c. Factor IX step in management?
d. Factor X a. Stop warfarin and give vitamin K
e. Factor V b. Stop warfarin and give protamine sulphate

78
Haematology 8

c. Stop warfarin and give fresh frozen plasma and a platelet count of 154 × 109/L. He undergoes
d. Stop warfarin completely bone marrow testing which reveals the presence
e. Reduce the current dose of warfarin of the Philadelphia chromosome. What is the most
likely diagnosis?
18. A middle-aged man presents with a sore throat a. Chronic myeloid leukaemia
and bruising of his lower limbs. On examination b. Acute promyelocytic leukaemia
you note evidence of lymphadenopathy and c. Acute lymphoblastic leukaemia
hepatosplenomegaly. A bone marrow aspirate d. Acute myeloid leukaemia
confirms the presence of abnormal myeloid cells. e. CLL
What additional finding is most likely to confirm a
diagnosis of acute myeloid leukaemia? 23. A 25-year-old male from Thailand was found to have
a. Blast cells mild microcytic anaemia with normal serum iron
b. Philadelphia chromosome levels. Further genetic analysis reveals that he has
c. Auer rods α-thalassaemia trait due to the presence of cis-
d. Smear cells deletion of the α-globin genes on chromosome 16.
e. T(15;17) translocation Which of the following is most likely to be seen on
the peripheral blood film?
19. A 45-year-old gentleman is diagnosed with acute a. Bite cells
leukaemia. The consultant explains to him that he b. Heinz bodies
will need to commence chemotherapy. He informs c. Pappenheimer bodies
the patient of the possibility of acute tumour lysis d. Schistocytes
syndrome as a consequence of tumour death e. Target cells
following treatment. The consultant then turns to
you and asks you to explain the most common 24. A 40-year-old man is diagnosed with leukaemia. The
biochemical disturbances of the syndrome to the consultant explains that his condition is associated
patient. The following are all common disturbances with involvement of the central nervous system.
EXCEPT? What is the most likely diagnosis?
a. Hyperkalaemia a. Chronic myeloid leukaemia
b. Hyperuricaemia b. Acute promyelocytic leukaemia
c. Hyperphosphataemia c. Acute lymphoblastic leukaemia
d. Hypocalcaemia d. Acute myeloid leukaemia
e. Hyponatraemia e. CLL

20. The defence system of the small intestine consists of 25. A 49-year-old man presents with fever, weight loss
T and B lymphocytes. B lymphocytes are responsible and night sweats. Bone marrow testing confirms the
for producing antibodies which act in the gut lumen. diagnosis of chronic myeloid leukaemia. What is the
Which is the main antibody that is produced? next most appropriate step in management?
a. Immunoglobulin G (IgG) a. Hydroxyurea
b. IgA b. Bone marrow transplantation
c. IgM c. Tyrosine kinase inhibitors
d. IgE d. Radiotherapy
e. IgD e. Alpha interferon

21. A middle-aged gentleman is diagnosed with acute 26. A 65-year-old woman presents with lethargy and
promyelocytic leukaemia. What is the next most a sore throat. On examination you note evidence
appropriate step in management? of bruising. Routine blood investigations reveal a
a. Cytosine arabinoside white cell count of 17 × 109/L, an Hb of 90 g/L and
b. Daunorubicin a platelet count of 75 × 109/L. A blood film reveals
c. Methotrexate the presence of excess lymphocytes. You suspect a
d. All-trans retinoic acid diagnosis of CLL. What additional finding would help
e. Bone marrow transplant to confirm such a diagnosis?
a. Blast cells
22. A 45-year-old man presents with fever, weight loss b. Philadelphia chromosome
and night sweats. On examination you note evidence c. Auer rods
of splenomegaly. Routine blood investigations reveal d. Smear cells
an Hb of 102 g/L, a white cell count of 105 × 109/L e. T(15;17) translocation

79 
Single best answer (SBA) questions

27. A 24-year-old woman presents with fever, night c. Burkitt lymphoma


sweats and weight loss. On examination you note d. MALT lymphoma
enlargement of her cervical lymph nodes. Which e. Sézary syndrome
investigation is most likely to lead to a diagnosis?
a. Blood count 32. A man with newly diagnosed Hodgkin disease
b. Liver function tests comes to see his GP. He has been reading about the
c. Computed tomography chest and abdominal condition on the Internet and came across a section
scan on ‘B symptoms’. He asks the GP what they are.
d. Chest X-ray What is the most likely example of a ‘B symptom’?
e. Lymph node biopsy and histology a. Chest pain
b. Cough
28. A 17-year-old girl from Nigeria is brought into c. Headache
the emergency department with 2-day history of d. Haematuria
worsening abdominal pain and bilateral hip pain e. Night sweats
in the absence of any trauma. This is her third
presentation with similar symptoms in 1 year. She 33. A woman with newly diagnosed Hodgkin disease is
required opioids for pain control during her previous being commenced on chemotherapy. The following
admissions. Her Hb has always been low and is are all important factors when determining treatment
70 g/L on this admission, with a high reticulocyte choice EXCEPT?
count. What is the most likely diagnosis? a. Stage
a. Spherocytosis b. Involved sites
b. Megaloblastic anaemia c. Age
c. Sickle cell disease (SCD) d. Bulk of lymph nodes involved
d. Sideroblastic anaemia e. Presence or absence of ‘B symptoms’
e. Thalassaemia
34. A 47-year-old man presents with lethargy and a sore
29. A 23-year-old man complains of fever and night throat. On examination you note evidence of bruising
sweats. On examination you note enlargement of and enlargement of cervical lymph nodes. A lymph
his cervical lymph nodes. He comments that when node biopsy is performed and histology confirms
he drinks alcohol he experiences pain at the site of the diagnosis as low-grade non-Hodgkin lymphoma.
these nodes. What is the most likely diagnosis? The following are all features of low-grade disease
a. Non-Hodgkin lymphomas EXCEPT?
b. Burkitt lymphoma a. Involvement of the bone marrow
c. Mucosa-associated lymphoid tissue (MALT) b. The disease is incurable with conventional
lymphoma chemotherapy
d. Hodgkin disease c. Middle-aged people are affected
e. Sézary syndrome d. Older-aged people are affected
e. Involvement of the gastrointestinal tract in all
30. A 17-year-old female presents to the GP cases
complaining of feeling tired all the time. On further
questioning, she admits to having heavy periods. 35. A 68-year-old woman presents to the GP feeling
Routine blood tests are done which show a Hb of lethargic and more breathless than usual. Routine
79 g/L and an MCV of 72.6 fL. What would the likely bloods are taken which show a Hb of 81 g/L and an
management plan be? MCV of 107.3 fL. What is the most likely cause of her
a. Vitamin B12 injections anaemia?
b. Thiamine tablets a. Iron-deficiency anaemia
c. Iron tablets b. Anaemia of chronic disease
d. Vitamin C supplements c. Pernicious anaemia
e. Folate supplements d. Haemolytic anaemia
e. Thalassaemia
31. A 28-year-old man is suspected of suffering from a
form of lymphoma. A lymph node biopsy is performed 36. A middle-aged woman complains of epigastric pain
and histology confirms the presence of Reed– in association with nausea and heartburn. Her GP
Sternberg cells. What is the most likely diagnosis? refers her for an endoscopy which reveals evidence
a. Non-Hodgkin lymphomas of gastric ulceration. Biopsies are taken and the
b. Hodgkin disease histological diagnosis is of a lymphoma. A diagnosis

80
Haematology 8

of lymphoma is made. What is the most likely type of request a blood film. The following are all likely to be
lymphoma? noted on the blood film EXCEPT?
a. Non-Hodgkin lymphomas a. Microcytosis
b. Hodgkin disease b. Macrocytosis
c. Burkitt lymphoma c. Hypochromia
d. MALT lymphoma d. Anisocytosis
e. Sézary syndrome e. Poikilocytosis

37. A 12-year-old boy of African origin has recently 42. A 35-year-old man with known Crohn disease
moved with his family to the UK. The family go to complains of feeling tired. Routine blood
see the GP as they are worried that their child is not investigations reveal a Hb of 102 g/L, a serum iron of
eating well. He comments that he has slight pain 7 μmol/L and a ferritin of 123 μg/L. What is the most
in his jaw and that it feels numb. After appropriate likely diagnosis?
investigation, he is found to have a lymphoma. What a. Anaemia of chronic disease
is the most likely type? b. Folate-deficiency anaemia
a. Non-Hodgkin lymphoma c. Iron-deficiency anaemia
b. Hodgkin disease d. Haemolytic anaemia
c. Burkitt lymphoma e. Aplastic anaemia
d. MALT lymphoma
e. Sézary syndrome 43. A 60-year-old vegan woman presents with
generalized fatigue and malaise. Routine blood
38. A 62-year-old man presents with nonspecific bone investigations reveal a Hb of 82 g/L. What is the
pain. He comments that his vision is not as good as most likely diagnosis?
it used to be and that he often gets up in the night a. Anaemia of chronic disease
to pass urine. Routine blood investigations reveal a b. Folate-deficiency anaemia
Hb of 102 g/L, a calcium of 4.2 mmol/L, a urea of c. Vitamin B12-deficiency anaemia
17.7 mmol/L and a creatinine of 175 μmol/L. What is d. Haemolytic anaemia
the most likely diagnosis? e. Aplastic anaemia
a. Waldenstrom macroglobulinaemia
b. Multiple myeloma 44. A 65-year-old woman is diagnosed with macrocytic
c. Acute-on-chronic renal failure anaemia. On examination you note a sore tongue. Her
d. Anaemia of chronic disease medical history includes autoimmune thyroid disease.
e. Benign prostatic hypertrophy Which investigation is most likely to lead to a diagnosis?
a. Blood count
39. You suspect multiple myeloma in a 67-year-old man. b. Vitamin B12 level
Which initial investigation is most likely to lead to a c. Serum folate level
diagnosis? d. Parietal cell autoantibodies
a. Urea and electrolytes e. Intrinsic factor autoantibodies
b. Bone profile
c. Platelet count 45. A 25-year-old woman presents with symptoms
d. Protein electrophoresis of anaemia. There is a family history of coeliac
e. Liver function tests disease. Investigations reveal a reduced serum B12
of 102 ng/L. What is the most likely cause of her
40. A 19-year-old girl complains of heavy periods. On anaemia?
examination you note that her hair and nails are a. Ileal Crohn disease
brittle. Routine blood investigations reveal a Hb of b. Pernicious anaemia
98 g/L. What is the most likely diagnosis? c. Coeliac disease
a. Vitamin B12-deficiency anaemia d. Small intestinal bacterial overgrowth
b. Folate-deficiency anaemia e. Dietary deficiency
c. Iron-deficiency anaemia
d. Anaemia of chronic disease 46. A 45-year-old man is diagnosed with anaemia. A
e. Aplastic anaemia blood film reveals the presence of megaloblasts.
What is the next most appropriate step in
41. A 25-year-old woman presents with fatigue and management?
lethargy. Routine blood investigations reveal a Hb of a. Vitamin B12 replacement
96 g/L, an MCV of 72 fL and a ferritin of 4 μg/L. You b. Folate replacement

81 
Single best answer (SBA) questions

c. Vitamin B12 and folate replacement 51. A patient with newly diagnosed autoimmune
d. Iron replacement haemolytic anaemia is started on treatment. The
e. Blood transfusion haematologist notes that the patient’s autoantibodies
attach to RBCs at a temperature of 29°C. Which
47. A middle-aged gentleman comes to see his GP management plan would you instigate first?
complaining of lethargy and shortness of breath. a. Steroids
He has recently undergone mitral valve replacement b. Splenectomy
following a diagnosis of mitral valve stenosis. Routine c. Azathioprine
blood investigations reveal he is anaemic. The d. Avoidance of cold exposure
GP suspects a diagnosis of haemolytic anaemia. e. Cyclophosphamide
The following are features of haemolytic anaemia
EXCEPT? 52. A 55-year-old woman with inguinal
a. Increased serum bilirubin lymphadenopathy presented to the GP, and a
b. Increased serum lactate dehydrogenase subsequent diagnosis of Hodgkin disease was
c. Decreased reticulocyte count made. Which of the following associated findings in
d. Low haptoglobins Hodgkin disease would give the worst prognosis?
e. Presence of methaemalbumin a. Cervical and inguinal lymphadenopathy
b. Just inguinal lymphadenopathy
48. A 2-year-old boy of Middle Eastern origin is c. Mediastinal and inguinal lymphadenopathy
referred to the paediatric outpatient department d. Mediastinal, inguinal lymphadenopathy and night
by his GP following an abnormal blood count. sweats
The GP mentions that there is a strong family e. Mediastinal, inguinal lymphadenopathy and
history of thalassaemia. On examination you pruritis
note evidence of an enlarged maxilla and
prominent frontal bones. What is the most likely 53. A 42-year-old man is diagnosed with warm antibody
diagnosis? autoimmune haemolytic anaemia. Which of the following
a. Alpha thalassaemia antibodies are components of this form of anaemia?
b. Beta thalassaemia minor a. IgG
c. Beta thalassaemia intermedia b. IgA
d. Beta thalassaemia major c. IgM
e. SCD d. IgD
e. IgE
49. A 19-year-old girl with known SCD presents to
the accident and emergency department with 54. A 35-year-old man presents with new-onset severe
severe bone and chest pain following a viral pains in his fingers when he goes outside in the cold,
infection. On examination you note she is mildly as well as going numb and cold. He had a recent
dehydrated. Her blood pressure and pulse are bout of pneumonia. Laboratory investigation reveals
stable and oxygen saturation is 99% on room the presence of IgM autoantibodies and a diagnosis
air. What is the next most appropriate step in of Raynaud phenomenon was made. Peripheral
management? blood film showed no abnormalities. What is the
a. Fluid replacement most likely diagnosis?
b. Analgesia a. Cold autoimmune haemolytic anaemia
c. Oxygen b. Malaria
d. Antibiotics c. Paroxysmal cold haematuria
e. Aspirin d. Paroxysmal nocturnal haematuria
e. Warm autoimmune haemolytic anaemia
50. A 34-year-old woman with systemic lupus
erythematosus is found to be anaemic. 55. A 44-year-old woman was diagnosed with type A
Which investigation is most likely to lead to a atrophic gastritis after presenting with tiredness for
diagnosis? over 6 months.
a. Coombs test a. Decreased serum ferritin
b. Serum ferritin level b. Decreased serum folate
c. Blood film c. Increased neutrophil segmentation
d. Vitamin B12 level d. Increased haematocrit
e. Serum folate level e. Microcytic RBCs

82
Haematology 8

56. A 30-year-old mother who is a carrier for haemophilia d. Leukaemia


A sees her GP with concerns that her 3-year-old e. Tuberculosis
child may have the condition. She reports that her f. HIV
child bruises easily and his knees get very swollen g. Toxoplasmosis
when he falls over. Which one of the following tests of h. Sarcoidosis
haemostasis is more sensitive for this disease? i. Carcinoma
a. Bleeding time j. Rheumatoid arthritis
b. PT
c. INR For each scenario below, choose the most likely
d. APTT corresponding option from the list given above.
e. Platelet count 1. A disease of young adults associated with a sore
throat, headache and a macular rash.
2. A viral infection associated with depletion of CD4
EXTENDED-MATCHING helper lymphocytes.
QUESTIONS (EMQs) 3. Characterized by malignant B and T lymphocytes
with symptoms of anaemia, infection and bleeding.
Anaemia 4. Associated with neoplastic cell formation in the
Each answer can be used once, more than once or not bone marrow as result of a variety of genetic and
at all. environmental factors.
a. Iron deficiency 5. A form of herpes virus that may result in fever,
b. Vitamin B12 deficiency lethargy, diarrhoea and vomiting.
c. Aplastic anaemia
d. Paroxysmal nocturnal haemoglobinuria Splenomegaly
e. Autoimmune haemolytic anaemia Each answer can be used once, more than once or not
f. Glucose-6-phosphate dehydrogenase deficiency at all.
g. SCD a. Schistosomiasis
h. Thalassaemia b. Leukaemia
i. Hereditary spherocytosis c. Myelofibrosis
j. Hereditary elliptocytosis d. Malaria
e. Tuberculosis
For each scenario below, choose the most likely f. Rheumatoid arthritis
corresponding option from the list given above. g. Gaucher disease
1. Associated with a pancytopenia and a h. Kala-azar
hypocellular bone marrow. i. Endocarditis
2. An autosomal dominant condition associated with j. Typhoid
increased red cell osmotic fragility.
3. Associated with jaundice and splenomegaly. The For each scenario below, choose the most likely
blood film may demonstrate the presence of cigar- corresponding option from the list given above.
shaped blood cells. 1. A disease associated with snail vectors which
4. An X-linked disorder associated with neonatal release cercariae that penetrate the skin to cause
jaundice and haemolysis due to ingestion of fava a papular rash.
beans. 2. A condition prevalent in Asia and associated with
5. A type of anaemia which occurs due to deficiency dry wart-like hyperpigmented skin lesions.
of CD59 and delay accelerating factor proteins. 3. A disease widespread in the tropics associated
with RBC rupture by sporozoites.
Lymphadenopathy 4. A lipid storage disease known to cause
Each answer can be used once, more than once or not thrombocytopenia, bone pain and
at all. neurodegenerative effects.
a. Epstein–Barr virus 5. A middle-aged woman presenting with abdominal
b. Cytomegalovirus pain, diarrhoea and confusion. On examination
c. Lymphoma you note the presence of rose spots on the trunk.

83 
This page intentionally left blank
SURGERY
QUESTIONS
Chapter 9

Trauma and surgical emergencies . . . . . . . . . . . 87


Chapter 10

Perioperative care . . . . . . . . . . . . . . . . . . . 93
Chapter 11

Gastrointestinal surgery . . . . . . . . . . . . . . . . 95
Chapter 12

Cardiothoracics and vascular surgery . . . . . . . . 105


Chapter 13

Orthopaedics . . . . . . . . . . . . . . . . . . . . . 109
Chapter 14

Ear, nose and throat . . . . . . . . . . . . . . . . . 115


Chapter 15

Urology . . . . . . . . . . . . . . . . . . . . . . . . 119
Chapter 16

Breast surgery . . . . . . . . . . . . . . . . . . . . 125


This page intentionally left blank
Trauma and surgical
emergencies 9
one-third of the distance from the anterior superior
SINGLE BEST ANSWER (SBA) iliac spine and umbilicus on the right side of his
QUESTIONS abdomen. What is being described here?
a. Grey Turner sign
1. A 23-year-old woman presents to accident and b. Cullen sign
emergency after being stabbed in the back with a c. McBurney point
knife. On examination you note weakness in her right d. Rovsing sign
limb and loss of proprioception below the level of the e. Murphy sign
injury site. You also note loss of temperature and pain
on the opposite side below the site of injury. What is 5. A 55-year-old man presents to accident and
the most likely diagnosis? emergency following a stabbing to his chest. On
a. Central cord syndrome examination you note an open wound on the right
b. Posterior cord syndrome side of his chest. The patient is severely breathless.
c. Anterior cord syndrome His oxygen saturation is 85% on room air. You apply
d. Brown-Séquard syndrome an occlusive dressing to his wound, taping three
e. Spinal shock of the four sides to the skin. His oxygen saturation
begins to improve. What is the most likely diagnosis?
2. A 5-year-old boy is admitted to accident and a. Tension pneumothorax
emergency following contact with a hot pan. On b. Open pneumothorax
examination you note erythema and mottling of his c. Massive haemothorax
right hand. His blood pressure is 100/60 mmHg d. Flail chest
and pulse rate 65 beats/min. You decide to initiate e. Cardiac tamponade
fluid resuscitation but find it difficult to obtain good
access in both his arms. You fast bleep the registrar 6. A 26-year-old woman presents with sudden-onset
and while waiting you note that his blood pressure is right iliac fossa pain. On examination you note that
quickly dropping. What is the next most appropriate when pressure is applied to her left iliac fossa, the
step in management bearing in mind the child’s age? patient complains of increased pain in her lower right
a. Central line side. What is being described here?
b. Venous cut down a. Grey Turner sign
c. Femoral line b. Cullen sign
d. Arterial line c. McBurney point
e. Intraosseous needle in the proximal tibia d. Rovsing sign
e. Murphy sign
3. A 29-year-old man was involved in a road traffic
accident during which he received a blow to the front 7. A 32-year-old woman presents with lower right-sided
of his chest from the steering column. On examination abdominal pain. She describes the pain as initially being
he had a tender sternum, the pulse was 120 beats/ centralized and associated with nausea. Routine blood
min, blood pressure 80/60 mmHg, jugular venous investigations reveal a white cell count of 23 × 109/L. A
pressure + 20 cm water and the heart sounds were urine dipstick reveals 3+ leucocytes. Which investigation
faint. What is most likely to confirm the diagnosis? is least useful in establishing the diagnosis?
a. Arterial blood gases a. Abdominal X-ray
b. Electrocardiogram (ECG) b. Abdominal ultrasound
c. Echocardiogram c. Pregnancy test
d. Serum creatinine d. Abdominal computed tomography (CT) scan
e. Ventilation–perfusion scan e. Pelvic ultrasound

4. A 32-year-old man presents with a 2-day history 8. A 45-year-old boxer is brought to accident and
of sudden-onset pain in the right iliac fossa. On emergency following a blow to his head which left
examination you note guarding and rebound him unconscious. In the department, he regains
tenderness localized to an area approximately consciousness but later deteriorates dramatically.

87 
Single best answer (SBA) questions

An urgent CT head scan is ordered which confirms d. Flail chest


the diagnosis. What is the most likely diagnosis? e. Cardiac tamponade
a. Extradural haematoma
b. Subdural haematoma 13. A 32-year-old man presents with severe abdominal
c. Contusion pain radiating to his back. On examination you note
d. Intracerebral haemorrhage notable tenderness and guarding in the epigastric
e. Axonal injury region. In addition, you note evidence of bruising
around the umbilicus. What is the term most likely
9. A 42-year-old man presents with generalized used to describe this finding?
abdominal pain and vomiting. He comments that he a. Grey Turner sign
has not opened his bowels for the past 2 days. On b. Cullen sign
examination you note severe abdominal distension c. McBurney point
and tinkling bowel sounds. An abdominal X-ray d. Rovsing sign
is ordered which reveals distended loops of small e. Murphy sign
bowel. What is the next most appropriate step in
management? 14. A 75-year-old chronic alcoholic presents to accident
a. Nasogastric decompression and normal saline and emergency following a fall. On examination
intravenously (IV) she appears drowsy and her Glasgow Coma Scale
b. Surgery (GCS) drops from 12 to 8. You order an urgent CT
c. Nasogastric decompression head scan. What is the most likely diagnosis?
d. Nasogastric decompression and dextrose IV a. Extradural haematoma
e. Normal saline IV b. Subdural haematoma
c. Contusion
10. A 67-year-old woman presents to the accident d. Intracerebral haemorrhage
and emergency department after collapsing in a e. Axonal injury
shopping centre. She is pale, sweaty and complains
of severe back pain. On examination she has a 15. A 42-year-old man presents with severe colicky pain
heart rate of 150 beats/min, blood pressure of in his right loin radiating to his groin. On examination
80/40 mmHg and a tender, expansile, pulsatile mass you note tenderness in the right loin. A urine dipstick
in her epigastrium. What is the most appropriate is performed which is positive for blood. What is the
management for this woman? next most appropriate initial investigation?
a. Abdominal CT scan a. Abdominal ultrasound
b. Abdominal ultrasound scan b. Abdominal X-ray
c. Erect chest X-ray and plain abdominal films c. Abdominal CT scan
d. Serum amylase d. 24-Hour urine collection
e. Take patient to theatre e. IV urogram

11. A 35-year-old chronic alcohol abuser complains 16. A middle-aged obese gentleman presents with
of upper abdominal pain radiating to his back. On severe right upper quadrant abdominal pain after
examination you note tenderness and guarding in a Chinese take away. On examination you note
the epigastrium. Which investigation is most likely to tenderness in his epigastrium and right upper
lead to a definitive diagnosis? quadrant. Which investigation is most likely to
a. Abdominal CT scan establish the actual diagnosis?
b. Abdominal X-ray a. White cell count
c. Chest X-ray b. Abdominal ultrasound
d. Abdominal ultrasound c. Liver function tests
e. Liver function tests d. Abdominal X-ray
e. Serum amylase
12. A 32-year-old woman is admitted to accident and
emergency following a stabbing to her left chest. On 17. A 52-year-old building contractor is brought to
examination she is notably breathless. Her left chest accident and emergency having fallen from a height
is dull to percussion and breath sounds are absent. onto his chest. He is notably breathless and in
What is the most likely diagnosis? severe pain. On examination you note crepitus on
a. Tension pneumothorax chest palpation and paradoxical movement of the
b. Open pneumothorax lower segment of his right chest. What is the most
c. Massive haemothorax likely diagnosis?

88
Trauma and surgical emergencies 9

a. Tension pneumothorax a. Pancreatitis


b. Open pneumothorax b. Large bowel obstruction
c. Massive haemothorax c. Large bowel perforation
d. Flail chest d. Cholecystitis
e. Cardiac tamponade e. Hepatitis

18. A 65-year-old hypertensive patient presents to 22. A 35-year-old woman presents with lower left-sided
accident and emergency with sudden-onset abdominal discomfort that is severe in nature. She
central abdominal pain radiating to his back. On denies any gastrointestinal symptoms. Abdominal
examination you note evidence of generalized severe examination confirms evidence of lower abdominal
abdominal tenderness and a pulsatile mass. Routine tenderness. Routine blood investigations reveal
observations reveal a pulse rate of 129 beats/min a normal white cell count and C-reactive protein.
and a blood pressure of 75/40 mmHg. What is the Serum beta human chorionic gonadotropin is
next most appropriate step in management? negative. What is the most likely diagnosis?
a. IV access and fluid resuscitation with Gelofusine a. Diverticulitis
b. IV access and blood resuscitation b. Ovarian cyst
c. Abdominal CT scan c. Uterine cyst
d. Abdominal ultrasound d. Sigmoid volvulus
e. Immediate transfer for surgery while e. Ectopic pregnancy
simultaneously performing resuscitation
23. A 15-year-old boy presents with sudden-onset
19. A 55-year-old woman presents with pain and severe pain in his left scrotum and vomiting. On
numbness in her right leg. Her past medical history examination you note marked tenderness on
includes atrial fibrillation. On examination of her right palpation. Elevation of the scrotum did not improve
leg you note absent pulses, pallor and a reduction the pain. A urine dipstick proves normal. What is the
in temperature as compared with her left limb. most likely diagnosis?
Routine observations reveal an irregular pulse of a. Epididymitis
125 beats/min, a blood pressure of 105/65 mmHg b. Hydrocele
and an oxygen saturation of 96% on room air. c. Orchitis
Following initial resuscitation, what is the next most d. Appendicitis
appropriate step in management? e. Testicular torsion
a. ECG
b. Digoxin 24. A 25-year-old sexually active male presents with scrotal
c. Embolectomy swelling and pain. On examination you note evidence
d. IV fluids of scrotal erythema. A urine dipstick proves positive for
e. Oxygen leucocytes. What is the most likely diagnosis?
a. Epididymitis
20. An elderly patient presents with left iliac fossa pain b. Hydrocele
and vomiting. On examination you note evidence c. Orchitis
of tenderness in the left iliac fossa and peritonism. d. Appendicitis
Routine blood investigations demonstrate a white e. Testicular torsion
cell count of 22 × 109/L. What is the most likely
diagnosis? 25. A 32-year-old man presents to accident and
a. Diverticulitis emergency following a stabbing to his left chest.
b. Ulcerative colitis On examination he is severely breathless with a
c. Sigmoid volvulus respiratory rate of 30 breaths/min. There is right-
d. Small bowel obstruction sided deviation of his trachea with hyperresonance
e. Large bowel obstruction on percussion and absent breath sounds on the left
side. You insert a large-bore IV cannula through the
21. A middle-aged obese gentleman presents with right second intercostal space midclavicular line. What is
upper quadrant pain. Abdominal examination reveals the most likely diagnosis?
evidence of severe tenderness in the right upper a. Tension pneumothorax
quadrant. Routine observations reveal a temperature b. Open pneumothorax
of 38.5°C. Blood tests confirm a white cell count c. Massive haemothorax
of 16 × 109/L and a C-reactive protein level of d. Flail chest
135 mg/L. What is the most likely diagnosis? e. Cardiac tamponade

89 
Extended-matching questions (EMQs)

e. Thoracic aortic aneurysm


EXTENDED-MATCHING f. Dissecting aortic aneurysm
QUESTIONS (EMQs) g. Peptic ulcer disease
h. Bowel obstruction
Trauma i. Biliary colic
Each answer can be used once, more than once or not j. Perforated viscus
at all.
a. Airway obstruction For each scenario below, choose the most likely
b. Cardiac tamponade corresponding option from the list given above.
c. Flail chest 1. Associated with colicky abdominal pain, vomiting
d. Haemothorax and absent bowel movement.
e. Myocardial infarction 2. A patient presenting with generalized abdominal
f. Open pneumothorax tenderness with evidence of free gas on an erect
g. Pulmonary contusion chest X-ray.
h. Ruptured diaphragm 3. Associated with colicky abdominal pain in the left
i. Ruptured oesophagus flank with radiation to the groin. Abdominal signs
j. Ruptured spleen are usually minimal. The pain may be so severe as
k. Tension pneumothorax to interrupt one’s breath.
l. Traumatic aortic rupture 4. A medical student presents with severe
upper abdominal tenderness and a serum
For each scenario below, choose the most likely amylase greater than four times the
corresponding option from the list given above. normal limit.
1. An unconscious 30-year-old male with weak pulse 5. Associated with sudden-onset severe tearing
who previously complained of left-sided pleuritic chest pain radiating to the back. A chest
chest pain. Examination reveals the left chest to X-ray demonstrates evidence of a widened
be hyperresonant to percussion, with decreased mediastinum.
breath sounds.
2. A 20-year-old girl is brought in with a stab wound Glasgow Coma Scale
to the right side of her chest. She is in respiratory Each answer can be used once, more than once or not
distress and air can be heard moving through the at all.
defect in the chest wall. a. 6
3. A 60-year-old man is hit by a lorry while out b. 7
walking his dog. He has injured his chest and is c. 8
in obvious respiratory distress. Initial observations d. 9
show that he has a pulse of 120 beats/min, a e. 10
blood pressure of 120/60 mmHg and a respiratory f. 11
rate of 45 breaths/min. Examination reveals g. 12
paradoxical chest wall movement. h. 13
4. A 3-year-old girl who has been eating peanuts and i. 14
who now has a cough and stridor. j. 15
5. A 38-year-old man is brought into accident
and emergency after a high-speed road traffic For each scenario below, choose the most likely
accident. He has chest pain and a blood pressure corresponding option from the list given above.
of 70/50 mmHg and a widened mediastinum on
1. Obeys commands, with normal speech and
his chest X-ray.
spontaneous eye movement.
2. Flexes to pain, with incomprehensible sounds and
Acute abdomen 1 no eye movement.
Each answer can be used once, more than once or not 3. Withdraws to pain with confused speech and eyes
at all. that open to pain.
a. Appendicitis 4. No motor response with normal speech and
b. Pancreatitis spontaneous eye movement.
c. Renal colic 5. Inappropriate speech associated with obeying
d. Abdominal aortic aneurysm (AAA) commands and eyes opening on command.

90
Trauma and surgical emergencies 9

Acute abdomen 2 nausea and vomiting and is clearly dehydrated. He


Each answer can be used once, more than once or not returned from holiday in Ayia Napa earlier today.
at all. 2. A 33-year-old woman is admitted with a 24-
hour history of nausea, vomiting and absolute
a. Acute pancreatitis
constipation. Her abdomen is tympanic with
b. Acute pyelonephritis
increased high-pitched bowel sounds.
c. Biliary peritonitis
3. A 57-year-old man is admitted with onset of
d. Mesenteric adenitis
abdominal pain initially in the left upper quadrant.
e. Perforated appendix
He has been self-medicating with ibuprofen
f. Perforated peptic ulcer
after spraining his left ankle playing squash. On
g. Perforated sigmoid diverticulus
examination the abdomen is rigid with no bowel
h. Ruptured AAA
sounds.
i. Small bowel obstruction
4. A 24-year-old woman is admitted with fever, rigors
j. Superior mesenteric artery occlusion
and severe back pain radiating to her left groin.
5. A 10-year-old boy is admitted with right iliac fossa
For each scenario below, choose the most likely
pain of several hours’ duration. Initially, he has
corresponding option from the list given above.
guarding and rebound tenderness. Pressing in
1. A 25-year-old man is admitted with severe upper the right iliac fossa also elicits the same pain. He
abdominal pain radiating to the back. He has rapidly progresses to having a tense abdomen.

91 
This page intentionally left blank
Perioperative care 10
4. You are a house officer in general surgery and are
SINGLE BEST ANSWER (SBA) clerking in a 65-year-old man who is being admitted
QUESTIONS electively for an inguinal hernia repair. His past medical
history includes chronic obstructive pulmonary
1. A 54-year-old woman is 2 days post an open disease. He states that he had a chest X-ray around
appendicectomy for perforated appendicitis. She 6 weeks ago. Which investigation is most highly
is reviewed on the ward round and comments recommended prior to his surgery?
that she is still experiencing abdominal soreness. a. Full blood count
On examination she is notably tender over the b. ECG
lower abdomen. The nurses inform you that she c. Chest X-ray
has a temperature of 38.9°C, a blood pressure of d. Urea and electrolytes
100/54 mmHg and a pulse rate of 95 beats/min. Her e. Clotting screen
blood results reveal a white cell count of 21 × 109/L
and C-reactive protein (CRP) of 154 mg/L. What is the 5. A 32-year-old man is complaining of increasing pain
most likely diagnosis? 2 days post inguinal hernia repair. He is currently
a. Cardiogenic shock prescribed paracetamol 1 g four times a day. What is
b. Hypovolaemic shock the next most appropriate step in management?
c. Septic shock a. Codeine
d. Obstructive shock b. Tramadol
e. Anaphylactic shock c. Morphine
d. Co-codamol
2. A 55-year-old woman with severe intermittent e. Diclofenac
claudication is admitted for an elective arteriogram
and possible angioplasty of her right leg. She has a
past medical history of rheumatoid arthritis and mildly
impaired renal function. Current medication includes EXTENDED-MATCHING
penicillamine. Admission blood investigations reveal QUESTIONS (EMQs)
a urea of 12.8 mmol/L and creatinine of 163 μmol/L
with previous results being in a similar range. However,
following the arteriogram, her urea and creatinine
Postoperative complications
Each answer can be used once, more than once or not
levels have increased to 32.8 mmol/L and 363 μmol/L,
at all.
respectively. What is the most likely cause of her acute
renal failure? a. Atelectasis
a. Age b. Anastomotic dehiscence
b. Minimal fluid intake c. Constipation
c. Radiological contrast nephropathy d. Deep vein thrombosis
d. Penicillamine e. Gravitational oedema
e. Past medical history of renal failure f. Haematoma
g. Pneumonia
3. You are a house officer in breast surgery and are h. Pseudo-obstruction
clerking in a 45-year-old woman who is being admitted i. Urinary retention
for excision of a benign breast lump. She has no known j. Wound infection
past medical or surgical history. Which investigation is
most highly recommended prior to her surgery? For each scenario below, choose the most likely
a. Full blood count corresponding option from the list given above.
b. Electrocardiogram (ECG) 1. A 45-year-old man complains of increasing pain
c. Chest X-ray in the groin following a right inguinal hernia repair
d. Urea and electrolytes 5 days previously. On examination, the wound is
e. None of the above red, swollen and tender.

93 
Extended-matching questions (EMQs)

2. A 70-year-old man who had a hip replacement c. Carotid Doppler scan


5 days previously complains of generalized d. Chest X-ray
abdominal discomfort. He has vomited four times, e. Clotting screen
his bowels have not functioned for 5 days and he f. Coronary angiogram
feels thirsty and has not passed urine for 8 hours. g. Echocardiogram
On examination, his abdomen is grossly distended h. ECG
and tympanitic, but there is no tenderness and i. Liver function tests
he has decreased bowel sounds. The rectum is j. Spirometry
empty. k. Troponin
3. An 83-year-old woman had an emergency
laparotomy for a bleeding duodenal ulcer 18 hours For each scenario below, choose the most likely
previously. She has become borderline pyrexial, corresponding option from the list given above.
and pulse oximetry shows saturation of 90% on 1. A 72-year-old woman has been admitted for a
2 L of oxygen. Her blood pressure is normal, but mastectomy. She gives a history of well-controlled
her pulse rate is 110 beats/min. Her respiratory hypertension for which she is on medication. On
rate is 20 breaths/min and she has decreased examination she looks well, and the examination
breath sounds at the right base. of the cardiovascular system reveals an ejection
4. A 72-year-old woman who has had an anterior systolic murmur in the aortic area.
resection of the rectum suddenly develops 2. A 50-year-old woman is admitted for a routine
severe abdominal pain 4 days postoperatively. repair of a femoral hernia. She has chronic
On examination, she is unwell, with a pyrexia asthma for which she has been taking medication
of 38°C, blood pressure of 100/60 mmHg, on a regular basis, but it is not always well
abdominal distension, a discharging wound and controlled.
peritonism. 3. A 76-year-old patient is admitted as an
5. A 62-year-old woman develops discomfort of the emergency with symptoms of obstructive
right lower leg 7 days after a hysterectomy for jaundice. An ultrasound shows the presence
carcinoma of the uterus. On examination, she has of a gallstone in the common bile duct. She is
a pyrexia of 37.5°C and bilateral ankle oedema about to undergo an endoscopic retrograde
(right > left). cholangiopancreatography.
4. A 66-year-old man is admitted for a coronary
Preoperative investigations artery bypass operation. During routine clerking,
Each answer can be used once, more than once or not he describes an episode when he temporarily
at all. lost the vision in his right eye and some
a. Arterial blood gases weakness of his arm, which has now recovered
b. Blood glucose completely.

94
Gastrointestinal surgery 11
5. A 46-year-old female presents to accident and
SINGLE BEST ANSWER (SBA) emergency (A&E) with severe upper abdominal
QUESTIONS pain. On examination she is tender in the right
upper quadrant and her sclera looks jaundiced. Her
1. A term baby is born with a vitelline duct still present. temperature is 38.2°C. Her blood samples showed a
His baby check was normal and he was discharged raised alkaline phosphatase and bilirubin. What is the
2 days after birth. He remains well until he attends most likely diagnosis?
university at 19 years of age, when he develops a. Liver abscess
colicky abdominal pain. What is the likely cause? b. Gallstone ileus
a. Appendicitis c. Appendicitis
b. Meckel diverticulum d. Viral hepatitis
c. Exomphalos e. Cholangitis
d. Gastroschisis
e. Atresia 6. A 68-year-old male was referred for an urgent clinic
appointment after a change in bowel habit. Colonoscopy
2. A 78-year-old female is admitted to the surgical and biopsy confirmed colorectal adenocarcinoma, which
assessment unit with severe, sudden-onset, was resected. Histology showed spread to the muscle
abdominal pain for the past 3 days, along with layer and bowel wall without any involvement of lymph
bloody diarrhoea for the past 24 hours. She has a nodes. What Dukes staging is this?
medical history of atrial fibrillation. Blood samples are a. Dukes A
unremarkable apart from an elevated lactate. What is b. Dukes B
the most likely diagnosis? c. Dukes C1
a. Ulcerative colitis d. Dukes C2
b. Colorectal carcinoma e. Dukes D
c. Angiodysplasia
d. Crohn disease 7. A 70-year-old male presented with central intermittent
e. Ischaemic bowel abdominal pain for the past 3 weeks. On examination
he had mild epigastric tenderness without any signs
3. A 10-month-old baby was admitted to the children’s of peritonitis. He is a known heavy smoker and
ward with vomiting. The parents said he seemed mentioned that he had recently had an ultrasound
generally unwell, crying more than usual and drawing scan done on his neck as an investigation for dizzy
his knees up to his chest. When the nurses changed spells. What is the most likely cause?
his nappy, they noticed that his stools were loose, and a. Mesenteric ischaemia
looked redder than normal. What is the diagnosis? b. Chronic pancreatitis
a. Pyloric stenosis c. Duodenal ulcer
b. Gastroenteritis d. Gastric carcinoma
c. Intussusception e. Appendicitis
d. Gastro-oesophageal reflux disease (GORD)
e. Duodenal atresia 8. A 5-week-old baby was brought into A&E vomiting.
The mother reported a 3-day history of vomiting large
4. A 21-year-old female presents to the hospital with amounts following every feed. The baby was born at
sudden-onset acute abdominal pain. On examination 39 weeks without any complications and had been
she is tender in the right iliac fossa with guarding. She putting weight on well postbirth. On examination the
has tachycardia and is hypotensive. What would be doctor could palpate a sausage-shaped mass in the
your initial management? right upper quadrant. What is the most likely diagnosis?
a. Abdominal X-ray a. Pyloric stenosis
b. Pregnancy test b. Neonatal jaundice
c. Intravenous (IV) fluids c. Duodenal atresia
d. Urine dipstick d. Diaphragmatic hernia
e. Abdominal ultrasound e. Acute gastritis

95 
Single best answer (SBA) questions

9. A 67-year-old male noticed a swelling in the right c. Surgery and adjuvant chemotherapy
groin region that appears to ‘come and go’. On d. Radiotherapy
examination there is a reducible 2 × 2 cm mass that e. Palliative treatment
appears to lie inferior to the inferior gastric vessels.
The hernia reappears on coughing, even when 14. A 24-year-old obese woman presents to A&E with
pressing. What hernia is this most likely to be? sudden-onset severe right upper quadrant pain. On
a. Incisional hernia examination you note notable tenderness in the right
b. Direct inguinal hernia upper quadrant with guarding. Which investigation is
c. Indirect inguinal hernia most likely to lead to a diagnosis?
d. Femoral hernia a. WCC
e. Umbilical hernia b. CRP
c. Liver function tests
10. A 46-year-old female presented to the general d. Abdominal X-ray
practitioner (GP) with a change in bowel habit for e. Abdominal ultrasound
the past 2 months. She is having looser stools than
normal with blood mixed in with the stools. There 15. A 25-year-old weightlifter presents with a lump near
is no history of any abdominal pain. What would be the right side of his groin. On examination you note
your first-choice investigation? the lump reduces upwards and laterally and moves
a. Abdominal X-ray downwards and medially on release. In addition,
b. Faecal occult blood you note the lump is controlled by pressure over the
c. Abdominal computed tomography (CT) deep ring. What is the most likely diagnosis?
d. Ultrasound abdomen a. Indirect inguinal hernia
e. Colonoscopy b. Direct inguinal hernia
c. Femoral hernia
11. A 58-year-old male, who has a long medical history d. Richter hernia
of achalasia, presents with symptoms of weight e. Sliding hernia
loss and worsening dysphagia. He is referred for
an urgent oesophagogastroduodenoscopy, which 16. A 53-year-old man presents with a lump near
shows a lesion in the upper third of the oesophagus his groin. On examination you note it reduces
suspicious for oesophageal cancer. What is the likely immediately on lying down and reaches full size
histological type to be shown on biopsy? immediately on standing. You ask the patient to
a. Adenocarcinoma cough and notice it protrudes straight. It is not
b. Lymphoma controlled by pressure over the deep ring. What is
c. Small cell carcinoma the most likely diagnosis?
d. Squamous cell carcinoma a. Indirect inguinal hernia
e. Melanoma b. Direct inguinal hernia
c. Femoral hernia
12. A 65-year-old woman presents with sudden-onset d. Richter hernia
left iliac fossa pain, fever and vomiting. Which e. Sliding hernia
investigation is most likely to lead to a diagnosis?
a. Colonoscopy 17. A 35-year-old woman presents with a lump in her
b. Abdominal ultrasound groin. On examination you note the lump is below
c. Abdominal and pelvic CT scan and lateral to the pubic tubercle. What is the most
d. Abdominal X-ray likely diagnosis?
e. White cell count (WCC) and C-reactive a. Indirect inguinal hernia
protein (CRP) b. Direct inguinal hernia
c. Femoral hernia
13. A 55-year-old man presents with rectal bleeding and d. Richter hernia
abdominal discomfort. A colonoscopy is performed e. Sliding hernia
which reveals the presence of a tumour in the
sigmoid colon. Further assessment and staging 18. A 25-year-old woman complains of pain at the navel.
confirm a colonic adenocarcinoma with probable On examination you note an obvious swelling at
local lymph node involvement. What is the next most the site of the umbilicus, which she says has been
likely step in management? present since birth. What is the most likely diagnosis?
a. Surgery a. Umbilical hernia
b. Chemotherapy b. Femoral hernia

96
Gastrointestinal surgery 11

c. Richter hernia of his abdomen demonstrates evidence of sigmoid


d. Sliding hernia volvulus. What is the next most appropriate step in
e. Paraumbilical hernia management to achieve definitive resolution?
a. Emergency laparotomy
19. A 43-year-old man presents with epigastric pain b. Flexible sigmoidoscopic decompression with a
and associated nausea. On examination you note a flatus tube
palpable swelling in the midline which is tender and c. IV fluids
irreducible. What is the most likely diagnosis? d. Nasogastric tube insertion
a. Epigastric hernia e. IV fluids and insertion of a nasogastric tube
b. Umbilical hernia
c. Paraumbilical hernia 24. A middle-aged woman presents with bleeding
d. Richter hernia when opening her bowels. She comments that the
e. Sliding hernia blood is bright red in nature and can be seen on
the toilet paper and in the pan following defecation.
20. A 42-year-old woman presents with a lump at the A proctoscopy is performed which helps confirm
lateral edge of the rectus sheath. What is the most the diagnosis. Which management plan would you
likely diagnosis? suggest first?
a. Epigastric hernia a. Review in 2 months
b. Umbilical hernia b. Barron banding
c. Spigelian hernia c. Surgery
d. Paraumbilical hernia d. Cryotherapy
e. Richter hernia e. Injection sclerotherapy

21. A 65-year-old woman presents with a 2-day history 25. A 55-year-old man presents to his GP complaining
of abdominal discomfort and vomiting. In addition, of perianal discomfort. On examination you note a
she complains of right-sided hip pain. An abdominal discrete swelling in the perianal margin. It is painful
X-ray is ordered which confirms evidence of small on palpation and has the appearance of a small
bowel obstruction. She undergoes an emergency blackcurrant. What is the most likely diagnosis?
laparotomy. What is the most likely diagnosis? a. Perianal haematoma
a. Epigastric hernia b. Haemorrhoid
b. Obturator hernia c. Thrombosed haemorrhoid
c. Umbilical hernia d. Tumour
d. Paraumbilical hernia e. Hidradenitis suppurativa
e. Richter hernia
26. A middle-aged man presents with pain on
22. An elderly man presents with left-sided abdominal defecation. He comments that the pain occurs
pain and dark red rectal bleeding. His medical during defecation and can continue for some time
history includes ischaemic heart disease. He is afterwards. On examination you note the presence
known to smoke 40 cigarettes per day over a of a midline fissure. He has had this symptom for the
30-year period. On examination you note obvious last 12 weeks. What is the next most appropriate
distension of his abdomen. Routine observations step in management?
demonstrate a blood pressure of 105/65 mmHg and a. Lateral internal sphincterotomy
a pulse rate of 130 beats/min. He is apyrexial. What b. Botulinum A toxin injection
is the most likely diagnosis? c. Oral diltiazem
a. Lymphocytic colitis d. Topical glyceryl trinitrate ointment
b. Ulcerative colitis e. Surgical sphincter dilatation
c. Ischaemic colitis
d. Diverticulitis 27. A 40-year-old woman is suspected of suffering from
e. Microscopic colitis faecal incontinence. Which investigation is most
likely to lead to a diagnosis?
23. A 65-year-old man presents with sudden-onset a. Abdominal ultrasound scan
abdominal pain. On further questioning, he b. Barium enema
comments that he has not opened his bowels for c. Anorectal physiology studies
3 days and has been vomiting intermittently. On d. Abdominal magnetic resonance imaging (MRI)
examination his abdomen is grossly distended and scan
he is notably tender throughout. A plain radiograph e. Colonoscopy

97 
Single best answer (SBA) questions

28. A middle-aged man presents with episodes of a. Abdominal CT scan


recurrent perianal pain and swelling. He reports b. Abdominal X-ray
ongoing purulent discharge between such episodes. c. Abdominal ultrasound scan
Inspection of the perineum reveals the presence d. Abdominal MRI scan
of circular granulation tissue which exudes pus on e. Endoscopic retrograde
digital compression from within the rectum. What is cholangiopancreatography (ERCP)
the most likely diagnosis?
a. Abscess 33. A 70-year-old woman presents with a 1-week history
b. Hidradenitis suppurativa of abdominal discomfort and weight loss. She has
c. Pilonidal cyst a known history of hepatitis C. On examination
d. Fistula in ano you note evidence of moderate hepatomegaly. An
e. Pilonidal sinus ultrasound scan demonstrates an isolated lesion in
her liver. Which additional investigation is most likely
29. A 35-year-old man presents to A&E at 3 am following to confirm a diagnosis?
an episode of severe vomiting. He complains of a. Serum bilirubin
sudden-onset chest and neck pain. On examination b. Serum alkaline phosphatase
you note the presence of crepitus in his suprasternal c. International normalized ratio
notch. What is the first initial investigation in the d. Prothrombin time
emergency department? e. Serum alpha-fetoprotein
a. Endoscopy
b. Anteroposterior and lateral chest X-ray 34. A 43-year-old man presents with right upper
c. Chest CT scan quadrant pain. On examination you note tenderness
d. Gastrograffin swallow and guarding in the right upper quadrant. Routine
e. Barium swallow observations reveal a temperature of 38.5°C.
Routine blood investigations demonstrate a WCC
30. A 10-year-old boy presents to A&E following ingestion of 17 × 109/L and CRP of 98 mg/L. What additional
of household bleach. He complains of severe pain sign is this patient most likely to demonstrate?
in his mouth and stomach. Routine observations a. Cullen sign
reveal a temperature of 38.5°C, a blood pressure of b. Grey Turner sign
105/65 mmHg and a pulse rate of 135 beats/min. c. Rovsing sign
What is the next most appropriate initial investigation? d. Boas sign
a. Endoscopy e. Trousseau sign
b. Chest CT scan
c. Chest X-ray 35. An 80-year-old woman presents with abdominal
d. Barium swallow pain and weight loss. An abdominal CT scan
e. Oesophageal manometry is performed which confirms the diagnosis of
pancreatic cancer with distant metastases. What is
31. A 45-year-old man presents complaining of difficulty the next most likely step in management?
in swallowing liquid and solid food. A barium swallow a. Surgery
is performed which demonstrates a narrowed portion b. Palliative therapy
within the lower end of his oesophagus. An upper c. Radiotherapy
gastrointestinal (GI) endoscopy excludes the presence d. Chemotherapy
of organic pathology. Which therapeutic option is e. Surgery and chemotherapy
most likely to relieve his symptoms in the long term?
a. Antispasmodics 36. A 50-year-old man presents with epigastric pain,
b. Balloon dilatation vomiting and weight loss. He is a heavy drinker and
c. Heller myotomy admits to consuming half a bottle of vodka each day
d. Botulinum toxin over a 10-year period. On examination you note a
e. Proton pump inhibitors purple-coloured periorbital skin rash and a palpable
lymph node in the left supraclavicular fossa. What is
32. A 48-year-old woman presents with right upper the next most appropriate investigation?
quadrant pain. On examination you note a slight a. Abdominal CT scan
yellow discolouration of her skin and sclerae. b. Abdominal ultrasound scan
Abdominal examination reveals tenderness across c. Endoscopy and biopsy
the upper abdomen. What is the next most d. Laparoscopy
appropriate initial investigation? e. Abdominal X-ray

98
Gastrointestinal surgery 11

37. An elderly man presents with rectal bleeding c. Paraumbilical


and weight loss. A colonoscopy is performed d. Sciatic
which demonstrates a mass in his sigmoid e. Cooper
colon. Biopsies are taken and histology confirms f. Incisional
the presence of a cancer that extends into the g. Richter
muscularis mucosa, with no involvement of h. Littre
regional lymph nodes or distant metastases. What i. Spigelian
is the most likely histological diagnosis? j. Diaphragmatic
a. Dukes stage A
b. Dukes stage B For each scenario below, choose the most likely
c. Dukes stage C corresponding option from the list given above.
d. Dukes stage D 1. Associated with abdominal pain, notable
e. Dukes stage E distension and vomiting. It may result in
perforation of the bowel.
2. Seen commonly in newborn babies where, typically,
EXTENDED-MATCHING the left side of the chest is occupied by the bowel.
3. A hernia occurring through a part of the
QUESTIONS (EMQs) abdominal wall between the lateral border of the
rectus abdominis and linea semilunaris, typically
Gastrointestinal bleeding diagnosed by ultrasound.
Each answer can be used once, more than once or not 4. A combined indirect and direct hernia whereby the
at all. hernial sac protrudes on either side of the inferior
a. Anal fissure epigastric vessels.
b. Colonic carcinoma 5. A lump located near the groin which protrudes
c. Crohn disease straight on coughing and is not controlled by
d. Diverticular disease pressure over the deep ring.
e. Duodenal ulcer
f. Gastric carcinoma Upper gastrointestinal disorders
g. Haemorrhoids Each answer can be used once, more than once or not at all.
h. Laryngeal carcinoma
a. Achalasia
i. Mallory–Weiss tear
b. Diffuse oesophageal spasm
j. Oesophageal varices
c. Pharyngeal pouch
d. Zollinger–Ellison syndrome
For each scenario described below, choose the SINGLE
e. Oesophagitis
most likely diagnosis from the above list of options. Each
f. Duodenal ulcer
option may be used once, more than once or not at all.
g. Adenocarcinoma
1. A 60-year-old man presents with altered bowel h. Squamous cell carcinoma
habit, crampy abdominal pain and blood mixed in i. Plummer–Vinson syndrome
with his stools. j. Atrophic gastritis
2. A 40-year-old woman presents with a long history
of indigestion, worse recently, black tarry stools For each scenario below, choose the most likely
and feeling light-headed. corresponding option from the list given above.
3. A 32-year-old woman presents to her GP
1. A condition known to cause severe
following childbirth, worried about the fresh,
gastroduodenal ulceration due to excessive
painless red blood on the toilet paper.
gastrin secretion.
4. A 22-year-old medical student presents to A&E
2. A tumour of the lower third of the oesophagus
with haematemesis and chest pain following
thought to be the result of longstanding reflux
celebrations on passing his finals.
disease.
5. A 17-year-old woman with cystic fibrosis presents
3. A motility disorder of the oesophagus
to A&E vomiting profuse volumes of fresh blood.
characterized by a ‘rat tail’ segment at the lower
oesophageal sphincter.
Hernia 4. Associated with severe epigastric discomfort
Each answer can be used once, more than once or not at all. particularly at night and relieved by food.
a. Inguinal 5. A condition associated with iron-deficiency
b. Pantaloon anaemia and swallowing difficulties.

99 
Extended-matching questions (EMQs)

Lower gastrointestinal disorders 2. A tumour presenting typically with painless


Each answer can be used once, more than once or not jaundice, known to result commonly from primary
at all. sclerosing cholangitis.
3. A patient presenting with tenderness and guarding
a. Perianal abscess
in the right upper quadrant with evidence of
b. Intussusception
hyperaesthesia below the right scapula and pyrexia.
c. Perianal haematoma
4. A patient presenting with abdominal pain, fever
d. Haemorrhoids
and jaundice as a result of Escherichia coli
e. Crohn disease
infection.
f. Angiodysplasia
5. A female patient presenting with anorexia, ascites
g. Collagenous colitis
and abdominal discomfort with a background
h. Ulcerative colitis
history of cirrhosis and long-term use of the oral
i. Anal fistula
contraceptive pill.
j. Ischaemic colitis

For each scenario below, choose the most likely Abdominal pain
corresponding option from the list given above. Each answer can be used once, more than once or not
1. Associated with vomiting and the passage of at all.
redcurrant-like stool in children. a. Acute appendicitis
2. A bluish tender perianal swelling often resulting b. Acute diverticulitis
from constipation and straining. c. Colorectal cancer
3. Typically affects the right colon and associated d. Crohn disease
with severe rectal bleeding. Colonoscopy may e. Familial adenomatous polyposis
reveal the presence of dilated tortuous vessels f. Perforated duodenal ulcer
and ‘cherry red’ areas. g. Renal colic
4. A pan-intestinal inflammatory disorder associated h. Strangulated inguinal hernia
with deep ulcers in the mucosa to give a i. Ulcerative colitis
cobblestone-like appearance. j. Testicular torsion
5. Associated with left-sided abdominal pain and
dark red rectal bleeding. Examination findings For each scenario below, choose the most likely
may demonstrate evidence of hypotension and corresponding option from the list given above.
abdominal distension. A barium enema may 1. A 19-year-old male who was previously fit and
demonstrate thickening and blunting of the well presents with generalized abdominal pain
mucosal folds of the bowel. for 3 days which is worsening, weight loss,
diarrhoea and dysuria. He is most tender in the
right iliac fossa. He has no appetite and has been
Hepatobiliary disorders
feverish.
Each answer can be used once, more than once or not
2. A 35-year-old man presents with 5 months
at all.
of colicky abdominal pain, weight loss and
a. Cholangitis bloody diarrhoea with mucus. He has also been
b. Hepatic haemangioma suffering from lower back pain for the last few
c. Focal nodular hyperplasia of the liver months.
d. Cholangiocarcinoma 3. A 37-year-old man presents with acute sudden
e. Hepatoma right testicular pain which radiates to the right
f. Cholecystitis groin, and has blood +++ in his urine.
g. Biliary colic 4. A 35-year-old man presents with red blood
h. Acalculous cholecystitis mixed in his stool and weight loss over the last
i. Pancreatitis 6 months. He had a colonoscopy 5 years ago, but
j. Hepatic abscess is an infrequent attender to his GP. His father died
of colorectal cancer aged 46.
For each scenario below, choose the most likely 5. A 60-year-old woman presents with a 2-day
corresponding option from the list given above. history of left iliac fossa pain and a significant (PR)
1. A patient presenting with recurrent pain in the right bleed of bright red blood which fills the pan. She
upper quadrant radiating to the tip of the scapula. has had alternating diarrhoea and constipation
There is no associated pyrexia. over the last year.

100
Gastrointestinal surgery 11

Hernias e. Biliary colic


Each answer can be used once, more than once or not f. Large bowel obstruction
at all. g. Perforated peptic ulcer
h. Perforated sigmoid diverticular disease
a. Congenital diaphragmatic hernia
i. Small bowel obstruction
b. Epigastric hernia
j. Urinary tract infection
c. Femoral hernia
d. Incarcerated hernia
For each scenario below, choose the most likely
e. Incisional hernia
corresponding option from the list given above.
f. Inguinal hernia
g. Obturator hernia 1. A 69-year-old woman presents with a 3-day
h. Rolling hiatus hernia history of constipation and constant pain in the
i. Sliding hiatus hernia left iliac fossa. The pain has suddenly become
j. Spigelian hernia much worse and she has collapsed and been
k. Umbilical hernia admitted to casualty. On examination she has a
tachycardia and is hypotensive. There is severe
For each scenario below, choose the most likely lower abdominal pain with guarding throughout
corresponding option from the list given above. the mid- and lower abdomen.
2. A 50-year-old obese woman presents with
1. A 64-year-old labourer presents to his GP with
epigastric pain. On examination, her temperature
increasing pain in his groin. He has had a lump for
is 38.5°C. She is tender in the upper abdomen
several years which usually disappears on lying. It
and Murphy sign is positive.
no longer does so. On examination you can feel a
3. A 22-year-old woman presents with pain in the
mass in the right groin arising above the inguinal
right iliac fossa. The patient is anorexic, has not
ligament. It is irreducible but bowel sounds are
vomited, but had some dysuria and frequency.
present.
Her temperature is 37.5°C. The patient is flushed
2. A 57-year-old woman is referred to the
and has localized guarding in the right iliac fossa
surgeon who performed her laparoscopic
and suprapubic region.
cholecystectomy 2 years previously. She has
4. A 72-year-old man presents with increasing
developed a bulge, which on examination is in
constipation. He has colicky lower abdominal
the midclavicular line in the right upper quadrant
pain and has not passed flatus for 24 hours.
of the abdomen. It disappears on lying down and
Examination reveals a distended abdomen which
‘gurgles’.
is tympanitic on percussion.
3. A 92-year-old frail lady is admitted to hospital
5. A 60-year-old man presents with a 48-hour history
with severe abdominal pain, colicky in nature,
of sudden-onset epigastric pain radiating through
with nausea and vomiting. A nasogastric tube
to the back after an alcoholic binge. Examination
is passed, but on abdominal X-ray, no bowel
reveals the patient to be apyrexial, tachycardic
distension is seen. Close inspection of the film
and normotensive. The patient is diffusely tender,
reveals the diagnosis.
with guarding in the epigastrium. An erect chest
4. A 31-year-old man presents to his GP with a mass
X-ray is normal, but the blood gas analysis reveals
in his left groin. It disappears on lying, is above the
hypoxia.
inguinal ligament and reappears on coughing. His
brother had the same problem.
5. A 29-year-old woman presents to her GP with Intestinal obstruction
an abdominal mass following the birth of her Each answer can be used once, more than once or not
first child. During her pregnancy, she had had an at all.
everted umbilicus and this has worsened after a. Adhesions
giving birth. b. Carcinoma caecum
c. Carcinoma rectum
Acute abdominal pain d. Carcinoma sigmoid
Each answer can be used once, more than once or not e. Gallstone ileus
at all. f. Intussusception
a. Acute appendicitis g. Pseudo-obstruction
b. Acute cholecystitis h. Sigmoid volvulus
c. Acute diverticulitis i. Strangulated femoral hernia
d. Acute pancreatitis j. Strangulated inguinal hernia

101 
Extended-matching questions (EMQs)

For each scenario below, choose the most likely mixture of fresh blood and clots. On this occasion,
corresponding option from the list given above. the bleeding has been severe enough to require
1. An 80-year-old man presents with a 4-day history a 4-unit blood transfusion. A barium enema is
of abdominal distension and pain. He is vomiting undertaken and is normal.
faeculent fluid. He has not been feeling well for 3. A 56-year-old man presents with anaemia and
3 months and has lost 2 stone in weight. Clinical weight loss. Examination reveals a mass in the
examination reveals visible peristalsis in the right iliac fossa and hepatomegaly.
midabdomen, distension and a mass in the right 4. A 50-year-old alcoholic presents with melaena.
iliac fossa. On examination the patient is drowsy and
2. A 68-year-old woman presents with colicky hypotensive. Examination of the abdomen shows
abdominal pain and vomiting. Examination shows splenomegaly.
that she is dehydrated, with abdominal distension 5. A 58-year-old man is admitted with acute onset
and a lower midline scar from a perforated appendix. of left iliac fossa pain. Recently, he has noticed
Plain abdominal film shows multiple distended small he has had some vague abdominal pain and felt
bowel loops and the presence of air in the biliary tree. more constipated. On examination he is pyrexial,
3. A 60-year-old man undergoes emergency lumbar pale and has localized peritonism in the left iliac
disc decompression. Postoperatively, he is fossa. His abdomen is distended. Investigations
immobile. He becomes increasingly constipated show a haemoglobin (Hb) of 7 g/dL (hypochromic,
and develops a distended abdomen. Plain microcytic) and a WCC of 18 × 109/L.
abdominal films show a grossly distended colon
down to the pelvic brim. Oesophagogastric disorders
4. A 78-year-old man presents with a 3-day history Each answer can be used once, more than once or not
of vomiting faeculent fluid. He has a grossly at all.
distended abdomen and a palpable mass in the
a. Achalasia
right groin. The mass is firm, slightly tender and
b. Duodenal ulcer disease
lies below and lateral to the pubic tubercle.
c. Gastric cancer
5. An 88-year-old woman with dementia is referred
d. Gastric ulcer disease
by her GP because she seems to be in pain and
e. GORD
unwell. She has a long history of constipation. On
f. Helicobacter atrophic gastritis
examination, she has a grossly distended tympanitic
g. Mallory–Weiss tear
abdomen which is nontender. The plain abdominal
h. Oesophageal adenocarcinoma
X-ray shows a large distended loop of colon.
i. Oesophageal squamous carcinoma
j. Pyloric stenosis
Gastrointestinal bleeding
Each answer can be used once, more than once or not For each scenario below, choose the most likely
at all. corresponding option from the list given above.
a. Angiodysplasia 1. A 45-year-old man presents with severe epigastric
b. Anterior duodenal ulcer pain. The pain is most severe when he is hungry
c. Caecal carcinoma and often wakes him from sleep at night. The
d. Crohn disease pain is relieved by eating and by taking milk and
e. Mallory–Weiss tear snacks. The patient has gained half a stone in
f. Oesophageal varices weight since his symptoms started.
g. Posterior duodenal ulcer 2. A 19-year-old student experiences worsening
h. Rectal carcinoma dysphagia for 3 months. She has lost a stone in
i. Sigmoid carcinoma weight and has had two courses of antibiotics for
j. Sigmoid diverticular disease a persistent chest infection.
3. A 55-year-old vagrant man has had a long history
For each scenario below, choose the most likely of recurrent episodes of epigastric pain. He
corresponding option from the list given above. presents with weight loss and severe vomiting.
1. An 82-year-old woman suffering with osteoarthritis On admission, he is noted to be dehydrated
of the hip is admitted with haematemesis and and abdominal examination demonstrates a
melaena. She has been taking ibuprofen in succussion splash.
increasing amounts to control her hip pain. 4. A 65-year-old man presents with a long history
2. A 77-year-old man presents with his fourth of indigestion self-treated with antacids, but
episode of acute rectal bleeding. The blood is a progressive dysphagia and weight loss for

102
Gastrointestinal surgery 11

3 weeks. On examination, the patient is anaemic g. Mucocoele of the gallbladder


and is markedly cachectic. h. Ovarian carcinoma
5. A 27-year-old man is admitted to the casualty with i. Ovarian cyst
frank haematemesis. He has been on his stag j. Sigmoid carcinoma
weekend and after consuming large amounts of k. Sigmoid volvulus
alcohol, has been vomiting repeatedly, and has l. Uterine fibroids
some epigastric pain.
For each scenario below, choose the most likely
Foregut investigations corresponding option from the list given above.
Each answer can be used once, more than once or not 1. A 69-year-old woman presents to her GP
at all. with a history of fatigue for 3 months, and
a. Abdominal ultrasound 4 days of colicky abdominal pain and
b. Barium meal vomiting. Examination shows that she is pale
c. Barium swallow and dehydrated, with a mass in the right iliac
d. Chest X-ray fossa.
e. CT scan 2. A 72-year-old woman presents with a 3-week
f. ERCP history of increasing discomfort in her lower
g. Endoscopic ultrasonography abdomen. She has not opened her bowels for
h. Gastroscopy 5 days but has passed flatus. On examination,
i. Laparoscopy she looks pale and dehydrated and the
j. Magnetic resonance scanning abdomen is distended and tympanitic. There is
an irregular nontender mass in the left iliac fossa
For each scenario below, choose the most likely region.
corresponding option from the list given above. 3. A 25-year-old woman presents with acute onset
of generalized abdominal pain. She has been
1. A 55-year-old man presents with flatulent
unwell for about 2 months and has lost half a
dyspepsia, fatty food intolerance and right upper
stone in weight. On examination, she looks unwell,
quadrant pain.
has a pyrexia and a tender mass felt in the right
2. A 65-year-old man is diagnosed with oesophageal
iliac fossa with diffuse tenderness in the lower
adenocarcinoma. The surgeon wishes to stage
abdomen.
the local extent of the tumour and to assess
4. A 67-year-old man is seen in the outpatient clinic
whether there is local lymph node spread or
following an episode of severe colicky upper
invasion of the thoracic aorta.
abdominal pain which has settled to leave some
3. A 70-year-old man presents with painless
discomfort. He has a history of ‘indigestion’ after
obstructive jaundice. He has undergone
fatty food and alcohol intake of at least 4 units
assessment by ultrasonography, showing a mass
a day. On examination, he is slightly jaundiced,
in the head of the pancreas. The surgeon wishes
apyrexial and, on abdominal examination,
to stage the tumour.
has a smooth, nontender mass in the right
4. An 80-year-old man presents with intermittent
hypochondrium.
cervical dysphagia and has a soft swelling on the
5. A 45-year-old woman complains to her
left side of his neck. He coughs and splutters after
GP of tiredness and a dragging sensation
eating and often aspirates liquids.
in the lower abdomen. She has noticed
5. A 59-year-old man has carcinoma of the stomach.
that her clothes do not fit as well. On
He is suitable for surgical resection but the
examination, she is anaemic and her
surgeon wants to exclude peritoneal seedlings
abdomen is distended with a mass
before undertaking a laparotomy.
consistent with the size of an 18-week
pregnancy in the suprapubic region.
Abdominal mass
Each answer can be used once, more than once or not
at all. Groin lump
a. Appendix mass Each answer can be used once, more than once or not
b. Caecal carcinoma at all.
c. Crohn disease a. False aneurysm
d. Diverticular abscess b. Femoral artery aneurysm
e. Empyema of the gallbladder c. Femoral hernia
f. Hepatomegaly d. Inguinal hernia

103 
Extended-matching questions (EMQs)

e. Lymphocoele Rectal bleeding


f. Lymphoma Each answer can be used once, more than once or not
g. Malignant lymphadenopathy at all.
h. Psoas abscess
a. Anal carcinoma
i. Obturator hernia
b. Anal fissure
j. Reactive lymphadenopathy
c. Anal skin tags
k. Saphena varix
d. Angiodysplasia
l. Soft tissue sarcoma
e. Crohn disease
f. Haemorrhoids
For each scenario below, choose the most likely g. Meckel diverticulum
corresponding option from the list given above. h. Peptic ulceration
1. An 80-year-old woman presents with a 2-day i. Rectal adenocarcinoma
history of a lump in the left groin. It has been j. Rectal villous adenoma
associated with colicky abdominal pain, k. Sigmoid carcinoma
vomiting and abdominal distension. Examination l. Sigmoid diverticular disease
reveals a red, tender, irreducible lump below m. Ulcerative colitis
and lateral to the pubic tubercle. The abdomen
is distended, with high-pitched ‘tinkling’ bowel For each scenario below, choose the most likely
sounds. corresponding option from the list given above.
2. A 75-year-old lady presents with a 3-month 1. A 60-year-old woman presents with fresh rectal
history of a right groin lump. It becomes painful bleeding, diarrhoea, profuse mucous discharge,
when she walks, but disappears when she rests. tenesmus and lethargy. Investigations show that
Examination reveals a right groin lump with cough she is anaemic (Hb 10 g/dL) and has a potassium
impulse. It reduces to a point above and medial of 2.2 mmol/L.
to the pubic tubercle. She also has long-standing 2. A 50-year-old woman presents with a short
varicose veins. history of painful, fresh rectal bleeding. She has a
3. A 70-year-old woman presents with a pretibial history of cervical intraepithelial neoplasia (CIN).
laceration but also mentions that she has a On examination, she has a single indurated painful
4-week history of a lump in the right groin. It is ulcer in the perianal region.
completely painless but has gradually increased 3. A 5-year-old girl’s mother brings her to the GP
in size during this period. She had a melanoma because of increasing problems with constipation
excised from her right calf 2 years previously. and she has noticed some fresh blood on her
Examination reveals a firm, nontender, irregular clothes.
lump below and lateral to the pubic tubercle. 4. An 80-year-old man is admitted as an emergency
4. An 80-year-old woman with recent onset of a with acute onset of profuse, painless, dark red
leg ulcer presents with a painless lump in her rectal bleeding. He is normally fit and well and has
right groin. She has recently had a femoral no history of bowel problems. On examination,
angiogram. On examination, there is a 3 × 2 cm he is pale, cold, clammy, hypotensive and his
pulsatile lump. abdomen is nontender.

104
Cardiothoracics and vascular
surgery 12
rate regular at 82 beats/min. What is the most likely
SINGLE BEST ANSWER (SBA) diagnosis?
QUESTIONS a. Thoracic aortic aneurysm
b. False aneurysm
1. A 75-year-old man, recently found to have an c. AAA
abdominal aortic aneurysm (AAA) on a routine d. Mycotic aneurysm
abdominal ultrasound scan, sees his general e. Popliteal aneurysm
practitioner (GP) to find out more about the risks of
the condition. Which of the following factors is not
5. A 42-year-old woman presents with sudden-onset
considered a high risk of rupture of AAA?
abdominal and back pain. An abdominal computed
a. Chronic obstructive pulmonary disease
tomography (CT) scan is arranged which confirms
b. Heavy smoking
the presence of an AAA. The patient is referred for
c. Fusiform shape of aneurysm
surgery. Postoperatively the patient complains of
d. Poorly controlled hypertension
weakness in her lower limbs. What is the most likely
e. Size increase by 0.6 cm/year
aetiological cause for this weakness?
a. Vertebral artery damage
2. A 65-year-old woman has a history of increasing
b. Cervical radicular artery damage
angina pectoris which led to coronary artery bypass
c. Thoracic radicular artery damage
surgery 3 years ago. She has generally done well
d. Iliac artery damage
since the surgery; she has reduced her cigarette
e. Artery of Adamkiewicz damage
smoking, but still smokes and leads a sedentary
lifestyle. In addition to emphasizing smoking cessation
and recommending an exercise program, which one 6. A 63-year-old woman with a history of ischaemic
of the following would be most effective for managing heart disease presents to the GP with pain in her
this patient’s intermittent claudication? legs. She states that she gets cramping pains in
a. Aspirin her buttocks and thighs when she tries to exercise.
b. Cilostazol On examination her femoral pulses are absent and
c. Clopidogrel there is wasting of her thighs and buttocks. Which
d. Ginkgo biloba of the following conditions has caused this patient’s
e. Pentoxifylline symptoms?
a. AAA
3. A 62-year-old man presents to his GP with a 1-month b. Aortoiliac occlusive disease
history of bilateral leg pain after walking 1 mi (1.6 km). c. Cauda equina syndrome
The pain disappears following a period of rest. He d. Inferior vena cava thrombosis
has mildly raised blood glucose and cholesterol e. Peripheral artery disease (PAD) affecting both
levels which are treated by lifestyle modification. He femoral arteries
is a smoker with a 40-pack year history and drinks
16 units of alcohol per week. His blood pressure is 7. A 35-year-old woman presents to accident and
135/85 mmHg. What is the most likely cause of this emergency (A&E) with a 3-hour history of acute left
condition in this patient? leg pains. Her past medical history is notable for
a. Alcohol previous ischaemic heart disease and diabetes.
b. Diabetes mellitus Which one of the following is a potential source of
c. Hypercholesterolaemia embolism causing acute ischaemia of the
d. Hypertension lower limb?
e. Smoking a. Amniotic fluid
b. Cerebral thrombus
4. A middle-aged man presents with sudden-onset back c. Fat embolism
pain and difficulty in swallowing. On examination his d. Gun shot
blood pressure is stable at 126/76 mmHg and pulse e. Mural thrombus

105 
Single best answer (SBA) questions

8. A 36-year-old man presents to his GP. On abdominal 13. A 54-year-old hypertensive patient presents with a
examination the GP notes a pulsatile mass. The GP tight cramp-like pain in his left calf. He comments that
suspects an AAA. What is the next most appropriate the pain is worse on walking and eases at rest. What
initial investigation? is the next most appropriate step in management?
a. Abdominal X-ray a. Cilostazol
b. Abdominal ultrasound b. Angioplasty
c. Abdominal magnetic resonance imaging c. Surgery to restore arterial inflow
scan d. Hypertension control
d. Abdominal CT scan e. Watchful waiting
e. Echocardiography
14. A 55-year-old man is brought to A&E with his wife as
9. You are on a post-take ward round when you assess a result of sudden-onset right-sided weakness. His
a patient recently diagnosed with an aneurysm. Your past medical history includes diabetes, hypertension,
consultant informs you that the patient presented hypercholesterolaemia and one previous stroke.
with a painful, tender pulsatile mass associated with On examination you note the presence of a bruit
a temperature of 38.2°C. What is the most likely in his carotids. You arrange an urgent carotid
diagnosis? doppler which confirms carotid artery stenosis. You
a. Thoracic aortic aneurysm refer the man to the on-call vascular surgeon who
b. False aneurysm agrees to perform a carotid endarterectomy. You
c. AAA explain to the patient’s wife that there is a risk of
d. Mycotic aneurysm potentially damaging his cranial nerves. What is the
e. Popliteal aneurysm most likely cranial nerve complication of a carotid
endarterectomy?
a. Cranial nerve V
10. A 65-year-old male is diagnosed with an AAA b. Cranial nerve VII
following an asymptomatic screen. It is 4.2 cm in c. Cranial nerve III
diameter. Which of the following is the best form of d. Cranial nerve XI
management for this patient? e. Cranial nerve XII
a. Annual abdominal ultrasound scans
b. Annual abdominal CT scans 15. A 75-year-old male presents to his GP with a
c. Endovascular aneurysm repair dull backache that has been worsening over the
d. No further treatment required last 3 months. On examination a pulsatile and
e. Open surgical repair expansile mass can be felt at the level of the
umbilicus. Which of the following investigations
11. You are on a ward round when your consultant would be most sensitive in confirming the
shows you a patient recently diagnosed with an underlying diagnosis?
aneurysm. He tells you that the patient developed a. Abdominal X-ray
the aneurysm in his femoral artery following a recent b. Abdominal ultrasound
angiogram. c. Ankle brachial pressure index
a. Thoracic aortic aneurysm d. CT scan of the abdomen
b. Popliteal aneurysm e. CT venogram
c. AAA
d. Mycotic aneurysm 16. A 65-year-old woman presents with an aching
e. False aneurysm sensation in both her lower legs. She comments
that this is mainly relieved by elevating her legs.
12. A 45-year-old smoker presents with a tight On examination you note the presence of tortuous
cramp-like pain in his left calf. He comments dilated veins on the back of her legs. You suspect a
that the pain is worse on walking and eases at diagnosis of varicose veins. What is the next most
rest. What is the next most appropriate initial appropriate initial investigation?
investigation? a. Handheld Doppler assessment of sapheno-
a. Duplex ultrasound femoral competence
b. Ankle brachial pressure index b. Venous duplex imaging
c. Angiography c. Venography
d. Magnetic resonance (MR) angiography d. Plethysmography
e. Intravascular ultrasound e. Leg X-ray

106
Cardiothoracics and vascular surgery 12

17. A 55-year-old obese woman presents with an


aching sensation in both her lower legs. On
EXTENDED-MATCHING
examination you note the presence of varicose QUESTIONS (EMQs)
veins. Which management plan would you
instigate first? Vascular disorders
a. Surgery Each answer can be used once, more than once or not
b. Endovenous laser therapy at all.
c. Radiofrequency ablation a. AAA
d. Compression stockings b. Thoracic aortic aneurysm
e. Regular exercise c. Dissecting aortic aneurysm
d. Compartment syndrome
18. A middle-aged, overweight man presents with e. Thromboangiitis obliterans
bilateral leg swelling. On examination you note f. Raynaud disease
evidence of ulceration and fissuring. There is no g. Raynaud phenomenon
notable evidence of infection. He comments that h. Varicose veins
there is a family history of Milroy disease. What is the i. Thrombophlebitis migrans
next most appropriate step in management? j. Deep venous thrombosis
a. Antibiotics
b. Compression stockings For each scenario below, choose the most likely
c. Homans procedure corresponding option from the list given above.
d. Charles procedure
1. Typically affects young men who smoke. The
e. Liposuction
condition is progressive unless one stops
smoking.
19. An 87-year-old woman with known atrial fibrillation 2. A disease of young women whereby arteries
(AF) presents to A&E with severe diffuse abdominal of the fingers and toes are reactive and enter
pain and tenderness. The patient has a blood spasm in response to the cold. There is often no
pressure of 87/56 mmHg and a heart rate of 145 underlying vascular disease.
beats/min. Initial blood tests show an increasing 3. Inflammation of a vein associated with pain,
serum lactate and a raised serum amylase. Which tenderness and redness. There is often an
of the following investigations is most useful in association with connective tissue disease.
confirming the underlying diagnosis? 4. Associated with a pulsatile mass on abdominal
a. Abdominal X-ray and erect chest X-ray examination with possible evidence of calcification
b. Colonoscopy with biopsy on abdominal X-ray.
c. CT scan abdomen without contrast 5. A condition associated with severe tearing central
d. Flexible sigmoidoscopy chest pain radiating to the back. Blood pressure is
e. MR angiography often unequal in both arms.

107 
This page intentionally left blank
Orthopaedics 13
c. Ewing sarcoma
SINGLE BEST ANSWER (SBA) d. Osteoclastoma
QUESTIONS e. Osteosarcoma

1. A 13-year-old male who is active in sports most of


5. A 48-year-old woman known to have systemic
the year presents with bilateral anterior knee pain that
lupus erythematosus (SLE) presents to her general
is worse in the right knee. An examination reveals
practitioner (GP) with a 3-month history of progressive
tenderness and some swelling at the tibial tubercles.
right hip pain, which is most severe during physical
Which one of the following is true regarding this
activity. She denies a history of trauma. She has been
patient’s condition?
taking paracetamol for 2 months with partial relief only.
a. Bilateral symptoms are unusual
She has also been on 20 mg of prednisolone for a
b. Corticosteroid injection of the tibial tubercle is a
lupus flare 6 months ago. Clinical examination shows
safe and effective treatment
pain with movement, but otherwise unremarkable.
c. It is almost never seen in adults
A plain X-ray of the pelvis and hip is unremarkable.
d. Radiographs should always be ordered to rule out
Which of the following is the most appropriate next
other conditions
step in the management of this patient?
e. Treatment with a straight leg cylinder cast for
a. Reassurance that pain will resolve
6 weeks is often needed
b. Advise patient to continue taking regular
paracetamol and follow-up in 1 month
2. A 14-year-old boy presents to accident and
c. Discontinue the steroids and start methotrexate
emergency (A&E) complaining of sudden-onset pain in
d. Switch to intravenous steroids
his right limb. On examination you note his right limb
e. Order an MRI of the hip
is red, tender and notably swollen. In addition, you
observe he is unwilling to move his limb. Routine blood
investigations reveal a white cell count of 23 × 109/L 6. A 45-year-old manual worker presents to his GP with
and C-reactive protein (CRP) of 98 mg/L. What is the a painful right shoulder made worse by movement.
most common aetiological cause for this presentation? On examination you note palpable crepitus; his pain
a. Streptococcus pyogenes is reproduced on cross body stressing of the right
b. Haemophilus influenzae shoulder. What is the most likely diagnosis?
c. Salmonella a. Subacromial impingement
d. Staphylococcus aureus b. Acromioclavicular osteoarthritis
e. Brucella c. Rotator cuff tear
d. Calcific tendinitis
3. An 8-year-old boy presents with a painful left limb. On e. Frozen shoulder
examination you note he is pyrexic with a temperature of
38.2°C. You suspect a diagnosis of acute osteomyelitis. 7. A woman presents to her GP with a painful right
Which investigation is most likely to lead to a diagnosis? shoulder. On examination you note that her pain can
a. Magnetic resonance imaging (MRI) of the affected be reproduced with shoulder abduction and external
limb rotation. What is the most likely diagnosis?
b. Plain limb X-ray a. Subacromial impingement
c. Erythrocyte sedimentation rate (ESR) b. Acromioclavicular osteoarthritis
d. White cell count c. Rotator cuff tear
e. CRP d. Calcific tendinitis
e. Frozen shoulder
4. A 69-year-old man with Paget disease presents with
a painful right leg. He comments that it is particularly 8. A 70-year-old woman complains of bilateral gradual
painful when walking. A plain limb X-ray reveals evidence knee pain on most days of the past few months. She
of bone destruction and a ‘sunray’-like appearance of complains of stiffness in the morning that lasts for a
his right femur. What is the most likely diagnosis? few minutes and a creaking sensation at times in the
a. Osteoid osteoma right knee. On examination there is a small effusion,
b. Chondroma diffuse crepitus and limited flexion of both knees.
109 
Single best answer (SBA) questions

Joint tenderness is more prominent over the 13. An 11-year-old overweight boy presents with a limp
medial joint line bilaterally. Which one of the in his right leg and groin pain. Physical examination
following is characteristic of osteoarthritis of the reveals that his right leg is shorter than his left. X-ray
knee? reveals widening of the growth plate. What is the
a. Greater frequency in men than in women most likely diagnosis?
b. Increased pain with rest a. A slipped femoral epiphysis
c. A direct correlation between radiographic b. Perthes disease
changes and pain severity c. Osteoarthritis
d. Reduction of pain with repair of associated d. Osteogenic sarcoma
meniscal tears e. Osteomyelitis
e. Reduction of pain with muscle strengthening
14. A man presents to his GP complaining of being
9. A rugby player presents to his GP complaining of unable to extend his right wrist. What is the most
a painful left shoulder following a local match. On likely nerve involved in this presentation?
examination you note reduced active but full passive a. Radial
movement. In addition, you note evidence of muscle b. Ulnar
wasting and tenderness on palpation. What is the c. Median
most likely diagnosis? d. Brachial
a. Subacromial impingement e. Musculocutaneous
b. Acromioclavicular osteoarthritis
c. Rotator cuff tear 15. A middle-aged chronic alcohol abuser complains of
d. Calcific tendinitis being unable to extend his ring and little finger of his
e. Frozen shoulder right hand. What is the most likely diagnosis?
a. Ulnar nerve palsy
10. A middle-aged woman presents with a painful right b. Dupuytren contracture
elbow. She comments that the pain starts over c. Brachial nerve palsy
the outer side of her elbow and radiates down her d. Carpal tunnel syndrome
forearm. In addition, she states that she finds it e. Trigger finger
difficult to open door handles. What is the next most
appropriate step in management? 16. A middle-aged woman presents to her GP
a. Steroid injections complaining that her right middle finger ‘catches’ as
b. Paracetamol she bends it. What is the most likely diagnosis?
c. Nonsteroidal antiinflammatory drugs (NSAIDs) a. Ulnar nerve palsy
d. Surgery b. Dupuytren contracture
e. Tramadol c. Brachial nerve palsy
d. Carpal tunnel syndrome
11. An 80-year-old woman falls in the geriatrics ward e. Trigger finger
whilst mobilizing with the physiotherapist. A lower
limb fracture is suspected. On examination her leg 17. A 69-year-old retired construction worker presents
and foot are externally rotated compared with the with a 2-week history of lower back pain. He
opposite limb and there is some shortening. What is comments that it is severe in nature and radiates
the most likely affected structure? down both legs. Today, he is unable to pass urine
a. Ankle and has reduced perianal sensation on neurological
b. Neck of the femur testing. Which investigation is most likely to lead to a
c. Rami of the pelvis diagnosis?
d. Shaft of the femur a. Plain spine X-ray
e. Tibia b. Computed tomography (CT) spine
c. MRI spine
12. A man presents to his GP complaining of numbness d. Myeloma screen
in his ring and little finger of his right hand. What is e. Prostate-specific antigen
the most likely nerve involved in this presentation?
a. Radial 18. An 18-year-old man presents with knee pain. This
b. Ulnar came on suddenly as a sharp pain while he was
c. Median turning quickly during a football match 1 week ago.
d. Brachial Since then, his knee has been swollen, he has been
e. Musculocutaneous unable to completely straighten his leg and has

110
Orthopaedics 13

experienced painful locking of his knee. Which of the 23. A middle-aged woman presents to A&E with pain
following is the most likely diagnosis? in her left knee following a head-on car crash. On
a. Anterior cruciate ligament tear examination you note displacement of the tibia on
b. Meniscal tear application of a posterior force. What is the most
c. Osteoarthritis likely diagnosis?
d. Patella tendon rupture a. Peripheral meniscal tear
e. Suprapatellar effusion b. Capsular tear
c. Anterior cruciate ligament rupture
19. A 65-year-old woman presents with pain in her d. Osteochondral fracture
right hip. She comments that the pain is particularly e. Posterior cruciate ligament rupture
worse on walking and at the end of the day. On
examination you note crepitus on movement and 24. A 65-year-old retired physical education teacher
reduced active and passive range of movement. presents to her GP with lower back pain, radiating
You suspect the possibility of osteoarthritis. Which into her thighs and buttocks. The pain is worse on
investigation is most likely to lead to a diagnosis? walking downhill and alleviated by cycling. Lower
a. Plain hip X-ray limb neurological examination reveals diminished
b. ESR ankle and knee reflexes. Which of the following
c. CRP investigations is most likely to confirm the diagnosis?
d. Rheumatoid factor a. X-ray of the lumbar region
e. Full blood count b. CT scan of the lumbar region
c. MRI of the lumbar region
20. A 35-year-old man is diagnosed with right knee d. Rheumatoid factor
haemarthrosis following a football injury. What e. Electromyography
is the most common aetiological cause for this
condition? 25. An 8-month-old boy is brought to the GP for a
a. Peripheral meniscal tear postnatal check. On examination the GP notes
b. Capsular tear asymmetry of groin skin creases. Further assessment
c. Anterior cruciate ligament rupture reveals an unstable dislocatable right hip. An
d. Osteochondral fracture ultrasound scan confirms the diagnosis. What is the
e. Posterior cruciate ligament rupture next most appropriate step in management?
a. Pavlik harness
21. A 16-year-old male presents with ongoing pain b. von Rosen splint
around his knee. On examination there is clear c. Femoral osteotomy
localized swelling and tenderness. An X-ray of the d. Open/closed reduction and immobilization
knee shows periosteal elevation (known as Codman e. Plaster immobilization
triangle) and underlying bone destruction with new
bone formation, described as ‘sunray spicules’. 26. A 9-year-old boy presents with a limp and
Which of the following is the most likely underlying generalized ache of his right hip. A hip X-ray reveals
condition? a smaller femoral head epiphysis and widening of
a. Ewing sarcoma the joint space on the right side. What is the most
b. Giant-cell tumour likely diagnosis?
c. Osgood–Schlatter disease a. Developmental dysplasia of the hip
d. Osteosarcoma b. Perthes disease
e. Secondary metastases to the bone cortex c. Slipped upper femoral epiphysis (SUFE)
d. Irritable hip
22. A teenager presents to A&E with severe pain in his e. Osteoarthritis
right knee following a football injury. He comments
that he heard a loud crack and then found it too 27. A 12-year-old boy of African descent presents with
difficult to run. On examination you note the right acute-onset left-sided hip pain and a limp. A hip
knee is red and swollen in appearance. What is the X-ray reveals displacement of the femoral head.
most likely diagnosis? What is the most likely diagnosis?
a. Peripheral meniscal tear a. Developmental dysplasia of the hip
b. Capsular tear b. Perthes disease
c. Anterior cruciate ligament rupture c. SUFE
d. Osteochondral fracture d. Irritable hip
e. Posterior cruciate ligament rupture e. Osteoarthritis

111 
Extended-matching questions (EMQs)

28. Which of the following nerves is most at risk of being 1. An elderly woman complaining of a productive
damaged following a shoulder dislocation? cough and shortness of breath following a hip
a. Accessory nerve fracture.
b. Axillary nerve 2. Occurs typically on day 3 after the fracture is
c. Median nerve sustained. Associated features include confusion,
d. Radial nerve dyspnoea and hypoxia.
e. Ulnar nerve 3. Failure of fracture healing within the expected
period. Typically seen with scaphoid fractures.
4. A serious condition associated with increased
EXTENDED-MATCHING tissue pressure, vascular occlusion and soft tissue
QUESTIONS (EMQs) necrosis.
5. A condition whereby a fracture unites in a
nonanatomical position and is associated with
Fractures poor fracture healing.
Each answer can be used once, more than once or not at all.
a. Clavicle Orthopaedic clinical examination
b. Humeral neck Each answer can be used once, more than once or not
c. Humeral shaft at all.
d. Scaphoid
a. Detects osteoarthritis of the thumb
e. Distal radius
b. Discriminates between a muscular and a bony flat
f. Pelvis
foot deformity
g. Femoral shaft
c. Identifies an Achilles tendon rupture
h. Intracapsular neck of femur
d. Tests for a fixed flexion deformity of the hip
i. Tibial
e. Tests for a fixed flexion deformity of the lumbar
j. Scapula
spine
f. Tests for laxity of the anterior cruciate ligament
For each scenario below, choose the most likely
g. Tests for torn medial menisci
corresponding option from the list given above.
h. Tests for a torn posterior cruciate ligament
1. Commonly seen in osteoporotic postmenopausal i. Tests for an unstable shoulder girdle
ladies who fall on an outstretched hand. j. Tests for weak ipsilateral hip abductor muscles
2. Occurs as a result of falling on an outstretched
arm. May be associated with radial nerve injury. For each scenario below, choose the most likely
3. Associated with external rotation and shortening corresponding option from the list given above.
of the leg.
1. Anterior draw test
4. Swelling and pain on wrist movement. Notably
2. Thomas test
difficult to diagnose on an X-ray.
3. Trendelenburg test
5. Associated with damage to the medial femoral
4. Schober test
circumflex artery.
5. Apprehension test
6. Lachman test
Fracture complications 7. McMurray test
Each answer can be used once, more than once or not
at all.
Knee injuries
a. Crush syndrome Each answer can be used once, more than once or not
b. Compartment syndrome at all.
c. Algodystrophy
a. Anterior cruciate ligament rupture
d. Nonunion
b. Bucket handle tear of meniscus
e. Malunion
c. Dislocation of the knee
f. Contracture
d. Lateral collateral ligament rupture
g. Compensation neurosis
e. Medial collateral ligament rupture
h. Fat embolus
f. Meniscal tear
i. Pneumonia
g. Patella fracture
j. Renal stone
h. Patella ligament rupture
i. Posterior cruciate ligament rupture
For each scenario below, choose the most likely
j. Tibial plateau fracture
corresponding option from the list given above.

112
Orthopaedics 13

For each scenario below, choose the most likely Skin, joint and bone infections
corresponding option from the list given above. Each answer can be used once, more than once or not
1. A 13-year-old girl presents with locking of the at all.
knee, having sustained previous knee injuries a. Escherichia coli
while playing netball. Arthroscopy diagnoses and b. Pseudomonas aeruginosa
treats the condition successfully. c. Proteus mirabilis
2. A 37-year-old man is hit side on by a van while d. Group A streptococcus
riding his motorbike. You, the orthopaedic senior e. S. aureus
house officer, are called to see the man after several f. Coliforms
attempts at reducing his patella have failed. There is g. Pasteurella multocida
obvious deformity of the knee. X-ray shows medial h. Mycobacterium tuberculosis
displacement of the femur on the tibia. i. Streptococcus pneumoniae
3. A 52-year-old lady is involved in a car accident. j. Bacteroides fragilis
She is hit by an uninsured driver as she crosses
the road to work. Her knee is very painful and For each scenario below, choose the most likely
she is unable to bear weight. X-ray demonstrates corresponding option from the list given above.
the abnormality, which is reported as ‘Schatzker
1. A builder with olecranon bursitis.
grade 3’.
2. Necrotizing fasciitis of the leg following a trivial
4. A 32-year-old man is brought into A&E having
injury.
injured his right knee in a strong tackle playing
3. Prosthetic hip infection.
football. The mechanism of injury is unclear.
4. An infected cat bite on the hand.
On examination pulses are present, there is no
5. A foot infection after stepping on a nail while
posterior sag but anterior draw is positive.
wearing trainers.
5. A 30-year-old professional woman is brought into
A&E. She fell down some stairs at a conference,
landing on her right knee. She is in pain and is
unable to straight leg raise.

113 
This page intentionally left blank
Ear, nose and throat 14
d. High-dose pseudoephedrine and increase fluid
SINGLE BEST ANSWER (SBA) intake
QUESTIONS e. Ordering sinus X-rays (XRs) to determine the most
appropriate course of care
1. A 54-year-old woman recently travelled to Cornwall
for a scuba diving trip. Since her return, she has noted 5. A 75-year-old man with a 20-year history of type 2
brief intermittent episodes of vertigo not associated diabetes mellitus presents to his general practitioner
with nausea or vomiting. She is concerned, however, (GP) with constant and severe right ear pain.
because these episodes occurred after sneezing or Otoscopic examination shows offensive-smelling
coughing and then a couple of times after straining purulent otorrhoea. There is also a red lesion at
while lifting something. She has had no hearing the external auditory canal. A biopsy of this lesion
loss, and no vertigo with positional changes such as is subsequently performed, and histology reveals
bending over or turning over in bed. The most likely presence of granulation tissue. Which of the following
cause of this patient’s vertigo is which of the following? agent is the most likely cause?
a. Benign paroxysmal positional vertigo a. Escherichia coli
b. Ménière disease b. Pseudomonas aeruginosa
c. Multiple sclerosis triggered by a rapid change in c. Haemophilus influenzae
climate d. Proteus vulgaris
d. Perilymphatic fistula e. Staphylococcus aureus
e. Vestibular neuronitis
6. A middle-aged man presents with a lump in his neck.
2. A 35-year-old man presents with a lump between On examination you note it is oval, nontender and
his thyroid notch and hyoid bone which moves pulsatile in nature. What is the most likely diagnosis?
upwards on tongue protrusion. What is the most likely a. Lymphoma
diagnosis? b. Thyroglossal cyst
a. Lymphoma c. Branchial cyst
b. Thyroglossal cyst d. Carotid body tumour
c. Branchial cyst e. Carcinoma
d. Carotid body tumour
e. Carcinoma 7. A 76-year-old man complains of swollen neck glands.
On examination you note swelling and tenderness of
3. An 18-year-old woman presents with a cyst in the both his parotid glands and the presence of purulent
upper anterior border of her sternocleidomastoid saliva on gentle palpation of his parotids. What is the
muscle. What is the most likely diagnosis? most likely diagnosis?
a. Lymphoma a. Lymphoma
b. Thyroglossal cyst b. Pleomorphic adenoma
c. Branchial cyst c. Parotitis
d. Carotid body tumour d. Parotid calculi
e. Carcinoma e. Adenolymphoma (Warthin tumour)

4. A 45-year-old male presents with a 3-day history 8. A 65-year-old man presents with a painless mass
of increasing cough, sinus pressure and low-grade in his right parotid gland. He comments that he has
fever. His past medical history is positive only for noticed the mass grow slowly over time. What is the
hypertension. What is the most appropriate initial step most likely diagnosis?
in patient care? a. Lymphoma
a. 7-Day course of amoxicillin b. Pleomorphic adenoma
b. 10-Day course of levofloxacin c. Parotitis
c. Explaining that most cases of sinusitis resolve d. Parotid calculi
without treatment e. Adenolymphoma (Warthin tumour)

115 
Single best answer (SBA) questions

9. A patient presents complaining of facial flushing c. Ibuprofen


following parotid surgery to remove a pleomorphic d. Paracetamol
adenoma. She states that whenever she is eating, e. Amoxicillin
she develops an erythematous rash on her cheek
and starts sweating. She also experiences a burning 14. A 6-year-old girl is brought to her GP by her
pain in the distribution of the rash. What is the most mother. Her mother comments that her right
likely underlying diagnosis? ear is giving her trouble and that she appears
a. Facial nerve palsy inattentive at times. On examination the GP notes
b. Frey syndrome the eardrum is yellow in colouration and that it
c. Heerfordt syndrome has been drawn inwards. What is the most likely
d. Salivary fistula diagnosis?
e. Staphylococcal infection a. Otitis externa
b. Otitis media
10. A 67-year-old woman presents with a painless mass c. Mastoiditis
in her left parotid gland. She comments that she d. Glue ear
has noticed the mass grow slowly over time but did e. Otosclerosis
not feel it was anything to worry about as it was not
painful in nature. What is the most likely diagnosis? 15. A middle-aged man complains of hearing loss
a. Lymphoma and ringing in his right ear. You first perform the
b. Pleomorphic adenoma Rinne test and note that air conduction is better
c. Parotitis than bone conduction, but both are notably
d. Parotid calculi reduced as compared with normal. On performing
e. Adenolymphoma (Warthin tumour) the Weber test, you note that the sound is
localized to his left ear. What is the most likely
11. A 25-year-old man presents with mild discomfort diagnosis?
and occasional pain in his right ear. On examination a. Ménière disease
you note some discharge from his ear. He b. Otitis externa
comments that there has been no change in his c. Acoustic neuroma
hearing. What is the most likely diagnosis? d. Otitis media
a. Otitis externa e. Otosclerosis
b. Otitis media
c. Mastoiditis 16. A 45-year-old man presents to his GP with
d. Glue ear tinnitus, hearing loss and a sensation that
e. Otosclerosis the room is spinning around him. The GP
prescribes an antihistamine which helps to
12. A 4-year-old boy is brought to his GP by his mother. relieve his symptoms. What is the most likely
She comments that he has been tearful and diagnosis?
complains of pain in his right ear. She also mentions a. Ménière disease
that it seems as if he is unaware of his surroundings. b. Otitis externa
On examination the GP notes a red-coloured, tense- c. Acoustic neuroma
looking ear drum. What is the most likely diagnosis? d. Otitis media
a. Otitis externa e. Otosclerosis
b. Otitis media
c. Mastoiditis 17. A 36-year-old man presents to accident and
d. Glue ear emergency with a nose bleed. As you begin to
e. Otosclerosis assess the patient you notice the bleeding has
increased in severity. He comments that he is feeling
13. A 5-year-old boy is brought to his GP by his mother slightly dizzy and light-headed. He is hypotensive
after complaining of pain in his right ear. She and tachycardic. Which management plan would
comments that he is continually crying and tugging you instigate first?
on his right ear. On examination the GP notes a. Topical cocaine
a red-coloured ear drum. Routine observations b. Cauterization of the bleeding vessel
demonstrate a temperature of 38.5°C. What is the c. Ribbon gauze
next most appropriate step in management? d. Large-bore cannula insertion and blood samples
a. Clarithromycin for urgent cross match
b. Erythromycin e. Introduction of an epistaxis balloon

116
Ear, nose and throat 14

18. A 15-year-old woman presents with sneezing and e. Sjögren syndrome


nasal discharge having been playing in the nearby f. Pleomorphic adenoma
fields. Her GP informs her that she is suffering from g. Adenolymphoma
allergic rhinitis. Which management plan would you h. Carcinoma
instigate first? i. Thyroglossal cyst
a. Intranasal antrostomy j. Cystic hygroma
b. Radical antrostomy
c. Antibiotics For each scenario below, choose the most likely
d. Oral antihistamines corresponding option from the list given above.
e. Steroid nasal spray 1. An autoimmune condition associated with dry
eyes and parotid swelling.
19. An elderly gentleman presents with difficulty in 2. A hard, irregular, craggy lump noted within the
swallowing. He states that his initial mouthful parotid gland. There is often associated facial
of either solid or liquid food is fine, but then nerve involvement.
progressively gets more difficult. In addition, he 3. A slow-growing mass which pulsates and has a
states experiencing bouts of coughing episodes at potential for malignant transformation.
night. What is the most likely diagnosis? 4. A discharging cavity which presents anteriorly to
a. Achalasia the lower third of the sternocleidomastoid muscle.
b. Diffuse oesophageal spasm 5. A soft painless mass which transilluminates
c. Oesophageal carcinoma brightly and typically presents in the anterior
d. Pharyngeal pouch triangle of the neck.
e. Systemic sclerosis
Neck lumps
20. An elderly gentleman is diagnosed with squamous cell
Each answer can be used once, more than once or not
carcinoma of the neck. What is the most common
at all.
risk factor for development of such a condition?
a. Epstein–Barr virus (EBV) a. Branchial cyst
b. Smoking b. Carotid artery aneurysm
c. Human papillomavirus c. Carotid body tumour
d. Gastro-oesophageal reflux disease d. Cervical rib
e. Asbestos e. Cystic hygroma
f. Lymph nodes
21. An 18-year-old patient has a smooth lump on the left g. Pharyngeal pouch
side of his neck. It is situated in the anterior triangle, h. Thyroglossal cyst
under the anterior border of the sternocleidomastoid i. Thyroid goitre
muscle and roughly one-third of the way down the j. Torticollis
sternocleidomastoid muscle. Which of the following
is the most likely diagnosis? For each scenario below, choose the most likely
a. Brachial cyst corresponding option from the list given above.
b. Carotid artery aneurysm 1. A midline lump that moves on swallowing and
c. Chondroma upon tongue protrusion. It has been present since
d. Dermoid cyst birth.
e. Thyroglossal cyst 2. Multiple lumps in the anterior triangle which do not
move upon swallowing. There are similar lumps in
the axilla.
EXTENDED-MATCHING 3. A painless, transilluminable lump in a 25-year-
old man which arises from beneath the anterior
QUESTIONS (EMQs) border of sternocleidomastoid in its upper third.
4. An anterior triangle lump in a 70-year-old man
Neck swellings which does not move up on swallowing. The
Each answer can be used once, more than once or not patient presented with a recent history of
at all. dysphagia and had not noticed the lump before.
a. Branchial cyst 5. A swelling which crosses the midline and has an
b. Branchial sinus irregular surface. It moves on swallowing but not
c. Carotid body tumour on tongue protrusion. The patient is thin, anxious
d. Parotitis and tachycardic.

117 
Extended-matching questions (EMQs)

Neck swellings 2. A 70-year-old woman presents with a 2-month


Each answer can be used once, more than once or not history of a swelling in the front of the neck. There
at all. has been a rapid increase in the size of the lump.
She has become hoarse. On examination, she is
a. Anaplastic carcinoma of the thyroid
noted to have an audible wheeze and there is a large
b. Benign tumour of parotid (pleomorphic adenoma)
hard mass in the lower anterior part of the neck.
c. Branchial cyst
3. A 60-year-old man presents with a 4 × 2 cm2
d. Papillary carcinoma of the thyroid
mass to the left of the midline in the lower neck.
e. Pharyngeal pouch
The mass is oval with a smooth surface. It moves
f. Riedel thyroiditis
upwards on swallowing.
g. Solitary thyroid nodule
4. A 14-year-old girl presents with a swelling on the
h. Submandibular calculus
left side of the neck. It is painless but has been
i. Thyroglossal cyst
slowly increasing in size. On examination, there is
j. Virchow node
a smooth 5 × 5 cm2 swelling arising from beneath
the anterior aspect of the upper third of the
For each scenario below, choose the most likely
sternomastoid muscle.
corresponding option from the list given above.
5. A 65-year-old man presents with difficulty
1. A 56-year-old woman complains of an intermittent with swallowing, weight loss and vomiting. On
swelling under her jaw on the left side. The examination he is cachexic, with a firm swelling in
swelling increases in size and becomes painful the left supraclavicular fossa.
when she eats.

118
Urology 15
normal and the PSA level is 4.2 μg/L (3.0 nanograms/
SINGLE BEST ANSWER (SBA) mL). Which amongst the following options is the correct
QUESTIONS treatment for benign prostatic hyperplasia (BPH)?
a. α-Adrenergic antagonists that reduce the smooth
1. A 47-year-old married male comes to your clinic muscle tone
complaining of right testicular pain that developed b. β-Reductase agonist that increase smooth muscle
quickly over the last 24 hours. He has a recent history tone
of having a catheter put in post-appendectomy. c. β-Reductase agonist that reduce smooth muscle
He reports no fever, dysuria, or urethral discharge. tone
His abdominal examination is normal but he has an d. 5α-Reductase inhibitors that reduce smooth
enlarged, exquisitely tender right hemi-scrotum. A muscle tone
urinalysis is normal and his white blood cell count is 13 e. 5α-Reductase inhibitors that increase smooth
× 109/L. Testicular ultrasonography shows increased muscle tone
blood flow in the affected testicle and epididymis. What
is the most appropriate initial treatment in this patient? 5. A 23-year-old woman presents with urinary frequency
a. Ceftriaxone and dysuria. Routine observations confirm a
b. Clindamycin temperature of 38.7°C. Blood cultures are taken
c. Famciclovir which demonstrate the growth of gram-negative
d. Ofloxacin organisms. Which management plan would you
e. Urgent urology review instigate first?
a. Oral antibiotics
2. A 75-year-old male with a past medical history of gout b. Intravenous antibiotics
and a recent chemotherapy regimen presents with c. Oral fluids
right flank pain. He tells you the pain was rapid in onset d. Urinary catheterization
and radiates to his groin. Laboratory analysis shows e. Analgesia
microscopic haematuria. A renal ultrasound (USS KUB)
is negative but a noncontrast computed tomography 6. A previously fit and well 22-year-old student presents
(CT) scan is diagnostic. Which of the following should to his general practitioner (GP) with an incidental
be used to prevent this condition from occurring again? finding of a painless testicular lump. He has a past
a. Citrate medical history of an undescended testis at birth.
b. Sodium restriction Clinical examination shows a hard, nontender mass
c. Penicillamine on his left testis which does not transilluminate. Which
d. Thiazide diuretics of the following tests will have to be performed to
e. Surgery investigate this mass?
a. Testicular ultrasound
3. A 62-year-old man presents with sudden onset loin b. CT scan of lungs, abdomen and pelvis
pain, urinary frequency and dysuria. On examination c. Alpha-fetoprotein (AFP)
he is pyrexic, with a temperature of 38.2°C. He d. β-Human chorionic gonadotropin (β-HCG)
appears notably confused. What is the next most e. All of the above
appropriate initial investigation?
a. Urinalysis and culture 7. A 45-year-old man presents with urinary frequency,
b. Abdominal ultrasound dysuria and haematuria. On examination you note
c. Intravenous urogram suprapubic tenderness. What is the most likely
d. Abdominal X-ray diagnosis?
e. Prostate specific antigen (PSA) a. Pyelonephritis
b. Interstitial cystitis
4. An 82-year-old man presents with a 3-month history c. Prostatitis
of weak stream, straining and hesitancy. Examination d. Acute cystitis
reveals a severely enlarged prostate. A urinalysis is e. Epididymo-orchitis

119 
Single best answer (SBA) questions

8. A previous fit and well 45-year-old man presents to c. PSA is elevated in prostatitis
the emergency department with a 1-day history of d. PSA is elevated in BPH
severe left sided loin pain radiating to his groin. Plain e. PSA is used to monitor the recurrence of
abdominal radiograph reveals a left ureteral calculus. prostate cancer
Urinalysis shows a trace of blood but nil else. What
is the most likely composition of his calculus? 13. A 25-year-old sexually active man presents
a. Cystine with testicular pain and vomiting. On examination
b. Uric acid you note his testicles are swollen and tender.
c. Calcium oxalate A urine dipstick reveals 3+ leucocytes, 2+
d. Magnesium-ammonium phosphate nitrites but no blood. What is the most likely
e. Iron deposits diagnosis?
a. Pyelonephritis
9. A 41-year-old man presents with urinary frequency, b. Interstitial cystitis
dysuria and urgency. Urine cultures are taken which c. Prostatitis
reveal no bacterial organisms. What is the most likely d. Acute cystitis
diagnosis? e. Epididymo-orchitis
a. Pyelonephritis
b. Interstitial cystitis 14. A 70-year-old man sees his GP with a 1-month
c. Prostatitis history of painless haematuria, urinary frequency
d. Acute cystitis and mild pelvic and lower back discomfort. He
e. Epididymo-orchitis recalls a transient episode of haematuria in his
childhood when he lived in West Africa. Clinical
10. A 62-year-old man presents with urinary frequency examination is unremarkable. What is the most
and dysuria. On examination you note tenderness in likely diagnosis?
the lower abdomen. On rectal examination you note a. Transitional cell carcinoma of urinary
tenderness on palpation of his prostate. What is the bladder
most likely diagnosis? b. Adenocarcinoma of urinary bladder
a. Pyelonephritis c. Acute infection of urinary bladder
b. Interstitial cystitis d. Squamous cell carcinoma of urinary bladder
c. Prostatitis e. Stone in the urinary bladder
d. Acute cystitis
e. Epididymo-orchitis 15. A 49-year-old woman presents with loin pain and
haematuria. On examination you note a mass in her
11. A 31-year-old man presents with an acute onset right loin. Which investigation is most likely to lead to
severe, colicky right flank pain radiating down into a diagnosis?
the groin. Over the past 3 days he has increased a. Urea and electrolytes
frequency, dysuria and haematuria. Which of the b. Abdominal CT scan
following is the most common metabolic abnormality c. Chest X-ray
associated with this patient’s condition? d. Intravenous urogram
a. Hypercalciuria e. Urinalysis
b. Increased urine citrate
c. Hyperuricaemia 16. A 50-year-old man presents to his GP with a
d. Alkaline urinary pH 1-month history of dull ache within the right
e. Hypernatraemia side of his scrotum, which he describes as
a ‘dragging sensation’. Clinical examination
12. A fit and well 55-year-old man sees his GP for reveals a nontender, twisted mass along the right
advice. Both his father and uncle suffer from spermatic cord that feels like a bag of worms,
prostate cancer, and he is concerned that he might which does not change in size in the supine
also be at risk. He has read some information position. The rest of the scrotal examination
regarding PSA and is requesting the blood test to is unremarkable. What is the most important
screen for prostate cancer. Which of the following investigation in this case?
statements is not true regarding PSA? a. PSA blood test
a. PSA is a protein produced by cells of the b. Plain chest radiograph
prostate gland c. Plain abdominal radiograph
b. PSA is sensitive and specific for prostate d. Urinalysis
cancer e. CT of the abdomen

120
Urology 15

17. A middle-aged man is recently diagnosed with renal d. Abdominal magnetic resonance imaging scan
cell carcinoma of his left kidney without evidence e. Abdominal CT scan
of distant metastasis. What is the next most
appropriate step in management? 22. A 60-year-old man presents with painless
a. Embolization of the renal artery haematuria. A cystoscopy confirms the presence
b. Radiotherapy of a tumour in his bladder, which is malignant on
c. Chemotherapy biopsy. He undergoes a staging CT scan which
d. Radical nephrectomy confirms involvement of the superficial muscle and
e. Hormonal treatments no evidence of distant metastases. What is the next
most appropriate step in management?
18. A 26-year-old man presents with severe colicky pain a. Radical cystectomy
in his right loin which radiates to his right groin. A b. Radiotherapy
urine dipstick is positive for blood. An intravenous c. Systemic chemotherapy
urogram is requested which demonstrates a d. Transurethral resection
ureteric stone approximately 3 mm in size. Following e. Watchful waiting
appropriate analgesia, which management plan is
most appropriate in the first instance? 23. A 72-year-old man presents with urinary frequency
a. Extracorporeal shockwave lithotripsy and nocturia. You suspect a diagnosis of BPH.
b. Percutaneous nephrolithotomy According to the International Prostate Symptom
c. Open surgical removal Score, his symptoms are classified as mild. What is
d. Ureteric stenting the next most appropriate step in management?
e. Allow for the stone to pass spontaneously a. Watchful waiting
b. Radical prostatectomy
19. A 26-year-old man presents with severe colicky pain c. Radiotherapy
in his right loin which radiates to his right groin. A d. 5α-Reductase inhibitors
urine dipstick proves positive for blood and nitrites. An e. α-Blockers
intravenous urogram is requested which demonstrates
a ureteric stone approximately 4 mm in size. Which 24. You are called to see a 65-year-old man following a
management plan would you instigate first? transurethral resection of the prostate (TURP). He
a. Analgesia has lost consciousness and is having uncontrolled
b. Commence an α-blocker rhythmic tonic clonic movements of his arms and
c. Open surgical removal legs. Which of the following is most likely to be the
d. Arrange referral for extracorporeal shockwave cause of his symptoms?
lithotripsy a. Hypernatraemia
e. Antibiotics b. Hyperkalaemia
c. Hyponatraemia
20. A 68-year-old man presents to the urology d. Hypokalaemia
outpatients clinic with urinary urgency and frequency e. Withdrawal of anaesthetic drugs
which is worse when he exercises. This has
gradually worsened over a period of weeks. What is 25. A 35-year-old man presents with a lump in his scrotum.
the most likely diagnosis? On examination the lump is translucent and nontender.
a. A bladder cancer You find it difficult to identify the testes and epididymis
b. A bladder stone separately. What is the most likely diagnosis?
c. A renal stone a. Hydrocele
d. Prostatitis b. Epididymo-orchitis
e. Urinary infection c. Epididymal cyst
d. Testicular tumour
21. A 65-year-old smoker presents with painless e. Testicular torsion
haematuria. Routine examination proves
unremarkable. Routine blood investigations reveal a 26. A 24-year-old man presents with a lump in his scrotum.
urea of 15.4 mmol/L and creatinine of 154 μmol/L. On examination the lump is non-translucent and
You suspect a diagnosis of bladder cancer. What is extremely tender. You find it difficult to identify the testes
the next most appropriate initial investigation? and epididymis. What is the most likely diagnosis?
a. Cystoscopy and biopsy a. Hydrocele
b. Chest X-ray b. Epididymo-orchitis
c. Intravenous urogram c. Epididymal cyst

121 
Extended-matching questions (EMQs)

d. Testicular tumour d. Testicular tumour


e. Testicular torsion e. Testicular torsion

27. A 56-year-old male presents to his GP with a fever and


backache. He also complains of a poor stream when EXTENDED-MATCHING
urinating. A rectal (PR) examination reveals a tender QUESTIONS (EMQs)
and boggy prostate which is of a regular size. Which of
the following drugs should this patient be prescribed? Scrotal swellings
a. Amoxicillin Each answer can be used once, more than once or not
b. Ciprofloxacin at all.
c. Doxazosin
a. Direct inguinal hernia
d. Finasteride
b. Epididymal cyst
e. Trimethoprim
c. Hydrocele
d. Hydrocele of the cord
28. A 60-year old male patient presents with painless
e. Indirect inguinal hernia
haematuria after being referred by the GP. Urinary
f. Lipoma
cytology was positive. What is the most likely
g. Lymph node
diagnosis?
h. Spermatocele
a. A clear cell renal cancer
i. Testicular tumour
b. A renal transitional cell cancer
j. Varicocele
c. Adenocarcinoma of the bladder
d. Prostatic adenocarcinoma
For each scenario below, choose the most likely
e. Transitional cell cancer (TCC) of the bladder
corresponding option from the list given above.

29. A 32-year-old man presents with a lump in his 1. A 40-year-old man presents with a tense lump
scrotum. On examination the lump is translucent, in the scrotum which has appeared over the last
separate from the testes and is nontender. What is 3 weeks and is mildly tender. It is transilluminable
the most likely diagnosis? and the testis cannot be felt separately.
a. Hydrocele 2. A 22-year-old man presents with a tender lump
b. Epididymo-orchitis in his scrotum that his girlfriend noticed in the
c. Epididymal cyst shower. He has lost a stone in weight over the last
d. Testicular tumour 2 months, and clinically, there is a lump attached
e. Testicular torsion to the testis.
3. A 30-year-old man presents with a large lump
30. A 76-year-old man was seen in accident and in the scrotum that came on a few days ago. It
emergency with abdominal discomfort. On is large, tense and you cannot feel above it. It is
examination he has a nontender, dome-shaped cystic nontender and non-transilluminable, and it has a
mass dull to percussion reaching to the umbilicus. cough impulse.
Investigation reveal Hb is 96g/L, serum urea is 4. A 35-year-old man presents with a left-sided
15 mmol/L, serum creatinine is 160 mmol/L. What is lump in the scrotum. He says that it disappears
the most appropriate next step in patient care? when he lies down. Upon palpation, the lump is
a. Abdominal CT scan inconsistent in nature and feels very mobile.
b. Sigmoidoscopy
c. Suprapubic aspiration Lower urinary tract symptoms
d. X-ray of the abdomen Each answer can be used once, more than once or not
e. Urinary catheterization at all.
a. Acontractile bladder
31. A 16-year-old boy presents with sudden onset b. Benign prostatic hypertrophy
severe pain in his left scrotum and vomiting. On c. Bladder neck stenosis
examination you note evidence of scrotal erythema d. Low-pressure, low-flow voiding
and tenderness on palpation; upon elevation of the e. Multiple sclerosis
scrotum, there is no relief of pain. A urine dipstick f. Prolapsed intervertebral disc
proves negative. What is the most likely diagnosis? g. Prostatic cancer
a. Hydrocele h. Spinal canal stenosis
b. Epididymitis i. Stress incontinence
c. Epididymal cyst j. Urethral stricture

122
Urology 15

For each scenario below, choose the most likely 3. A middle-aged man presenting with urinary
corresponding option from the list given above. hesitancy, a poor stream and terminal
1. A 70-year-old man presents with a 5-year history dribbling postmicturition with evidence of renal
of a poor flow, hesitancy, frequency, nocturia and dysfunction.
postmicturition dribbling. A flow rate showed a 4. A 75-year-old man presenting with urinary
Q max of 7 mL/s with a voided volume of 270mL frequency, nocturia and weight loss. An irregular
and a residual of 100mL. mass is evident on the prostate on digital
2. A 40-year-old woman presents with urinary examination.
incontinence when coughing, sneezing and during 5. A middle-aged female who has had five vaginal
her fitness class. She has had three children deliveries presenting with urinary leakage when
delivered vaginally. The first labour was prolonged coughing or laughing.
and she had a repair of a second-degree tear.
3. A 16-year-old man presents with a long history Urgent urological referrals
of hesitancy and worsening urinary stream. He Each answer can be used once, more than once or not
also complains of postmicturition dribbling, which at all.
may occur some minutes after he completes his a. Benign prostatic hypertrophy
normal void. He had a significant fall astride a bar b. Cystitis
injury as a child. c. Epididymitis
4. A 40-year-old woman presents with urinary frequency, d. Lymphoma
urgency and episodes of urge incontinence. She had e. Priapism
a transient episode of unilateral blindness 2 years prior f. Prostate cancer
to these new symptoms. g. Pyelonephritis
5. A 40-year-old man has a long history of intermittent h. Seminoma of testis
low back pain but develops acute onset of difficulty i. Teratoma of testis
passing urine. On examination he has a palpable j. Testicular torsion
bladder and an area of saddle anaesthesia. k. Ureteric colic
l. Urinary tract infection
Urological investigations
Each answer can be used once, more than once or not For each scenario below, choose the most likely
at all. corresponding option from the list given above.
a. Urine dipstick 1. A 19-year-old man presents with a firm, nontender
b. Cystoscopy lump in his right testicle.
c. Serum PSA 2. A 23-year-old woman presents with right-sided
d. Renal ultrasound scan loin pain which radiates to the groin. The pain
e. Abdominal CT scan comes and goes in waves. She also complains of
f. Serum urea and creatinine frequency of micturition. Dipstick urine test reveals
g. Urodynamic studies (cystometrogram) 4+ blood. The only abnormality on examination is
h. Abdominal X-ray of kidneys, ureters and tenderness in the right loin.
bladder (KUB) 3. An 88-year-old man presents via his GP with
i. Intravenous urography suprapubic pain and a history of frequency and
j. Renal aortography passing only very small amounts of urine for
1 week. He has had long-standing problems
For each scenario below, choose the most likely with nocturia and poor stream. He has a tender
corresponding option from the list given above. palpable mass arising from the pelvis. His PSA is
1. A patient presenting with dysuria and frequency. < 5ng/mL (NR 0–4ng/mL).
Routine observations reveal a temperature of 4. A 52-year-old man presents with a 3-day history
38.6°C. of dysuria and frequency and a gradual onset
2. A 25-year-old long-distance runner presenting of left testicular pain and swelling. Dipstick urine
with severe right-sided loin pain and vomiting. reveals 3+ blood, 2+ leukocytes and nitrites.

123 
This page intentionally left blank
Breast surgery 16
the production of a straw-coloured fluid. What is the
SINGLE BEST ANSWER (SBA) most likely diagnosis?
QUESTIONS a. Localised fibroadenosis
b. Fibroadenoma
1. A 45-year-old woman presents to her general practitioner c. Fat necrosis
(GP) with a unilateral erythematous, itchy rash around her d. Carcinoma
nipple. On examination her nipple is inverted. Which of e. Cyst
the following is the most likely diagnosis?
a. Ductal carcinoma in-situ 6. A 34-year-old woman smoker presents with a
b. Infective mastitis green-coloured discharge from both her nipples. On
c. Lobular carcinoma in-situ examination you note evidence of nipple retraction and
d. Medullary carcinoma breast lumpiness. What is the most likely diagnosis?
e. Nipple eczema a. Mammary duct ectasia
b. Duct papilloma
2. A 35-year-old woman presents to her GP complaining c. Galactorrhoea
of a lump in the upper outer quadrant of her right d. Duct carcinoma
breast. She comments the lump becomes particularly e. Paget disease
tender before the start of her menstrual cycle. What is
the most likely diagnosis? 7. A middle-aged woman presents with a serous discharge
a. Localised fibroadenosis from her right nipple. On examination the breast appears
b. Fibroadenoma normal but on deep palpation the discharge is produced.
c. Cyst What is the most likely diagnosis?
d. Fat necrosis a. Mammary duct ectasia
e. Carcinoma b. Duct papilloma
c. Galactorrhoea
3. A 31-year-old woman presents complaining of a lump d. Duct carcinoma
in her left breast. On examination you note the lump is e. Paget disease
nontender, rubbery and fairly mobile. What is the most
likely diagnosis? 8. A 57-year-old woman has been recently diagnosed
a. Localised fibroadenosis with breast cancer. Her GP comments she was at
b. Cyst high risk due to her strong family history of breast
c. Fat necrosis cancer. The GP explains that particular chromosomes
d. Fibroadenoma are linked to the condition. Which chromosome is
e. Carcinoma most likely to be associated with breast cancer?
a. Chromosome 14
4. A 67-year-old woman had her breast cancer removed b. Chromosome 15
with breast conserving surgery 2 weeks ago. c. Chromosome 16
Subsequently, she notices a painless lump beneath d. Chromosome 17
the incision. On examination she is apyrexial, the lump e. Chromosome 18
is nontender, firm and well circumscribed. What is the
most likely diagnosis? 9. A 45-year-old woman is concerned about developing
a. Abscess breast cancer. She comes to see you asking when
b. Cystic disease she will be called for a ‘screening mammogram’. At
c. Fat necrosis what ages do women in the UK undergo screening as
d. Fibroadenoma part of a National Programme?
e. Recurrence of the cancer a. 50–70
b. 45–59
5. A 35-year-old woman presents with a lump on her c. 55–69
right breast. On examination you note the lump is d. 46–60
tense and mobile. On fine-needle aspiration you note e. 56–70

125 
Extended-matching questions (EMQs)

10. A 54-year-old woman presents with a 5 cm lump 14. A 39-year-old woman is diagnosed with left-sided
in her right breast. On examination you note the breast cancer. The tumour is confirmed as an
presence of fixed, right-sided axillary nodes. There oestrogen-receptor-negative, well-differentiated
are no palpable supraclavicular or infraclavicular adenocarcinoma, approximately 6.5 cm in size.
nodes. You refer her for tissue biopsies. The There is involvement of ipsilateral axillary nodes but
histology report confirms the presence of a no evidence of distant metastasis. She is referred to
moderately differentiated adenocarcinoma with no a breast surgeon who explains possible treatment
evidence of distant metastasis. According to the options. What is the next most appropriate step in
International Union against Cancer, what is the most management?
likely staging classification of this presentation? a. Neoadjuvant chemotherapy and surgical
a. Stage 1 excision with axillary clearance
b. Stage 2 b. Primary surgical excision of tumour without
c. Stage 3 axillary dissection
d. Stage 4 c. Radiotherapy
e. Stage 5 d. Tamoxifen
e. Herceptin
11. A 52-year-old woman presents with a lump
approximately 4 cm in size in her right breast. On 15. A middle-aged woman presents with a scaly red
examination you note no palpable nodes in her coloured rash affecting her right nipple and areola.
axilla, supraclavicular or infraclavicular region. Wide She comments it is sore and has a tendency to
local excision and axillary node clearance confirm bleed. What is the most likely diagnosis?
the presence of a well-differentiated cancer with no a. Mammary duct ectasia
evidence of lymph node involvement. According to b. Duct papilloma
the International Union against Cancer, what is the c. Galactorrhoea
most likely 5-year survival rate for this presentation? d. Duct carcinoma
a. 84% e. Paget disease
b. 71%
c. 59% 16. A 52-year-old woman presents with a fairly large
d. 48% painless lump in her left breast. On examination you
e. 18% note it appears fleshy and has a characteristic tear
drop shaped appearance. What is the most likely
12. A 64-year-old woman presents with a lump in her diagnosis?
left breast. On examination the lump is nontender a. Fibroadenoma
and painless. In addition, you note dimpling of the b. Duct papilloma
skin and retraction of her left nipple. You refer her for c. Phyllodes tumour
urgent tissue biopsies, which confirm the presence d. Duct carcinoma
of an adenocarcinoma with associated fibrosis. e. Paget disease
Having explained the diagnosis, you explain that this
tumour is the most common type of tumour to affect
women. What is the most likely diagnosis? EXTENDED-MATCHING
a. Ductal
b. Lobular
QUESTIONS (EMQs)
c. Medullary
d. Colloid Management of breast cancer
e. Sarcoma Each answer can be used once, more than once or not
at all.
13. A 28-year-old woman presents with a lump on her a. Cyclophosphamide
right breast. On examination you note the lump b. Trastuzumab (Herceptin)
is tense, discrete and mobile. You suspect the c. Tamoxifen
possibility of a breast cyst. Which investigation is the d. Anastrozole
most appropriate in this scenario? e. Doxorubicin
a. Mammography f. Raloxifene
b. Computed tomography chest scan g. Vincristine
c. Magnetic resonance imaging chest scan h. Dexamethasone
d. Ultrasound i. Paclitaxel
e. Chest X-ray j. Faslodex

126
Breast surgery 16

For each scenario below, choose the most likely i. Mastitis


corresponding option from the list given above. j. Papilloma
1. A therapeutic agent highly recommended in the
treatment of lymph-node-positive HER2-positive For each scenario below, choose the most likely
breast cancer. corresponding option from the list given above.
2. A selective oestrogen-receptor modulator drug 1. A 19-year-old woman presents with a highly
useful in the treatment of breast cancer but mobile lump in her breast. Fine-needle aspiration
associated with risks of endometrial cancer and shows no malignancy.
pulmonary embolism. 2. A 28-year-old woman, four weeks postpartum,
3. An aromatase inhibitor useful in the treatment of presents to her GP with a tender lump in the
early breast cancer in postmenopausal women. upper outer quadrant of her left breast. On
4. A therapeutic agent useful in the treatment of examination it is ill-defined, erythematous and hot
breast cancer but known to cause left ventricular to touch.
failure and pulmonary toxicity. 3. A 38-year-old very anxious woman presents
5. A therapeutic agent useful in the treatment of with a hard, irregular lump in her breast. On
breast cancer but known to cause bladder cancer. examination it is craggy and well-defined.
Fine-needle aspiration shows no evidence of
malignancy. On further questioning, she reveals
Breast lumps that she was involved in a car crash 6 months
Each answer can be used once, more than once or not previously.
at all. 4. A 42-year-old woman presents with a yellow-
a. Breast abscess brown discharge from her nipples. On examination
b. Cyst her discharge appears to be coming from multiple
c. Duct ectasia ducts. Examination is otherwise normal and
d. Fat necrosis mammography is unremarkable.
e. Fibroadenoma 5. A 32-year-old woman presents with a
f. Fibrocystic change lump in her breast which is painless. On
g. Galactorrhoea examination it is well demarcated, fluctuant and
h. Lipoma transilluminable.

127 
This page intentionally left blank
MEDICINE ANSWERS
Chapter 1

Cardiology . . . . . . . . . . . . . . . . . . . . . . 131
Chapter 2

Gastroenterology . . . . . . . . . . . . . . . . . . . 139
Chapter 3

Respiratory . . . . . . . . . . . . . . . . . . . . . . 147
Chapter 4

Renal . . . . . . . . . . . . . . . . . . . . . . . . . 153
Chapter 5

Neurology . . . . . . . . . . . . . . . . . . . . . . 157
Chapter 6
Rheumatology . . . . . . . . . . . . . . . . . . . . 165
Chapter 7

Endocrine and diabetes . . . . . . . . . . . . . . . 169


Chapter 8

Haematology . . . . . . . . . . . . . . . . . . . . . 177
This page intentionally left blank
Cardiology 1
SVT; adenosine is used instead. IV lignocaine can be
SBA ANSWERS given for VT with pulse if bisoprolol is contraindicated.

Question 1 Question 6
D. Hypertrophic cardiomyopathy (HCM) is the most
C. Narrow complex regular tachycardia is invariably
common cause of sudden cardiac death in young
SVT. Slowing down the rhythm with IV adenosine
athletes in the UK. It is autosomal dominant with
may break the re-entrant circuit and back into
variable penetrance. Long QT syndrome is a cause
normal rhythm. If there is response to adenosine,
of sudden death in the young athlete, but is less
then the rhythm may slow down if it is atrial flutter
common. Wolff–Parkinson–White is caused by
(to reveal sawtooth pattern ECG). Lack of response
the presence of an abnormal accessory electrical
to adenosine means underlying rhythm is likely sinus
conduction pathway between the atria and the
tachycardia or paroxysmal atrial tachycardia.
ventricles. Electrical signals travelling down this
abnormal pathway (known as the bundle of Kent)
may stimulate the ventricles to contract prematurely,
Question 7
B. First-degree heart block is the result of delayed
resulting in a unique type of supraventricular
atrioventricular conduction and is characterized by a
tachycardia (SVT) (referred to as an atrioventricular
PR interval greater than 0.22 seconds.
(AV) reciprocating tachycardia) but is less common
than HCM.
Question 8
E. Digoxin, with warfarin. Rate control is good for elderly
Question 2 patients, but this patient has a CHADSVASC risk
D. Treatment with spironolactone reduces mortality
score that needs anticoagulation too. The elderly
along with beta-blockers, ACE I, angiotensin receptor
patient is asymptomatic and has enlarged left
blockers (ARBs) and other aldosterone antagonists.
atrium, which means that he has a smaller chance to
Beta-blockers and ACE I reduce the degree
cardiovert out of.
of neurohormonal activation in heart failure.
Beta-blockers are contraindicated in acute heart
failure (HF), but are often used during stable period of
Question 9
D. Second-degree block is characterized by an
HF to modulate the sympathetic system.
intermittent failure or interruption of AV conduction but
there is no increasing PR interval as is the case with
Question 3 Mobitz type 1 block.
A. Sick sinus syndrome is characterized by a time interval
of greater than 2 seconds between consecutive P Question 10
waves. The very slow P rate and relatively stable D. Treatment with statins as primary prevention should
ventricular rate suggest a sinoatrial problem rather be offered to those at the highest risk of developing
than an AV nodal problem. Thromboembolism is a coronary heart disease. This patient is at risk now
feature and patients often require anticoagulation. because of this mild elevation in lipid levels, borderline
blood pressure and overweight. Alcohol consumption
Question 4 is associated with an increase in high-density
C. Also known as Wenckebach phenomenon. This lipoprotein cholesterol levels of as much as 9 to
arrhythmia is characterized by an increasing PR interval 13 mg/dL (0.23 to 0.34 mmol/L) when nondrinkers are
until the P wave no longer conducts to the ventricles. compared with the highest consumers. The patient’s
hyperlipidaemia should be controlled by lifestyle
Question 5 modifications in the beginning for 4 to 6 months.
D. Adenosine should be given to stop SVT or to
slow down the conduction to reveal underlying Question 11
rhythm. IV amiodarone can often be given for any E. Typical symptoms include blackouts and dizzy
supraventricular or ventricular tachycardias upon spells. Pacemaker insertion is the mainstay form of
cardiology advice. However, it is not used to terminate treatment.

131 
SBA answers

Question 12 large area of myocardium or hemodynamic instability if


B. If someone is in ventricular fibrillation (on monitor PCI is not available.
or defib pads), then advanced life support (ALS)
guidelines say that you need to shock in the first Question 19
instance. Continue CPR until you establish the charge C. The diagnosis is an SVT. Carotid sinus massage is
needed. Calcium chloride/gluconate and sodium helpful in the acute situation. Other manoeuvres may
bicarbonate IV administration are for hyperkalaemia. include the Valsalva manoeuvre. Adenosine is used
Carotid artery compression has no place in the in the treatment of SVT but is contraindicated in this
ALSalgorithm. patient because of asthma.

Question 13 Question 20
A. This is atrial flutter with 2:1 block and characterized by A. The diagnosis is pulseless electrical activity due
sawtooth flutter waves on the ECG. to hypovolaemia. This is a nonshockable rhythm.
Administration of colloid, saline or blood would be the
Question 14 next most appropriate step in management, followed
D. Selective coronary angiography is the definitive by adrenaline 1 mg IV.
diagnostic test for evaluating the anatomic extent and
severity of coronary artery disease. More importantly, Question 21
you can treat it with balloon angioplasty to treat the B. The main features of cholesterol embolism include
lesion as well! eosinophilia, purpura, livedo reticularis and renal
injury. Usually occur in arteriopaths, or as a
Question 15 complication of coronary angiography, or following
E. The diagnosis is atrial flutter of recent onset and the commencement of anticoagulants/ thrombolytics.
treated with electrical cardioversion in the first Treatment is supportive.
instance.
Question 22
Question 16 B. Serial assessments of serum digoxin levels are not
E. Gastrointestinal distress such as bloating or necessary in most patients. The radioimmunoassay was
indigestion, along with muscular imbalances and developed to assist in the evaluation of toxicity, rather
poor posture, can also irritate the vagus nerve than the efficacy of the drug. There appears to be little
causing palpitations, but this patient is not relating her relationship between serum digoxin concentration and
symptoms to meals or to any intestinal problems. the drug’s therapeutic effect. The dosage of digoxin
should be 125–250 mcg daily in the majority of patients.
Question 17 Higher doses (e.g., 375–500 mcg) are rarely needed,
A. The diagnosis is AF. Current National Institute for especially as initial therapy for chronic heart failure.
Health and Clinical Excellence (NICE) guidelines
state that beta-blockers are the initial choice of Question 23
management for rate control. A. Typical features of mitral stenosis. Additional signs
include a malar flush and tapping apex beat.
Question 18
C. Thrombolytic therapy is indicated in patients with Question 24
evidence of ST-segment elevation myocardial C. This patient had a syncopal episode, which can
infarction (STEMI) or new left bundle-branch block be cardiogenic or neurogenic in origin. After the
presenting within 12 hours of the onset of symptoms, history review and physical examination, blood tests
if there are no contraindications to fibrinolysis. are usually sent to rule out anaemia, infections,
Thrombolysis remains the treatment of choice hypocalcaemia or hypomagnesemia and an ECG
in STEMI when primary percutaneous coronary must be performed. An ECG may show evidence of
intervention (PCI) cannot be performed within 90 cardiac abnormalities such as Wolf–Parkinson–White
minutes. The benefits of fibrinolytic therapy are well syndrome, idiopathic hypertrophic subaortic stenosis
established during the initial 12 hours after symptom or congenital prolonged QT syndrome.
onset. The new guidelines mention that you should
consider administration of a fibrinolytic agent in Question 25
symptomatic patients presenting more than 12 to 24 B. Typical features of mitral regurgitation. Additional signs
hours after symptom onset with STEMI affecting a include a displaced apex beat.

132
Cardiology 1

Question 26 Question 34
D. Thiazide diuretics such as bendrofluazide can cause E. The diagnosis is mitral stenosis. A widened angle of
hypokalaemia. Afro-Caribbeans should have calcium carina (representing enlarged left atrium) on the chest
channel blocker (CCB) as first-line antihypertensives. X-ray can be seen.
In cases of renal artery stenosis, ACE I is
contraindicated as it may result in acute kidney injury. Question 35
CCBs cause ankle swelling as a side effect. A. When patients present with severely elevated blood
pressure (systolic blood pressure of 180 mmHg or
Question 27 greater, or diastolic blood pressure of 110 mmHg or
C. NICE no longer recommends aspirin as monotherapy greater), doctors need to differentiate hypertensive
to prevent stroke in people with AF (small benefit is emergency from severely elevated blood pressure
offset by risk of bleeding). If the CHADSVASC score without signs or symptoms of end-organ damage
is 1 or greater, then NICE recommends offering (severe asymptomatic hypertension). Most patients
anticoagulation. In this case, we have no documented who are asymptomatic but have poorly controlled
AF on the ECG, and we just have one episode of hypertension do not have acute end-organ damage
self-reported palpitations. He should have a longer and, therefore, do not require immediate workup or
Holter monitor to see whether this is paroxysmal AF. treatment. Rapidly lowering blood pressure in A&E is
His CHADSVASC score is 0 and does not warrant unnecessary in asymptomatic patients and may be
anticoagulation currently. harmful (can cause watershed infarcts).

Question 28 Question 36
C. Additional signs would include a slow rising carotid B. The echocardiogram is the most useful investigation and
pulse. may show regional wall abnormalities with ischaemia,
valve abnormalities and pericardial thickening. A CXR
Question 29 will usually, but not always, show typical features of
E. This is a hypertensive ‘urgency’ with minor end-organ heart failure but will not usually demonstrate the cause.
damage (without papilloedema). The hypertension is
longstanding, given by the ECG changes showing left Question 37
ventricular hypertrophy. This will require a reduction in C. The patient is suffering from left-sided heart failure.
blood pressure (over hours) using oral agents with the CXR features include fluid in the right horizontal
goal of normal blood pressure within 24 to 48 hours. fissure, seen as a white line running over the anterior
If there was haematuria (renal damage), papilloedema end of the fourth rib. There is no horizontal fissure in
and mental changes, then IV therapy would need to the left lung as there are only two lobes.
be commenced instead.
Question 38
Question 30 A. Ischaemic heart disease is the most common cause
D. Additional signs would include a collapsing pulse, a of heart failure in the Western world.
displaced apex beat and a mid-diastolic murmur over
the cardiac apex. Question 39
A. ACE I should be first-line therapy for someone with
Question 31 congestive heart failure. Propranolol is a rather dirty
C. Typical features of tricuspid regurgitation. Abdominal beta-blocker, and bisoprolol/metoprolol can be added
examination is likely to reveal an enlarged pulsatile liver. once patient stabilized on ACE I. Verapamil (CCB) is
not first-line to slow down the heart rate, beta-blockers
Question 32 are. Disopyramide is an antiarrhythmic medication
E. Enalapril is an ACE inhibitor. ACE inhibitors delay used in the treatment of ventricular tachycardia.
progression of diabetic kidney disease and are more Phosphodiesterase inhibitors play no role in congestive
effective than other medications in delaying the onset heart failure. However, hydralazine/nitrates can be
of kidney failure in patients who have hypertension considered if patients cannot tolerate ACE I/ARB.
and type 1 diabetes with macroalbuminuria.
Question 40
Question 33 E. The diagnosis is heart failure. Current guidelines state
B. An echo is the first-line investigation in the diagnosis that ACE inhibitors are typically the first-line treatment
of cardiac murmurs. in heart failure. However, as this patient is severely

133 
SBA answers

fluid overloaded, diuretics such as furosemide would elevates the pressure, which prevents blood outflow
be the first-line of treatment. from neck veins into the mediastinum. Another feature
is pulsus paradoxus, which is a blood pressure drop
Question 41 greater than 12 mmHg during inspiration. Normally,
B. This class of drug is known to cause hyperkalaemia. inspiration leads to a systolic blood pressure drop of
up to 10 mmHg. Another commonly tested condition
that can cause pulsus paradoxus is asthma or chronic
Question 42 obstructive pulmonary disease.
D. The diagnosis is pulmonary oedema. Furosemide
is the most appropriate treatment. IV GTN (glyceryl
Question 50
trinitrate) may be added in if he does not respond to
B. The patient is experiencing angina, which is best
diuretics to reduce the cardiac preload.
treated by GTN in the first instance.

Question 43 Question 51
E. This patient exhibits atypical (noncardiac) chest pain B. Q waves indicate old infarcts and II, III and aVF indicate
and has no risk factors for coronary artery disease. that it involves the right coronary artery, an inferior MI.
Because the likelihood of coronary disease is very low,
stress testing is not indicated.
Question 52
E. Typical ECG changes of a posterior infarct.
Question 44
E. Low intake of folic acid is a risk factor for ischaemic
heart disease.
Question 53
B. At present, the ICD is the most effective modality
for preventing sudden death in high-risk patients
Question 45 with asymptomatic HCM. Because of its effects on
E. A patient with positional chest pain (occurs in decreasing dysrhythmias, amiodarone may decrease
conjunction with a viral infection) along with ECG the risk of sudden death; however, the ICD is the most
changes (widespread ST changes), and a pericardial effective modality for the prevention of sudden death.
rub are consistent with a diagnosis of acute pericarditis. Medications such as verapamil, beta-blockers and
diltiazem are used for symptom management, but do
Question 46 not decrease the risk of sudden death.
C. Such a syndrome is thought to be due in part to an
abnormal coronary microcirculation. Angiograms are Question 54
always normal. A. Troponin is the most appropriate investigation in
individuals with chest pain as it is highly specific and
Question 47 sensitive for cardiac damage.
C. Hypertension may be primary, which may develop
as a result of environmental or genetic causes, or Question 55
secondary, which has multiple aetiologies, including B. ST elevation is an indication for PCI; however, if there
renal, vascular and endocrine causes. Primary or are no PCI centres within 90 minutes of symptoms,
essential hypertension (undetermined cause) accounts then thrombolysis is second-line. Q waves suggest an
for 90% to 95% of adult cases and a small percentage old infarct. ST depression (V1–V3) might indicate acute
of patients (2%–10%) have a secondary cause. true posterior MI; however, widespread STD is not an
acute indication as it is likely to represent digitalis effect
Question 48 (it would be highly unlikely that an obstruction would
E. Enoxaparin 1 mg/kg twice daily is the recommended affect so many coronary vessels globally).
dose in patients with evidence of acute coronary
syndrome Question 56
D. Aspirin 300 mg is the recommended immediate
Question 49 management in a suspected MI.
D. This patient presents with a stab wound to the chest
and highly suspicious for penetrating cardiac trauma Question 57
and cardiac tamponade. Always consider cardiac A. According to the Duke criteria, the presence of a new
tamponade in a patient who presents with hypotension murmur (or changing characteristic of an existing one)
and silent heart sounds. Other features are neck vein is a major criterion for infective endocarditis. Along
distension – build-up of blood in the pericardium with positive blood cultures, it would be diagnostic.

134
Cardiology 1

Question 58 Question 69
D. Pericarditis is an early complication of an MI and D. Symptoms of aortic stenosis are syncope, angina
typically occurs within the first 2–4 days. and dyspnoea. Usually presents in elderly patient.
ECG shows left ventricular hypertrophy. Once you
Question 59 suspect aortic stenosis and patient has symptoms
B. Congestive heart failure can cause mild jaundice due of angina, do echocardiography before ordering
to hepatic congestion from excess fluid with hepatic stress test as there is risk of sudden death. If
venous system (deranged liver function tests will stenosis is severe, take patient to cardiac cath to
normalize once patient is off-loaded and back to dry assess concomitant coronary artery disease and if
body weight). Consider also eventual cardioversion for planned for surgery, then both valvular surgery and
AF, which will increase ejection fraction and improve coronary artery bypass grafting can be done at the
symptoms. same time.

Question 60 Question 70
D. This is the most likely organism to cause infective B. This patient has developed a pericardial effusion. Echo
endocarditis in intravenous drug abusers and those is diagnostic.
with central venous lines.
Question 71
C. According to current guidelines, ACE inhibitors are
Question 61
the first-line management of hypertension in diabetic
A. This is the most likely offending organism associated
patients.
with dental diseases or procedures.

Question 72
Question 62 E. The Duke diagnostic classification for IE divides signs
E. Whilst all of the conditions give the patient a higher
and symptoms into major and minor criteria. IE is
risk for infective endocarditis, NICE guidelines (2015)
diagnosed if patients have two major criteria, or one
do not recommend prophylaxis antibiotics.
major and three minor criteria or five minor criteria.
Microbiological evidence that is considered as minor
Question 63 criteria includes the following: positive blood culture
A. A splenic abscess is more likely to be due to that does not meet the requirements of a positive
embolization of vegetations of fibrin and platelets. blood culture as mentioned in the major criteria,
or serological evidence of active infection with an
Question 64 organism consistent with IE.
E. The most likely diagnosis is infective endocarditis
typically confirmed by three sets of blood cultures in Question 73
the first instance. C. Additional features include ‘cotton wool’ exudates.

Question 65 Question 74
A. Classic features of a dilated cardiomyopathy. D. Infective endocarditis is most commonly due to
Streptococcus viridans, which is a normal flora of the
Question 66 mouth and thus frequently enters the bloodstream
C. Patients with Marfan syndrome often develop mitral after dental procedures. Preexisting valvular heart
valve prolapse or ascending aortic aneurysms (which disease increases the risk of endocarditis and a new
can lead to fatal aortic dissections). As the ascending regurgitation murmur should raise suspicion as the
aorta dilates, the aortic valve annulus also stretches pathogen can destroy valve leaflets.
and may lead to regurgitation.
Question 75
Question 67 C. Many medications interfere with the absorption,
B. Additional features include a jerky carotid pulse and metabolism and pharmacological action of ACE
an ejection systolic murmur. inhibitors. Of the choices given, the one most likely
to do this is ibuprofen, a commonly used NSAID.
Question 68 NSAIDs may attenuate the haemodynamic actions of
A. Classic presentation of pericarditis. Management ACE inhibitors. NSAIDs reduce renal excretion of ACE
is usually with nonsteroidal antiinflammatory drugs inhibitors, with a corresponding increase in circulating
(NSAIDs) or corticosteroids. drug concentrations.

135 
EMQ answers

Question 76 2. E  ACE inhibitors such as captopril can affect renal


E. Enalapril is an ACE inhibitor. ACE inhibitors function in patients with renovascular disease. This is
delay progression of diabetic kidney disease due to elimination of efferent vasoconstriction which
and are more effective than other medications leads to a decrease in the glomerular filtration rate.
in delaying the onset of kidney failure in patients 3. D  Typical side effects of CCBs.
who have hypertension and type 1 diabetes with 4. C  Aspirin is regarded as the gold standard in the
macroalbuminuria. CCBs reduce cardiovascular management of acute coronary syndrome.
events in patients with diabetes and hypertension; 5. A  This is due to bronchoconstrictive and
however, they may be inferior to other agents in vasoconstrictive effects.
some cardiovascular outcomes. Dihydropyridine
and nondihydropyridine CCBs are less effective Clinical features of cardiac disease
than ACE inhibitors and ARBs in slowing 1. D  Classic features of mitral regurgitation.
progression of diabetic kidney disease. Because 2. E  Additional features would include Roth spots and
CCBs may be inferior in some patient-oriented Osler nodes.
outcomes in those with diabetes, they should be 3. F  The pink-coloured rings are characteristic of
reserved for patients who cannot tolerate preferred erythema marginatum seen typically in rheumatic
agents or those who need additional agents to fever. One would also expect to suffer from arthritis
achieve target blood pressure. and cardiac murmurs.
4. A  Classic presentation of aortic stenosis.
EMQ ANSWERS 5. I  Additional features include chest pain and
dyspnoea.
Cardiac murmurs and added sounds
1. A  Aortic regurgitation. The early diastolic murmur
is best heard at the left sternal edge, with the
Murmurs
1. H  Tricuspid regurgitation. He may have caught
patient leaning forward and the breath held in
schistosomiasis in Egypt, affecting the tricuspid
expiration.
valve.
2. E  Mitral stenosis. Acquired mitral stenosis is usually
2. F  Mitral regurgitation. The apex is displaced.
due to rheumatic fever, now a very rare disease
3. J  Ventricular septal defect. Down syndrome with
in the West, but still common in the developing
congenital heart defects.
world.
4. A  Aortic regurgitation. Murmurs can be discriminated
3. B  Aortic stenosis. Patients may also present with
by the pattern of murmur, where it is loudest, and
angina, even in the absence of severe coronary
any features that make it louder (in this case sitting
arterial disease, due to a combination of increased
forward).
oxygen requirements and fixed flow obstruction.
5. E  Graham Steell murmur. An uncommon murmur,
4. D  Mitral regurgitation. This often occurs because of
typically associated with pulmonary regurgitation.
papillary muscle or chorda rupture.
5. I  Ventricular septal defect. A more common
presentation is an asymptomatic ejection systolic Symptoms of heart failure
murmur detected on routine examination. 1. G  Pulmonary fibrosis. This patient has been treated
with diuretics and is hypovolaemic. He may
Chest pain have had pulmonary oedema; however, this has
1. A  Typical presentation of angina pectoris. resolved. Pulmonary fibrosis may give very similar
2. C  Classic features of a myocardial infarction. auscultatory findings to pulmonary oedema.
Additional features would include sweating, 2. E  Paroxysmal nocturnal dyspnoea. These are
breathlessness, nausea and vomiting. episodes of acute breathlessness that usually
3. D  There may also be unequal blood pressure in both occur at night and may result in the patient
arms. getting out of bed and pulling the window
4. H  Classic description of costochondritis. open. It happens in left heart failure due to fluid
5. B  Pain may also radiate to the neck and shoulders. redistribution, but there are other causes.
3. D  Orthopnoea. Breathlessness occurring on lying flat;
Drugs it is often described in terms of how many pillows
1. E  Captopril is an ACE inhibitor, which can lead to a a patient requires to get to sleep (e.g., ‘four-pillow
cough due to inhibition of bradykinin metabolism. orthopnoea’). It is seen in left heart failure.
Other side effects include renal failure and 4. B  Cough. ACE inhibitors may cause a chronic cough
angioedema. that can persist for up to 1 month after cessation

136
Cardiology 1

of treatment; this is likely due to decreased 2. B  Aortic stenosis. This gentleman has all three of the
bradykinin breakdown. classical symptoms of aortic stenosis. He should
5. C  Hepatomegaly. Hepatomegaly can be seen in right be assessed for valve surgery, as the presence
heart failure due to hepatic venous congestion. of these symptoms increases the risk of sudden
This can lead to a macroscopic ‘nutmeg’ cardiac death in these patients.
appearance. 3. G  Fibrinous pericarditis. This is suggested by the
speed of postmyocardial infarction onset, relief by
Electrocardiograph findings NSAIDs and classical ST changes.
1. G  Posterior myocardial infarction. Because of 4. F  Dilated cardiomyopathy. This is associated with
lead placement, posterior myocardial infarction excess alcohol consumption. The clinical findings
changes are the reverse of those elsewhere: are classic with the mitral murmur caused by
dominant R waves equivalent to Q waves and ST dilatation of the annulus.
depression instead of elevation. 5. E  Cor pulmonale. This is suggested by the ankle
2. F  Pericarditis. These changes may be less marked swelling (sign of right heart failure) on top of
as T-wave inversion. The widespread nature and chronic lung disease.
saddle shape allow distinction from myocardial
infarction. Chest pain
3. E  Inferior myocardial infarction. 1. B  Pulmonary embolus. Sinus tachycardia,
4. D  Digitalis effect. Complete (also known as third- right bundle-branch block and pleuritic
degree) heart block: the ventricular escape rate of chest pains should point you towards pulmonary
45 beats/min, broad complex QRS and presence embolus.
of P waves (which will be at a rate of 70 beats/min) 2. F  Gastro-oesophageal reflux disease (GORD)
confirm this diagnosis. (secondary to NSAID). Taking NSAIDs and
5. C  Atrial flutter with 2:1 block. Any rhythm at 150 having large body mass index are risk factors for
beats/min should prompt a search for ‘sawtooth’ GORD.
P waves to exclude flutter. 3. C  Pericarditis.
4. E  Rib fracture (with associated pneumothorax). CXR
Heart disease shows pneumothorax, which can give a pleuritic
1. H  Infective endocarditis. Known valve disease plus pain.
a minor procedure leading to fever should be 5. A  Myocardial infarct (lateral). Elevated troponin and
considered infective endocarditis until proven ST elevation point to lateral MI.
otherwise.

137 
This page intentionally left blank
Gastroenterology 2
SBA ANSWERS Question 5
B. Vitamin B1. Vitamin B1 is also known as thiamine.
Question 1 Thiamine deficiency is very common in people who
D. Pancreatic cancer. The most likely diagnosis here drink excess alcohol due to several factors. These
is carcinoma of the head of the pancreas. The patients generally have a poor oral diet, and thiamine
tumour obstructs the common bile duct causing is also a coenzyme in alcohol metabolism. It is
jaundice and dark urine because of conjugated essential that it is given quickly in patients who have
hyperbilirubinaemia. The faeces are pale due to a decompensated liver failure as thiamine deficiency can
lack of stercobilinogen. Painless jaundice is typically lead to Wernicke encephalopathy. It is always given
the presenting feature of carcinoma of the head of intravenously in the acute presentation as it is poorly
the pancreas, along with weight loss. Hepatocellular absorbed orally in people who are alcohol dependent.
carcinoma would not cause the change in stool
colour. Question 6
D. Oesophageal candidiasis. Patients with diabetes are
Question 2 more prone to getting candidiasis as high sugar levels
A. Columnar cells. Barrett oesophagus is when lead to better conditions for the yeast to grow. It is
prolonged injury (due to acid reflux) results in also only a short 10-day history, which also favours
the normal squamous epithelium of the lower candidiasis over some of the more chronic conditions
oesophagus being replaced by columnar epithelium. listed.
This metaplastic change may be followed by
dysplastic change, which predisposes to malignant Question 7
transformation. The majority of patients present with a A. Oesophageal varices. The examination findings here
long history of reflux. show evidence of liver disease. This helps point to
the diagnosis of oesophageal varices as the cause
Question 3 of the upper GI bleed. In patients with cirrhosis of
A. Primary biliary cirrhosis. PBC is an autoimmune the liver and portal hypertension, the raised pressure
disorder and predominantly affects women, especially in the portal system causes the site of a connection
those aged 40–50. All patients with PBC have between the systemic and portal venous systems to
antimitochondrial autoantibodies and this, together open up and enlarge. The enlarged veins protrude into
with raised serum alkaline phosphatase, is diagnostic the lumen of the lower oesophagus and may burst,
for the condition. Patients often present with pruritus resulting in haematemesis. A man with this history
and fatigue, and jaundice can be present at a later could also be prone to a Mallory–Weiss tear without
stage; some patients have xanthelasma. varices but there would usually be a history of retching
or vomiting before blood appears and it would rarely
cause large-volume haematemesis.
Question 4
E. Haemochromatosis. Primary haemochromatosis is
an autosomal recessive disorder characterized by the Question 8
absorption of too much iron, which then accumulates B. Hepatic encephalopathy. Hepatic encephalopathy is
(as haemosiderin) in the liver, pancreas, heart, pituitary a neurological disorder caused by metabolic failure of
and joints. The most common presentation is an the hepatocytes and the shunting of blood around the
incidental finding of abnormal liver enzymes or raised liver (due to cirrhosis or after portocaval anastomosis).
ferritin level. Symptoms are rare in women of child- It may occur in both chronic and acute liver failure,
bearing age as menstrual losses and pregnancy and results in the exposure of the brain to abnormal
compensate for the excess iron absorption. metabolites, causing oedema and astrocyte changes.
Symptomatic presentation in males is usually in the Symptoms include disturbances in consciousness
fourth or fifth decade and can be obscure, such (ranging from confusion to coma and death), asterixis
as loss of libido and hypogonadism, secondary to (coarse flap of outstretched hand) and fluctuating
dysfunction of the pituitary gland or can include neurological signs including seizures, muscular rigidity
diabetes mellitus or arthritis. and hyperreflexia.

139 
SBA answers

Question 9 Question 14
C. Gilbert syndrome. The isolated rise in the bilirubin E. Mallory–Weiss tear. A Mallory–Weiss tear is a
within the LFTs is suggestive of Gilbert syndrome. This tear at the gastro-oesophageal junction. It is
is a congenital disorder caused by patients having caused by prolonged retching or coughing and
a reduced amount of the enzyme UDP-glucuronyl a sudden increase in intraabdominal pressure. It
transferase (UGT-1), which conjugates bilirubin with is most common in alcoholics and presents as
glucuronic acid. There is only a slight increase in serum haematemesis. All the other answers can cause an
bilirubin (unconjugated), often after prolonged starvation upper GI bleed, but the history given of prolonged
or intercurrent illness. The syndrome is asymptomatic, vomiting preceding haematemesis is suggestive of a
although some patients complain of fatigue. Mallory–Weiss tear.

Question 10 Question 15
A. Giardiasis. Giardiasis, prevalent in the tropics, is an C. Viral illness. The raised WCC and raised lymphocyte
important cause of traveller’s diarrhoea. It is caused count is suggestive of an acute viral illness, whilst
by the flagellate protozoan Giardia lamblia, which lives the rise in the LFTs, in particular the transaminases,
in the duodenum and jejunum and is transmitted by suggests a hepatitis type of picture. There are several
the faecal–oral route. Patients may be asymptomatic, causes of a viral hepatitis, which include Epstein–
or alternatively, symptoms of diarrhoea, malabsorption Barr virus and hepatitis A. Drugs such as antibiotics
and abdominal pain or bloating may develop within and alcohol can also cause a noninfective hepatitis.
1 or 2 weeks of ingesting cysts. Diagnosis is by stool Further investigations to determine the exact cause
microscopy, and treatment is with metronidazole. would be required. In general, the treatment would be
Hepatitis A can also be picked up when travelling supportive.
abroad; however, this is usually asymptomatic, or
presents as fever, jaundice and malaise. Question 16
D. CT abdomen. When acute pancreatitis is diagnosed,
Question 11 the focus should be on the acute management.
D. α1-Antitrypsin deficiency. The history here of both Once the patient has recovered from the initial attack,
chronic obstructive pulmonary disease and deranged he/she requires further imaging to assess whether
LFT is suggestive of α1-antitrypsin deficiency. there has been any damage to the pancreas. In
α1-Antitrypsin is a serum protein that is produced particular, the consultant is looking for any evidence of
in the liver and has antiprotease effects. One in ten pancreatic necrosis. CT scan is the most appropriate
northern Europeans carries a deficiency gene, which investigation to view the pancreas properly and any
is autosomal recessive in inheritance. Symptoms soft tissue damage. CT is normally done prior to an
include basal emphysema in about 75% of MRI due to the lower cost and the lower levels of
homozygotes and liver cirrhosis in approximately 10%. radiation to the patient.
Heterozygotes have an increased risk of developing
emphysema if they smoke. None of the other Question 17
diagnoses given have a link with respiratory disease. B. Chronic pancreatitis. Chronic pancreatitis is an on-
going inflammation of the pancreas accompanied
Question 12 by irreversible architectural changes. Alcohol
E. Gastric cancer. An enlarged supraclavicular lymph consumption causes more than 85% of cases
node of the left side of the body is also known as and is the most common cause of pancreatitis in
Virchow node. This is highly suggestive of carcinoma developed countries. High-fat and high-protein
of the stomach. Patients with a palpable Virchow diets amplify the damage done by alcohol. Other
node should be referred for an upper GI endoscopy causes include idiopathic chronic pancreatitis;
for further investigation. trauma and scar formation, leading to the
obstruction of the main pancreatic duct; previous
Question 13 episodes of acute pancreatitis predispose to
A. Hepatocellular carcinoma. A raised AFP is associated others although they may often be subclinical;
with hepatocellular carcinoma. In younger males, a cystic fibrosis resulting in protein plugs in the
raised AFP along with a raised β-human chorionic duct system; and hereditary forms of pancreatitis.
gonadotropin is also indicative of germ cell tumours. Patients normally present with a history of
However, with hepatocellular carcinoma, the rest of prolonged ill-health with chronic epigastric pain,
the LFTs are often deranged as well. usually radiating through to the back.

140
Gastroenterology 2

Question 18 however, cephalosporins are considered the


D. Crohn disease. In a young adult, the history of highest-risk group. Cefotaxime is a third-generation
diarrhoea and weight loss should be investigated cephalosporin antibiotic.
by a colonoscopy to look for inflammatory bowel
disease (IBD). The presence of skip lesions in the Question 23
colon indicates that Crohn is the most likely diagnosis. A. Tropical sprue. Weight loss, diarrhoea and steatorrhea
Crohn disease may affect any part of the GI tract from are classic symptoms for malabsorption. The
the mouth to the anus. Endoscopy usually shows skip colonoscopy findings of villous atrophy are suggestive
lesions and deep ulcers and fissures in the mucosa. initially of coeliac disease; however, you would expect
Ulcerative colitis usually starts in the rectum and it the patient’s symptoms to improve with a gluten-
may extend proximally, although never beyond the free diet. Tropical sprue is a chronic, progressive
colon. Unlike Crohn disease, which commonly shows malabsorption in patients from the tropics (mainly the
skip lesions, ulcerative colitis is continuous. Areas of West Indies and Asia) associated with abnormalities
normal gut are not found between lesions. of small intestinal structure and function. It is thought
to have an infective cause. Like coeliac disease, there
Question 19 is villous atrophy, although obviously a gluten-free
E. Cyclizine. Cyclizine is an antiemetic from the H1 diet is of no help. Treatment is with broad-spectrum
receptor antagonist class. These are effective against antibiotics.
motion sickness, as these receptors are found in the
vestibular nuclei, and against vomiting caused by Question 24
substances acting in the stomach. B. Sodium docusate. This patient’s constipation has
been caused by the strong opioids he is taking. His
Question 20 main problem is hard stools, so the first-line laxative
C. Serum amylase. Diagnosis of acute pancreatitis is by should be a faecal softener. Sodium docusate is the
measurement of serum amylase. It is normally greatly only faecal softener in the answers given. If he is still
elevated (five times or more) in acute pancreatitis. having problems with constipation, then it would be
The rest of the investigations are relevant once acute appropriate to add in a different class of laxative.
pancreatitis has been diagnosed. The U&Es and
arterial blood gases should be done to ascertain the Question 25
severity of the disease, using the Glasgow criteria. C. The diagnosis is GORD which is usually investigated
AXR may show an absent psoas shadow and an air- by upper GI endoscopy as a first-line. However,
filled dilatation of the proximal jejunum. CT is typically in patients less than 55 years and without red flag
done after the acute event to assess the degree of symptoms or signs, it is reasonable to have a trial of
damage to the pancreas. treatment before proceeding with investigations.

Question 21 Question 26
A. Duodenal ulcer. The positive 14C urea breath test D. Proton-pump inhibitors, such as lansoprazole or
is indicative of the presence of Helicobacter pylori, omeprazole, are usually first choice for GORD.
which is closely associated with peptic ulcer disease. H2 receptor antagonists can be added, but they
The history given means the diagnosis is most likely to suffer from tachyphylaxis, which means their utility
be a duodenal ulcer. With duodenal ulcers, classically diminishes with time and are often stopped after
the epigastric pain is said to be relieved by food or 2 weeks.
antacids and exacerbated by hunger. The epigastric
pain with gastric ulcers is characteristically associated Question 27
with food. A. A classic description of achalasia. This is a motility
disorder associated with aperistalsis and failure of
Question 22 relaxation of the lower oesophageal sphincter. The
A. Cefotaxime. Pseudomembranous colitis is caused by tapered portion of the oesophagus is commonly
an overgrowth of Clostridium difficile, a bacillus which referred to as a beak deformity.
is carried by 2% of the population, asymptomatically.
It usually occurs postantibiotic therapy, which Question 28
suppresses the normal colonic flora allowing C. E. GORD is a risk factor for adenocarcinoma and not
difficile to proliferate. Pseudomembranous colitis is squamous cell carcinoma. The rest are risk factors for
associated mainly with broad-spectrum antibiotics; squamous cell carcinoma.

141 
SBA answers

Question 29 irritable bowel syndrome, which does not cause iron


C. A ‘corkscrew’ appearance is diagnostic of diffuse and folate deficiency.
oesophageal spasm. Treatment often involves the use
of calcium-channel blockers. Systemic sclerosis can Question 38
cause peptic stricturing due to chronic GORD. A. The first-line investigation to confirm coeliac disease
is via endoscopy and biopsy. Positive IgA and tTG are
Question 30 not enough to confirm the diagnosis. In people who
C. Management of Helicobacter pylori infection involves have IgA deficiency, a serologically positive result can
a proton-pump inhibitor and two antibiotics. Common be derived from any one of the IgG antibodies.
regimens include omeprazole, metronidazole
and clarithromycin (penicillin allergic patients), or Question 39
omeprazole, amoxicillin and clarithromycin. C. These symptoms suggest small bowel bacterial
overgrowth following recent surgery. The hydrogen
breath test helps to demonstrate the presence of such
Question 31 organisms. Upon ingestion of glucose the bacteria will
D. The diagnosis is most likely gastric cancer. The
metabolize the sugar and produce hydrogen, which
peri-orbital skin rash is classic of dermatomyositis
can then be subsequently measured in exhaled air.
which is associated with an underlying malignancy in
some cases. An upper GI endoscopy for biopsy and
diagnosis is the most appropriate investigation. Question 40
B. In cases of acute severe ulcerative colitis (by Truelove
and Witt’s severity index), IV hydrocortisone is
Question 32 the first-line treatment to induce remission, with
E. Tar-coloured stool is commonly known as melaena,
ciclosporin as second-line if patients do not tolerate
which is partially digested blood, and signifies an
IV steroids. For patients not tolerating ciclosporin,
upper GI bleed. The most common cause of such a
infliximab is indicated instead.
bleed is a peptic ulcer.

Question 41
Question 33 A. His symptoms suggest a flare-up of his Crohn disease
A. Severe vomiting often leads to a tear in the oesophagus and steroids are the most appropriate treatment to
(Mallory–Weiss) and a subsequent bleed. Conservative induce remission.
approach is used initially, but if bleeding does not settle,
then endoscopic management is indicated.
Question 42
D. Although dietary and lifestyle factors are thought to
Question 34 play an aetiological role, family history is the most
C. Diclofenac is a nonsteroidal antiinflammatory drug important aetiological factor in this case.
and can cause peptic ulceration which may be
complicated by bleeding or perforation.
Question 43
A. An ileoscopy would assess for recurrent Crohn
Question 35 disease, which is the most likely diagnosis.
D. This patient is haemodynamically unstable and hence
the first-line treatment would be IV access and urgent Question 44
fluid resuscitation with saline or O-negative blood. D. Sepsis is common with TPN. To confirm a source,
Once haemodynamic stability has been achieved, both peripheral and central (Hickman) line blood
upper GI endoscopy is needed. cultures should be taken and TPN stopped until
cultures are deemed negative.
Question 36
C. This patient has iron-deficiency anaemia and the initial Question 45
investigation is an upper and lower GI endoscopy (top A. This patient has developed jaundice. Following liver
and tail) to look for a source of GI blood loss such as biochemistry, a USS is the initial investigation to
a malignancy. assess for biliary duct dilatation.

Question 37 Question 46
E. Coeliac disease is the most likely as it is common (1% B. Hepatitis A is commonly due to ingestion of
of the population) and causes these symptoms. Other contaminated water or food such as shellfish.
listed diagnoses are far less common, other than Symptoms such as abdominal pain, diarrhoea and

142
Gastroenterology 2

dark urine are characteristic of the condition, as is the The diagnosis is usually clinical, but an EEG will
timeline of 3 days. show characteristic changes if there is diagnostic
uncertainty.
Question 47
E. This patient has developed jaundice as a result Question 56
of hepatitis C. Hepatitis C is known to cause D. The history suggests PBC, a condition known to
cryoglobulinemia-mediated glomerulonephritis, which affect middle-aged women. Serum AMAs are the first-
explains this patient’s haematuria. Hepatitis B causes line investigation as they are elevated in 90% to 95%
membranous and mesangiocapillary glomerulonephritis, of PBC with a specificity of 98%. Serum ANA can be
which does not commonly result in haematuria. elevated in 20%. IgM is elevated but is nonspecific.

Question 48 Question 57
D. This patient most probably has a diagnosis of E. The diagnosis in this case is haemochromatosis
hepatitis A, which is commonly due to consumption based on the history and examination. From the
of contaminated food such as shellfish or clams. listed options, serum ferritin is the most appropriate
Management is often conservative. diagnostic tool.

Question 49 Question 58
C. Grade 3 encephalopathy comprises features of C. The diagnosis is alcoholic hepatitis in view of the
drowsiness, stupor and no evidence of communication. presence of Mallory bodies. Stopping alcohol
completely is the mainstay form of treatment to
Question 50 prevent the development of alcoholic cirrhosis.
B. The suspected diagnosis is autoimmune hepatitis,
which is more common in patients with other Question 59
autoimmune conditions. This condition is associated D. This patient is likely to be suffering from a primary
with high titres of antinuclear antibodies. PBC causes hepatocellular carcinoma in view of his weight loss,
elevated IgM levels. presence of ascites and raised AFP level. USS or CT
is the most appropriate diagnostic tool.
Question 51
D. The best assessment of liver function in cirrhotic Question 60
patients is prothrombin time. Therefore if there are no D. The diagnosis is chronic pancreatitis. As a result of
acute bleeding issues, gastroenterologists tend not pancreatic lipase secretion reduction, this patient
to correct the international normalized ratio (INR) with has developed steatorrhoea. Chronic pancreatitis is
vitamin K (within limits). most appropriately investigated by abdominal
CT scan.
Question 52
A. With any GI bleed, an urgent endoscopy is the most Question 61
appropriate initial management following adequate B. Pancreatic carcinoma has a poor prognosis. This
resuscitation of the patient. lady is elderly and frail with metastatic disease and
therefore is likely to have palliative treatment only.
Question 53
C. This patient has developed ascites. The initial
EMQ ANSWERS
management is with diuretics, such as spironolactone,
and sodium restriction. Of course, alcohol cessation Liver problems
would be useful here. 1. A  Alcoholic liver disease. Classic signs of hepatic
failure and hepatic encephalopathy.
Question 54 2. G  Metastatic liver disease. The patient needs an
B. This patient has developed spontaneous bacterial urgent ultrasound examination and possible liver
peritonitis which is a complication of ascites in chronic biopsy; LFTs would not be particularly helpful at
liver disease. The most common organism in this this stage.
condition is Escherichia coli. IV Tazocin is first-line therapy. 3. B  Biliary atresia. Prolonged jaundice (>2 weeks)
should be investigated. Bilirubin accumulation will
Question 55 eventually cause hepatic failure if left untreated.
E. The diagnosis is hepatic encephalopathy, confirmed Physiological jaundice should disappear by day 8
by features of asterixis, fetor hepaticus and confusion. in normal births.

143 
EMQ answers

4. D  Hepatitis A. The disease is endemic in many parts Drugs


of the world, and transmission is via the orofaecal 1. D  Acamprosate is known to help reduce the craving
route, usually from consumption of contaminated experienced by alcohol-dependent patients. It is
water or food. thought to interact with glutamate and gamma-
5. C  Gallstones. In the classic presentation, the aminobutyric acid (GABA) neurotransmitters to
patient might also have dietary fat intolerance and restore neuronal excitation and inhibition balance.
flatulence. 2. F  Acute severe exacerbations involve the use of
IV steroids in almost all cases. Ciclosporin can
Hepatobiliary disorders be given for patients who do not tolerate/refuse
1. G  Haemochromatosis. This condition is associated steroids.
with a triad of skin pigmentation, diabetes and 3. B  In view of his stroke, this patient is likely to have
hepatomegaly. Treatment is with venesection, been prescribed aspirin. Aspirin is an antiplatelet
initially weekly and then less frequently to maintain drug and may be associated with an increased
iron depletion. risk of GI bleeding.
2. B  α1-Antitrypsin deficiency. This disorder can result 4. A  Omeprazole. This patient is likely to be suffering
in liver cirrhosis and emphysema. There is no from GORD. Agents such as proton-pump
specific treatment available. inhibitors or alginate containing antacids are
3. J  PBC. Such a disorder is most appropriately beneficial in such cases.
diagnosed by the presence of antimitochondrial 5. E  The diagnosis is Crohn disease in view of
antibodies. Treatment plans include the use of histological findings. Mesalazine induces remission
ursodeoxycholic acid and cholestyramine to help in this condition.
relieve pruritus.
4. A  Wilson disease is associated with an error of Dysphagia
copper metabolism. Treatment consists of 1. I  Candida. This patient is likely to be taking
copper chelating agents such as penicillamine immunosuppressants to prevent rejection
or trientine. following her recent transplant. Candida infection
5. F  Primary sclerosing cholangitis is diagnosed is common in such patients and can lead to
following a liver biopsy which demonstrates swallowing difficulties.
evidence of polymorph infiltration of bile 2. G  Oesophageal web. These features are consistent
ducts and is associated with IBDs. An ERCP with Plummer-Vinson syndrome. This condition
may also be useful in such cases and may is associated with ulceration and subsequent
demonstrate a bead-like appearance within structuring or web-like formations in the upper
the bile ducts as a result of stricture oesophagus resulting in dysphagia.
formation. 3. E  Pharyngeal pouch. Muscle dysmotility can lead
to mucosal out pouching, which is commonly
Gastrointestinal bleeding referred to as a pharyngeal pouch. Treatment is
1. C  Haemorrhoids. Additional symptoms may include primarily surgical.
perianal irritation and itch. Treatment involves the 4. C  Benign oesophageal stricture. Additional causes
use of sclerosant injections, elastic band ligation or include reflux disease, ingestion of caustic
surgical resection. substances and previous radiotherapy.
2. A  This is a classic presentation of a Mallory–Weiss 5. A  Achalasia. Such radiological findings are classic of
tear. Forceful vomiting has led to a tear in the achalasia. A barium swallow often demonstrates a
oesophagus and a subsequent bleed. tapered lower end of the oesophagus.
3. B  Angiodysplasia. There is a strong association
with underlying cardiac disease, particularly aortic Gastroenteritis
stenosis, possibly due to the shearing of the von 1. H  Staphylococcus aureus. This organism is known
Willebrand factor as they go across the stenosed to produce such symptoms as a result of the
valves. Replacement of the aortic valve can cure production of a heat-stable endotoxin. Treatment
the problem. is usually supportive.
4. G  Reflux oesophagitis. Classic description of reflux 2. C  Bacillus cereus. Vomiting in this case is commonly
disease. due to production of an emetic toxin. Diarrhoea
5. E  Duodenal ulcer. The gram-negative bacterium typically occurs 8–16 hours following vomiting.
is most probably Helicobacter pylori, which is 3. B  Campylobacter. Symptoms last for approximately
commonly associated with both gastric and 1 to 2 weeks. Treatment involves the use of
duodenal ulcers. antibiotics such as erythromycin.

144
Gastroenterology 2

4. E  Escherichia coli. Additional features may include Altered bowel habit


renal failure and anaemia. Avoidance of antibiotics 1. G  Diverticulitis
is key as certain agents may lead to toxin 2. D  Irritable bowel syndrome
production. 3. B  Crohn disease
5. F  Yersinia enterocolitica. Diagnosis is often by 4. A  Ulcerative colitis
serology and treatment is typically supportive. 5. C  Colorectal carcinoma

Pancreatitis Jaundice
1. E  Hyperlipidaemia. This is an uncommon cause of 1. D  Pancreatic carcinoma
pancreatitis and is frequently seen in conjunction 2. C  Hepatocellular carcinoma
with alcohol excess. High triglycerides in the 3. I  Chronic pancreatitis
blood causes pseudohyponatraemia as only 4. E  Gallbladder carcinoma
the sodium in the serum, rather than lipid, is 5. J  Liver abscess
measured.
2. D  Grey-Turner sign. Grey-Turner described his Dysphagia
sign in acute haemorrhagic pancreatitis. This 1. D  Pharyngeal pouch. Pharyngeal pouches occur
is an uncommon presentation now, but the most commonly in elderly patients (over 70 years)
sign may still be seen in trauma where there is and typical symptoms include dysphagia,
retroperitoneal bleeding, in this case due to trauma regurgitation, chronic cough, aspiration and
of the pancreas as it comes into contact with the weight loss. The aetiology remains unknown but
vertebral bodies. theories centre upon a structural or physiological
3. F  Pancreatic divisum. Pancreatic divisum is relatively abnormality of the cricopharyngeus.
common and most patients are asymptomatic. 2. J  Oesophageal web. These webs are thin (2–3 mm)
Drainage of the ducts of Santorini and Wirsung membranes of normal oesophageal tissue
by a minor papilla can lead to increased intraduct consisting of mucosa and submucosa that can
pressure leading to pancreatitis. partially protrude/obstruct the oesophagus. They
4. A  Alcohol. This is the most common cause of are mainly observed in Plummer-Vinson syndrome,
pancreatitis. Presentation may be delayed after a which is associated with chronic iron-deficiency
significant bout of drinking, as in this case. anaemia.
5. C  Gallstones. This is the other common cause of 3. C  Oesophageal carcinoma. Prominent symptoms
pancreatitis (alcohol and gallstones making up usually do not appear until the cancer has
around 95% of pancreatitis). The patient gives a infiltrated over 60% of the circumference of the
history of biliary colic as her gallstone impacts in oesophagus. Dysphagia is experienced first with
Hartmann’s pouch. solid foods and later with softer foods and liquids.
If the cancer has spread elsewhere, symptoms
Diarrhoea related to metastatic disease may appear.
1. G  Irritable bowel syndrome. This is the likely 4. A  Achalasia. It is characterized by incomplete
diagnosis in this patient; however, he should be lower oesophageal sphincter (LES) relaxation,
monitored as he is the right age for a primary increased LES tone and lack of peristalsis of the
presentation of IBD. oesophagus.
2. H  Metformin. This is the likely culprit. Many drugs are 5. I  Retrosternal thyroid. A goitre with a portion of
associated with diarrhoea; however, it is common its mass located in the mediastinum causes
with metformin, especially at higher doses. symptoms of hyperthyroidism and difficulty
3. E  Diverticular disease. Colonoscopy is indicated to swallowing.
rule out a malignancy due to the gentleman’s age,
change in bowel habit and haematochezia. Biliary disease
4. I  Ulcerative colitis. In a patient with the same 1. D  Biliary peritonitis. This patient has initially
symptoms but aged 60+, the consideration of developed acute cholecystitis, but due to his
malignancy would be much higher. This young age and comorbidities, such as ischaemic heart
lady has IBD. Left iliac fossa tenderness would disease and diabetes, he is at increased risk of a
suggest ulcerative colitis rather than Crohn perforation of the gallbladder. This happens when
disease, which more often affects the ileum. he suddenly develops generalized abdominal pain.
5. J  Zollinger–Ellison syndrome. This is characterized He has signs of peritonitis, with a tachycardia and
by a duodenal or pancreatic gastrin-secreting generalized abdominal tenderness, particularly on
tumour leading to profuse ulceration. the right side, and absent bowel sounds.

145 
EMQ answers

2. C  Biliary colic. This patient has a chronic history however, has remained obstructed and so has
of pain that occurs intermittently and resolves become swollen due to retention of fluid within
spontaneously. She has not required admission to it. This gives the palpable mass. The flu-like
hospital as the pain is due to biliary colic, where symptoms and a swinging pyrexia are consistent
the gallbladder is trying to contract against an with an abscess (i.e., an empyema of the
obstruction (i.e., the gallstone). It may last for gallbladder).
several hours. 5. E  Cholangiocarcinoma. This patient has painless
3. B  Acute pancreatitis. This patient is known to have obstructive jaundice. His confusion may be
gallstones. She has a classic history of acute due to the jaundice, secondary infection or
pancreatitis (i.e., pain radiating through to the associated renal impairment. He is afebrile
back and vomiting). She has signs of localized and abdominal examination does not show a
peritonism and systemic signs of dehydration and distended gallbladder, which might be present if
a tachycardia. his gallbladder was not diseased and was able
4. G  Empyema of the gallbladder. This is a chronic to distend secondary to obstruction of his bile
history which has been going on for several duct (i.e., Courvoisier’s law). In many cases,
weeks. The patient initially started with an however, although the bile duct is obstructed, the
episode of acute cholecystitis, which was gallbladder is fibrosed and fails to distend despite
probably unrecognized. The gallbladder, the obstruction.

146
Respiratory 3
SBA ANSWERS Question 7
D. Exacerbations of pulmonary symptoms in patients
Question 1 with cystic fibrosis must be recognized early and
D. Tobacco smoking is the most common cause of treated vigorously to maintain pulmonary function and
COPD. Other factors, such as air pollution and relieve symptoms. Antibiotic treatment is prescribed
genetics, play a smaller role. Long-term exposure to based on new symptoms or worsening of existing
these irritants causes an inflammatory response in the symptoms.
lungs, which results in the breakdown of lung tissue
and narrowing of airways. Diagnosis is based on lung Question 8
function tests. D. Because of defective chloride channels ((cystic fibrosis
transmembrane conductance regulator (CFTR)),
Question 2 the concentration of chloride in sweat is elevated in
C. When the patient arrives in hospital, arterial blood patients with cystic fibrosis. The sweat test measures
gases should be measured and the inspired oxygen the amount of chloride excreted in sweat. A chloride
concentration noted in all patients with an exacerbation level of greater than or equal to 60 mmol/L means that
of COPD, so that controlled oxygen therapy can be cystic fibrosis is likely to be diagnosed.
tailored. These measurements should be repeated
regularly, according to the response to treatment. Question 9
C. Farmer’s lung is a hypersensitivity pneumonitis
Question 3 induced by the inhalation of biologic dusts coming
B. Individuals with COPD typically depend on a degree from hay dust or mould spores or any other
of hypoxia to maintain respiratory drive, so it is best to agricultural products. Acute farmer’s lung starts
commence low oxygen concentrations via a Venturi about 4 to 8 hours after the person breathes in a
mask. The oxygen saturation and arterial blood gas large amount of dust from mouldy crops, causing
results should be regularly assessed to determine breathlessness, cough, nausea and fevers. Symptoms
correct oxygen therapy. usually decrease after 12 hours, but could be longer.

Question 4 Question 10
D. The diagnosis is obstructive sleep apnoea, caused B. The next step in the management plan is to add on a
by a combination of alcohol, which decreases muscle low-dose inhaled steroid. If this is inadequate, then a
tone, and obesity. All of the listed options can occur long-acting β2 agonist is added. If control of asthma
in obstructive sleep apnoea but excessive daytime symptoms is still inadequate, then a high-dose
sleepiness is the most common and is the hallmark of inhaled steroid may be added. Add-on therapy (e.g.,
the disease. leukotriene receptor antagonist or theophylline) or oral
steroids are for more advanced cases.
Question 5
A. A typical presentation of cystic fibrosis. The presence Question 11
of clubbing suggests a chronic respiratory disease E. Nebulized salbutamol or terbutaline in addition to
process as opposed to acute infections or inhaled oral steroids are the initial treatment of acute severe
foreign body. It is an autosomal recessive disorder asthma. Oral steroids are as effective as intravenous
which affects the lungs, pancreas, liver, kidneys and steroids. Intravenous magnesium is added in life-
gastrointestinal tract. threatening exacerbation.

Question 6 Question 12
E. High-resolution CT of the chest is the most frequently A. S. pneumoniae is the most common cause of
used imaging test to establish the diagnosis of community-acquired pneumonia where a pathogen
bronchiectasis. Other investigations may also need to is isolated, accounting for about 50% of cases.
be performed to determine the underlying cause (e.g., Other bacterial causes include H. influenzae, C.
sweat test for cystic fibrosis). pneumoniae, M. pneumoniae and S. aureus.

147 
SBA answers

Question 13 extrapulmonary features (e.g., TB meningitis, Pott


C. The mortality associated with community-acquired disease of the spine and osteomyelitis).
pneumonia is dependent on the CURB 65 score
as recommended by the British Thoracic Society. A Question 21
respiratory rate of 30 breaths/min is a key risk factor, B. TB infection begins when mycobacteria reach the
not 20. If the score is 1 or less, the patient could pulmonary alveoli, where they invade and replicate
potentially be managed at home. If it is a 2, a short within macrophages. Macrophages are responsible
stay in hospital is needed. If the score is 3 or more, for engulfing the bacilli in an attempt to eliminate them
hospitalization is recommended. by phagocytosis. The bacilli have a resistant mycolic
capsule that protects them from toxic substances in
Question 14 the macrophages. They are able to reproduce inside
D. M. pneumoniae causes a form of atypical pneumonia the macrophage and eventually kill the immune cell.
which can cause extra-pulmonary symptoms
including erythema multiforme, haemolytic anaemia, Question 22
Guillain–Barré syndrome, renal failure and pericarditis. B. Multiple sputum cultures are usually taken to look for
acid-fast bacilli if the patient is producing sputum.
Induced sputum samples can be taken if there is
Question 15 no spontaneous sputum production. Auramine–
A. C. psittaci is an intracellular bacterial species found in
rhodamine staining (fluorescence microscopy) is more
birds, cattle, pigs, sheep and horses. Transmission is
sensitive than Ziehl–Neelsen stain.
by inhalation, contact or ingestion among birds and
then to humans. This type of pneumonia in humans
Question 23
often starts with flu-like symptoms. Tetracyclines and
C. The recommended treatment of new-onset pulmonary
macrolides are used to treat this type of pneumonia.
TB is 6 months of quadruple therapy consisting of
rifampicin, isoniazid, pyrazinamide and ethambutol for
Question 16 the first 2 months, and dual therapy with rifampicin and
C. Intravenous drug users have a 10-fold increased risk isoniazid for the last 4 months. Streptomycin is used in
of community-acquired pneumonia compared with the patients who have previously been treated for TB as they
general population. S. aureus pneumonia is often seen in are more likely to have developed some drug resistance.
intravenous drug abusers, and can result in lung abscess.
Question 24
Question 17 A. Rifampicin can cause hepatotoxicity, fevers,
E. Legionella transmission is via inhalation of water droplets gastrointestinal disturbances and rashes. It can also
from a contaminated source that has allowed the cause certain bodily fluids, such as urine, sweat and
organism to grow and spread (e.g., cooling towers). tears, to become orange-red. This is sometimes used
It usually starts with flu-like symptoms, followed by to monitor effective absorption of the drug.
gastrointestinal tract and central nervous system
symptoms, causing diarrhoea, nausea and confusion. Question 25
E. This patient has a diagnosis of sarcoidosis, which
Question 18 is a systemic inflammatory disease that can affect
A. Mild pneumonia is usually managed at home with any organ. Up to 90% of patients complain of eye
treatment from GP. A common regimen is a 5–7-day involvement. Manifestations in the eye include uveitis
course of oral amoxicillin 500 mg three times a day. (commonest), uveoparotitis and retinal inflammation,
which can lead to loss of visual acuity.
Question 19
D. CXR is the gold-standard investigation in pneumonia. Question 26
The X-ray findings of pneumonia are airspace opacity, E. Tissue obtained from biopsy can be used to confirm
lobar consolidation/air bronchograms or interstitial the diagnosis of sarcoidosis and rule out an infective
opacities. TB, Klebsiella and staphylococcal infections cause. Biopsy shows evidence of noncaseating
can all cause lung abscesses. granulomas. Supporting investigations include CXR
showing hilar lymphadenopathy, hypercalcaemia,
Question 20 raised liver function tests and raised serum ACE.
D. M. tuberculosis is often seen in Asian immigrants.
Classic symptoms are chronic cough with blood- Question 27
containing sputum, fever, night sweats and weight D. All the above except D are causes of upper zone
loss. In 20% of active cases of TB, the infection has fibrosis. Cryptogenic fibrosing alveolitis is a disease

148
Respiratory 3

of unknown aetiology, which is characterized by a is a transudate or exudate. Glucose levels are usually
cellular alveolar infiltrate and later by fibrosis of the lower in infection. Amylase levels may be increased
alveolar walls. It usually presents with progressive with pancreatitis, oesophageal rupture or malignancy.
dyspnoea, persistent cough and finger clubbing.
Question 35
Question 28 A. Causes of transudative effusions include heart failure,
A. Extrinsic allergic alveolitis is an inflammation of the cirrhosis, hypoalbuminaemia/nephrotic syndrome,
alveoli within the lung caused by hypersensitivity to a hypothyroidism, peritoneal dialysis and Meigs
huge variety of inhaled organic dusts. Worldwide, there syndrome.
are differences, but forking mouldy hay (farmer’s lung)
is the most common cause from the listed options. Question 36
A. This patient has a pulmonary embolus best diagnosed
Question 29 by CTPA as it has the best diagnostic accuracy of all
E. Prolonged inhalation of asbestos fibres can cause the advanced noninvasive imaging methods. If there
serious and fatal illnesses including all of the above. is a contraindication to CTPA (e.g., contrast allergy/
Top at-risk occupations are construction workers, severe renal failure), then a ventilation–perfusion (V/Q)
firefighters, industrial workers, power plant workers scan may be performed.
and shipyard workers. Sufferers with any of the above
options are usually able to claim compensation for Question 37
past exposure. C. Ultrasound scan-guided tap is ‘diagnostic’ and will
result in pleural fluid for cultures and Gram stain.
Question 30 Features suggestive of an empyema on thoracic
A. Squamous cell carcinoma of the lung is closely ultrasound include the presence of echogenic fluid,
correlated with a history of tobacco smoking, and loculations and septations. The use of ultrasound
smoking is the most common cause of lung cancer. to guide thoracentesis (pleural tap/aspiration)
Although working in a shipyard puts the patient at risk reduces complications. Pleural tap revealed
of mesothelioma, this is not as common as SCC of >100,000 neutrophils/μL, bacteria seen on Gram
the lung. stain and pH <7.2, which may be presumed to be
empyema.
Question 31
C. Adenocarcinoma is a type of nonsmall cell lung Question 38
carcinoma. It is currently the most common type of B. The presence of bubbling in an underwater seal
lung cancer in lifelong nonsmokers. These account for means that there is a consistent air leak, and when
40% of all lung cancers. it stops bubbling it means that either the air leak
has stopped (pneumothorax resolved) or the drain
Question 32 is blocked. Thus the drain can only be removed
E. This patient has clinical features of Cushing syndrome. when it has stopped bubbling, and a CXR is done to
Because of its high-grade neuroendocrine nature, demonstrate that there is no re-expansion after the
small cell carcinomas can produce ectopic hormones drain is clamped (as a test).
including adrenocorticotropic hormone (as in this case)
and antidiuretic hormone. Lambert–Eaton myasthenic Question 39
syndrome (LEMS) is a well-known paraneoplastic C. Common complications include puncture of the
condition linked to small cell carcinoma. lungs resulting in a ‘pneumothorax’ especially with a
subclavian approach (as opposed to via an internal
Question 33 jugular approach).
A. All of the above can be found in lung cancers, but
clubbing is the most common extrapulmonary feature. Question 40
It is usually associated with nonsmall cell lung cancer B. The most characteristic findings of a pneumothorax
(SCLC) (>50% cases). SCLCs are also associated are hyperresonance and decreased breath sounds. A
with various paraneoplastic syndromes. tension pneumothorax may displace the mediastinum
to the unaffected side. This is associated with
Question 34 distended neck veins and hypotension. A needle
E. Transudate has low protein/albumin levels when insertion in the second intercostal space in the
compared with exudate. Light criteria are usually midclavicular line decompresses the system, is life
applied when trying to determine whether pleural fluid saving and provides relief to the patient.

149 
SBA answers

Question 41 processes that injure airways and cause luminal


D. When dyspnoea is accompanied by decreased breath dilatation. In addition to daily vicious, often purulent
sounds unilaterally and the tracheal deviation, the sputum production with occasional haemoptysis,
most likely diagnosis will be pneumothorax. In the wheezing and dyspnoea occurs in 75% of patients.
case of pneumothorax, there is no orthopnoea or Emphysema and chronic bronchitis, forms of COPD,
paroxysmal nocturnal dyspnoea and also S3 gallop is also cause a decreased FEV1:FVC ratio, but the
not a feature of pneumothorax. Pneumothorax is the baseline sputum is generally mucoid and luminal
collection of air in the thoracic cavity, which presents dilatation of bronchi is not characteristically present.
with diminished breath sounds and tracheal deviation
away from the pathological site. Question 47
A. CPAP is the most effective treatment for obstructive
Question 42 sleep apnoea in adults. Results with devices that
C. SCCs are associated with hypercalcaemia move the tongue or mandible forward are variable
(parathyroid hormone-related protein producing). and inconsistent. Uvulopalatal surgery often reduces
SCLC may be associated with syndrome of snoring but may not reduce the frequency of apnoeic
inappropriate antidiuretic hormone secretion (SIADH), episodes during sleep.
resulting in hyponatremia. Adenocarcinomas are
associated with hypertrophic osteoarthropathy Question 48
(clubbing). Lung cancer metastasizes to the adrenals, D. Radiation pneumonitis is common after radiotherapy,
brain, bone, liver and skin. resulting in a localized pneumonitis, which would
result in fibrosis eventually. TB would produce upper
Question 43 lobe cavitation rather than shadowing. Pulmonary
E. SCLC is strongly associated with smoking and, due lymphoma is extremely rare; it is the involvement
to its central location, results in recurrent or slowly of lung parenchyma from lymphoma, and usually
resolving pneumonia/lung collapse. A bronchoscopy bilateral. Tuberculosis presents with nonspecific
is warranted to view the obstruction and take clinical signs of cough, fever, haemoptysis.
biopsies.
Question 49
Question 44 C. Pulmonary fibrosis is associated with finger clubbing,
D. The large mass is consistent with an SCC, which is hypoxaemia and restrictive PFT picture as it is in
seen mainly in smokers. Mesothelioma is rare; it is this patient. Bronchial asthma is associated with an
characterized by a bulky pleural mass in a person obstructive PFT picture with reduced peak flows.
with prior asbestos exposure. P. carinii (jirovecii) It would not be associated with finger clubbing.
pneumonia is a diffuse process and occurs in Emphysema would be associated with an obstructive
immunocompromised patients. Haemoptysis does PFT, as well as quiet breath sounds.
not occur. Granulomatous inflammation is typical for
mycobacterial and fungal infections; though a solitary Question 50
granuloma may be present, it is usually not larger than B. Silicosis is a respiratory disease caused by inhalation
2 to 3 cm in size. of silica dust. The fact that the patient worked as
a rock miner is enough evidence that the patient is
Question 45 suffering from silicosis. In such conditions, there is
B. Extrapulmonary symptoms suggest atypical bacteria, bilateral upper lobe involvement accompanied by
and classically hotels (stagnant water source) are hilar lymphadenopathy and eggshell calcification of
a breeding ground for Legionella. H. influenzae is the lymph nodes. People who are involved in the
normally a community-acquired pneumonia affecting occupations of mining, quarrying, sand blasting, stone
COPD patients and children, via aerosol transmission. cutting and glass manufacturing fall easy prey to
The symptoms are more indicative of atypical bacteria silicosis. The symptoms of this disease include cough,
however. M. pneumoniae is an atypical bacterium; unexplained weight loss and SOB.
common in teens, mostly causing pharyngitis/
bronchitis only, which is not concordant with the Question 51
clinical picture in the question. C. Pneumoconiosis is the general term for lung disease
caused by inhalation and deposition of mineral
Question 46 dust, with asbestosis more specifically being
B. Bronchiectasis is an illness of the bronchi and pneumoconiosis caused by asbestos inhalation. The
bronchioles involving obstructive and infectious term refers to a group of naturally occurring,

150
Respiratory 3

heat-resistant fibrous silicates. Because the 3. H  The diagnosis is S. aureus pneumonia most
development of asbestosis is dose dependent, appropriately treated with flucloxacillin.
symptoms appear only after a latent period of 4. B  Additional side effects include rashes and
20 years or longer. This latent period may be shorter arthralgia.
after intense exposure. Dyspnoea upon exertion is 5. E  This individual is at risk of aspiration pneumonia;
the most common symptom and worsens as the most appropriately treated with metronidazole as
disease progresses. Patients may have a dry (i.e., infection is commonly due to anaerobes.
nonproductive) cough. A productive cough suggests
concomitant bronchitis or a respiratory infection. Lung cancer
Patients may report nonspecific chest discomfort, 1. E  This is especially associated with nonsmall cell
especially in advanced cases. lung carcinoma. These patients get clubbing and
increased bone deposition on long bones.
Question 52 2. G  The growing tumour can cause compression of
C. Absent breath sounds, decreased wheezing, pulsus the brachial plexus and cause pain in the shoulder
paradoxus, cyanosis, diaphoresis with inability to and inner arm.
speak in full sentences and increasing levels of CO2 in 3. D  This could be familial or secondary to endocrine
blood are all ominous signs of impending respiratory conditions (e.g., diabetes mellitus, obesity), drug
failure in the patients of asthma exacerbation. It is related (e.g., steroid use) or malignancy as a
important to be ready with rapid sequence intubation paraneoplastic syndrome.
in such patients. 4. F  Classic features of Horner syndrome as a result
of involvement of the preganglionic sympathetic
Question 53 nerves. In this case it is caused by a Pancoast
A. Asthma is typically associated with an obstructive tumour.
impairment that is reversible with short-acting 5. A  Symptoms are associated with a defective
bronchodilators. A reduced FEV1 and FEV1:FVC ratio release of acetylcholine at the neuromuscular
indicates airflow obstruction. A reduced FVC1 with a junction. About 60% of those with LEMS have an
normal or increased FEV1:FVC ratio is consistent with underlying malignancy, most commonly SCLC.
a restrictive pattern of lung function.
Diseases of the respiratory tract
EMQ ANSWERS 1. G  Classic definition of emphysema. Severe
destruction can lead to development of bullous
Pneumonia emphysema.
1. G  A common cause of pneumonia in cystic fibrosis 2. F  A viral infection associated with myalgia,
or individuals who are immunocompromised. headaches, cough and a sore throat. Management
More than 60% of adults with cystic fibrosis have is usually symptomatic relief.
Pseudomonas. 3. E  Features are essentially due to inflammatory
2. E  This is the most common cause of acute oedema involving the larynx.
exacerbations of COPD. Other common 4. A  Cystic fibrosis is the most common cause of
causes include S. pneumoniae and Moraxella bronchiectasis, which is a permanent dilatation of
catarrhalis. airways.
3. B  Seen particularly in those in close contact with 5. H  Typical features of acute bronchitis, which is a
infected parrots. It often starts with flu-like short-term inflammation of the bronchi causing
symptoms. Additional features include high fever cough, wheeze, breathlessness, fever and chest
and meningism. discomfort.
4. C  S. aureus is a common cause of pneumonia in
intravenous drug users or in patients with central Investigative findings
venous catheters. 1. I  This condition is associated with a ground-glass
5. D  Likely cause of pneumonia associated with appearance progressing to a honeycomb lung on
contaminated water systems in hotels. CXR.
2. G  Typical X-ray features of extrinsic allergic alveolitis,
Respiratory drugs an inflammation of the alveoli within the lung
1. D  This drug also causes pink coloured urine and caused by hypersensitivity to a large range of
body secretions. inhaled organic dusts.
2. A  Ethambutol is known to result in visual 3. A  Patients with COPD have partially reversible airflow
disturbances such as colour blindness and central limitation with an increase in FEV1 of less than
scotoma formation. 15% on inhalation of a β2 agonist.

151 
EMQ answers

4. J  Typical presentation of pneumonia, with S. 4. J  Spontaneous pneumothorax. This occurs in the


pneumoniae being the most common bacterial absence of underlying lung disease and is usually
cause. due to the formation of small sacs of air blebs in
5. D  This is an autoimmune, systemic, small to medium lung tissue that rupture, causing air to leak into the
vessel vasculitis affecting the upper respiratory pleural space. Risk factors include male gender,
tract, lungs and kidneys with antineutrophil smoking and a family history of pneumothorax.
cytoplasmic antibodies found in over 90% of 5. G  Bronchial carcinoma. This patient’s occupation
cases. puts her at risk of lung carcinoma. The most
common symptoms are cough, haemoptysis,
Lung diseases breathlessness, weight loss and chest pains.
1. D  Tar-stained fingers indicate long-term tobacco The majority of cases are caused by long-term
use, which results in the development of COPD. tobacco smoking. About 10% of cases occur in
2. J  Pneumothorax. A history of trauma, respiratory nonsmokers but those exposed to carcinogens
distress and unilateral loss of breath sounds (e.g., asbestos) or those with genetic factors.
suggests a pneumothorax.
3. F  Pleural effusion. Symptoms suggest that this is Respiratory infections
likely secondary to a lung malignancy. 1. C  A common gram-negative bacterium that is
4. I  Rheumatoid lung. This patient has underlying associated with hospital-acquired infections (e.g.,
rheumatoid arthritis. Rheumatoid lung disease ventilator-associated pneumonia). It is frequently
includes nodules, pulmonary fibrosis, pulmonary multidrug resistant and is difficult to treat.
hypertension, pleural effusions and bronchiolitis 2. B  Organisms that cause lung abscesses include S.
obliterans. aureus, Klebsiella, Aspergillus and Pseudomonas.
5. E  Cor pulmonale. The occupational history indicates Alcoholism is the most common condition
exposure to metal shards or chemicals, leading predisposing to lung abscesses.
to pulmonary fibrosis and subsequent right-sided 3. E  Legionella transmission is via inhalation of water
heart failure. droplets from a contaminated source that has
allowed the organism to grow and spread (e.g.,
Shortness of breath cooling towers). It usually starts with flu-like
1. B  Aortic stenosis. Patients present with angina, symptoms, followed by gastrointestinal tract
breathlessness and syncope. Causes include and central nervous system symptoms, causing
degeneration, bicuspid aortic valve and rheumatic diarrhoea, nausea and confusion.
fever. 4. D  E. coli is a gram-negative bacterium commonly
2. I  Right middle lobe pneumonia. The acute history, found in the gastrointestinal tract. Following
X-ray findings and raised inflammatory markers intraabdominal surgery, it is possible for the
all indicate a right-sided pneumonia. The most bacterium to track up to the lung to cause
common bacterial cause is S. pneumoniae. pneumonia and sepsis.
3. F  Pleural effusion. The underlying cause is likely to 5. F  This is a classic presentation of community-
be lung carcinoma given the significant smoking acquired pneumonia. S. pneumoniae is the most
history. This type of effusion is usually exudative. common bacterial cause.

152
Renal 4
SBA ANSWERS Question 7
D. People with estimated glomerular filtration rate
Question 1 less than 30 mL/min per 1.73 m2, with or without
E. The patient has prerenal renal failure. This is diagnosed, diabetes, should be referred for specialist
in combination with clinical examination, by a urine assessment. Patients should ideally be referred
osmolality greater than 500 mOsm/kg and a urinary at least 1 year before they might be anticipated
sodium of less than 20 mmol/L. Causes of prerenal renal to require dialysis. Conservative management
failure include hypovolaemia, decreased cardiac output, including vitamin D analogues, erythropoietin and
renal artery obstruction and liver failure. Acute tubular dietary control is also important but early referral for
necrosis is an example of intrinsic renal failure. specialist assessment is the most important step in
this patient’s management.
Question 2
A. All the options are used in patients with suspected Question 8
renal failure, but serum urea and electrolytes are the B. The most common complications from peritoneal
gold-standard initial investigation that can confirm dialysis are infections, for example, peritonitis.
suspected renal failure. Noninfectious complications include:
− Catheter related: perforation, haemorrhage,
Question 3 obstruction of flow, leakage
A. A potassium of 6.5 mmol/L is a medical emergency, − Related to increased intraabdominal pressure: hernia,
so must be treated as a matter of urgency. This is hydrothorax, back pain
done by intravenous (IV) administration of calcium − Metabolic: hyperglycaemia, hypertriglyceridaemia
gluconate 10% (10 mL) to stabilize the heart, followed − Others: haemoperitoneum, encapsulating peritoneal
by insulin and dextrose infusion to drive potassium sclerosis
into cells. Salbutamol nebulizers can also be used.
Cardiac monitoring is important throughout.
Question 9
D. PKD can be either autosomal dominant or recessive.
Question 4 Diagnosis is suspected if patient presents with
E. Refer patients for dialysis immediately if any of the
flank pain, haematuria, hypertension, headaches,
following are not responding to medical management:
abdominal pain and positive family history. This patient
hyperkalaemia, metabolic acidosis, symptoms
is likely to have PKD in view of his positive family
or complications of uraemia (e.g., pericarditis),
history and clinical presentation. Renal ultrasound is
encephalopathy, fluid overload and pulmonary
the most reliable and noninvasive way to diagnose the
oedema. Anaemia is not an indication.
condition.

Question 5
B. This can be acute or chronic. Common causes
Question 10
C. Autosomal dominant PKD is the most common
include infection, or reaction to medications, for
of all the inherited cystic kidney diseases, with
example, analgesia (NSAIDs), antibiotics (penicillin,
an incidence of 1:500 live births. The autosomal
cephalexin), proton-pump inhibitors, allopurinol,
recessive form is less common, with an incidence of
phenytoin and cimetidine. ACE inhibitors cause
1:20,000 live births and typically identified in the first
ischaemic nephropathy.
few weeks after birth.
Question 6
A. Renal osteodystrophy is the alteration of bone Question 11
morphology in patients with chronic kidney disease B. Renal cell carcinoma originates in the lining of the
(CKD). There are many radiological findings including proximal convoluted tubule. Symptoms include
osteopenia, salt and pepper skull, subperiosteal haematuria, flank pain, abdominal mass, weight
erosions, brown tumours, pseudofractures and loss, fevers, hypertension, night sweats and general
‘rugger jersey spine’ sign. Bamboo spine is a finding malaise. The recommended treatment is usually
seen in ankylosing spondylitis. surgical removal of all or part of the affected kidney.

153 
SBA answers

Question 12 This patient has gout, which is a risk factor for


B. This man is likely to be suffering from prostatic developing kidney stones due to high levels of uric
cancer. PSA is elevated in the condition and should acid. Most stones pass and therefore the most
be measured initially, but is not specific, as benign appropriate management is effective analgesia,
prostate hyperplasia can also cause PSA to be usually NSAIDs and opiates. Large stones that cannot
elevated. Transrectal ultrasound and prostate biopsy, pass naturally may require interventions, for example,
and magnetic resonance imaging are subsequently extracorporeal shockwave lithotripsy, ureteroscopy,
performed for disease confirmation and staging. percutaneous nephrolithotomy or even open surgery,
depending on the size and location of the stones.
Question 13
B. Teratomas are nonseminomatous germ cell tumours. Question 19
Most of them present with a lump in the testis, B. This patient has developed renal stones as a result of
which is usually painless, but can be painful or give a dehydration. Of the available options, abdominal X-ray
dragging sensation. Elevated levels of beta-hCG are is the most appropriate as it may reveal radio-opaque
almost always found with this type of tumour. Alpha- stones, but it may miss small stones or radiolucent
fetoprotein (AFP) level is usually also raised. These stones. A computed tomography KUB is a better
tumour markers are not raised in seminomas. imaging modality to investigate for renal stones and is
now the investigation of choice for suspected cases.
Question 14
B. Teratomas are nonseminomatous germ cell tumours. Question 20
Most of them present with a lump in the testis, A. Escherichia coli is the most common cause of
which is usually painless, but can be painful or give a community-acquired UTIs, with a frequency of over
dragging sensation. Elevated levels of beta-hCG are 80%. Staphylococcus saprophyticus causes 5%
almost always found with this type of tumour. AFP to 10% of cases. Catheter-associated UTIs can be
level is usually also raised. These tumour markers are caused by E. coli, Klebsiella, Pseudomonas, Candida,
not raised in seminomas. However, normal marker Proteus or Enterococcus.
levels do not exclude testicular cancer.
Question 21
Question 15 A. UTIs are generally rare in young men and may require
A. The man probably has a seminoma in view of no further investigations including HIV testing and STD
cystic spaces. It has metastasized to his lungs screening. Dysuria is the most frequently presenting
causing him to be short of breath. Following complaint. Urine microscopy and culture are the first-
orchidectomy, seminomas with metastases are best line investigations.
treated by chemotherapy. Metastases below the
diaphragm are best treated with radiotherapy. Question 22
E. This patient is likely to suffer from a severe UTI. IV
Question 16 antibiotics (e.g., gentamicin) is most appropriate.
C. Stress incontinence is due to the insufficient strength Trimethoprim/nitrofurantoin/amoxicillin are generally
of the closure of the bladder, resulting in loss of urine for uncomplicated UTIs where the patients are not
when coughing, laughing, sneezing, exercising or any acutely unwell.
movement that increases intraabdominal pressure.
Risk factors in females are pregnancy, childbirth and Question 23
menopause. A. This child is likely to suffer from nephrotic syndrome.
Patients present with a triad of significant proteinuria,
Question 17 hypoalbuminaemia and oedema. In children, the most
B. This patient is likely to suffer from overflow incontinence, common cause is minimal change disease. A 24-hour
which is characterized by the involuntary release of urine collection and serum albumin measurement will
urine from an overfull bladder. It occurs in people with a confirm the diagnosis. A renal biopsy may be indicated
blockage of the bladder outlet (e.g., from benign prostate to investigate the underlying cause. Treatment is usually
hyperplasia or prostate cancer). The gold standard for all with a course of oral corticosteroids.
urinary incontinence is a urodynamic study.
Question 24
Question 18 C. Minimal change disease is by far the most common
C. This patient has renal colic, commonly caused by cause of nephrotic syndrome in children aged
kidney stones. The pain is classically loin to groin. between 1 and 7. The hallmarks of the disease are

154
Renal 4

diffuse loss of podocyte foot processes, vacuolation Question 32


and the appearance of microvilli on histology. C. In bilateral renal artery stenosis, use of an ACE
inhibitor can cause AKI and lead to flash pulmonary
Question 25 oedema, which is what this patient has presented
B. Dalteparin. This patient has nephrotic syndrome, with. It presents with cough, dyspnoea, orthopnoea
which causes a hypercoagulable state due to loss of and bilateral fine inspiratory crackles. Other causes
antithrombin III in the urine. This patient has developed of flash pulmonary oedema include acute myocardial
a renal vein thrombosis. This presents with loin pain, infarction, acute respiratory distress syndrome, heroin
haematuria, a palpable kidney and sudden deterioration and cocaine use.
of renal function. This requires treatment with low-
molecular-weight heparin such as dalteparin. Following Question 33
this, warfarin should be given for at least 3 months. C. The build-up of urine has caused hydronephrosis,
which presents with enlarged kidneys, bladder and
Question 26 renal failure. Any cause of obstruction can lead to
B. This patient has developed nephrotic syndrome as a hydronephrosis. Causes include renal calculi, benign
result of focal segmental glomerulosclerosis, which is prostatic hyperplasia, malignancy (bladder cancer,
associated with both HIV and heroin use, but can also prostate cancer and urethral cancer), urethral stenosis
be idiopathic. and retroperitoneal fibrosis.

Question 27 Question 34
D. This patient has developed pericarditis as a result of E. ECG findings associated with hyperkalaemia include
uraemia, which is an indication for haemodialysis, prolonged PR interval, tall T waves, widened QRS
which would remove urea and therefore relieve this complexes, absent P waves and eventually sinusoidal
patient’s symptoms. waves (severe hyperkalaemia).

Question 28 Question 35
A. Normally in CKD, patients become anaemic due to E. On the contrary, hypercoagulability state is present
reduced production of erythropoietin. However, in in nephrotic syndrome. Venous thrombosis and
PKD there is production of erythropoietin from the pulmonary embolism are well-known complications
multiple cysts that form in the kidneys. Therefore of the nephrotic syndrome. Hypercoagulability in
these patients are less likely to become anaemic. these cases appears to derive from urinary loss of
anticoagulant proteins, such as antithrombin III and
Question 29 plasminogen, along with the simultaneous increase in
C. Ramipril. This patent has developed proteinuria as clotting factors, especially factors I, VII, VIII and X.
a result of nephrotic syndrome. Proteinuria is best
treated with an ACE inhibitor or an angiotensin II Question 36
receptor blocker. Human albumin solution would not A. Carpal tunnel syndrome is the tingling and pain of
reduce loss of albumin via the kidneys. the hands due to impingement of the median nerve
in the carpal tunnel of the hand. In dialysis patients,
Question 30 it is secondary to dialysis-related amyloidosis due to
A. This patient has developed rhabdomyolysis as a result deposition of β2-microglobulin.
of his prolonged muscular crush injury. The skeletal
muscle breakdown leads to myoglobin being released EMQ ANSWERS
into the bloodstream, which is nephrotoxic. First-
line treatment is IV fluid rehydration to prevent AKI. Renal disease
Haemodialysis may be required, but fluid rehydration 1. H  Typical presentation includes renal colic and
would occur first. haematuria. Diagnosis is usually made by
excretion urography.
Question 31 2. G  Typical features include acute loin pain,
A. Churg–Strauss syndrome is a small vessel P-ANCA- haematuria and abdominal discomfort. Individuals
positive (myeloperoxidase (MPO)-positive) vasculitis. It are at risk of developing hypertension.
can cause pulmonary–renal syndrome, which presents 3. A  The offending organism is typically β-haemolytic
with haemoptysis and haematuria. This patient has streptococci. Additional features include
developed nephritic syndrome and AKI as a result of a hypertension and oedema of the legs and
rapidly progressive glomerulonephritis. sacrum.

155 
EMQ answers

4. B  Management typically involves high-dose 3. F  The diagnosis is urinary tract obstruction, which
prednisolone. is most appropriately investigated by renal
5. D  NSAIDs are a common cause of tubulointerstitial ultrasound in the first instance.
nephritis. 4. C  This patient has a UTI. A urine dipstick, together
with urine microscopy and culture, is essential.
Complications of renal failure 5. J  This patient has stress incontinence, which is
1. D  Essentially osteosclerosis is an increase in bone most appropriately diagnosed by urodynamic
density. If present in the vertebrae it can give rise investigations.
to a ‘rugger jersey’ spine.
2. E  Congo red staining or tissue biopsy are the gold- Management of renal disease
standard methods of diagnosing amyloidosis. 1. D  This patient is likely to have benign prostatic
3. A  In this case, decreased erythropoietin formation hypertrophy. Alpha-blockers would help to relax
results in reduced red blood cell formation and the smooth muscle in the bladder neck and
subsequent anaemia. prostate.
4. F  This is commonly due to uraemia. Dialysis is 2. F  The diagnosis is renal colic. Analgesia is the
essential in such cases. mainstay form of management.
5. B  Classic description of osteomalacia. Renal failure 3. E  This patient is likely to be suffering from a UTI and
results in phosphate retention and impaired would require antibiotic therapy.
production of vitamin D. 4. G  The diagnosis is nephrotic syndrome most
appropriately treated with steroid therapy.
Clinical features of renal disease 5. C  The diagnosis is renal cell carcinoma most
1. B  Classic presentation of nephrotic syndrome. appropriately managed by surgical
2. G  Additional features include neurological and intervention.
cardiovascular abnormalities.
3. I  Patients may also experience notable weight loss Disease of the tubules and interstitium
and bone or chest pain due to metastases. 1. E  UTI. Common in pregnancy due to high levels of
4. E  The patient has gout, which may be associated progesterone and smooth muscle relaxation.
with uric acid stone formation. 2. K  Acute pyelonephritis. The ascending infection
5. A  Classic presentation of nephritic syndrome. causes the systemic signs.
3. J  Ischaemic acute tubular necrosis. The blood loss
Investigations following the trauma causes hypoperfusion and
1. D  Renal failure may be precipitated by ACE inhibitors ischaemia.
in patients with renal artery stenosis (pressured 4. D  Urate nephropathy. In tumour lysis syndrome, the
atheromations in this case). Magnetic resonance sudden breakdown of tumour cells releases large
angiography is the gold standard for the diagnosis quantities of uric acid. This can lead to AKI.
of renal artery stenosis. 5. G  Chronic pyelonephritis. Infection (usually in early
2. E  The diagnosis is renal colic, which may be childhood) results in chronic scarring and can lead
diagnosed by abdominal X-ray of the kidneys, to hypertension and chronic renal failure. This is a
ureters and bladder. T-cell-mediated inflammatory response.

156
Neurology 5
SBA ANSWERS Question 7
E. Motor neurone disease is a clinical diagnosis and
Question 1 needs no investigative involvement. Investigations are
A. The diagnosis is myasthenia gravis (MG), which is usually performed only to rule out other neurological
initially investigated by demonstrating the presence of problems with a similar presentation.
acetylcholine receptor antibodies. Single-fibre EMG
is more sensitive for diagnosis than acetylcholine Question 8
receptor antibodies but is less widely available. B. Classic presentation of Friedreich ataxia, which is an
autosomal recessive condition. Additional features
include pes cavus, scoliosis, diabetes and cardiac
Question 2
conditions (e.g., atrial fibrillation and hypertrophic
E. The patient is demonstrating myotonia: a delay
cardiomyopathy).
in muscle relaxation (prolonged contraction) after
voluntary contraction or electrical stimulation. Causes
include myotonic dystrophy, myotonia congenita and
Question 9
A. The likely diagnosis is syringomyelia, which is a
paramyotonia congenita.
disorder in which a cyst/cavity called a ‘syrinx’
forms within the spinal cord that can elongate over
Question 3 time. It leads to a cape-like bilateral loss of pain and
C. Typical presentation of the X-linked recessive
temperature sensation along the back and arms, and
condition Duchenne muscular dystrophy. Symptoms
also causes paralysis and weakness. The investigation
of muscle weakness usually present around the age
of choice is spinal MRI studies.
of 4 in boys and progress quickly. Muscle loss occurs
first in the upper legs and pelvis resulting in difficulty
Question 10
standing up. Becker muscular dystrophy usually
B. This patient has spinal cord compression, which is a
presents later but may have similar clinical features.
medical emergency requiring prompt diagnosis and
treatment to prevent irreversible spinal cord injury. The
Question 4 patient should be referred to the neurosurgical team
B. Alcoholics are often at risk of thiamine (vitamin B1) for surgical decompression.
deficiency, which can lead to neurological abnormalities
such as ataxia, nystagmus and ophthalmoplegia Question 11
(Wernicke encephalopathy). Long term, there is a risk of C. Typical presentation of a cluster headache. The
brain damage and memory loss (Korsakoff syndrome). cause is unknown but risk factors include exposure
to tobacco smoking and family history. Additional
Question 5 autonomic features include rhinorrhoea and ptosis.
E. The diagnosis is Guillain–Barré syndrome, which is an
autoimmune disorder that causes rapid-onset muscle Question 12
weakness starting from extremities and spreading B. Giant-cell arteritis is a form of vasculitis affecting
to respiratory muscles. Over 50% of patients large and medium vessels. Visual loss is due to
with Guillain–Barré syndrome have experienced occlusion of the ciliary and central retinal arteries.
an infection before the onset of the condition, for Prompt diagnosis is required as a delay could cause
example, gastroenteritis (Campylobacter jejuni) or irreversible ischaemia and blindness. Treatment, if
respiratory tract infection. clinically suspected, is with glucocorticoids.

Question 6 Question 13
C. Typical presentation of median nerve damage, which E. The diagnosis is giant-cell arteritis. High-dose
results in lack of ability to abduct and oppose the steroids (prednisolone) should be the first choice
thumb due to paralysis of thenar muscles, weakness of management in this case, which can reduce
in forearm pronation and wrist and finger flexion, inflammation and prevent vascular occlusion. The
sensory loss in the thumb, index finger, middle finger dose of prednisolone is then slowly tapered over a
and radial aspect of the ring finger. course of months to years.

157 
SBA answers

Question 14 cefotaxime or ceftriaxone as recommended by the


A. The diagnosis is giant-cell arteritis. This condition Meningitis Research Foundation.
leads to visual loss due to central retinal artery
occlusion. As a result, the retina appears white with a Question 22
cherry red spot on the macula. B. LP is the appropriate investigation in suspected
meningitis. However, LP is contraindicated if there
Question 15 is a space-occupying lesion or raised intracranial
D. Typical presentation of a migraine. This typically pressure as this may lead to brain herniation, so a CT
starts with a prodrome phase (e.g., altered mood is recommended prior to LP in these patients.
and craving for certain foods). This is followed by an
aura phase, which can be visual, sensory or motor in Question 23
nature. The pain phase follows, which is classically a E. Acute bacterial: low glucose, high protein,
unilateral, throbbing severe headache lasting hours. neutrophil predominant; acute viral: normal glucose,
The pain accompanies vomiting and photosensitivity. normal or high protein, lymphocyte predominant;
The attack ends with a postdrome, which is like a tuberculosis: low glucose, high protein, lymphocyte
‘hung over’ sensation. predominant.

Question 16 Question 24
A. The diagnosis is a migraine. This is believed to be a E. The diagnosis is multiple sclerosis (MS). The main
neurovascular disorder but many neurotransmitters, characteristics are the formation of plaques in the
such as serotonin, appear to play a role in the central nervous system, inflammation and destruction
disease. of myelin sheaths of neurons. The peripheral nervous
system is rarely involved.
Question 17
D. Because of his past medical history, ergotamine Question 25
and sumatriptan are contraindicated. The most A. The most likely diagnosis here is MS. History and
appropriate management would be analgesia and examination alone may be sufficient to diagnose
antiemetics. Pizotifen and methysergide are only MS but investigations may be required. This usually
useful if attacks are frequent in nature. begins with brain MRI, which may show areas of
demyelination.
Question 18
C. The frontal lobe plays a role in higher mental functions Question 26
(e.g., motivation, planning, social behaviour and E. Testing of CSF obtained from an LP can provide
speech production). Symptoms include movement evidence of chronic inflammation in the central
disorders (e.g., tremor and clumsiness), emotional nervous system. CSF is tested for oligoclonal bands
disorders (e.g., disinhibition and depression), of immunoglobulin G (IgG) on electrophoresis, which
behavioural disorders (e.g., compulsive eating) and are inflammatory markers found in over 70% patients
language disorders (e.g., aphasia). with MS.

Question 19 Question 27
B. Owing to distortion of normal structures at a distance D. This is the most common movement disorder. Over
from the expanding tumour, a third and sixth nerve half of cases are found to be autosomal dominant.
palsy are common additional findings. The tremor characteristically improves with alcohol
and propranolol and is made worse by any sort of
Question 20 physical/mental stress.
A. In adults, Neisseria meningitidis and Streptococcus
pneumoniae together cause 80% of bacterial Question 28
meningitis cases. In premature babies and newborns, A. Chorea is an abnormal involuntary movement
group B streptococci is common. Risk of infection with disorder, characterized by brief, semidirected, irregular
Listeria monocytogenes is higher in elderly individuals. movements that are not repetitive or rhythmic, but
appear to flow from one muscle to the next. This
Question 21 patient is most likely to be taking levodopa for his
B. Clinical findings are suggestive of meningococcal Parkinson’s, which is known to cause chorea as a
septicaemia, which requires immediate use of side effect.

158
Neurology 5

Question 29 Question 35
C. Myoclonus is a brief, involuntary twitching of a B. Also commonly known as a petit mal seizure.
muscle or a group of muscles. The most common This is characterized by a brief loss and return of
circumstance under which they occur is while falling consciousness, without a postictal state. These
asleep. It is seen in healthy people when it occurs absences can be easily induced by hyperventilation in
occasionally, but persistent myoclonus can be seen most cases.
in a wide variety of diseases (e.g., Parkinson disease,
Alzheimer disease (AD) and Creutzfeldt-–Jakob Question 36
disease (CJD)). C. The presentation is that of a Jacksonian seizure,
where a simple partial seizure spreads from the distal
Question 30 part of the limb to the rest of the ipsilateral side of
D. Dystonia is a neurological movement disorder body. It can lead to paralysis of involved limbs for
whereby sustained or repetitive muscle contractions several hours.
result in twisting and repetitive movements or
abnormal fixed postures. Causes include hereditary, Question 37
physical trauma, infection, lead poisoning and D. The diagnosis is a temporal lobe seizure due
reaction to neuroleptics. Botulinum helps to inhibit to experience of olfactory hallucinations. Visual
the release of acetylcholine from nerve endings into hallucinations and experience of déjà vu are also
muscle. common.

Question 31 Question 38
E. The diagnosis is Parkinson disease and this patient D. Sodium valproate commonly causes nausea,
is suffering from motor symptoms of the disease, vomiting, drowsiness, dizziness and weakness. It has
which is the result of reduced dopamine production a black box warning for hepatotoxicity, pancreatitis
in the basal ganglia. Levodopa is the initial treatment and teratogenicity.
of choice, which can pass through the blood–brain
barrier where it is readily converted to dopamine. Question 39
However, long-term use of levodopa leads to A. Common side effects include nausea, loss
development of dyskinesias and fluctuations in the of appetite, increased hair growth, gingival
effectiveness of the medication. hyperplasia. Other side effects include folate
deficiency leading to megaloblastic anaemia,
Question 32 cerebellar features (nystagmus, ataxia, tremor),
A. This patient has Parkinson disease. The pathogenesis vitamin D deficiency and osteomalacia. This is
of this condition is primarily the result of depletion of done by the induction of enzymes in the liver that
dopamine-containing neurones in the substantia nigra metabolize vitamin D.
and basal ganglia.
Question 40
Question 33 C. Ethosuximide is used in the treatment of absence
C. Parkinson plus syndromes are a group seizures. Common side effects include abdominal
of neurodegenerative diseases featuring pain, fatigue and loss of appetite. Serious side effects
the classical features of Parkinson disease include pancytopenia, suicidal thoughts, night terrors,
with additional features that distinguish them paranoid psychosis, systemic lupus erythematosus
from simple idiopathic Parkinson disease. This and Stevens–Johnson syndrome.
patient is likely to suffer from progressive
supranuclear palsy. Question 41
E. Diazepam given rectally is a useful alternative if
Question 34 intravenous access is difficult.
A. Tonic-clonic seizure is a type of generalized seizure
that affects the entire brain, consisting of a tonic Question 42
phase (tense skeletal muscles), clonic phase D. Blood glucose measurement should be
(violent shaking, eyes rolling back, tongue biting, taken immediately to ensure the patient is not
jaw contractions, peripheral cyanosis, urinary hypoglycaemic. A CT scan should then be arranged
incontinence) and postictal phase (drowsiness, to ensure there is no evidence of an intracranial
confusion, total amnesia). haemorrhage.

159 
SBA answers

Question 43 indicative of stenosis and there is a potential for


A. The most likely diagnosis is a subarachnoid carotid endarterectomy depending on the degree of
haemorrhage with features of raised intracranial stenosis.
pressure. An urgent CT head scan is the initial
investigation. Question 52
E. High-dose aspirin is the first-line management
Question 44 following a cerebral infarct. He has presented too late
B. A yellow-coloured supernatant is diagnostic of a to be considered for thrombolysis.
subarachnoid haemorrhage and is due to lysis of red
blood cells. Question 53
D. Opening eyes spontaneously = 4, Confused speech =
4, Withdrawing to pain = 4.
Question 45
C. Subdural haematoma is usually associated with
traumatic brain injury, causing tears in bridging veins Question 54
that cross the subdural space. Onset is slower than A. A scotoma is an area of depressed vision within the
other brain haemorrhages. They are often seen visual field. Such a finding is seen in lesions of the
in elderly or alcoholic patients who have cerebral optic nerve which are commonly associated with
atrophy. This is because the length the bridging veins demyelination, as is the case with MS.
have to traverse between the two meningeal layers is
longer, therefore increasing the likelihood of shearing Question 55
forces causing a tear. A. The nerve also supplies the levator palpebrae
superioris (responsible for upper eyelid) and sphincter
Question 46 pupillae (responsible to pupil constriction). Therefore
E. The diagnosis is most likely to be an extradural third nerve palsy results in pupillary dilatation.
haemorrhage due to temporal bone fracture and
subsequent rupture of the middle meningeal artery. Question 56
D. The diagnosis is a third nerve palsy. Aneurysms of
Question 47 the posterior communicating artery are the third most
D. Such neurological symptoms that improve within common circle of Willis aneurysm and this can lead to
24 hours are indicative of TIAs. The risk of stroke oculomotor nerve palsy.
occurring after a TIA can be predicted using the
ABCD2 score. Question 57
C. Ramsay Hunt syndrome is the reactivation of herpes
zoster in the geniculate ganglion. It results in a lower
Question 48
motor neurone facial palsy, deafness, vertigo and
E. Transient global amnesia is associated with the
pain. The triad of ipsilateral facial paralysis, ear pain,
vertebrobasilar vasculature and not the carotids.
and vesicles on the face/ear/in the ears is the typical
presentation.
Question 49
B. The most likely diagnosis is a cerebral hemisphere
Question 58
infarct caused by occlusion of the middle cerebral
B. This patient is experiencing vertigo as a result of
artery.
gentamicin toxicity to the vestibular apparatus. Vertigo
indicates a disturbance of the inner ear, eight cranial
Question 50 nerve or brainstem.
D. The diagnosis is lateral medullary syndrome caused
by occlusion of the PICA. Additional features may Question 59
include palatal paralysis, nystagmus and Horner E. Pseudobulbar palsy is the result of damage of
syndrome. motor fibres travelling from the cerebral cortex
to the lower brainstem. Patients suffer from
Question 51 slow and indistinct speech, dysphagia, brisk
A. The most likely diagnosis is an ischaemic stroke. A jaw jerk, dysarthria. In pseudobulbar palsy, the
head CT scan is the initial investigation to exclude a tongue is small and spastic. There are no notable
haemorrhage, although an infarct may not be shown fasciculations. Causes include progressive
in the early stages. In the case of an infarct, carotid supranuclear palsy, amyotrophic lateral sclerosis,
dopplers would then be performed as the bruit is MS and various motor neuron disease.

160
Neurology 5

Question 60 60–90 seconds and is followed by brief postictal


B. As MS may affect the cerebellum, horizontal confusion. However, generalized weakness, asthenia
nystagmus is typically seen. Vertical nystagmus is and fatigue may last for a few days.
seen in syringobulbia and Budd–Chiari malformation.
Question 64
Question 61 E. The frontal lobe is the area of the brain most closely
B. The clinical manifestations presented in this case associated with personality. Patients who develop
study are indicative of fat embolism. Fat embolism is a neoplasm in this area often present with profound
a common accompaniment of fractures of long bones disturbances in their personality; loved ones will
and is usually asymptomatic. It mainly affects the mention a distinct change and uncharacteristic
lungs and brain and is characterized by dyspnoea, behaviour.
tachycardia and changes in the mental state of the
individual. In rare cases, this condition can lead to Question 65
death. Fat embolism in the lungs can be histologically A. CJD is a degenerative disorder of the central
visualized. Presence of multifocal petechiae in the nervous system that is caused by accumulation of
white matter of the brain region represents the most abnormally folded protein (PrPsc) particles termed
profound pathological change. ‘prions’. Normal prion protein is termed PrPc
(cellular), whereas an abnormal, pathogenic isoform
Question 62 of the prion protein is designated PrPsc. It usually
C. NPH is not a disease but a clinical symptom presents in late middle age (50–75 years), with rapidly
characterized by abnormal gait, urinary incontinence progressive dementia, ataxia, dysarthria, myoclonic
and dementia. This occurs due to impaired fasciculations, somnolence and eventually death,
reabsorption of CSF at the arachnoid villi. Often, NPH usually following pneumonia, within a year of onset.
is misdiagnosed as AD or Parkinson disease due to MRI typically shows bilateral areas of increased
the nature of the symptoms and the chronic nature intensity, predominantly in the caudate and putamen.
of the disease. NPH is a reversible condition and can On light microscopy, the pathologic hallmarks of
be easily cured with treatment. One of the treatment CJD are spongiform degeneration, astrogliosis and
modes include insertion of a ventricular shaft to drain the lack of an inflammatory response. Spongiform
off the excess CSF. There have been some pieces of changes occur in the putamen, caudate nucleus,
evidence suggesting the development of NPH several cerebral cortex, thalamus and cerebellum. The
years after sustaining a subarachnoid haemorrhage, amyloid plaques that are seen in about 10% of cases
head injury, cranial surgery or central nervous system are histologically different from those seen in scrapie
infection. or Kuru. There is no known effective therapy for
treating or preventing CJD, with the exception of the
Question 63 prevention of iatrogenic cases.
C. Seizures originating in one area of the brain that
affect consciousness are labelled as complex partial Question 66
seizures. In most patients, complex partial seizures D. AD is initially associated with memory impairment
represent underlying temporal lobe epilepsy. The that progressively worsens. Vascular dementia results
predominant symptoms occurring during a seizure in a stepwise deterioration. Repetitive statements
event determine the seizure type. Symptoms start or movement and occasional muscle twitches are
with aura (a simple partial seizure), which is a features of the moderate presentation of AD. Agnosia
subjective sensation and is the initial part during means failure to identify or recognize objects despite
which the patient is aware. Typically, it is of brief intact sensory function. It is one of the criteria in
duration, rarely lasting longer than seconds. Followed cognitive disturbances.
by impaired consciousness, this implies decreased
responsiveness and awareness of one’s self and Question 67
surroundings. Usually, during a complex partial A. Migraine is a chronic neurological disorder
seizure, a patient is unresponsive and does not characterized by recurrent moderate to severe
remember events that occurred. Automatisms are headaches often in association with a number of
nonpurposeful, stereotyped and repetitive behaviours autonomic nervous system symptoms. Typically, the
that commonly accompany complex partial seizures. headache is unilateral (affecting one-half of the head)
The most common automatisms, at least in temporal and pulsating in nature, lasting from 2 to 72 hours.
lobe epilepsy, are oral (e.g., lip smacking, chewing, In more than 40% of cases, however, the pain may
swallowing) and manual (e.g., picking, fumbling, be bilateral, and neck pain is commonly associated.
patting). A typical complex focal seizure lasts about Associated symptoms may include nausea,

161 
EMQ answers

vomiting, photophobia (increased sensitivity to light), Question 71


phonophobia (increased sensitivity to sound) and C. The radial nerve is usually injured due to compression
the pain is generally aggravated by physical activity. in the spiral groove of proximal humerus as it winds in
Up to two-thirds of people with migraine headaches the groove due to improper positioning of the arm for
perceive no aura. long periods. The symptoms of compression are wrist
drop, inability to extend fingers, thumb abduction and
Question 68 sensory loss in dorsal web between the thumb and
A. Cluster headache is a rare form of primary index finger.
headache. It occurs in recurrent bouts of deep
pain, usually retro-orbital and often excruciating in Question 72
intensity, nonfluctuating and explosive in quality. D. MG is a disease of the motor endplate, most often
Episodes occur more frequently during the night. affecting the cranial nerves, especially the oculomotor
The typical cluster headache patient has daily nerves and the eyelids. The disease begins with
bouts of one to two attacks of relatively short- weakness and fatigue but can progress over days to
duration, unilateral pain for a dozen weeks a year; months. Confirmatory testing consists of acetylcholine
this is usually followed by a pain-free interval that receptor antibody testing or EMG when the situation
averages a little less than 1 year. Patients are is not urgent. When the diagnosis must be made
generally perfectly well between episodes. Patients promptly, the old-fashioned Tensilon test gives
with cluster headache tend to move about during immediate and dramatic results.
attacks, pacing, rocking or rubbing their head for
relief; some may even become aggressive during Question 73
attacks. A. MG is an autoimmune disease. Antibodies bind to
the nicotinic receptor located at the postsynaptic
Question 69 neuromuscular junction. Patients usually progress
E. Viral labyrinthitis (vestibular neuronitis) can present with progressive muscular weakness. Over 80%
suddenly, with a prodrome of viral symptoms as of patients with MG have thymic abnormalities,
is in this case. The vomiting is as a cause of the with 15% having a thymoma or thymic malignancy.
vertigo symptoms. Tertiary syphilis (late symptomatic Expansion of thymic tissues results in a widened
syphilis) consists of end-organ damage (neurosyphilis, mediastinum.
cardiovascular syphilis, gummatous syphilis).
Neurosyphilis can result in damage of the dorsal EMQ ANSWERS
columns of the spinal cord (tabes dorsalis). Features
include ataxia, incontinence, Argyll Robertson pupils Delirium
and dorsal column loss (loss of proprioception and 1. B  Diabetic ketoacidosis. This is a classic first
vibration). Brain involvement can result in personality presentation of diabetes, with children more likely
changes, memory impairment, seizures and to complain of abdominal pain.
confusion. 2. E  Hypercalcaemia. This is a medical emergency,
the mainstay of treatment being rehydration and
Question 70 bisphosphonates.
C. The accessory nerve is a motor nerve. It consists 3. F  Hypercapnia. Respiration in this situation is driven
of a small cranial root that is distributed through by hypoxia, highlighting the potential dangers of
the branches of the vagus nerve to the muscles oxygen therapy.
of the soft palate, pharynx and larynx, and a large 4. K  Subdural haematoma. A classic presentation which
spinal root that innervates the sternocleidomastoid is common in elderly individuals and alcoholics;
and trapezius muscles. There are many causes for often insidious in onset, and easily missed.
injury to the accessory nerve or its spinal branch 5. C  Encephalitis. Herpes simplex is the commonest
including lymph node biopsy in the posterior cause of viral encephalitis in the UK, but insect-
triangle of the neck, radical neck dissection and borne causes are important in other parts of the
penetrating injuries. Shoulder pain is the most world.
common presenting symptom, and limited or loss
of sustained abduction of the shoulder is the most Cranial nerve lesions
common sign. The ipsilateral shoulder may droop 1. C  Characteristic features of a third nerve palsy.
(trapezius causes elevation of the whole shoulder 2. G  Additional features include undue sensitivity to
girdle) and scapular winging or prominence of the sound.
medial border of the scapula and protraction may be 3. D  Typical presentation of damage to the trochlear
found. nerve.

162
Neurology 5

4. B  This visual loss is commonly known as a scotoma, 2. H  This is usually caused by infarction or
which may occur centrally or paracentrally. haemorrhage in the contralateral subthalamic
5. E  Such lesions may also be associated with jaw nucleus.
deviation to the side of the lesion. 3. G  An autosomal dominant condition where typical
features include chorea, personality change and
Diseases of the peripheral nerves dementia.
1. F  Also seen in patients with hypothyroidism. Carpal 4. J  Sudden, repetitive, nonrhythmic motor movement/
tunnel syndrome may also be associated with vocalization involving discrete muscle groups.
sensory loss of the palm and radial three and a Common tics including eye blinking and throat
half fingers. clearing.
2. D  Hereditary (duplication at chromosome 17) 5. B  Sites typically affected include the optic nerves,
motor and sensory neuropathies of the peripheral the brainstem, the cerebellum and the cervical
nervous system. spinal cord.
3. E  Autoimmune disorder that causes rapid-onset
muscle weakness beginning distally and spreading Headache 1
proximally, affecting respiratory muscles. 1. H  Subarachnoid haemorrhage. This type of
4. B  Chronic thiamine deficiency can lead to Wernicke headache is often described as ‘the worst type of
encephalopathy and Korsakoff syndrome. pain ever felt’ or as ‘feeling like you have been hit
5. A  Isoniazid is associated with vitamin B6 (pyridoxine) over the back of the head by a cricket bat’.
deficiency, which can lead to skin rashes, atrophic 2. J  Tension headache. Commonest type of primary
glossitis with ulceration and sensory neuropathy. headache. The lack of associated symptoms and
stable nature of the headache suggest this benign
type of headache. The headaches usually respond
Diseases of the muscle well to simple analgesia.
1. J  Duchenne muscular dystrophy is an X-linked
3. C  Giant-cell arteritis. Medium to large vessel
recessive condition caused by a mutation on
vasculitis usually in elderly patients causing
chromosome 21. Onset is usually 4 years of age
headache, jaw claudication and tenderness on the
and death is common in the late teens.
scalp. It may lead to visual loss if treatment with
2. C  Additional features include fatigability of
high-dose steroids is delayed.
extraocular muscles and muscles of facial
4. E  Meningitis. Most infections are caused by viruses
expression.
(enteroviruses, herpes simplex virus, Varicella
3. D  Additional features include cardiomyopathy,
zoster virus), followed by bacterial (Streptococcus
glucose intolerance and hypogonadism.
pneumoniae, N. meningitidis) and fungal
4. E  Either autosomal dominant or recessive. The
(Cryptococcus neoformans) agents. The absence of
myotonia is often made worse by periods of inactivity.
neck stiffness does not alter the working diagnosis.
5. A  An autosomal recessive condition that can
progress to severe disability within 20 years.
Nerves of the upper limb
1. C  Lower brachial plexus. This boy has arrested his
Epilepsy fall with an outstretched arm and there has been
1. A  Tonic-clonic seizures are also frequently
stretching of the lower brachial plexus. This may
associated with jaw claudication and tongue biting
be a neuropraxia which will hopefully resolve.
and urinary incontinence.
2. E  Median nerve. This lady describes a classical
2. B  Absence seizures cause a brief loss and return
carpal tunnel syndrome. Symptoms may be worse
of consciousness, usually without an obvious
at night due to fluid redistribution (especially in
postictal state.
pregnancy) and flexion of the wrist.
3. D  A simple partial seizure that can lead to paralysis
3. H  Radial nerve. This injury is associated with
of the involved limb(s) for several hours.
fractures of the proximal humerus. This gentleman
4. A  Carbamazepine or sodium valproate are also
has both motor and sensory loss.
useful agents.
4. A  Axillary nerve. This injury is also associated with
5. E  Additional features include olfactory and visual
fractures of the proximal third of the humerus. The
hallucinations/distortions.
anaesthesia over the regimental patch can vary
considerably.
Extrapyramidal diseases 5. B  Long thoracic nerve of Bell. The long thoracic
1. D  This is progressive supranuclear palsy, which is a nerve passes through the axilla and innervates
Parkinson plus syndrome. There is often a poor serratus anterior which, when injured, causes
response to levodopa in these cases. winging of the scapula.

163 
EMQ answers

Headache 2 3. F  PICA occlusion. Also known as the lateral


1. B  Benign intracranial hypertension. Headache medullary syndrome or Wallenberg syndrome, a
worsens with activities that further increase very specific constellation of symptoms makes up
intracranial pressure (e.g., coughing and this syndrome.
sneezing). This is often described as affecting 4. H  TIA. This lady has an episode of focal neurology,
young obese women; the underlying cause is of presumed vascular origin, that lasted less
unknown. than 24 hours. She requires a full work up and
2. F  Giant-cell arteritis. This is a vasculitic disorder treatment of her vascular risk factors.
affecting the temporal artery and classically causes 5. B  PACI. This man has two of the three factors for
thick pulseless temporal arteries. Diagnosis is diagnosis of a total anterior syndrome, and a CT
confirmed by biopsy, though sampling error is scan demonstrating an infarct makes this a PACI
common as the vasculitis does not affect the rather than a PACS.
whole vessel. Treatment is with high-dose steroids
and should not be delayed, tapering down over Headache 3
months to weeks. 1. F  Extradural haematoma. Between the dura
3. E  Cluster headache. These are migrainous mater and the skull; usually with lucid interval
headaches that occur in clusters, which may immediately following the trauma before
be separated by months or years. The patient symptoms occur. About 20% of cases are fatal.
describes fortification spectra preattack. It is often 2. K  Viral encephalitis. Clinical presentation may be
accompanied by eye watering, eye swelling and similar to bacterial meningitis; hence CSF sampling
nasal congestion. must be obtained to differentiate the two.
4. J  Tension headache. This gentleman is suffering Common viruses include enteroviruses, herpes
from tension headaches, or muscle contraction simplex and varicella zoster virus.
headaches as they are also known. This seems 3. J  Idiopathic intracranial hypertension. Cause
to be brought on by stress in his current post, unknown but more common in obese females. If
although other causes should be excluded. uncontrolled, can lead to visual loss. Treatment is
Treatment with simple analgesia is usually mainly via reduction of CSF pressure (e.g., by LP)
effective. and lifestyle modification (e.g., weight loss).
5. A  Bacterial meningitis. This is more common among 4. E  Subdural haematoma (chronic). More common
those living in crowded conditions (such as in alcoholics and elderly individuals. Onset can
students). Kernig sign, headache and photophobia be gradual and progressive because the lower
should always be treated as meningitis until pressure bridging veins bleed at a slower rate than
proven otherwise. The CSF findings here are arteries.
consistent with bacterial cause. 5. D  Tension headache. Commonest type of primary
headache; no pathology on CT; caused by stress,
Strokes sleep deprivation and eye strain. Treatment with
1. G  TACS. This man has the required three features to simple analgesia is usually effective.
make up a total anterior circulation syndrome. 6. H  Cerebral infarct. This patient has a history of
2. A  Lacunar infarct. Caused by occlusion of a single angina and peripheral vascular disease; hence
deep penetrating artery that supplies the brain’s she is a high risk for developing a stroke. Lesions
deep structures (e.g., internal capsule, pons, in the internal capsule cause contralateral
corona radiata, basal ganglia and thalamus). hemiparesis/hemiplegia.

164
Rheumatology 6
SBA ANSWERS Question 6
E. Methotrexate is a disease-modifying antirheumatic
Question 1 drug (DMARD), which helps to minimize symptoms,
B. Osteoarthritis is a degenerative joint disease that prevent irreversible joint damage and maintain
commonly affects the following: DIP joints and first activities of daily living. NSAIDs and COX-2 inhibitors
carpometacarpal joints of the hand, cervical and have no disease-modifying effect and are used to
lumbar spine, knee and hip joints. manage pain. Tumour necrosis factor-α blockers are
monoclonal antibodies, which are also DMARDs, but
they are not first-line treatment. Surgery is confined to
Question 2 very severe situations.
D. The likely diagnosis is osteoarthritis. Stiffness is often
worse in the morning and typically lasts less than 30
Question 7
minutes. The most useful investigation is an X-ray
A. Ankylosing spondylitis is an autoimmune disease
of the knee, which may demonstrate the following
primarily affecting the joints of the spine. Onset is
typical osteoarthritic changes: joint space narrowing,
usually <40 years of age. It initially presents with
subchondral sclerosis, subchondral cysts and
pain and stiffness of the lower back. As disease
osteophytes.
progresses, patients experience loss of mobility in the
spine and chest expansion, resulting in a kyphotic
Question 3 posture. Extraarticular manifestations include uveitis,
E. Osteoarthritis can result in the formation of hard, bony aortic valve incompetence and pulmonary fibrosis.
outgrowths on the PIP joints (Bouchard nodes) or
DIP joints (Heberden nodes) of the hands. Lifestyle
Question 8
changes, exercise, physiotherapy and analgesia are
D. Reactive arthritis, also known as Reiter syndrome, is
the mainstay of treatment. Paracetamol is the first
a type of inflammatory arthritis that develops following
line of treatment in terms of analgesia as it is the
an infection, commonly urinary tract infection. The
simplest and safest. NSAIDs are the next step in
triad of symptoms consists of arthritis of large joints,
treatment if pain is not controlled by paracetamol.
urethritis and conjunctivitis/uveitis.
Heat and hydrotherapy may be used in conjunction
with analgesia. Surgery and steroids would only be
Question 9
considered if the aforementioned modalities fail to
B. Psoriatic arthritis is a chronic inflammatory arthritis
control symptoms.
in patients with psoriasis. There are five main types
of the disease depending on the joints involved:
Question 4 oligoarticular, polyarticular (rheumatoid arthritis-like),
C. This is a case of rheumatoid arthritis. It commonly DIP joint predominant, spondyloarthritis and arthritis
affects younger women, and primarily presents mutilans. Psoriasis can also cause nail changes
with pain, stiffness and swelling of the PIP joints (e.g., pitting and onycholysis).
and wrist joints symmetrically. It can also affect
multiple organs in the body, such as the skin causing Question 10
rheumatoid nodules over the elbows and heels. RF E. This is a case of septic arthritis and is a medical
and anticyclic citrullinated peptide antibodies are the emergency. The joint inflammation is caused by invasion
best investigations, but up to 25% of cases can be of an infectious agent. The most common cause in
seronegative. adults (as in this case) is Staphylococcus aureus.
Prompt diagnosis is required and treatment with
Question 5 antibiotics should be initiated if clinically suspected.
A. Rheumatoid arthritis is a multiorgan disease. Common
signs in the joints include swelling of the PIP joints, ulnar Question 11
deviation, swan neck deformity, boutonniere deformity B. The patient is probably suffering from SLE. Up to 70%
and Z thumb. Anaemia is a common extraarticular of patients have skin manifestations including malar
manifestation of the condition, where chronic rash, discoid lupus and photosensitivity. ANAs are
inflammation results in anaemia of chronic disease. the mainstay of serological testing.

165 
EMQ answers

Question 12 medications such as thiazide use. The treatment of


C. The woman is suffering from scleroderma, which is choice is with NSAIDs or colchicine if NSAIDs are
an autoimmune disease of the connective tissue. contraindicated.
The limited form is called CREST syndrome, which
consists of calcinosis, Raynaud phenomenon, Question 18
oesophageal dysfunction, sclerodactyly and D. This woman has pseudogout secondary to
telangiectasia. The diffuse form also affects the haemochromatosis. It is a form of inflammatory
gastrointestinal tract, kidneys, heart and lungs. arthritis formed from the deposition of calcium
Diagnosis is usually achieved by clinical examination pyrophosphate crystals in and around the joint. The
and the presence of autoantibodies in the blood, presentation is similar to gout, but in pseudogout joint
specifically anticentromere antibodies and fluid microscopy would demonstrate brick-shaped
antitopoisomerase antibodies. crystals, which would be positively birefringent under
polarized light.
Question 13
A. Dermatomyositis is a chronic autoimmune Question 19
inflammatory disorder of the muscles. It presents D. This woman suffers from osteomalacia, which is
with a periorbital heliotrope rash around the eyes a form of metabolic bone disease caused from an
and proximal myopathy. Other features include inadequate mineralization of bones in the body.
erythematous plaques over the fingers and knuckles This is due to a deficiency in calcium, vitamin D
called Gottron papule, arthritis and respiratory or phosphate, or increased calcium resorption.
disorders. Symptoms include generalized arthralgia and bony
pains, muscle weakness (especially the proximal
Question 14 muscles), causing a difficulty in climbing stairs or rising
E. Polymyositis causes proximal muscle weakness and up from squatting position, and a waddling gait.
myalgia. There is no skin involvement in polymyositis (as
opposed to dermatomyositis). Systemic features include Question 20
conduction abnormalities of the heart, interstitial lung D. Lifestyle changes such as smoking cessation
disease and increased risk of malignancy. Diagnosis and increasing weight-bearing exercise are very
is by history and clinical examination, combined with important in the management of osteoporosis, but
creatine kinase levels and EMG. Confirmation of disease bisphosphonates are the most appropriate step in the
is by muscle biopsy, which demonstrates inflammation management of this patient as they act to decrease
of the endomysium of muscle. the risk of further fractures in those who have already
sustained a fracture secondary to osteoporosis.
Question 15
A. This patient is likely to suffer from Sjögren syndrome, Question 21
a chronic autoimmune disorder affecting primarily B. Paget disease of the bone is a type of metabolic
sweat and tear glands, resulting in dry eyes and dry bone disease, where there is excessive breakdown
mouth. Anti-Ro and anti-La antibodies are present in and disorganized remodelling of the bone. The
60% to 70% of patients. most common symptom is bony pain. Diagnosis
is achieved by an elevated level of serum ALP in
Question 16 the presence of normal calcium, phosphate and
B. Henoch–Schönlein purpura is a type of systemic alanine aminotransferase levels. Plain X-rays show
vasculitis. The classic triad of symptoms are arthritis, characteristic changes, which can be analyzed in
purpura and abdominal pain. The purpuric rash more detail by performing a full body skeletal survey.
is often on the legs and buttocks. About 40% will
also present with haematuria. The condition is more EMQ ANSWERS
common in children, and usually follows an upper
respiratory tract infection. Limp
1. D  Fractured neck of femur. This is a classic
Question 17 presentation and must always be considered in an
D. This man is suffering from acute gout, which is elderly patient who has pain or mobility problems
an inflammatory arthritis commonly affecting the after a fall, however trivial.
metatarsal phalangeal joint of the big toe, but any joint 2. E  Gout. Thiazide (loop) diuretics cause
may be affected. Risk factors include the metabolic hyperuricaemia, which in turn can lead to gout. It
syndrome, alcohol consumption, renal failure, can also cause hyperglycaemia and hypokalaemia.

166
Rheumatology 6

3. I  Slipped upper femoral epiphysis. This is a fracture Arthritis


through the growth plate, resulting in a slippage of 1. F  Additional features include involvement of the
the overlying end of the femur. It causes groin pain first metatarsophalangeal joint of the foot, bony
on the affected side and/or knee or thigh pain with swellings at the DIP joints (Heberden nodes) and
a painful limp. Avoid the pitfall of labelling such at the PIP joints (Bouchard nodes).
presentation as ‘growing pains’. 2. H  Additional features include ulnar deviation,
4. B  Compartment syndrome. Also known as ‘shin hyperextension of the PIP joint (swan neck
splints’, this is due to increased pressure in the deformity), flexion of the PIP joint (boutonniere
anterior tibial compartment as a result of swelling deformity) and Z thumb.
or bleeding from a muscle tear. 3. G  There are five main types of the disease
5. F  Osteoarthritis of hip. Another classic presentation depending on the joints involved: oligoarticular,
with referred pain from the hip to the knee. The polyarticular (rheumatoid arthritis-like) DIP joint
patient’s knee would be normal on examination predominant, spondyloarthritis and arthritis
(unless he also had osteoarthritis in that joint!). mutilans. Psoriasis can also cause nail changes
(e.g., pitting and onycholysis).
Finger clubbing 4. E  This inflammatory arthritis develops following an
1. G  Pulmonary abscess. This pus-filled cavity is usually infection, commonly urinary tract infection. The
caused by aspiration. It is a progressive disease triad of symptoms consists of arthritis of large
and patients are typically cachectic on presentation. joints, urethritis and conjunctivitis/uveitis.
2. B  Crohn disease. The terminal ileum and right side 5. A  Ankylosing spondylitis is linked to the development
of the colon are the sites most frequently affected of inflammatory bowel disease, psoriasis and
by the disease, and malabsorption is a key feature. uveitis.
3. E  Fibrosing alveolitis. This is an insidious,
progressive disease, and for good reason labelled Connective tissue
‘cryptogenic’. 1. C  Features include discoid rash, malar rash,
4. F  Infective endocarditis. An increasingly common photosensitivity, arthritis, pleural effusions,
presentation with a wide range of potential causative neurological abnormalities, pericarditis, renal
agents. Intravenous drug users commonly introduce failure, anaemia and antiphospholipid syndrome.
S. aureus into the right-sided valves. 2. D  Discoid lupus is a chronic photosensitive skin
5. D  Familial clubbing. There is no direct association eruption characterized by persistent scaly, disc-
between asthma and clubbing. like plaques on the scalp, face and ears that may
cause scarring and alopecia.
The painful joint 3. A  Additional features include an increased risk of
1. A  Ankylosing spondylitis. The majority of deep vein thrombosis, stroke and myocardial
affected persons carry the HLA-B27 antigen, infarction, thrombocytopenia and livedo
with a male:female ratio of 4:1. Extraarticular reticularis.
manifestations include uveitis/iritis, aortic valve 4. F  Dermatomyositis causes a periorbital heliotrope
incompetence and pulmonary fibrosis. rash around the eyes and proximal myopathy.
2. J  SLE. Up to 70% patients have skin manifestations Other features include erythematous plaques over
including malar rash, discoid lupus and the fingers and knuckles called Gottron papule,
photosensitivity. SLE can also affect other organs arthritis and respiratory disorders.
(e.g., lungs, kidneys, heart and brain). 5. B  The limited form is called CREST syndrome, which
3. G  Reiter syndrome. There is a strong association consists of calcinosis, Raynaud phenomenon,
with the HLA-B27 antigen, as there is for oesophageal dysfunction, sclerodactyly and
ankylosing spondylitis. The classical triad of telangiectasia. The diffuse form also affects the
symptoms consists of arthritis of large joints, gastrointestinal tract, kidneys, heart and lungs.
urethritis and conjunctivitis/uveitis.
4. D  Osteoarthritis. Osteoarthritis is by far the most Vasculitis
common form of arthritis, shows a strong 1. A  Polymyalgia rheumatica causes pain and stiffness
association with ageing and is a major cause of in the shoulders/pelvic girdle, fatigue and loss
pain and disability in the elderly. of appetite. Associated with temporal arteritis.
5. C  Gout. Gout is a crystal arthropathy with a strong Treatment is with corticosteroids.
(over 10:1) male:female predominance. Loop 2. B  Temporal arteritis causes headaches, scalp
diuretics (e.g., thiazides) are known to be a risk tenderness, jaw claudication and visual loss. It is
factor for developing gout. associated with an increased risk of stroke.

167 
EMQ answers

3. H  Additional features may include mononeuritis 2. C  Most common forms are cervical and lumbar
multiplex, hypertension and weight loss. There is a spinal stenosis. Main cause is ageing. Most
strong link with hepatitis B antigen with regard to appropriate investigation is MRI.
pathogenesis. Small aneurysms are strung like the 3. A  Treatment typically involves bed rest, analgesia
beads of a rosary (rosary sign). and physiotherapy.
4. J  A small-vessel vasculitis that causes 4. B  This is commonly due to the slipping of one
systemic symptoms and renal failure. Rapidly vertebra on another, most commonly L4/L5.
progressive glomerulonephritis may occur. 5. D  Additional features include loss of lumbar lordosis,
Management typically involves prednisolone and kyphosis and reduced chest expansion.
cyclophosphamide.
5. I  Typical features include painful mouth and genital Joint pain
ulcers, inflammatory eye disease, arthritis and 1. E  Pseudogout. Deposition of calcium pyrophosphate
erythema nodosum. dihydrate crystals in joints, commonly the knee
joint. Common in older patients. Fluid microscopy
Drugs demonstrates brick-shaped crystals, which are
1. B  Additional side effects include muscle weakness, positively birefringent under polarized light.
double vision and a yellow discolouration of the 2. G  Septic joint. A medical emergency that requires
skin. prompt diagnosis and treatment with antibiotics if
2. E  Hydroxychloroquine is specifically associated with clinically suspected.
retinopathy. 3. A  Rheumatoid arthritis. About 30% of cases are
3. F  A well-recognized side effect of auranofin. seronegative.
4. D  Additional side effects include blood dyscrasias, 4. B  Psoriatic arthritis. There are five patterns of
gastrointestinal disturbances and central nervous joint involvement: oligoarticular, polyarticular
system reactions. (rheumatoid arthritis-like), DIP joint predominant,
5. G  Additional side effects include thrombocytopenia spondyloarthritis and arthritis mutilans.
and hepatoxicity. 5. J  SLE. Multiorgan involvement. The arthritis is
nonerosive, unlike in rheumatoid arthritis.
Back pain 6. F  Gout. Loop diuretics (e.g., thiazides) are known to
1. G  A medical emergency that requires urgent cause gout.
neurosurgical input. Main features are saddle 7. E  Pseudogout. This patient has an underlying
anaesthesia, severe lower back pain, bowel and diagnosis of haemochromatosis, which is known
bladder dysfunction, sexual dysfunction. to precipitate pseudogout.

168
Endocrine and diabetes 7
symptoms in this case suggest a prolactinoma and an
SBA ANSWERS MRI of the pituitary will then be necessary.
Question 1
C. Pituitary tumours typically cause a bitemporal
Question 7
B. The diagnosis is acromegaly. The majority of cases
hemianopia due to compression of the optic nerve by
are due to an overproduction of growth hormone by
the tumour at the optic chiasm. The temporal visual
a pituitary adenoma. IGF-1 is the most sensitive test
field on both sides becomes affected.
for diagnosis of the condition. This is followed by a
growth hormone suppression test following an oral
Question 2 glucose bolus, which confirms the diagnosis following
A. FSH and LH are usually affected first, with TSH and
a positive screening test for IGF-1. MRI of the pituitary
ACTH affected last. Causes of hypopituitarism include
will confirm the size and location of the tumour.
tumours (e.g., pituitary adenomas), infections (e.g.,
meningitis), radiation, Empty sella syndrome and
Question 8
pituitary apoplexy.
E. Iodine deficiency is the most common cause of
primary hypothyroidism and endemic goitre in
Question 3 developing countries. Hashimoto thyroiditis is the
E. The most common cause of hypopituitarism is a most common cause in countries with sufficient
tumour in over 70% of cases. Most pituitary tumours dietary iodine. It is an autoimmune condition where
are adenomas, which are usually benign in nature. there is infiltration of thyroid gland with T lymphocytes
Pituitary tumours account for 8% of all brain tumours. and autoantibodies against thyroid peroxidase,
thyroglobulin and TSH receptor.
Question 4
C. Glossitis. This is inflammation of the tongue. It may Question 9
occur in anaemia and certain other deficiency states, B. Anaemia in individuals who are hypothyroid is typically
most notably vitamin B12 deficiency. The most normocytic or macrocytic. Other causes of macrocytic
common symptoms are difficulty with chewing and anaemia include B12 deficiency, pernicious anaemia
swallowing foods, tender tongue, smooth tongue and alcoholism.
swelling and a change in colour (is paler if caused by
pernicious anaemia). Question 10
D. Cortisol is essential for life and therefore replacement
Question 5 is of critical importance. This patient has a background
A. Most pituitary hormones can be replaced indirectly of hypopituitarism and should be treated promptly with
by administering the products of the effector glands IV hydrocortisone in the first instance. Stress dosing
orally. Levothyroxine is used for hypothyroidism; with hydrocortisone is always mandatory during major
hydrocortisone is used for adrenal insufficiency; surgery, trauma or severe illness.
testosterone for male hypogonadism; oestradiol for
female hypogonadism. Cortisol is essential for life and Question 11
therefore should be replaced first. From the list above, D. Graves disease is the most common cause of
thyroid hormone replacement is next important. It hyperthyroidism worldwide. It is an autoimmune disease
is important that ACTH deficiency is diagnosed and where thyroid-stimulating immunoglobulin (IgG antibodies
treated appropriately prior to starting levothyroxine in with similar effect to TSH) binds to TSH receptor and
order not to provoke an Addisonian crisis. causes overproduction of thyroid hormones.

Question 6 Question 12
D. Galactorrhoea in a man is due to hyperprolactinaemia E. Ophthalmoplegia is associated with Graves disease.
either due to increased secretion or decreased Additional features include exophthalmos, thyroid
elimination (renal or hepatic disease). The first step acropathy and pretibial myxoedema. All the other
in diagnosis is always to measure serum prolactin features listed in the question can occur with any
concentrations (at least three times). The neurological cause of thyrotoxicosis.

169 
SBA answers

Question 13 resistance to cancer treatment. Surgical resection


D. Anti-TSH receptor antibodies are present in almost is indicated for the few cases that are confined to
all cases of Graves disease and are diagnostic of the thyroid gland. Treatment is generally palliative,
the disorder. Thyroid growth immunoglobulins and consisting of radiation therapy combined with
thyrotropin binding-inhibiting immunoglobulins are also chemotherapy.
recognized autoantibodies to the TSH receptor.
Question 22
Question 14 E. High cortisol levels in Cushing syndrome cause a
A. Carbimazole takes approximately 120 days to take suppression of innate immune and T-cell responses,
effect. Propranolol will help to provide symptomatic resulting in lymphopenia, and increasing the patient’s
control and will relieve symptoms of tachycardia/ risk of severe viral and opportunistic infections.
palpitations almost immediately.
Question 23
D. The diagnosis is Addison disease. In suspected cases,
Question 15 demonstration of low adrenal hormone levels using
A. The most common side effect is a rash, which is
the ACTH stimulation test (synacthen test) is used to
often treated with antihistamines. The most serious
confirm the diagnosis. The synthetic pituitary ACTH
side effect is bone marrow suppression leading to
called tetracosactide is used for this investigation.
neutropenia and agranulocytosis.

Question 24
Question 16 A. Autoimmune adrenalitis is the most common cause
C. Pregnancy is an absolute contraindication for
of adrenal insufficiency (90% of all cases). The
radioactive iodine treatment as it is teratogenic.
destruction of the adrenal cortex is caused by an
Surgery is possible but unlikely to be necessary in a
immune reaction against the enzyme 21-hydroxylase.
pregnant patient.

Question 25
Question 17 C. The diagnosis here is Addisonian crisis, which is a
E. The diagnosis is thyroid crisis following radioactive medical emergency and potentially life-threatening.
iodine. This is much less common now that patients It may be the result of a previously undiagnosed
are adequately prepared. As the patient is severely or untreated Addison disease, suddenly stopping
dehydrated, IV fluids would be the first most glucocorticoids or an intercurrent infection/trauma.
appropriate step in management. The first-line management is IV hydrocortisone
administered with fluid support immediately after taking
Question 18 a blood sample for random cortisol. Intramuscular
E. One should aim to normalize thyroid status. hydrocortisone can be used if there is no venous access.
Hypothyroidism should be avoided as this may
exacerbate visual disturbance. Question 26
E. The diagnosis here is secondary hypoadrenalism
Question 19 as a result of long-term steroid use. Use of high-
D. An ultrasound is performed in the first instance to dose steroids for more than 1 week begins to
confirm the presence of a thyroid nodule and to suppress adrenal glands because the exogenous
assess the rest of the thyroid gland. To obtain a glucocorticoids suppress hypothalamic corticotropin-
definitive diagnosis, a fine-need aspiration for cytology releasing hormone and ACTH. This is best diagnosed
is performed. with a long tetracosactide test.

Question 20 Question 27
D. Medullary cell thyroid cancers originate from D. This woman has Cushing syndrome. Of the given
parafollicular cells (C cells) which are responsible options, the most common cause is a pituitary
for the production of calcitonin, which results in adenoma which secretes ACTH. This is also called
hypercalcaemia. It is the third most common of all Cushing disease, which is responsible for 70% of
thyroid cancers and 25% are genetic in nature. endogenous Cushing syndrome.

Question 21 Question 28
A. Anaplastic carcinomas are associated with a poor D. The raised ACTH suggests that the high cortisol
prognosis due to their aggressive behaviour and level is not of adrenal origin. Suppression of plasma

170
Endocrine and diabetes 7

cortisol following dexamethasone confirms a pituitary- Question 37


dependent aetiology, typically an adenoma. D. This is a typical presentation of a
phaeochromocytoma and the suspected diagnosis
Question 29 is confirmed by a measurement of 24-hour urinary
E. Surgical treatment is the best approach for pituitary catecholamines and metanephrines, the breakdown
tumours, most commonly via a transsphenoidal product of adrenaline.
approach. Most patients require postoperative steroid
replacement in the interim, as long-term suppression Question 38
of pituitary ACTH and normal adrenal tissue does not A. This is type 2 diabetes mellitus. Of the listed options,
recover immediately. diet therapy is always the first-line management,
particularly in elderly individuals. However, metformin
Question 30 is the first-line pharmacological agent and is usually
C. The criteria for diagnosis include a low serum sodium, used as well.
low plasma osmolality, a urine osmolality higher than
plasma osmolality and an elevated urinary sodium. Question 39
B. Metformin is particularly useful in obese individuals
Question 31 as it does not increase appetite and result in weight
E. The diagnosis here is SIADH and is treated initially gain. The most common side effect is gastrointestinal
with fluid restriction, usually starting at 500 mL/day symptoms (e.g., diarrhoea and abdominal cramps).
of water with adjustments based on serum sodium The most serious side effect is lactic acidosis.
levels. Long-term fluid restriction of 1200–1800 mL/
day may maintain a symptom-free state. Question 40
E. The most common side effect is gastrointestinal
Question 32 symptoms (e.g., diarrhoea, abdominal cramps,
E. A side effect of lithium is diabetes insipidus. This patient nausea and vomiting). The most serious side effect is
is likely to have hypernatraemia, dilute urine with low lactic acidosis.
specific gravity and low urine osmolarity. Diagnosis is
obtained with water deprivation test with desmopressin. Question 41
C. This patient is likely to have type I diabetes mellitus.
Question 33 Many patients present with diabetic ketoacidosis
B. Secondary hyperparathyroidism is a physiological (DKA), as in this case. Patients with type I diabetes
response to hypocalcaemia, which in this case is due always need treatment with insulin, in additional to diet
to a low vitamin D. This condition is seen especially in modification.
patients with chronic renal failure.
Question 42
Question 34 C. Hypoglycaemia in diabetic patients is usually due
B. Hypercalcaemia, if untreated, can lead to arrhythmias to overmedication. In an unconscious patient it is
and cardiac arrest. It is treated with IV saline as the always treated with IV dextrose initially. Intramuscular
first-line management. Bisphosphonates are then glucagon is used if venous access is not possible.
used. These are taken up by osteoclasts and inhibit
osteoclastic bone resorption. Question 43
A. The diagnosis is DKA, which is a medical emergency
Question 35 as it is a potentially life-threatening complication
E. Hypocalcaemia causes numbness, muscle spasms, of diabetes. The most important initial therapeutic
seizures, confusion or cardiac arrest. A diagnosis of intervention in DKA is appropriate fluid replacement
hypocalcaemia is most commonly due to renal failure. followed by fixed-rate IV insulin administration. The
From the given list, renal failure is the most common main aims for fluid replacement are restoration
cause of hypocalcaemia from hypoparathyroidism. of circulatory volume, clearance of ketones and
correction of electrolyte imbalance.
Question 36
A. A high aldosterone-to-renin ratio confirms the Question 44
diagnosis of primary hyperaldosteronism, where C. Preproliferative retinopathy describes increasing
renin is suppressed and aldosterone is increased. If a ischaemia superimposed on background diabetic
diagnosis is confirmed, imaging is then used to look retinopathy. Cotton wool spots are soft exudates
for underlying tumour. that represent microinfarcts in the nerve fibre

171 
SBA answers

layer. Close monitoring is required even if the should be fulfilled for a diagnosis of SIADH to be
patient is asymptomatic due to the high risk of made:
neovascularization. • plasma sodium concentration <135 mmol/L
• plasma osmolality <280 mOsmol/kg
Question 45 • urine osmolality >100 mOsmol/kg
E. Symmetrical sensory neuropathy affects the toes • urinary sodium concentration >30 mmol/L
and soles of the feet initially. Involvement of the • patient clinically euvolaemic
hands does occur, which results in sensory loss in a • absence of clinical or biochemical features of
characteristic ‘glove and stocking’ distribution. adrenal and thyroid dysfunction
• no diuretic use (recent or past)
Question 46
E. Diabetes in pregnancy can lead to neonatal Question 53
hypoglycaemia not hyperglycaemia due to E. This patient has a diagnosis of osteoporosis but
hypersecretion of insulin from foetal islets cells. with no previous fractures. Diet supplementation
with calcium and vitamin D is intended to prevent
Question 47 osteoporosis and reduce the incidence of fracture.
A. This patient has a pancreatic insulinoma. These are Bisphosphonates reduce risk of fracture in patients
rare neuroendocrine tumours arising from the Islet with prior fragility fractures.
cells. They are usually benign in nature. The definitive
treatment is surgical removal of the tumour, which Question 54
may involve part of the pancreas as well. D. In patients with diabetes symptoms (e.g., polyuria,
polydipsia) diagnosis of type 2 diabetes mellitus can
Question 48 be obtained from:
E. Pituitary apoplexy is caused by impaired blood supply - a random venous plasma glucose concentration
of the pituitary gland. This is usually in the presence ≥11.1 mmol/L or
of a pituitary tumour. Diagnosis is achieved with MRI - a fasting plasma glucose concentration ≥7.0
of the pituitary gland. Treatment is by correction of mmol/L (whole blood ≥6.1 mmol/L) or
hormone deficiencies, and in many cases surgical - 2-hour plasma glucose concentration ≥11.1
decompression is required. mmol/L two hours after 75 g anhydrous glucose in
an oral glucose tolerance test.
Question 49 As this patient is confused, a random blood glucose is the
D. Conn syndrome (primary hyperaldosteronism) is the most appropriate investigation.
excess production of aldosterone by the adrenal
glands, usually by an adrenal adenoma. This is a Question 55
benign tumour of the adrenal cortex. B. About half of the patients with carcinoid syndrome
develop cardiac abnormalities, classically a
Question 50 secondary restrictive cardiomyopathy caused
B. Cushing disease is a cause of Cushing syndrome by serotonin-induced fibrosis of the valvular
characterized by increased secretion of ACTH from endocardium, notably tricuspid and pulmonary
a pituitary adenoma. The first line of treatment is valves. This is usually tricuspid insufficiency and
surgical resection of the tumour via transsphenoidal pulmonary stenosis.
surgery.
Question 56
Question 51 A. PCOS develops when the ovaries are stimulated to
D. Neurogenic diabetes insipidus is due to a lack produce excessive amounts of androgenic hormones,
of ADH production in the brain. Causes include especially testosterone, by the release of excessive
tumours, infections (e.g., encephalitis), trauma or LH by the anterior pituitary or through high levels of
neurosurgery. About 25% of cases are of unknown insulin in the blood.
aetiology. Treatment is with ADH analogues (e.g.,
desmopressin). Question 57
B. During acute intercurrent illnesses, patients will
Question 52 have higher need for insulin. In general patients
C. SIADH is characterized by excessive insuppressible already on insulin will need to have regular
release of ADH either from the posterior pituitary gland insulin, or to increase their dosages to prevent
or from a nonpituitary source. The following criteria hyperosmolar hyperglycaemic state (HHS) or DKA.

172
Endocrine and diabetes 7

Metformin can cause lactic acidosis, especially in Question 62


dehydrated patients, and so should be avoided. D. Hypercalcaemia can be treated with IV saline and
Short-acting insulin should be given if their original furosemide. Fluid replacement with IV saline and
oral antihypoglycaemics do not control the high forced diuresis with saline and a loop diuretic, such
blood sugar levels (BMS), not starting an oral as furosemide, is a rapid and safe way to lower
sulphonylurea. serum calcium and should be the initial approach
to therapy. This patient has lung cancer and is
Question 58 probably exhibiting a paraneoplastic secretion of
B. National Institute for Health and Care Excellence parathyroid-related hormone, which is making her
(NICE) guidelines suggest that type 2 diabetes hypercalcaemic.
mellitus should be aimed for <130/80 mm/Hg if
any sign of vascular damage is noted (i.e., diabetic Question 63
retinopathy); otherwise, blood pressure targets C. Vitamin E is fat soluble and may become deficient in
should be 140/80 mm/Hg. the setting of cholestasis. Tremor and hyporeflexia
are classical findings in the setting of vitamin E
Question 59 deficiency. All patients with cystic fibrosis must
A. No clear evidence for prevention of macrovascular receive supplementation with fat-soluble vitamins
disease, but there is evidence of long-term (A, D, E and K). B vitamins are water soluble
prevention of microvascular complications of type and cholestasis is not a risk factor for deficiency.
2 diabetes. Sulphonylureas act on the membranes B-vitamin deficiency may cause ataxia, memory
of beta cells, causing closure of ATP-sensitive problems and paraesthesia. Tremor and hyporeflexia
potassium channels. When the potassium channels are not common findings.
are closed, it results in opening of calcium channels
and exocytosis of insulin. Question 64
A. Beta cells secrete insulin in response to increases in
Question 60 blood glucose. The resulting increase in insulin acts
D. Vitamin C deficiency causes scurvy (typically presents to lower blood glucose back to normal levels at which
with ecchymosis, petechiae, bleeding gums, impaired point further secretion of insulin is stopped. By contrast,
wound healing) as it is in this patient. Gingivitis is the secretion of insulin by insulinomas is not properly
inflammation of the gingiva. The majority of cases are regulated by glucose and the tumours will continue
related to bacteria-induced inflammation caused by to secrete insulin, causing glucose levels to fall further
the dental plaque causing inflammation of the gingiva than normal. Patients with insulinomas present with
and bleeding on tooth-brushing. symptoms of hypoglycaemia, such as tremulousness,
shakiness, sweating, which are improved by eating.
The diagnosis of an insulinoma is usually made
Question 61 biochemically with low blood glucose, elevated
E. This patient has the triad of signs consistent with insulin, proinsulin and C-peptide levels and confirmed
hyperaldosteronism: hypertension, hypokalaemia by localizing the tumour with medical imaging or
and metabolic alkalosis. In this patient, the primary angiography. Surgery is the most appropriate treatment.
aldosteronism is caused by bilateral adrenal
hyperplasia (bilateral adrenal enlargement in the
CT scan). The treatment of choice in patients with Question 65
primary hyperaldosteronism and bilateral adrenal C. Hyponatraemia and hyperkalaemia unresponsive to
hyperplasia is an aldosterone receptor antagonist IV rehydration should lead the clinician to think about
such as spironolactone/eplerenone. Even though Addison disease (primary adrenal insufficiency). A
other antihypertensive agents can also be used low serum cortisol and markedly elevated ACTH
to control the blood pressure in these patients, level confirm the diagnosis. DHEA production starts
it is believed that the effect of high aldosterone to increase in midchildhood around the onset of
itself is harmful in the long term. Blocking its effect adrenarche. In an adult or adolescent, a normal DHEA
using an aldosterone receptor antagonist therefore level suggests a functional adrenal gland but the test
would be the treatment of choice. For patients with is very unreliable in a young child prior to the onset of
unilateral adenoma causing hyperaldosteronism, adrenarche.
removal of the adrenal gland on that side is the
preferred treatment. This patient has bilateral adrenal Question 66
hyperplasia, however, and therefore medical therapy D. Hypercalcaemia of malignancy should be treated
is the treatment of choice. initially with aggressive rehydration, followed

173 
EMQ answers

by diuresis with furosemide, phosphorus illness. According to current data, thyroid hormone
replacement if hypophosphataemia is present and replacement therapy has not been shown to be of
IV bisphosphonates. Adjunctive therapies include benefit in the vast majority of these patients.
dialysis, glucocorticoids, calcitonin, plicamycin and
gallium nitrate. EMQ ANSWERS
Question 67 Pituitary disease
A. Diagnosis achieved by excluding other aetiologies 1. D  Kallmann syndrome is associated with
such as pregnancy, lactation, hypothyroidism gonadotrophin deficiency with absent sense
and medications before starting the work-up for of smell (anosmia), colour blindness and renal
hyperprolactinemia. If basal fasting morning prolactin impairment.
levels fall between >100–200 mg/L in nonpregnant 2. B  This condition is associated with pituitary infarction
woman such as in our presenting patient, that will following postpartum haemorrhage.
indicate MRI of the pituitary. Prolactin level more than 3. A  Infarction or haemorrhage into a pituitary
100 ng/mL suggests probable pituitary adenoma, and tumour. Headaches and visual loss are typical
hence the MRI indication. symptoms.
4. C  The sella turcica appears devoid of pituitary tissue
Question 68 on radiological imaging, and the area is filled with
A. Graves disease, also known as toxic diffuse goitre cerebrospinal fluid instead.
is an autoimmune disease that affects the thyroid. It 5. E  Classic presentation of acromegaly, which is
frequently results in hyperthyroidism and an enlarged almost always due to a pituitary tumour-secreting
thyroid. Signs and symptoms of hyperthyroidism growth hormone.
may include irritability, muscle weakness, sleeping
problems, fast heartbeat, heat intolerance, diarrhoea Thyroid disease
and weight loss. Other symptoms may include 1. G  Inflammation of the thyroid gland due to a viral
thickening of the skin on the shins, known as pretibial infection. Often associated with fever, neck pain
myxoedema, and eye problems such as bulging, a and malaise.
condition known as Graves ophthalmopathy. About 2. E  Graves disease is associated with the binding of
25% to 80% of people develop eye problems. This autoimmune IgG antibodies to the TSH receptor,
presentation might be confused with postpartum causing the overproduction of thyroid hormones,
thyroiditis but the presence of bruit is significant for resulting in symptoms of hyperthyroidism.
Graves disease but not for postpartum thyroiditis. 3. A  Hashimoto thyroiditis is an autoimmune disease
that is a common cause of hypothyroidism
Question 69 where autoantibodies against thyroid peroxidase,
A. Thyroid malignancy presents with solid cold nodules thyroglobulin and TSH receptors are developed.
in the thyroid glands. Treatment involves removal 4. H  Likely to represent a thyroid cancer, specifically
of the nodules through surgery. Surgery would be medullary cell carcinoma. The increased calcium
effective when the cancer has not spread to other in these tumours is due to coexisting increased
body parts. These nodules can be felt in the neck. parathyroid hormone and some cases are part of
The nodules present a significant degree of concern multiple endocrine neoplasia type 2 (MEN-2).
when diagnosed in individuals under the age of 5. I  A rare complication associated with deterioration
20 years. The solid cold nodules are diagnosed during of hyperthyroidism following intercurrent infection,
physical examination and the patient is then referred stress or radioactive iodine therapy.
to an endocrinologist for further examination. The
symptoms include difficulty in swallowing, pain in the Diabetic complications
neck, vocal changes and swollen lymph nodes. It 1. E  Additional features include numbness, tingling and
has been estimated that women have a three times pain particularly at night.
greater risk of developing thyroid cancer as compared 2. C  In addition to background diabetic retinopathy (dot
with men. and blot haemorrhages, microaneurysms, hard
exudates), the retina also develops multiple cotton
Question 70 wool spots. The patient is at risk of developing
E. It remains controversial whether development of these proliferative retinopathy.
changes in thyroid metabolism reflects a protective 3. A  Patients are usually asymptomatic at this
mechanism or a maladaptive process during stage.

174
Endocrine and diabetes 7

4. I  More common in type 2 diabetes. Symptoms requires fluid resuscitation, insulin and replacement
include pain in buttocks, hips, thighs or legs, of potassium.
muscle weakness, fasciculations and diminished 4. H  Random serum glucose >7.1 mmol/L but below
knee reflexes. 12 mmol/L. These are the levels for impaired
5. G  Symptoms are typically worse at night and are glucose tolerance. Sugars should be monitored
best treated with tricyclic antidepressants. as the patient may go on to develop full-blown
diabetes.
Drugs 5. I  Type I diabetes mellitus. This most commonly
1. D  Typical side effects of acarbose. It inhibits alpha- presents in the young and is characterized by
glucosidase, an intestinal enzyme that releases absolute insulin deficiency. Type II usually presents
glucose from larger carbohydrates. in the older population (middle aged onwards) and
2. C  Typical side effects of metformin. Its main is more characterized by relative insulin lack or
effect is to decrease liver glucose production resistance.
(gluconeogenesis).
3. C  Used in obese individuals as it does not increase The pancreas and diabetes
appetite or cause weight loss. 1. A  Type 2 diabetes mellitus is due to insufficient
4. A  A sulphonylurea that acts via closure of K+ ATP insulin production and insulin resistance.
channels, leading to calcium influx and insulin Treatment is with lifestyle modification and oral
release. Unlike tolbutamide, it is long acting and hypoglycaemic agents in the first instance.
therefore avoided in the elderly due to the risk of Insulin is only used in the advanced stages of
hypoglycaemia. disease.
5. F  An insulin-releasing agent, which helps to lower 2. F  MODY describes hereditary forms of diabetes
postprandial hyperglycaemia. mellitus due to ineffective insulin production or
release by pancreatic beta cells. Defects are
Investigation mutations of transcription factor genes (e.g.,
1. B  The diagnosis is acromegaly. IGF-1 and growth glucokinase gene).
hormone suppression test following an oral 3. E  This consists of central obesity, hypertension,
glucose load are used to diagnose the condition. hyperglycaemia/insulin resistance and
2. G  The diagnosis is Cushing syndrome. This test will hypercholesterolaemia. It is associated with
help confirm the raised cortisol. development of cardiovascular disease and type 2
3. J  Presentation of a phaeochromocytoma diagnosed diabetes mellitus.
by measurement of urinary catecholamines and 4. D  Type 1 diabetes usually presents with DKA,
their breakdown products. which is a medical emergency requiring prompt
4. E  This is Addison disease and is usually diagnosed treatment with fluid rehydration and insulin and
via the short synacthen test. potassium replacement.
5. H  This is likely to be diabetes insipidus. This is 5. B  This is especially common during the third
usually first investigated by measurement of urine trimester of pregnancy. Infants born to
and plasma osmolality, to look for psychogenic mothers with gestational diabetes are at risk
polydipsia, but will need a supervised water of macrosomia, intrauterine growth restriction,
deprivation test to confirm the diagnosis. polyhydramnios, neonatal hypoglycaemia and
respiratory distress syndrome.
Diabetes
1. B  Diabetes insipidus. This can be confused with The adrenal glands
diabetes mellitus due to the polydipsia and 1. F  Primary adrenal insufficiency; the most common
polyuria. The normal blood glucose and reduced cause is autoimmune adrenalitis.
serum osmolarity should point towards the 2. B  Primary hyperaldosteronism caused by adrenal
diagnosis. adenoma.
2. F  Hyperosmolar nonketotic coma. This gentleman 3. A  A neuroendocrine tumour that secretes high
has presented in hyperosmolar nonketotic coma amounts of catecholamines.
brought on by his choice of sugary fluid to 4. D  The most common cause of Cushing syndrome
rehydrate himself. He requires rehydration and is the long-term use of glucocorticoids, as in this
cautious correction of his blood sugar. case (iatrogenic Cushing).
3. C  DKA. This boy has presented in DKA, which may 5. E  The most common type of congenital adrenal
have been precipitated by his chest infection. He hyperplasia involves the gene for 21-hydroxylase.

175 
This page intentionally left blank
Haematology 8
SBA ANSWERS Question 7
C. MCV is very sensitive for alcohol intake. There
Question 1 will be thrombocytopaenia due to splenic
A. In this case, the INR is 7.2 and the patient does not sequestration and liver failure; however, it is a very
report of any bleeding. In situations when blood INR late sign. White cell count is inversely associated
is between 5 and 9 and bleeding is absent, then with alcohol intake. However, it is very nonspecific.
warfarin should be stopped with immediate effect. A blood film may show thrombocytopaenia,
macrocytosis and haemolysis, but they are
nonspecific. Erythrocyte sedimentation rate does
Question 2
not increase with alcohol intake.
D. Pro-inflammatory cytokines cause thrombocytosis
whereas all else cause low platelets
Question 8
D. Factor VII initiates the clotting process following tissue
Question 3
damage.
D. The diagnosis is polycythaemia vera based on the
blood count, splenomegaly and raised red cell
volume. This is usually managed via venesection in the Question 9
first instance. There is no absolute cure. A. This patient is likely to be suffering from autoimmune
thrombocytopaenic purpura. Platelet count is the first-
line investigation.
Question 4
C. ‘Tear drop’-shaped red cells are found in myelofibrosis
and can also be found in other myeloproliferative Question 10
disorders, pernicious anaemia, thalassaemia, myeloid C. Multiple myeloma often has plasma cell infiltrates
metaplasia and some haemolytic anaemias. >10%, with rouleaux formations (stacks of RBCs)
and lytic lesions on skeletal survey. International
Myeloma Working group states that MGUS bone
Question 5
marrow biopsy should reveal <10% plasma cells
D. Determination of full blood count is the first essential
and asymptomatic MGUS may have a chance of
test. The hallmark of immune thrombocytopaenic
developing into multiple myeloma, and so monitoring
purpura (ITP) is isolated thrombocytopaenia. Careful
is required.
examination of the peripheral smear is essential in
a patient with thrombocytopaenia. Peripheral blood
smear would show that the morphology of red blood Question 11
cells (RBCs) and leukocytes is normal, and that of D. This patient is suffering from haemophilia A, which
platelets is typically normal, with varying numbers of is due to deficiency of factor VIII. The history
large platelets. Some persons with acute ITP may suggests mild haemophilia and the treatment in
have megathrombocytes or stress platelets, reflecting this case is replacement of factor VIII for bleeding
the early release of megakaryocytic fragments into episodes.
the circulation. In addition, careful examination
of the smear is essential to exclude thrombotic Question 12
thrombocytopaenic purpura (TTP) and rare instances E. Vitamin K is necessary for the activity of prothrombin
of acute leukaemia presenting as thrombocytopaenia. II, VII, IX and X. Absorption of vitamin K and other fat-
In TTP, a striking degree of RBC fragmentation is seen soluble vitamins will be impaired by cholestasis and
in addition to thrombocytopaenia. hence lead to impaired coagulation by altering activity
of these factors.
Question 6
A. This patient is experiencing an anaphylactic Question 13
reaction best managed with 0.5-mg adrenaline A. The diagnosis is a probable deep vein thrombosis.
intramuscularly. Doppler ultrasound is useful for iliofemoral thrombosis.
Calf vein thrombosis is best assessed by venography.

177 
SBA answers

Question 14 Question 23
C. The endemic form of Burkitt lymphoma is associated E. Target cells are due to disproportional cell membrane
with Epstein–Barr virus. Facial involvement in 50% surface area-to-volume ratio. Can also be due to an
of patients. Immunodeficiency-associated Burkitt increase in cell surface membrane (liver disease) or
lymphoma rarely involves facial masses. HTLV-1 is decreased haemoglobin content (thalassaemias).
thought to be associated with adult T-cell lymphoma/ Schistocytes are red cell fragments and are an
leukaemia. indicator of mechanical, heat, toxin damage to RBCs.

Question 15 Question 24
C. APTT is used for monitoring of unfractionated C. In addition to the central nervous system, this form of
heparin. PT and INR are utilized for warfarin leukaemia may affect the testes in males.
monitoring.
Question 25
Question 16 C. Tyrosine kinase inhibitors such as imatinib are
A. CLL is a disease of the elderly presenting with the initial treatment of choice for chronic myeloid
enlarged, ‘rubbery’, nontender lymph nodes. Hodgkin leukaemia and has better outcomes than alpha-
lymphoma commonly affects young adults presenting interferon plus cytarabine, which was the previous
with night sweats, weight loss, fevers (B symptoms); standard of care.
it also presents with anaemia, splenomegaly and
multiple lymph node enlargements. Histology shows Question 26
Reed–Sternberg cells. D. Smear cells are characteristic of CLL. They reflect
fragility and distortion of B-CLL cells while the film is
Question 17 being made.
D. The warfarin should be stopped and reinstituted when
the INR has fallen within the therapeutic range. Other Question 27
measures are usually only necessary if the INR is very E. This patient is likely to be suffering from Hodgkin
high (>9) or if there is bleeding. disease, which is most appropriately diagnosed by
lymph node biopsy and histology.
Question 18
C. Auer rods are characteristic of acute myeloid Question 28
leukaemia. C. SCD is characterized by RBCs that assume an
abnormal, rigid, sickle shape. Sickling decreases the
Question 19 cells’ flexibility and results in a risk of various life-
E. Hyponatraemia does not occur in acute tumour lysis threatening complications. SCD usually manifests early
syndrome. in childhood. The most common clinical manifestation
of SCD is vasoocclusive crisis. A vasoocclusive crisis
Question 20 occurs when the microcirculation is obstructed by
B. IgA. The defences of the small intestine are in the sickled RBCs, causing ischemic injury to the organ
most part due to T and B lymphocytes. The main supplied and resultant pain. Pain crises constitute the
type of antibody produced by intestinal B cells is most distinguishing clinical feature of SCD and are the
IgA, the major Ig of external secretions. IgA prevents leading cause of emergency department visits and
microorganisms from entering the gut lumen as it hospitalizations for affected patients.
binds and neutralizes them directly without the need
for other effector systems. A much smaller amount of Question 29
IgM is also secreted by the intestinal B cells. D. Characteristic presentation of Hodgkin disease.

Question 21 Question 30
D. All-trans retinoic acid is proven to induce remission in C. Iron tablets. The low haemoglobin and low MCV
this form of leukaemia. suggest microcytic anaemia. In females of menstrual
age, the most common cause of this is iron-deficiency
Question 22 anaemia caused by menorrhagia, sometimes on a
A. The Philadelphia chromosome is invariably present in background of an iron-deficient diet. Patients should
chronic myeloid leukaemia but can occasionally occur have further blood samples sent off for iron studies
in acute lymphoblastic leukaemia. to confirm the diagnosis, and then be started on iron

178
Haematology 8

tablets, such as ferrous sulphate, if appropriate. If Question 42


patients had either vitamin B12 or folate deficiency, A. This is commonly seen in chronic illnesses, such as
then the MCV should be high. Crohn disease, renal failure and tuberculosis. Laboratory
findings include a low serum iron, low serum iron-
Question 31 binding capacity and an increased or normal serum
B. Sternberg–Reed cells are malignant B lymphocytes ferritin. Treatment is of the underlying cause.
and are diagnostic of Hodgkin disease.
Question 43
Question 32 C. Vitamin B12 deficiency is commonly seen in vegans
E. Additional ‘B’ symptoms include fever and weight loss. who do not consume animal products.

Question 33 Question 44
C. Age is not an important factor when determining E. In view of her autoimmune thyroid disease, age and
treatment in the vast majority of cases. microcytosis, the most likely diagnosis is pernicious
anaemia, an autoimmune disease. She will be
Question 34 vitamin B12 deficient but the key to the diagnosis
E. The gastrointestinal tract may be affected but not in all is the presence of intrinsic factor antibodies, which
cases. are more specific for the disease than parietal cell
autoantibodies.
Question 35
C. Pernicious anaemia. The low haemoglobin and high Question 45
MCV are indicative of macrocytic anaemia. Pernicious C. All of these could cause vitamin B12 deficiency but the
anaemia is the only answer given here that is a cause history makes coeliac disease the most likely.
of macrocytic anaemia. Iron-deficient anaemia and
thalassaemia are both causes of microcytic anaemia, Question 46
whereas anaemia of chronic disease and haemolytic C. Megaloblastic anaemia is seen in vitamin B12 and
anaemia are causes of normocytic anaemia. folate deficiency. In this case the cause is not clear, so
replacement with both vitamin B12 and folate is given
Question 36 pending the results for measurement of vitamin B12 and
D. MALT lymphoma typically affects the stomach. serum folate. Folate alone has been shown to worsen
the neurological complications of vitamin B12 deficiency.
Question 37
C. Burkitt lymphoma typically affects African children and Question 47
can result in jaw tumours. C. The reticulocyte count is increased in haemolytic
anaemia.
Question 38
B. Key features of multiple myeloma. Additional features Question 48
include gangrene and bleeding. D. The clinical features are characteristic of beta
thalassaemia major.
Question 39
D. The initial test is protein electrophoresis, which will Question 49
reveal the presence of a monoclonal protein (‘M’ B. This patient is experiencing a sickle cell crisis. As
bands). A bone marrow examination showing 10% her oxygen saturation is currently within normal
or more clonal bone marrow plasma cells will be range, analgesia would be the most suitable
necessary to make the diagnosis. management step.

Question 40 Question 50
C. Heavy menstruation is known to cause iron-deficiency A. Systemic lupus erythematosus is associated with
anaemia. autoimmune haemolytic anaemia. Coombs test is
diagnostic in this form of anaemia.
Question 41
B. This patient has iron-deficiency anaemia in view of Question 51
her low ferritin. The blood film will show all except D. The diagnosis is cold antibody haemolysis, which is
macrocytosis. best managed by avoidance of cold exposure.

179 
EMQ answers

Question 52 2. I  Common in Northern Europe, this form of anaemia


D. Stage 3B disease (lymph node areas on both sides of can result in splenomegaly and leg ulcers.
the diaphragm) and constitutional symptoms. Treatment is usually a splenectomy.
3. J  Cigar-shaped RBCs are characteristic of hereditary
Question 53 elliptocytosis. Treatment is usually not required.
A. Warm antibody haemolytic anaemia is characterized 4. F  Haemolysis is the result of oxidant red cell
by IgG antibodies. Cold antibody haemolytic anaemia damage. In addition to fava beans, drugs such as
is characterized by IgM antibodies. sulphonamides may precipitate haemolysis.
5. D  This condition may eventually progress to
myelodysplasia and leukaemia.
Question 54
A. Cold autoimmune haemolytic anaemia involves
IgM autoantibodies that are most active at 4°C Lymphadenopathy
to 6°C, resulting in transient ischemia of Raynaud 1. A  Epstein–Barr virus is typically transmitted in saliva.
phenomenon in cold weather. Mycoplasma In rare cases, there may be splenic rupture,
pneumonia is often associated with cold agglutinins. myocarditis and meningitis.
2. F  The virus may be transmitted sexually or via
contaminated blood products in the main. HIV is
Question 55 an RNA virus which binds to CD4 cells via gp120
C. Atrophic gastritis is often followed by pernicious
to cause cellular damage.
anaemia. Megaloblastic changes result in
3. C  Characteristic description of a lymphoma.
hypersegmented neutrophils (5+ lobes)
4. D  Genetic factors such as the Philadelphia
chromosome are responsible for the majority of
Question 56 chronic myeloid leukaemia cases. Environmental
D. The PTT or APTT is a performance indicator factors include radiation exposure and drugs such
measuring the efficacy of both the intrinsic and as chemotherapy.
the common coagulation pathways. Apart from 5. B  Additional features may include retinitis, loss of
detecting abnormalities in blood clotting, it is also visual acuity and orbital pain.
used to monitor the treatment effects with heparin,
a major anticoagulant. It is used in conjunction with
Splenomegaly
the PT, which measures the extrinsic pathway. It
1. A  This is characteristic of schistosomiasis, which is
is significantly prolonged in severe haemophilia,
most prevalent in Africa.
but may be normal in mild or even moderate
2. H  Characteristic features of kala-azar. Additional
haemophilia. The diagnosis is suggested by an
features include pyrexia, sweats, a burning
elevated APTT level in a male patient with a positive
sensation in the feet and joint pain.
family history. Laboratory studies for suspected
3. D  Four main malarial parasites exist. These include
haemophilia include a complete blood cell count,
Plasmodium falciparum, P. vivax, P. ovale and P.
coagulation studies and a factor VIII assay.
malariae. Clinical features of falciparum malaria
include impaired consciousness, uraemia,
EMQ ANSWERS respiratory distress syndrome and splenic rupture.
4. G  A classic description of Gaucher disease of which
Anaemia there exists three types. Type 2 and type 3 have a
1. C  Such a form of anaemia is best treated propensity to involve the central nervous system.
supportively with red cell and platelet transfusion in 5. J  Additional features may include epistaxis and
addition to antibiotic use for infections associated bruising. Ciprofloxacin is the mainstay form of
with neutropenia. treatment.

180
SURGERY ANSWERS
Chapter 9

Trauma and surgical emergencies . . . . . . . . . . 183


Chapter 10

Perioperative care . . . . . . . . . . . . . . . . . . 187


Chapter 11

Gastrointestinal surgery . . . . . . . . . . . . . . . 189


Chapter 12

Cardiothoracics and vascular surgery . . . . . . . . 197


Chapter 13

Orthopaedics . . . . . . . . . . . . . . . . . . . . . 199
Chapter 14
Ear, nose and throat . . . . . . . . . . . . . . . . . 203
Chapter 15

Urology . . . . . . . . . . . . . . . . . . . . . . . . 207
Chapter 16

Breast surgery . . . . . . . . . . . . . . . . . . . . 211


This page intentionally left blank
Trauma and surgical
emergencies 9
SBA ANSWERS Question 8
A. Extradural haematomas are often associated with
Question 1 a head injury and are classically depicted by a brief
D. Classic hemisection of the spinal cord. A rare period of unconsciousness followed by a lucid
occurrence but resulting in ipsilateral power and interval of recovery. A CT head is the gold-standard
proprioceptive loss below the lesion with contralateral investigation.
loss of pain and temperature sensation.
Question 9
Question 2 A. The ‘drip and suck’ technique. Patients are severely
E. For children under the age of 6, an intraosseous dehydrated and require IV saline for adequate fluid
needle in the proximal tibia is a useful emergency resuscitation.
route for fluid administration in times of difficult
access. Question 10
E. This is simply a life or death situation. When there
Question 3 is a clinical suspicion of abdominal aortic aneurysm
C. Echocardiogram is the diagnostic test of choice, and (AAA) you should proceed ahead straight to theatres;
earliest finding is usually collapse of right atrium and unoperated upon, AAA rupture results in 100%
ventricle. ECG shows electrical alternans (swinging mortality.
QRS complexes) as the heart moves in the fluid-
filled pericardial sac. However, this is a late and Question 11
nonsensitive sign. A. This patient is likely to have acute pancreatitis best
diagnosed by an abdominal CT scan.
Question 4
C. McBurney point is the name given to the point Question 12
over the right side of the abdomen that is one- C. A massive haemothorax is associated with the
third of the distance from the anterior superior iliac accumulation of over 1500 mL of blood in the chest.
spine to the umbilicus (navel). This point roughly The chest is notably dull to percussion and breath
corresponds to the most common location of the sounds are absent.
base of the appendix where it is attached to the
cecum. Question 13
B. Cullen sign is superficial oedema and bruising in
Question 5 the subcutaneous fatty tissue around the umbilicus.
B. An open pneumothorax allows air to be sucked into This sign takes 24 to 48 hours to appear and can
the pleural cavity on inspiration and forced out on predict acute pancreatitis, with mortality rising from
expiration. Immediate management is to close the 8–10% to 40%. It may be accompanied by Grey
defect with occlusive dressing. Turner sign (bruising of the flank), which may then be
indicative of pancreatic necrosis with retroperitoneal or
Question 6 intraabdominal bleeding.
D. Rovsing sign is a sign of appendicitis. If palpation
of the left lower quadrant of a person’s abdomen Question 14
increases the pain felt in the right lower quadrant, the B. Subdural haematomas are commonly seen in elderly,
patient is said to have a positive Rovsing sign and alcoholic individuals. These individuals are more
may have appendicitis. susceptible on the background of their atrophic
brains, which make the connecting veins more likely
Question 7 to rupture.
A. The diagnosis is acute appendicitis. Abdominal
CT scan or ultrasound is the most preferred Question 15
investigative tools. Abdominal X-ray is of no E. The diagnosis is renal colic, which is best assessed by
diagnostic benefit. an IV urogram.

183 
EMQ answers

Question 16 cavity. Immediate management involves passing


B. This patient is demonstrating evidence of gall bladder a large-bore IV cannula into the pleural cavity
disease best diagnosed by abdominal ultrasound in through the midclavicular line second intercostal
the first instance. space.

Question 17 EMQ ANSWERS


D. The fall has resulted in a flail chest. This occurs
when one or more ribs are fractured in more than Trauma
one place. The broken segment of chest wall moves 1. K  Tension pneumothorax. This develops
independently from the rest of the chest resulting in if the communication between the pleura
a paradoxical motion. The underlying lung is often and lung is small and acts as a one-way
severely contused as a result. valve, allowing air to enter the pleural space
during inspiration, but preventing it from
Question 18 escaping.
E. Presentation of a ruptured AAA. Surgery is the most 2. F  Open pneumothorax. This occurs when the
appropriate intervention. communication between the lung and pleural
space does not seal and allows air to transfer
Question 19 freely between the two.
C. Presentation of lower limb ischaemia on the 3. C  Flail chest. This is a clinical syndrome resulting
background of atrial fibrillation. In view of her from major trauma to the chest walls, sufficient
satisfactory blood pressure and oxygen saturation, to cause fracture of several ribs in at least two
an emergency embolectomy would be the next most places, resulting in paradoxical movement of part
appropriate management step. of the chest on respiration.
4. A  Airway obstruction. An inhaled foreign
body is more likely to enter the right main
Question 20
bronchus.
A. The diagnosis is acute diverticulitis. Additional features
5. L  Traumatic aortic rupture. Similar radiological
would include rectal bleeding and altered bowel habit.
findings in association with severe chest
Such a condition is best diagnosed by abdominal
pain radiating to the back are found in aortic
CT scan.
dissection.

Question 21
D. The diagnosis is cholecystitis in view of his right upper Acute abdomen 1
quadrant pain, pyrexia and elevated inflammatory 1. H  A classic presentation of bowel obstruction.
markers. The mainstay form of treatment is regular 2. J  Free gas on an erect chest X-ray is diagnostic of a
analgesia and antibiotic use. perforation.
3. C  Typical presentation of renal colic.
Question 22 4. B  An abnormal amylase together with abdominal
C. Additional features may include menstrual cycle tenderness is a key feature of pancreatitis.
irregularities and micturition abnormalities. 5. F  The diagnosis is a dissecting aortic aneurysm
in view of symptomatology and a widened
Question 23 mediastinum.
E. Additional features would include swelling of the
testicle and no relief of pain upon elevation of the Glasgow Coma Scale
scrotum. This is unlike epididymitis where elevation 1. J  Obeys commands = 6, normal speech = 5,
may help improve the pain. spontaneous eye movement = 4.
2. A  Flexes to pain = 3, incomprehensible sounds = 2,
Question 24 no eye movement = 1.
A. Classical presentation of epididymitis. Antibiotics and 3. D  Withdraws to pain = 4, confused speech = 4,
analgesia are the mainstay form of treatment. eyes open to pain = 2.
4. D  No motor response = 1, normal speech = 5,
Question 25 spontaneous eye movement = 4.
A. A tension pneumothorax occurs when the pleura 5. G  Inappropriate speech = 3, obeys commands = 6,
is breached, allowing air to escape into the pleural eyes opening on command = 3.

184
Trauma and surgical emergencies 9

Acute abdomen 2 3. F  Perforated peptic ulcer. This is suggested by the


1. A  Acute pancreatitis. This is consistent with the left upper quadrant pain and recent nonsteroidal
abdominal pain experienced by the patient, as antiinflammatory drug use. He is also peritonitic.
well as dehydration, nausea, vomiting and the 4. B  Acute pyelonephritis. This is suggested by the
implied excess alcohol consumption that often flank pain radiating to the groin and associated
accompanies holidays. rigors.
2. I  Small bowel obstruction. This lady has absolute 5. E  Perforated appendix. This young man has features
constipation, nausea, vomiting and high-pitched of peritonism related to a right iliac fossa tenderness.
bowel sounds, all consistent with small bowel Other causes of right iliac fossa pain and peritonism
obstruction. This is a surgical emergency. would be unusual in a boy of this age.

185 
This page intentionally left blank
Perioperative care 10
ileus, with vomiting and constipation. The
SBA ANSWERS appearance of a grossly distended abdomen
that is tympanitic and nontender is associated
Question 1
with pseudo-obstruction. If he had just been
C. This patient has notable features of sepsis, namely
constipated, the rectum would have been
pyrexia, an elevated white cell count and CRP. The
loaded and the distension would not have been
patient could have concurrent hypovolaemic shock
as significant. He has probably not passed urine
due to sepsis and postoperative timeline, but with a
for 8 hours because of dehydration.
fever and elevated inflammatory markers, sepsis is the
3. A  Atelectasis. This patient has had emergency upper
most urgent diagnosis.
abdominal surgery. She is probably not breathing
satisfactorily because of postoperative pain. She
Question 2 therefore has a reduced oxygen saturation. At this
C. Contrast used for radiological investigations/
stage (i.e., less than 24 hours postoperatively),
procedures is notorious for adversely affecting renal
the alveoli have collapsed because of inadequate
function as the contrast is nephrotoxic. Precontrast
ventilation. At the present time, she has not
(and postcontrast) hydration with normal saline is
developed secondary infection but, if she receives
used; some centres use sodium bicarbonate as
good pain relief and physiotherapy, one should be
well. Nephrotoxics should be stopped pre- and
able to avoid secondary infection of the collapsed
postcontrast injection for a few days until the
alveoli.
creatinine resolves.
4. B  Anastomotic dehiscence. The history of
sudden-onset severe abdominal pain 4 days
Question 3 postoperatively suggests peritonitis. The patient is
E. According to current National Institute of Health and showing signs of septicaemia (i.e., a temperature
Clinical Excellence (NICE) guidelines, none of the listed of 38°C and hypotension). The most likely cause
investigations are recommended prior to surgery. is an anastomotic dehiscence. This is a significant
risk after a low anterior resection if the blood
Question 4 supply to the anastomosis is inadequate or there
B. According to the NICE, an ECG is highly has been poor operative technique.
recommended. If he had not had a prior chest X-ray, 5. D  Deep vein thrombosis. Patients who are having
then that would be warranted too. surgery for malignancy and pelvic surgery are
at increased risk of deep vein thrombosis.
Question 5 Poor mobility and low albumin and protein
E. According to the World Health Organization (WHO) levels postoperatively may cause bilateral
analgesic ladder, diclofenac would be the next ankle oedema. The presence of a pyrexia and
recommended choice of drug. tenderness in the right calf suggests a deep vein
thrombosis.
EMQ ANSWERS
Preoperative investigations
Postoperative complications 1. G  Echocardiogram. Examination of the patient
1. J  Wound infection. This wound discomfort is due suggests a diagnosis of aortic stenosis. This
to infection rather than a simple haematoma. needs to be confirmed by an echocardiogram
A haematoma would have been apparent so that the anaesthetist is aware and maintains
immediately postoperatively. A wound infection, a good blood pressure throughout the
however, takes several days to develop. operation.
Infection may have been precipitated by the 2. J  Spirometry. This patient needs assessment of
presence of a haematoma, as it is a good her respiratory function, rather than just a
culture medium. chest X-ray.
2. H  Pseudo-obstruction. This patient has been 3. E  Clotting screen. This patient is to undergo an
immobilized in bed following orthopaedic invasive procedure of which one complication
surgery. He has developed a postoperative is internal bleeding. As she has been jaundiced,

187 
EMQ answers

her clotting may be abnormal because of failure 4. C  Carotid Doppler scan. The history is suggestive of
to absorb vitamin K and produce some clotting transient ischaemic attacks, secondary to carotid
factors, such as II, VII, IX and X. Before the artery stenosis. This needs to be known about
procedure, her clotting should be normal or before coronary artery surgery as hypotension
corrected appropriately. may precipitate a further event.

188
Gastrointestinal surgery 11
a rise in the liver function test results, particularly
SBA ANSWERS bilirubin and alkaline phosphatase. The other diseases
do not cause jaundice with pain.
Question 1
B. Meckel diverticulum. Meckel diverticulum is the
remnant of the vitelline duct present in embryonic Question 6
life. It connects the developing embryo with the B. Dukes B. Dukes classification is most widely used
yolk sac. It is often asymptomatic; however, acute to stage and predict the prognosis of colorectal
inflammation of the diverticulum may occur, mimicking carcinoma. Dukes stages are as follows: stage A,
acute appendicitis. The mucosa at the mouth of invaded submucosa and muscle layer of the bowel,
the diverticulum may become inflamed and lead to but confined to the wall; stage B, breached the
intussusceptions and obstruction, or the diverticulum muscle layer and bowel wall, but no involvement
may perforate and cause peritonitis. More common in of lymph nodes; stage C1, spread to immediately
males. draining pericolic lymph nodes; stage C2, spread
to higher mesenteric lymph nodes; stage D, distant
visceral metastases.
Question 2
E. Ischaemic bowel. Ischaemic bowel is difficult to
diagnose and commonly missed. It usually presents Question 7
as abdominal pain out of proportion to clinical A. Mesenteric ischaemia. The key to diagnosis here
findings and investigations, and is often a diagnosis is in the history. The neck ultrasound scan is likely
of exclusion; however, the presence of metabolic to be a carotid Doppler scan, suggesting that there
acidosis helps. It rarely occurs in patients less than may be a history of ischaemia. Smoking is also
60 years old. considered a risk factor for this. Mesenteric ischaemia
generally only affects people over 50 years of age
and is associated with conditions causing arterial
Question 3
emboli and thrombosis. It presents as a colicky poorly
C. Intussusception. Intussusception occurs when one
localized pain with minimal tenderness in the early
segment of the bowel slides inside the adjacent
stages, leading eventually to signs and symptoms of
segment, like a telescope. The most common site
peritonitis.
is at the ileocaecal valve. Patients typically present
aged 5–12 months of age, with periods of screaming,
vomiting, blood in the faeces (redcurrant jelly) and Question 8
drawing up of the legs. The child is pale and a A. Pyloric stenosis. Pyloric stenosis, or narrowing, occurs
sausage-shaped mass may be felt on palpation of the in about 1 in 150 male infants and 1 in 750 female
abdomen. infants. Stenosis is caused by hypertrophy of the
circular muscles of the pylorus, obstructing the pyloric
Question 4 canal and the flow of contents from the stomach into
C. IV fluids. There is clinical evidence of shock with the the duodenum. Typically, it presents 4 to 6 weeks after
patient being tachycardic and hypotensive, so the birth with projectile vomiting within half an hour of a
initial management should be resuscitation with IV feed. Note that there is no bile in the vomit because
fluids. All of the other investigations are relevant, and the obstruction is proximal to the ampulla of Vater.
should be done once the patient is haemodynamically Peristaltic waves are visible in the child, a sausage-
stable. shaped mass (the enlarged pylorus) can be felt in the
right upper quadrant and the hypertrophied pyloric
muscle can be observed using ultrasonography.
Question 5
Treatment is surgical.
E. Cholangitis. The triad of symptoms which includes
fever, abdominal pain and jaundice is typical in
cholangitis. Cholangitis occurs when gallstones Question 9
become impacted in the bile duct; the bile, which B. Direct inguinal hernia. A hernia is the protrusion of
is unable to escape, becomes concentrated and any organ or tissue through its coverings and outside
infected, resulting in acute cholecystitis. This causes its normal body cavity. If reducible, hernias can be

189 
SBA answers

pushed back into the compartment from which they the inferior epigastric artery. They do not protrude into
came. Inguinal hernias are the most common and the scrotum.
may be direct (protruding through the posterior wall
of the inguinal canal), or indirect (passing through Question 17
the inguinal canal). Indirect hernias are much more C. Femoral hernias are four times more common in
common, and lie lateral to the inferior epigastric women and present below and lateral to the pubic
vessels. Direct inguinal hernias are less common, and tubercle.
lie medial to the inferior epigastric vessels.
Question 18
Question 10 A. A classic description of an umbilical hernia. Such
E. Colonoscopy. This history of a change in bowel habit hernias are at great risk of strangulation.
for greater than 6 weeks with rectal (PR) bleeding is
one of the criteria of the National Institute for Health Question 19
and Care Excellence (NICE) guidelines for an urgent A. Epigastric hernias typically present in the midline and
2-week referral for patients of any age. This should are often tender and irreducible.
include an urgent colonoscopy and review by a
colorectal surgeon. Question 20
C. Spigelian hernias present as lumps at the lateral
Question 11 margin of the rectus sheath. Strangulation or
D. Squamous cell carcinoma. Achalasia is a known obstruction often occurs.
risk factor for the development of squamous cell
carcinoma of the oesophagus. This is thought to be Question 21
caused by chronic inflammation and stasis within the B. Obturator hernias are typically seen in elderly women.
oesophagus. Other risk factors for the development of The hernia occurs through the obturator canal within
squamous cell carcinoma are smoking, alcohol, tylosis the pelvis, so the patient often presents with small
and Plummer–Vinson syndrome. bowel obstruction. There may also be pain along the
medial aspect of the thigh due to pressure on the
Question 12 obturator nerve.
C. The most likely diagnosis is acute diverticulitis.
Abdominal and pelvic CT scan is the most appropriate Question 22
investigation. Ultrasound would be poorly tolerated C. Left-sided abdominal pain and dark red rectal
and would not be specific. bleeding are diagnostic features of ischaemic colitis.
Except in the most severe cases, ischemic colitis is
Question 13 treated with supportive care.
C. The staging investigations suggest Dukes stage
C cancer. Surgery in addition to postoperative Question 23
chemotherapy is the most effective form of B. This is the most definitive step in the treatment for
management. sigmoid volvulus. Caecal volvulus requires immediate
surgery.
Question 14
E. The clinical diagnosis is acute cholecystitis most Question 24
appropriately investigated by abdominal ultrasound E. The diagnosis is haemorrhoids. Injection sclerotherapy
scan. Blood tests would show an inflammatory is the preferred management option in such a
response but would not be specific. condition, with an overall success rate of up to 85%.

Question 15 Question 25
A. A classic presentation of an indirect inguinal hernia. A. Classic presentation of a perianal haematoma. There
Such hernias pass through the internal ring lateral is often a history of straining prior to the perianal pain.
to the inferior epigastric artery, along the canal to
emerge at the external ring above the pubic crest and Question 26
tubercle. D. Glyceryl trinitrate is regarded as the first-line form
of management in those with a chronic anal fissure.
Question 16 It helps to relax the internal anal sphincter, allowing
B. A description of a direct inguinal hernia. Such hernias fissure healing and thus relieving the pain. For more
bulge through the posterior wall of the canal, medial to acute anal fissures, managing constipation is first line.

190
Gastrointestinal surgery 11

Question 27 EMQ ANSWERS


C. Such studies help to measure internal and external
sphincter pressures and assess integrity of the Gastrointestinal bleeding
sphincter complex using endoanal ultrasound, hence 1. B  Colonic carcinoma. The prognosis is good if
confirming such a diagnosis. detected early, so prompt referral is necessary.
2. E  Duodenal ulcer. Altered blood almost always
Question 28 comes from haemorrhage in the upper GI tract.
D. Classic presentation of an anal fistula. Diagnosis may 3. G  Haemorrhoids. Haemorrhoids commonly follow,
be aided by performing a fistulogram, proctoscopy or are exacerbated by, childbirth but often resolve
and/or sigmoidoscopy. spontaneously.
4. I  Mallory–Weiss tear. Another classic presentation
Question 29 due to tearing at the gastro-oesophageal junction
B. The diagnosis is an oesophageal perforation in view of as a result of excess vomiting.
the presence of crepitus in the suprasternal notch. This 5. J  Oesophageal varices. Oesophageal varices are
is best investigated by a chest X-ray in the first instance. associated with portal hypertension caused, in
turn, by hepatic cirrhosis.
Question 30
A. The diagnosis is corrosive oesophagitis which involves
an urgent endoscopy in the first instance to assess
Hernia
1. G  Richter. A partially obstructed hernia which can
extent of damage.
lead to bowel perforation through ischaemia.
2. J  Diaphragmatic. Such hernias occur when part of
Question 31
the stomach or intestine protrude into the chest
C. The diagnosis is achalasia. The most effective
cavity through a defect in the diaphragm.
management plan is a Heller myotomy, which involves
3. I  Spigelian. Such hernias are often difficult to
division of the muscles of the lower oesophageal
distinguish from inguinal hernias and may lead to
sphincter.
strangulation or obstruction.
4. B  Pantaloon. Such hernias are essentially a
Question 32
combination of an indirect and direct hernia.
C. This patient has evidence of jaundice. An abdominal
5. A  Inguinal. A classic description of a direct inguinal
ultrasound is of key importance in such cases to
hernia.
assess evidence of bile duct dilatation.

Question 33 Upper gastrointestinal disorders


E. The diagnosis most likely is liver cancer. Serum alpha- 1. D  Zollinger–Ellison syndrome. Management typically
fetoprotein is highly sensitive and specific for such involves the use of high-dose proton-pump
cancers. inhibitors or surgery.
2. G  Adenocarcinoma. Squamous cell carcinomas
Question 34 usually affect the middle third of the oesophagus
D. The diagnosis in this case is acute cholecystitis. and are due to a high intake of alcohol, smoking
Patients may often demonstrate Boas sign, which is and salted fish consumption.
hyperaesthesia below the right scapula. 3. A  Achalasia. Chest X-ray features may show
evidence of a fluid level behind the heart.
Question 35 Management typically involves surgical division of
B. Taking into account evidence of distant metastases, the lower oesophageal sphincter.
the mainstay form of management would be palliative. 4. F  Duodenal ulcer. Characteristic features of a
Resection of such a tumour with the hope for cure is duodenal ulcer. Gastric ulcers typically result in
less than 15%. pain after food.
5. I  Plummer–Vinson syndrome. Such swallowing
Question 36 difficulties are due to a web-like formation in the
C. The diagnosis is gastric cancer, most appropriately upper part of the oesophagus.
investigated by endoscopy and biopsy.
Lower gastrointestinal disorders
Question 37 1. B  Intussusception. Classic presentation of
B. Dukes stage B cancers extend into the muscularis or intussusception. Additional features may include a
into or through the serosa. The latter is referred to as mass in the right iliac fossa. Management typically
stage B2 and the former B1. involves surgery.

191 
EMQ answers

2. C  Perianal haematoma. Classic presentation of a 5. K  Umbilical hernia. These hernias usually resolve
perianal haematoma. within 2 years of birth but leave a weak spot; they
3. F  Angiodysplasia. In addition to colonoscopy, can reappear in the elderly and in postpartum
a diagnosis of angiodysplasia is often made females.
following arteriography.
4. E  Crohn disease. Additional macroscopic features Acute abdominal pain
include the presence of skip lesions. 1. H  Perforated sigmoid diverticular disease. This can
5. J  Ischaemic colitis. Complications include gangrene, occur acutely but is often proceeded by left iliac
perforation and stricture formation. fossa pain and constipation due to oedema of the
sigmoid colon. Constant pain implies diverticulitis
Hepatobiliary disorders with localized inflammation. A sudden change and
1. G  Biliary colic. Classic presentation of biliary colic. increase in severity of the pain with associated
The pain is often quite severe in nature and hypotension and tachycardia imply perforation
associated with episodes of vomiting. causing generalized peritonitis.
2. D  Cholangiocarcinoma. Such tumours carry a 2. B  Acute cholecystitis. Gallbladder disease is
poor prognosis with a 5-year survival rate of common in overweight women. The symptoms
approximately 35% following surgery. are not those of biliary colic because she has signs
3. F  Cholecystitis. Such a finding is known as of inflammation with pyrexia and a positive Murphy
Boas sign and is characteristic of acute sign, which is due to localized peritonism in the
cholecystitis. right hypochondrium.
4. A  Cholangitis. This triad of features is known as 3. A  Acute appendicitis. The symptoms and signs are
Charcot triad and is characteristic of cholangitis. typical of acute appendicitis. The symptoms of
5. E  Hepatoma. Other predisposing factors may dysuria and frequency are not uncommon with
include hepatitis or haemochromatosis. Treatment appendicitis, as the appendix can be irritating the
often involves surgical resection. bladder.
4. F  Large bowel obstruction. The symptoms are
of increasing constipation and colicky lower
Abdominal pain abdominal pain, so that the patient has now
1. A  Acute appendicitis. The pelvic symptoms developed absolute constipation. This is due to
suggest a pelvic appendicitis causing an unusual large bowel obstruction rather than small bowel
presentation, as it irritates the rectum and bladder. obstruction, because the abdomen is grossly
However, the causes of such a history in a young, distended and tympanitic, and there is no history
previously well man are limited, as is right iliac of vomiting.
fossa pain in such people. 5. D  Acute pancreatitis. This patient describes sudden
2. I  Ulcerative colitis. The age is suggestive of ulcerative onset of epigastric pain. This history may suggest
colitis, which is associated with sacroiliitis. a perforated duodenal ulcer. An erect chest X-ray
3. G  Renal colic. It can present as testicular or labial which does not show gas under the diaphragm
pain; haematuria makes the diagnosis more likely. does not rule this out, as 30% of perforated
Torsion is uncommon after the age of 20. duodenal ulcers do not give free gas. If, however,
4. E  Familial adenomatous polyposis. The family history the diagnosis was that of perforation, the patient
and previous colonoscopy make this more likely. would have been cold, clammy and hypotensive,
5. B  Acute diverticulitis. A diverticular bleed is likely. with generalized guarding and rigidity of the
abdomen. Diagnosis is therefore that of alcohol-
Hernias induced pancreatitis.
1. D  Incarcerated hernia. This gentleman has
an irreducible hernia that has no signs of Intestinal obstruction
strangulation. 1. B  Carcinoma caecum. This history of abdominal
2. E  Incisional hernia. These are not uncommon after distension, pain and feculent vomiting suggests
abdominal surgery (≈2%). small bowel obstruction. Examination reveals
3. G  Obturator hernia. These are uncommon but do abdominal distension, with visible peristalsis,
occur in thin women and can present with features suggestive of small bowel obstruction. The most
of bowel obstruction. likely diagnosis in someone aged 80 years, with
4. F  Inguinal hernia. This is more common in those with a history of malaise and weight loss, is caecal
a family history. Differentiating between direct and carcinoma.
indirect hernias clinically can be attempted, but is 2. E  Gallstone ileus. Small bowel obstruction is
not evidence based. suggested by the history. This could be due to

192
Gastrointestinal surgery 11

adhesions from previous abdominal surgery, but oesophageal varices are a possibility. Drowsiness
the abdominal X-ray shows air in the biliary tree. implies hepatic encephalopathy precipitated by the
This implies the presence of a cholecystoduodenal bleeding.
fistula, which is due to a gallstone eroding into the 5. I  Sigmoid carcinoma. The history suggests a
duodenum and causing small bowel obstruction perforated sigmoid carcinoma. The anaemia
when it lodges in the ileum. is due to chronic blood loss from the sigmoid
3. G  Pseudo-obstruction. Immobilized patients often carcinoma. This has also caused a recent change
develop an ileus, particularly after orthopaedic of bowel habit with an increase in constipation.
surgery. This is usually painless and may be The presence of pyrexia and localized peritonism
associated with decreased potassium levels. The in the left iliac fossa suggests the presence of
abdominal X-ray shows a grossly distended colon inflammation. This diagnosis could be diverticular
with no cut-off to imply obstruction. disease, but a localized perforation of the
4. I  Strangulated femoral hernia. This patient has obstructed carcinoma would fit with the iron-
symptoms of small bowel obstruction. There is a deficiency anaemia.
mass in the right groin, which in a man is usually
due to irreducible inguinal hernia, but men can Oesophagogastric disorders
also develop femoral hernias, and this is implied 1. B  Duodenal ulcer disease. This patient’s symptoms
by the fact that swelling is below and lateral to the are typical of a duodenal ulcer. He has increased
pubic tubercle. weight because of drinking milk and eating to
5. H  Sigmoid volvulus. This is an elderly patient relieve his epigastric pain.
with dementia who often suffers with chronic 2. A  Achalasia. This diagnosis is suspected as
constipation. In view of the dementia, she is not the patient is only 19 years old and has had
able to give a good history, but is just unwell with symptoms over the last few months. The
malaise and is obviously in some discomfort. recurrent chest infections are due to aspiration
The presence of a very distended tympanitic pneumonias, as food and fluid collect in a dilated
abdomen is usually indicative of sigmoid volvulus oesophagus above a ‘rat-tail’ stricture of the lower
and this is compatible with the abdominal X-ray oesophagus.
appearance. 3. J  Pyloric stenosis. This man has a long history of
peptic ulceration which has been left untreated. It
Gastrointestinal bleeding has therefore healed with scarring, to cause pyloric
1. G  Posterior duodenal ulcer. This is an elderly person stenosis. He is dehydrated because of vomiting.
who is on nonsteroidal antiinflammatory drugs The succussion splash is suggestive of it. The
and therefore is at increased risk of developing an classic biochemical abnormality is hypochloraemic
ulcer. Ulcers that bleed are usually in the posterior alkalosis.
part of the duodenum because the bleeding is 4. H  Oesophageal adenocarcinoma. This man has a
from the erosion of the gastroduodenal artery. history of reflux that has been self-treated with
2. A  Angiodysplasia. The differential diagnosis is antacids. This does put him at risk of developing
diverticular disease or angiodysplasia. Both can a benign oesophageal stricture secondary to
produce a significant GI blood loss, which is his oesophagitis. He is also at increased risk of
painless. By contrast, a sigmoid carcinoma would oesophageal carcinoma, particularly in a Barrett
not usually bleed enough to require a transfusion, oesophagus which is related to reflux. However,
but would be associated with chronic blood the presence of an iron-deficiency anaemia and
loss causing anaemia. Angiodysplasia is only cachexia is more compatible with oesophageal
diagnosed by colonoscopy. Diverticulae would be adenocarcinoma. This is also related to the
seen on a barium enema. fact that he has progressive dysphagia and
3. C  Caecal carcinoma. This patient has presented weight loss, whereas with a benign oesophageal
with anaemia, which suggests chronic blood stricture, the dysphagia may not be so
loss. The most likely diagnosis is that of a caecal progressive.
carcinoma, which is associated with a mass in the 5. G  Mallory–Weiss tear. This man has had
right iliac fossa and weight loss. Hepatomegaly haematemesis, which could be due to gastritis,
implies metastatic disease. peptic ulcer disease or oesophageal varices. He
4. F  Oesophageal varices. An alcoholic person has been drinking heavily and has subsequently
with haematemesis and melaena could have a vomited. The haematemesis developed after
duodenal ulcer, gastritis or oesophageal varices. the vomiting, which suggests it is due to trauma
The presence of splenomegaly, however, suggests caused from the vomiting that has caused a minor
that he probably has portal hypertension, and tear in the lower oesophagus.

193 
EMQ answers

Foregut investigations perforation of Crohn disease. The differential is an


1. A  Abdominal ultrasound. The history is suggestive appendix mass, when the history is usually a few
of gallstones, and the simplest test to make this days to 10 days. On examination, the patients are
diagnosis is an abdominal ultrasound scan. usually fairly well, with a slightly tender mass in the
2. G  Endoscopic ultrasonography. This patient has an right iliac fossa.
oesophageal carcinoma. It could be staged by 4. G  Mucocoele of the gallbladder. This man has a
doing a CT scan, which will show liver metastases, history of biliary pain. The gallstone has become
but a more accurate way of assessing whether stuck in the neck of the gallbladder, causing
the carcinoma has spread to the local structures, obstruction and then the gallbladder distends.
including the thoracic aorta, is by using an The jaundice is due to partial obstruction of the
endoscopic ultrasound. common bile duct by the distended gallbladder.
3. E  CT scan. The history is suggestive of pancreatic It is not an empyema as he would have a
carcinoma. An ultrasound scan has shown pyrexia.
a dilated common bowel duct. The possible 5. L  Uterine fibroids. These are often associated with
diagnosis is carcinoma of the head of the menorrhagia, causing anaemia and therefore
pancreas. A CT scan would be the best way tiredness. The ovarian cyst and carcinoma are less
of staging this. This would show the evidence likely to be associated with anaemia. The mass
of liver metastases and whether the pancreatic arising suprapubically is consistent with a uterine
carcinoma involves the superior mesenteric mass.
vessels.
4. C  Barium swallow. The history is suggestive Groin lump
of a pharyngeal pouch. It is not appropriate 1. C  Femoral hernia. This patient has symptoms of
to do an endoscopy because of the risk of obstruction due to a strangulated hernia. It is a
perforation. A barium meal looks at the stomach femoral hernia because of its classic position,
and duodenum, not the oesophagus. A barium below and lateral to the pubic tubercle. An
swallow is therefore the most appropriate obturator hernia is internal and is not seen
investigation, and will show the function and externally.
anatomy of the oesophagus. 2. D  Inguinal hernia. This is because it is above and
5. I  Laparoscopy. This man has carcinoma of the medial to pubic tubercle. It goes away when she
stomach. A CT scan shows no evidence of liver rests. A saphena varix is below and lateral to
metastases and an endoscopy shows a localized pubic tubercle, is compressible and has a cough
lesion in the antrum of the stomach. A CT scan impulse.
suggests operability, but is not appropriate for 3. G  Malignant lymphadenopathy. Although the
diagnosing peritoneal metastases as they may not patient presents with an acute pretibial
be evident on CT. A laparoscopy is, therefore, the laceration, the lump has been present for a
appropriate way forward. few weeks. She has had a previous melanoma
excised which is likely to recur. It is an irregular
Abdominal mass lump which is more likely to be malignant rather
1. B  Caecal carcinoma. This often presents insidiously than reactive nodes.
with fatigue due to anaemia before there are any 4. A  False aneurysm. This woman has had a
bowel symptoms. The colicky abdominal pain and radiological intervention to investigate the cause
vomiting imply obstruction developing. The age of her leg ulcer. Damage has been caused to the
of the patient makes the diagnosis more likely to artery by the procedure, so a false aneurysm has
be caecal carcinoma, rather than Crohn disease. developed. This is more likely than a femoral artery
The gynaecological pathologies are not likely to aneurysm.
present with obstruction.
2. J  Sigmoid carcinoma. This patient has symptoms Rectal bleeding
of a change of bowel habit of gradual onset. 1. J  Rectal villous adenoma. The history of rectal
She also has anaemia consistent with bleeding, diarrhoea and tenesmus could be due
malignancy. The mass is nontender but a to a rectal carcinoma, but a villous adenoma
diverticular mass is usually tender and not produces mucus and causes potassium
associated with anaemia. depletion.
3. C  Crohn disease. Although this woman has an 2. A  Anal carcinoma. Fresh rectal bleeding can be due
acute presentation, this is on a chronic history of to haemorrhoids, anal fissure or rectal carcinoma,
malaise and weight loss, which fits with a localized but only anal carcinoma causes an indurated,

194
Gastrointestinal surgery 11

perianal ulcer. Anal carcinomas are also thought to 4. D  Angiodysplasia. This is more common in elderly
be linked aetiologically with CIN. people and can be associated with significant
3. B  Anal fissure. Chronic constipation and bleeding to cause hypotension. Diverticulosis can
straining are causes of an anal fissure. It also cause spontaneous bleeding but patients
can occur at all ages and can cause some usually have a history of some bowel symptoms.
rectal bleeding. Haemorrhoids could also This man was previously fit and well and this
cause rectal bleeding but not likely in a relates to the fact that he is on no medication,
5-year-old child. which could predispose to peptic ulceration.

195 
This page intentionally left blank
Cardiothoracics and vascular
surgery 12
of features: claudication in the buttocks and thighs,
SBA ANSWERS absent or decreased femoral pulses and erectile
dysfunction in males. PAD affecting both femoral
Question 1
arteries would cause claudication in the thighs, but
C. Clinical opinion also holds that eccentric or
not the buttocks.
saccular aneurysms represent greater rupture
risk than more diffuse, cylindrical aneurysms.
Using computer modelling, Wall stress is Question 7
substantially increased by an asymmetric E. The mural thrombus is usually located in the
bulge in AAA. abdominal aorta, but, albeit infrequently, it can occur
in the thoracic aorta. Generally, thromboembolic
events are associated with advanced age, with
Question 2
the thrombus arising from complex and ulcerated
B. Cilostazol is a phosphodiesterase inhibitor
atherosclerotic plaques. Fat embolism syndrome
with therapeutic focus on cyclic adenosine
typically presents 24 to 72 hours after the initial
monophosphate. It inhibits platelet aggregation and
injury. Patients present with a classic triad:
is a direct arterial vasodilator. Its main effects are
respiratory changes, neurological abnormalities and
dilatation of the arteries supplying blood to the legs
petechial rash.
and decreasing platelet coagulation. A meta-analysis
of eight randomized, placebo-controlled trials of
cilostazol for moderate to severe claudication found Question 8
that 100 mg of the drug twice daily increased maximal B. The first most appropriate investigation is ultrasound.
and pain-free walking distances by 50% and 67%, CT scanning is only useful later to determine whether
respectively. endovascular or open repair is appropriate.

Question 3 Question 9
E. Smoking is the number one risk factor for peripheral D. Typical presentation of a mycotic aneurysm.
arterial disease (PAD), and smoking even a few Such aneurysms arise from infection within the
cigarettes a day can interfere with PAD treatment. vessel wall.
Smoking increases the risk for PAD by 2–25 times,
with the danger being higher when other risk factors Question 10
are present. Between 80% and 90% of patients with A. Annual abdominal ultrasound scans. All males at the
PAD are current or former smokers. Progression to a age of 65 will have an abdominal ultrasound. If this
more critical state of illness is likely for patients who shows evidence of an AAA, then it will be confirmed
continue to smoke. using a CT scan of the abdomen. If the AAA is
between 3 cm and 4.4 cm, then they will have an
Question 4 annual ultrasound. If it is between 4.5 cm and 5.4 cm,
A. Back pain is characteristic of a thoracic aortic then they will have three-monthly ultrasound scans;
aneurysm. Swallowing impairment is typical due to however, if it is 5.5 cm or bigger, then patients are
compression of the cervical oesophagus. This is, considered for surgery.
however, rare.
Question 11
Question 5 E. Such aneurysms occur when an artery has been
E. This artery is commonly damaged in AAA repair and is damaged. In this case, the femoral artery has been
most likely responsible for paraplegia postoperatively. punctured for an angiogram but has failed to seal.

Question 6 Question 12
B. Aortoiliac occlusive disease. This patient has B. The diagnosis is intermittent claudication which is
developed Leriche syndrome, which is caused by most likely investigated via measurement of ankle
peripheral artery disease affecting the abdominal aorta brachial pressure index. This is often reduced by
or the common iliac arteries. It presents with a triad approximately 60%–70%.

197 
EMQ answers

Question 13 Question 18
D. Risk factor modification is always the first-line B. The diagnosis here is most likely to be lymphoedema
management of intermittent claudication, in this case, with treatment being in the form of compression
hypertension. Drug therapy and surgery are rarely stockings in the main.
common practice in the first instance.
Question 19
Question 14 E. This is mesenteric ischaemia. This should be
E. The hypoglossal nerve may be damaged as it crosses suspected in any patient who has AF and presents
the internal carotid artery. with abdominal pain (due to systemic emboli).
Diagnosis can be confirmed by arteriography or MR
Question 15 angiography, but most are made during a laparotomy,
D. This patent has a symptomatic AAA and needs a which reveals areas of necrotic bowel.
CT scan of the abdomen. If asymptomatic, then an
abdominal ultrasound could be used; however, even if EMQ ANSWERS
this test is positive, patients are still sent for a CT scan
of the abdomen to confirm the diagnosis as CTs are
Vascular disorders
1. E  Also known as Buerger disease. This condition is
more sensitive for diagnosis of smaller-sized AAAs.
characterized by segmental thrombotic occlusions
An abdominal X-ray would often be normal; however,
of small- and medium-sized vessels in both upper
sometimes if an aneurysm is chronic, there may
and lower limbs.
be evidence of calcification around the walls of the
2. F  Treatment typically involves avoidance of cold and
aneurysm.
stopping smoking. Medical treatment includes the
use of nifedipine.
Question 16 3. I  This is essentially recurrent episodes of superficial
A. Handheld Doppler helps to confirm sapheno-femoral thrombophlebitis which may also precede clinical
and sapheno-popliteal incompetence. Other useful manifestations of malignancy.
tests include venography and plethysmography, but 4. A  Typical features of an AAA. Surgery is typically
these are rarely needed. indicated for aneurysms which are symptomatic
or those which are asymptomatic but greater than
Question 17 5 cm.
D. Compression stockings are the initial choice of 5. C  Diagnosis is often confirmed via chest X-ray
management. Surgery is a useful second option. which typically shows a widened mediastinum.
Laser therapy and radio frequency ablation are Management involves surgery and urgent blood
considered if the above options have failed. pressure control.

198
Orthopaedics 13
SBA ANSWERS Question 8
E. Rest improves the pain of osteoarthritis, and
Question 1 increasing muscle strength improves joint stability and
C. Osgood–Schlatter disease is encountered in patients reduces pain.
between ages 10 and 15. These patients are often
active in sports that involve a lot of jumping. It is Question 9
thought to be secondary to repetitive microtrauma C. Typical presentation of a rotator cuff tear. Usually
and traction apophysitis of the tibial tuberosity. This associated with traumatic injury or dislocation.
condition is usually self-limited, and most patients Management is usually through surgery in the young.
are able to return to full activity within 2 to 3 weeks.
Treatment includes rest, ice, antiinflammatory Question 10
medications, a rehabilitation program and an C. The diagnosis is tennis elbow or lateral epicondylitis
infrapatellar strap during activities. best treated with NSAIDs initially. With lateral
epicondylitis, there is degeneration of the tendon’s
Question 2 attachment, weakening the anchor site and placing
D. Likely diagnosis of acute osteomyelitis most greater stress on the area. This can lead to pain
commonly due to S. aureus in over 80% of cases. associated with activities in which this muscle is active,
such as lifting, gripping and/or grasping. Sports such
as tennis are commonly associated with this, but the
Question 3 problem can occur with many different activities.
A. MRI has become the gold-standard investigation but
may need sedation at this age. Plain limb X-rays may
be normal for up to 10 days.
Question 11
B. Neck of femur fracture, which can be intracapsular
(subcapital, transcervical, basicervical) or
Question 4 extracapsular (intertrochanteric, subtrochanteric);
E. A tumour derived from osteoblasts. The ‘sunray’-like
common in osteoporotic elderly after a fall; affected
appearance is due to the formation of bony spicules.
limb is shortened and externally rotated.

Question 5 Question 12
E. In a patient who has nontraumatic hip pain and is on B. Usual features of ulnar nerve neuropathy. If the median
chronic steroid therapy for a disease such as SLE, nerve is damaged, the ability to abduct and oppose
one of the major complications is avascular necrosis the thumb may be lost due to paralysis of the thenar
(AVN). In the setting of AVN, MRI is the most sensitive muscles.
imaging modality and the diagnostic method of choice
when clinical suspicion is high. MRI results will show
Question 13
marked bone marrow oedema even when the X-ray is
A. SUFE: overweight boy with pain in groin, front of
totally normal and is the only objective test to rule out
thigh or knee and limping. Normally ages 10–14 and
the disease in this early stage.
<10 years associated with hypothyroidism. Pain
comes on with movement. Affected leg is shorter and
Question 6 turns outwards. X-ray shows widened growth plates
B. Classic presentation of acromioclavicular which are denser/lucent.
osteoarthritis. Excision of the outer end of the clavicle
may help to relieve symptoms. Question 14
A. The radial nerve may be damaged anywhere in its
Question 7 course. It is most commonly affected in the upper arm
A. This is a common problem to affect the shoulder best where it winds round the humerus and in the extensor
treated with NSAIDs or steroid injections. The pain is muscle compartment of the forearm affecting the
often worsened by shoulder overhead movement and posterior interosseous branch. Finger and wrist drop
may occur at night, especially if the patient is lying on are common, their severity depending upon the site of
the affected shoulder. the lesion.

199 
SBA answers

Question 15 that are characterized by multinucleate giant cells.


B. This condition is typically due to thickening They mainly occur in patients aged between 20 and
and shortening of the palmar fascia which 45 and form around the epiphysis of the bones.
becomes adherent to the skin. As a result, The knee is a common site for them to form.
the ring and little finger develop a fixed flexion These tumours often progress slowly, and X-rays
deformity. of the area would show osteolysis and may show
pathological fractures.
Question 16
E. Results from localized thickening of the flexor tendon Question 22
with associated sheath narrowing causing the finger C. Typical presentation of an anterior cruciate ligament
to catch as it is flexed. injury. It is usually detected in approximately 70%
of cases.
Question 17
C. The diagnosis is most likely to be cauda equina Question 23
syndrome. MRI is the gold-standard investigation in E. Here the tibia has been forced posteriorly resulting in
such cases. such an injury.

Question 18 Question 24
B. Meniscal tear. This is a common sporting injury, C. The patient has lumbar spinal stenosis. MRI of
which classically occurs while performing a sharp the lumbar region is the only test to diagnose
turn. It presents with pain and swelling of the knee. lumbar spinal stenosis. Lumbar spinal stenosis
Patients complain of locking of the knee joint and is characterized by narrowing of the spinal canal,
a reduced range of movement. On examination an leading to pressure on the spinal cord which results
effusion may be present and McMurray test may in pain occurring when the back is in extension.
be positive. During an anterior cruciate ligament The characteristic features of pain in lumbar spinal
tear, patients often state that they hear a ‘pop’ as stenosis include pain while walking, radiating into the
the ligament ruptured. It causes pain, swelling and buttocks and thighs bilaterally. Pain gets worse when
instability. On examination the anterior draw test is walking downhill and better when sitting. Unsteady
positive. Patella tendon rupture is a less common gait as well as leg weakness while walking may
injury. This would lead to an upward displacement of also result. Pedal pulses and ankle/brachial index
the patella, and an inability to extend the knee joint are normal and about quarter of the patients have
and inability to ‘straight leg raise’. diminished lower extremity reflexes. Pain is much
less with activities that involve leaning forward of the
patients (e.g., cycling).
Question 19
A. Plain X-ray is the best-choice investigation from the
listed options. Question 25
D. As the child is over 6 months, this is the most suitable
Question 20 management. Before 6 months, a splint is typically
C. Anterior cruciate ligament rupture is the most likely used.
cause in approximately 40% of cases. Symptoms
include pain, a popping sound during injury, instability Question 26
of the knee and joint swelling. Swelling generally B. Typical presentation of Perthes disease. A
appears within a couple of hours. childhood hip disorder initiated by a disruption
of blood flow to the femoral head causing
Question 21 osteonecrosis and restricts growth. Over time,
D. Osteosarcoma is the most common primary healing occurs by new blood vessels infiltrating the
malignant bone tumour. It mainly occurs in dead bone and removing the necrotic bone which
adolescents, and a common site is around the leads to a loss of bone mass and a weakening of
knee. X-ray would show bone destruction and new the femoral head.
bone formation (known as ‘sunray spicules’), along
with uplifting of the periosteum (known as Codman Question 27
triangle). Ewing sarcoma is a rare primary bone C. Usual findings of a SUFE best treated with surgery.
tumour that occurs in children and forms from the SUFE refers to a fracture through the physis, which
round cells of the long bones. Giant-cell tumours are results in slippage of the overlying end of the femur
also known as osteoclastomas and are rare tumours (epiphysis).

200
Orthopaedics 13

Question 28 dislocation. Do not push too far or the shoulder


B. The axillary nerve passes around the neck of the will redislocate.
humerus, so any displacement of the proximal 6. F  Tests for laxity of the anterior cruciate ligament. An
humerus can lead to damage to the axillary nerve. alternative to the anterior draw test, though it is
In any patient who has dislocated the shoulder, it is slightly harder to perform.
important to check sensation over the C5 dermatome. 7. G  Tests for torn medial menisci. Uncommonly asked
for in an objective structured clinical examination
EMQ ANSWERS (OSCE), as if positive, it is painful for the patient.

Fractures Knee injuries


1. E  Classic history of a distal radius fracture. 1. B  Bucket handle tear of meniscus. Meniscal injuries
Associated with a backward angulation and are common in sportsmen and women; the
displacement producing a dinner fork wrist. history of locking suggests that there is a meniscal
2. C  Radial nerve injury may lead to a wrist drop. fragment being trapped in the joint.
Immobilization is the mainstay form of treatment. 2. C  Dislocation of the knee. This is exceedingly rare
3. H  This occurs just below the femoral head. but does happen; in this case, the pulses and
4. D  Scaphoid fractures result in tenderness on direct ligaments should all be checked as the patient
pressure approximately 2 cm distal to the Lister may require emergency surgery to repair the
tubercle of the radius and on proximal pressure on vascular supply.
the extended thumb. If one suspects this fracture, an 3. J  Tibial plateau fracture. These are common and
additional oblique scaphoid view should be requested. occur with trauma to the knee in which the femoral
5. H  Presents with external rotation and shortening condyles impact upon the tibia. Schatzker refers
of the leg. Medial femoral circumflex artery to a grading system for these types of fracture.
involvement may lead to ischaemic necrosis of the 4. A  Anterior cruciate ligament rupture. This is
femoral head. demonstrated by the anterior draw test; other
ligaments may be injured at the same time.
Fracture complications 5. G  Patella fracture. Falling onto the knee may fracture
1. I  Hip fractures are commonly associated with the the patella; this is supported by the inability to
development of pneumonia. The severity of such a straight leg raise.
condition is based on the CURB-65 score, which
assesses the presence of confusion, serum urea, Skin, joint and bone infections
respiratory rate, blood pressure and age. 1. E  Approximately 20% of all bursitis cases are
2. H  Classic features of a fat embolism. caused by bacterial infections; rarely, bursitis
3. D  Nonunion is commoner in cortical bone rather is caused by fungi and algae. S. aureus/S.
than in cancellous bone. Predisposing factors epidermidis as the offending organism in 90%
include malignancy and infection. of cases and Streptococcus species in 9% of
4. B  This complication is associated with fluid loss and cases.
acute tubular necrosis as a result of myoglobin 2. D  Necrotizing fasciitis causes include group
release. An urgent fasciotomy may be required. A Streptococcus (group A strep), Klebsiella,
5. E  Classic description of malunion. The resulting Clostridium, E. coli, S. aureus, Aeromonas
deformity may be of length, angulation or rotation. hydrophila and others. Group A strep is
considered the most common cause of
Orthopaedic clinical examination necrotizing fasciitis.
1. F  Tests for laxity of the anterior cruciate ligament. 3. E  Prosthetic joint infections (PJIs) occur in 1.5% to
‘Give’ of more than 1 cm is considered abnormal. 2.5% of all hip arthroplasties. Pain is the most
2. D  Tests for a fixed flexion deformity of the hip. This consistent symptom. Staphylococcus species are
can be a difficult test to explain to the patient and so the most common organisms isolated from PJI sites.
good communication skills and practice are required. 4. G  P. multocida infection in humans is often
3. J  Tests for weak ipsilateral hip abductor muscles. If associated with an animal bite, scratch or lick, but
the patient is unsteady on the left foot and falls to infection without epidemiologic evidence of animal
the right side, the problem is with the left hip. contact may occur.
4. E  Tests for a fixed flexion deformity of the lumbar 5. B  Puncture wounds of the foot can frequently
spine. This is a relatively simple test but should be become infected with Pseudomonas species
practised as it can seem complex. and the patient will present with drainage with
5. I  Tests for an unstable shoulder girdle. This a sweet, fruity-smelling discharge. Cellulitis and
tests for an unstable shoulder following a prior osteomyelitis are common complications.

201 
This page intentionally left blank
Ear, nose and throat 14
SBA ANSWERS Question 8
B. Pleomorphic adenoma is a common benign salivary
Question 1 gland neoplasm characterized by neoplastic
D. A perilymphatic fistula between the middle and inner proliferation of parenchymatous glandular cells along
ear may be caused by barotrauma from scuba diving, with myoepithelial components, having a malignant
as well as by direct blows, heavy weightbearing and potentiality. It is the most common type of salivary
excessive straining (e.g., with sneezing or bowel gland tumour and the most common tumour of the
movements). This patient’s recent trip involved two of parotid gland.
these potential factors.
Question 9
Question 2 B. Frey syndrome is a condition that occurs following
B. Presentation is usually during the second or third trauma to the parotid gland. The auriculo-temporal
decade of life. Surgery is the mainstay form of branch of the trigeminal nerve sends parasympathetic
management. fibres to the parotid gland, and sympathetic fibres to
the facial sweat glands. Trauma to this area means
that these nerves must then regrow, and often they
Question 3 switch places. This results in gustatory sweating,
C. This cyst is typically smooth and mobile. Diagnosis
meaning that when the patient eats, the sweat glands
is often by fine-needle aspiration which produces a
on the cheek become stimulated, resulting in sweating
creamy-coloured fluid.
and erythema.

Question 4 Question 10
C. In patients with a sinus infection, acute bacterial E. This is a benign tumour which may occur bilaterally in
rhinosinusitis should be diagnosed and treated with approximately 10% of cases. Warthin tumour primarily
antibiotics only if symptoms have not improved after affects older individuals (age 60–70 years). There is a
10 days. slight male predilection according to recent studies.
The tumour is slow growing, painless and usually
Question 5 appears in the tail of the parotid gland near the angle
B. Otitis externa, or infection of the external ear canal, of the mandible. In 5% to 14% of cases, Warthin
can be caused by a variety of organisms, notably tumour is bilateral, but the two masses usually are
including E. coli, P. aeruginosa, P. vulgaris and S. at different times. Warthin tumour is highly unlikely to
aureus. There is, however, a severe subtype of otitis become malignant.
externa, malignant otitis externa, of which you should
be aware. This form is specifically caused by P. Question 11
aeruginosa, and tends to affect elderly diabetics and A. Typical presentation of otitis externa. Management
acquired immunodeficiency syndrome (AIDS) patients, involves acetic acid topically in mild cases to antibiotic
causing the findings illustrated in the question stem. drops in moderate to severe cases.
E. coli can cause both otitis externa and acute otitis
media, but does not usually cause malignant otitis
Question 12
externa.
B. Rare after the age of 5. H. influenzae is the main
organism responsible for such a condition in
Question 6 approximately 40% of cases. Complications include
D. Auscultation may often reveal a bruit. Surgical excision effusion or scarring of the tympanic membrane.
is the preferred treatment option.
Question 13
Question 7 E. This child has otitis media. Antibiotics are typically
C. A plain XR may help to reveal a stone in the parotid administered for a 7- to 10-day period due to fever. If
duct. Treatment is with mouth care, rehydration and patient is afebrile, then conservative management is
antibiotics. indicated.

203 
EMQ answers

Question 14 EMQ ANSWERS


D. Glue ear is essentially otitis media with effusion and
typically occurs between the age of 2 and 6. In over Neck swellings
90% of cases, the effusion resolves spontaneously. 1. E  Additional features include a lack of saliva
production. Management is usually symptomatic.
Question 15 2. H  Salivary gland malignancies are typically
C. This condition results in unilateral sensorineural carcinomas and usually affect middle- to older-
hearing loss and tinnitus. Surgery is the mainstay aged people.
form of treatment. When nerve deafness is present, 3. C  Such tumours are highly vascular and hence their
then the note is audible at the external meatus, as propensity to pulsate.
air and bone conduction are reduced equally, so that 4. B  Management is typically surgical.
the air conduction is better (as is normal) than bone 5. J  Such swellings are typically large and thick walled
conduction: this is termed ‘Rinne positive’. in nature. They may lead to stridor and cyanosis in
certain cases.
Question 16
A. Tinnitus, hearing loss and vertigo are associated with Neck lumps
Ménière disease. Antihistamines have been shown to 1. H  Thyroglossal cyst. Midline, moves on swallowing
decrease middle ear labyrinth excitability and block and tongue protrusion: These are the classic
conduction in the middle ear vestibular cerebellar features of a thyroglossal cyst. It has been present
pathways which primarily aid in reducing vertigo. since birth, which confirms its congenital nature.
2. F  Lymph nodes. There are multiple small lumps,
Question 17 and their presence in the axilla also confirms the
D. In view of his haemodynamic instability and ongoing likelihood of lymph nodes; you should check the
blood loss, intravenous access for urgent fluid entire neck, axilla and groin, as well as suggesting
resuscitation (in this case with blood) is the initial step a breast examination in a woman (for lumps).
in management. 3. A  Branchial cyst. These typically affect young men,
and appear in this characteristic location.
Question 18 4. G  Pharyngeal pouch. A new-onset history of
E. Oral antihistamines may be added following steroid dysphagia with a new neck lump in a previously
usage. Surgery is often the last resort. well elderly man suggests this; the lump should
not move on swallowing or tongue protrusion as it
Question 19 is not attached to the thyroid (if it does, reconsider
D. Pharyngeal pouches are also associated with the diagnosis!).
repeated chest infections and regurgitation of food. 5. I  Thyroid goitre. The features of the lump suggest a
Management is typically surgical. multinodular thyroid (it is large and has an irregular
surface). Furthermore, the patient has signs of
Question 20 systemic hyperthyroidism, which should be clearly
B. Smoking and alcohol are the most common risk mentioned for extra marks!
factors in the development of such a condition, but
there is an association with EBV. Neck swellings
1. H  Submandibular calculus. This is a typical history
Question 21 of a submandibular stone. On eating, the salivary
A. A branchial cyst is a congenital cyst that arises due to glands are stimulated to produce saliva. As
either failure to obliterate the second branchial cleft or there is a calculus present in the duct, the gland
failure of the fusion of the second and third branchial swells because it is obstructed. This occurs most
clefts. It presents with a fluctuant lump on the side commonly in the submandibular gland. The stone
of the neck, at the level where the upper third of the may be palpable in the floor of the mouth.
sternocleidomastoid muscle meets the middle third 2. A  Anaplastic carcinoma of the thyroid. This is a very
of sternocleidomastoid muscle. A dermoid cyst is a short history of 2 months. The rapid increase in
midline lump which often occurs in young patients, size suggests malignancy. The other features that
and when excised, contains dermal structures, such are compatible with this are the hoarse voice due
as teeth or hair. A chondroma is a benign cartilaginous to involvement of the recurrent laryngeal nerve;
tumour, which can form a hard lump in the midline of the audible wheeze will also be due to pressure on
the neck. the trachea and paralysis of the recurrent laryngeal

204
Ear, nose and throat 14

nerve will also have effects on the vocal cord. The 4. C  Branchial cyst. This usually presents in young
hard mass in the lower anterior part of the neck people. It may not be apparent until an infection
is suggestive of anaplastic carcinoma. Riedel develops, and the classic position is at the anterior
thyroiditis would also produce a hard, irregular aspect of the upper third of the sternomastoid
swelling of the thyroid gland, but would have a muscle.
longer history. 5. J  Virchow node. The history is suggestive of gastric
3. G  Solitary thyroid nodule. This swelling is benign, carcinoma at the cardia with secondaries in the
as is it oval and smooth. It is obviously connected supraclavicular nodes (i.e., Virchow node). The
with the thyroid because it moves on swallowing. dysphagia, weight loss and vomiting would be
It is not a thyroglossal cyst because it is to the left compatible with a pharyngeal pouch, but this
of the midline. It is not a branchial cyst because it would produce a soft swelling of variable size in
moves on swallowing. the left side of the neck.

205 
This page intentionally left blank
Urology 15
and staging. Testicular cancer is the most common
SBA ANSWERS cancer in men between 20 and 40 years of age.
Lifetime incidence is 0.4% for white males and 0.08%
Question 1
in black males. Cryptoorchidism is the biggest risk
D. This patient has epididymo-orchitis. This condition is
factor for testicular cancer and it increases the risk
most often caused by coliform bacteria in men over the
about fourfold. Nearly 95% of all malignant testicular
age of 35. Non-sexually transmitted epididymitis is usually
cancers are of the germ cell type that are divided into
caused by enteric organisms and is often associated with
the seminoma and nonseminoma. Various tumour
recent urinary tract instrumentation, systemic disease
markers can help distinguish between the two types
or immunosuppression. These organisms are effectively
of germ cell tumours (e.g., β-HCG and AFP).
treated with ofloxacin. Those less than 35 years with
this condition have a higher chance of having a sexually
transmitted disease (STD) and would benefit from
Question 7
D. Urinary symptoms and suprapubic tenderness are key
intramuscular ceftriaxone and oral doxycycline. However,
features of acute cystitis.
these medications would not adequately cover coliform
bacteria. Clindamycin and famciclovir would also be
ineffective. Testicular torsion is another possible cause
Question 8
C. Calcium oxalate containing stones account for
of testicular pain with a quick onset, and would require
approximately 60% of all kidney stones. These stones
urgent urology surgery consultation. However, in cases
are radio-opaque, (visible on X-ray). Hypercalciuria is
of torsion, the examination usually reveals an elevated
the direct precursor to most calcium stone formations.
testicle and testicular ultrasonography would show
This increase of calcium levels in the urine can occur
decreased or absent blood flow on the affected side.
from increased intestinal absorption, decreased renal
absorption, or increased bone resorption. The patient
Question 2 in this case has no medical problems and his urine is
A. Uric acid stone formation can be prevented by
not suggestive of any infection. His stone is visible on
alkalinizing the urine with citrate.
x-ray and is most likely to be ‘calcium oxalate’.

Question 3 Question 9
A. Urinalysis and culture are essential here. The patient B. Urinary symptoms with a sterile culture is diagnostic
has a urinary tract infection as noted by his urinary of interstitial cystitis. Referral onwards to urology is
symptoms and associated pyrexia. indicated for consideration of cystoscopy.

Question 4 Question 10
A. BPH is a condition characterized by enlargement of C. Prostatitis manifests as urinary frequency and dysuria
the prostate gland that makes urination difficult in in addition to prostatic tenderness. It is commonly
males. α-Adrenergic antagonists block the receptors caused by Escherichia coli or Staphylococcus aureus.
in the bladder neck and prostrate and work by
reducing the smooth muscle tone. This improves the Question 11
urine flow rate and reduces the symptoms associated A. The clinical presentation is that of renal stones, and
with the disease. the most common stone are calcium oxalate stones.
Risk factors include low urine volume, hypercalciuria,
Question 5 hyperuricosuria, hyperoxaluria and hypocitraturia.
B. This is a severe urinary tract infection with associated Reducing dietary calcium is NOT an effective method
pyrexia. Intravenous antibiotics such as cefuroxime or of prevention; low dietary calcium leads to increased
gentamicin may be of use. gastrointestinal oxalate absorption and higher urine
levels of calcium oxalate
Question 6
E. A non-painful testicular mass that does not Question 12
transilluminate is most likely testicular cancer and a B. PSA is sensitive and specific for prostate cells, but not
full evaluation must be performed for proper diagnosis specific to the prostate cancer cells.

207 
SBA answers

Question 13 Question 22
E. Epididymo-orchitis is commonly seen in young A. This is a T2 tumour, best treated with radical
sexually active males. The offending organism is cystectomy.
commonly Chlamydia.
Question 23
Question 14 A. Individuals with mild symptoms are best managed
D. Squamous cell carcinoma. Squamous cells appear by watchful waiting as the risks of treatment usually
in your bladder in response to infection and irritation. outweigh the benefits.
Over time they can become cancerous. Squamous cell
bladder cancer is rare (5%–7%). It is more common in Question 24
the parts of the world where a certain parasitic infection C. This patient has developed post-TURP
(schistosomiasis) is a prevalent cause of bladder syndrome. This is caused by systemic
infections. The species, Schistosoma haematobium absorption of the irrigation fluids used during
(SH), is the type that causes urinary bladder infestation, TURP and these lead to dilution of the plasma,
and the most distinguishing symptom is painless hyponatraemia and hypothermia. Hyponatraemia
haematuria, more common in children who play can lead to seizures.
barefoot in the marshes where SH is endemic.

Question 15 Question 25
A. A classic description of a hydrocele.
B. A CT scan is the most useful here in diagnosing a
possible renal tumour.
Question 26
B. Epididymo-orchitis is commonly seen in young males
Question 16
and is particularly tender in nature.
E. A secondary varicocele is due to compression of the
venous drainage of the testicle. A pelvic or abdominal
malignancy is a definite concern when a right-sided Question 27
varicocele is newly diagnosed in a patient older than B. Ciprofloxacin. This patient has prostatitis. This
40 years of age. The most common cause is renal presents with a fever, back pain, urinary symptoms
cell carcinoma, followed by retroperitoneal fibrosis or (dysuria, frequency and haematuria), haematospermia
adhesions. and a swollen prostate. Ciprofloxacin is a quinolone
antibiotic and used because it is able to penetrate the
Question 17 prostate well. Doxazosin is an α-blocker, which can
D. The only curative treatment is a nephrectomy. be used to treat BPH. Finasteride is a 5α-reductase
inhibitor also used in BPH.
Question 18
E. Most stones pass spontaneously. Stones greater than Question 28
5 mm require treatment. E. Painless haematuria is a buzzword to associate
with bladder cancer (of which TCC is the most
Question 19 common).
A. This is a classic presentation of ureteric colic.
Analgesia is of huge importance in helping to relieve Question 29
the patient's symptoms. C. A common presentation of an epididymal cyst.

Question 20 Question 30
B. Bladder stones can be asymptomatic but can cause E. Acute urinary retention requires immediate urinary
irritation to the bladder wall or neck, resulting in catheterization as complications include renal failure
suprapubic pain (can be referred to the tip of penis, and hydronephrosis. Suprapubic aspiration only
scrotum, perineum), dysuria, urinary frequency, if symptomatic, and if urinary catheterization has
urgency, nocturia and haematuria. Pain is often failed. In the longer term, treatment depends on the
aggravated by sudden movements or exercise. underlying cause.

Question 21 Question 31
A. The definitive investigation in bladder cancer. Biopsy E. Additional features would include swelling of the
will allow for staging assessment and ultimately choice testicle. This is unlike epididymitis, where elevation
of treatment. may help improve the pain.

208
Urology 15

possibility of prolapsed intervertebral disc. He


EMQ ANSWERS suddenly develops acute onset of difficulty in
passing urine, which is due to a central disc
Scrotal swellings
prolapse compressing the spinal cord and
1. C  Hydrocele. A rapid onset, tense lump which is not
the sacral nerve roots. This fits with an area
particularly tender but is transilluminable suggests
of saddle anaesthesia and decreased anal
a hydrocele.
sphincter tone.
2. I  Testicular tumour. The lump arises from the testis
itself and the young man has been losing weight;
this is cancer until proved otherwise. The other Urological investigations
testis must be palpated, the groin checked for 1. A  The diagnosis is a urinary tract infection most
lymphadenopathy (although the first point of lymph appropriately confirmed by a urine dipstick or urine
drainage for the testis is the paraaortic nodes) microscopy and culture.
and tumour markers should be sent (β-HCG and 2. H  The diagnosis is renal colic. The clue is with
alpha-fetoprotein). regards to his long distance running and hence
3. E  Indirect inguinal hernia. Direct hernias very rarely tendency to become severely dehydrated. An
enter the scrotum. abdominal KUB is the most appropriate diagnostic
4. J  Varicocele. The classic statement is that when tool.
standing it feels like ‘a bag of worms’, and 3. D  The diagnosis is urinary tract obstruction,
it disappears when lying down as blood is specifically at the bladder. An ultrasound scan will
redistributed. Remember 95% are left-sided due help to detect evidence of obstruction and likely
to venous drainage into the left renal vein (where hydronephrosis.
there is turbulent blood flow) as opposed to 4. C  The diagnosis is most probably prostatic
directly into the inferior vena cava on the right side carcinoma. Serum PSA is likely to be notably
(where flow is less resistant). elevated.
5. G  The diagnosis is stress incontinence most
Lower urinary tract symptoms appropriately diagnosed by urodynamic studies.
1. B  BPH. This patient has a chronic history of
symptoms suggestive of outflow obstruction. The Urgent urological referrals
tests of flow rate confirm this picture. 1. I  Teratoma of testis. A painless swelling of testis is
2. I  Stress incontinence. This woman has typical likely to be a tumour and, in a 19-year-old, the
symptoms of stress incontinence (i.e., on most likely diagnosis is a teratoma. Seminomas
coughing and sneezing). The risk factors for its and lymphomas occur in older men.
development are three vaginal deliveries with a 2. K  Ureteric colic. The history is of severe, colicky loin
prolonged first labour. to groin pain. Although there is some dysuria and
3. J  Urethral stricture. This young man has symptoms frequency, there is no fever to suggest a urinary
of obstruction. There is a past history of a fall tract infection or pyelonephritis. The dipstick
astride a bar as a child, which may have led to a shows blood only and no evidence of infection.
urethral injury that was initially undetected. This 3. A  BPH. This patient has acute-on-chronic urinary
has healed to form a urethral stricture. outflow obstruction. He has some symptoms
4. E  Multiple sclerosis. This patient has symptoms of suggestive of a urinary infection, but these are
bladder instability, but the associated past medical also symptoms of obstruction. As he has a long
history is that of unilateral transient blindness, history of nocturia and poor stream and his PSA is
which implies the possible diagnosis of multiple normal, it is more likely to be BPH than cancer.
sclerosis. 4. C  Epididymitis. This is not a simple urinary tract
5. F  Prolapsed intervertebral disc. This patient has infection as he has testicular pain and swelling
a history of low back pain, which suggests the as well.

209 
This page intentionally left blank
Breast surgery 16
SBA ANSWERS Question 7
B. A description of a duct papilloma. The discharge may
Question 1 be serous or blood stained. Intraductal papillomas
A. This patient has developed Paget disease of the breast, of the breast are benign lesions with an incidence
which is caused by an underlying ductal carcinoma of approximately 2%–3% in humans. Two types of
in-situ. It presents with a unilateral erythematous rash intraductal papillomas are generally distinguished.
around the nipple, which does not respond to normal The central type develops near the nipple. They are
eczema treatment (such as emollients). There may also usually solitary and often arise in the period nearing
be nipple retraction and inversion, and straw-coloured menopause. On the other hand, the peripheral type
discharge. Lobular carcinoma in-situ does not present are often multiple papillomas arising at the peripheral
with a rash around the nipple. The main differences breasts, and are usually found in younger women.
are that it tends to be multifocal and there is no The peripheral type is associated with a higher risk of
microcalcification seen on the mammogram. malignancy.

Question 2 Question 8
A. A classic description of localised fibroadenosis. D. BRCA1 on chromosome 17Q and BRCA2 on
Triple assessment would be required to establish the chromosome 13Q are associated with breast
diagnosis. cancer.

Question 3 Question 9
D. This lady has a fibroadenoma, commonly nick-named A. In the UK, women between the ages of 50 and 70 are
‘breast mice’ due to their mobility within the breast called every 3 years for screening. However, the NHS
tissue. Fibroadenomas of the breast are benign is in the process of extending the programme as a
tumours characterized by an admixture of stromal and trial, offering screening to some women aged between
epithelial tissue. 47 and 73

Question 4 Question 10
C. Fat necrosis is where trauma has led to fibrosis and C. Stage 3 comprises of any sized tumour,
calcification of the breast tissue. It can occur following involvement of fixed ipsilateral axillary nodes or
surgery. It may be difficult to differentiate clinically from ipsilateral supraclavicular or infraclavicular nodes
breast cancer. This is the most likely diagnosis in this and no evidence of distant metastasis. It may
patient as she has just recently had breast surgery. also comprise of any sized tumour, no palpable or
Recurrence of breast cancer is unlikely to occur that palpable lymph nodes and no evidence of distant
quickly after surgery. metastasis.

Question 5 Question 11
E. Breast cysts are discrete mobile lumps which produce
B. This is a stage 2 cancer with a 5-year survival rate
a straw-coloured fluid on aspiration. Breast cysts can
of 71%.
be painful and may be worrisome but are generally
benign. They are most common in pre-menopausal
women in their 30s or 40s. They usually disappear Question 12
after menopause, but may persist or reappear when A. Ductal cancers are the most common breast cancers,
using hormone therapy. with a frequency of 80%.

Question 6 Question 13
A. Mammary duct ectasia presents with nipple retraction D. Ultrasound helps to distinguish a solid mass from a
as a result of fibrosis and a milky or dirty green cystic mass and is preferable to mammography in
discharge. However, formal triple assessment must be women under the age of 35 years in whom the breast
undertaken to exclude malignancy. tissue is very dense.

211 
EMQ answers

Question 14 factor to HER2 and thus prevents the division and


A. This is a locally advanced breast cancer (stage 3) growth of breast cancer cells.
and is best treated with neoadjuvant chemotherapy, 2. C  Additional side effects include flushes, hot sweats
followed, if possible, by breast conserving surgery and and irregular periods.
axillary clearance. As it is oestrogen-receptor-negative, 3. D  Such drugs prevent the conversion of androgens
tamoxifen would be of no use. Herceptin would only into oestrogen, which is needed for the growth of
be utilised if the cancer were HER2-positive. breast cancer.
4. B  Other common side effects include diarrhoea,
Question 15 tumour pain and headaches.
E. Paget disease of the breast is a type of cancer that 5. A  This agent is also commonly associated with
outwardly may have the appearance of eczema, with tumours of the kidney.
skin changes involving the nipple of the breast. The
condition is an uncommon disease, accounting for Breast lumps
1% to 4.3% of all breast cancers 1. E  Fibroadenoma. Fibroadenoma or ‘breast mouse’
is common in young women as a discreet highly
Question 16 mobile mass.
C. A classic description of a Phyllodes tumour. 2. I  Mastitis. This may precede breast abscess
Histologically, the tumour would have a characteristic and is not uncommon in breastfeeding
leaf like appearance. This is predominantly a tumour mothers.
of adult women, with very few examples reported 3. D  Fat necrosis. This is common after trauma,
in adolescents. Patients typically present with a especially in the breasts. This may be seen after
firm, palpable mass. These tumours are very fast- road traffic accidents as the seat-belt passes over
growing, and can increase in size in just a few weeks. the breast.
Occurrence is most common between the ages of 40 4. C  Duct ectasia. This is common around the time
and 50, prior to menopause. of the menopause, and is due to a dilated, fluid-
filled milk duct or ducts. These can become
EMQ ANSWERS infected.
5. B  Cyst. This is unlikely to be serious but should be
Management of breast cancer aspirated if it becomes symptomatic or there are
1. B  Also known as herceptin, this drug interferes worrying features on ultrasound.
with the attachment of human epidermal growth

212
Index

A adrenal insufficiency, 175 aortoiliac occlusive disease, 105, 197


adrenaline, 77, 177 APTT. See activated partial thromboplastin
α-adrenergic antagonists, 119, 207 adrenocorticotropic hormone (ACTH) time (APTT)
α1-antitrypsin deficiency, 20, 31, 140, 144 levels, 72, 173 arterial blood gas, 31, 147
abdominal aortic aneurysm (AAA), 183, 197 agnosia, 161 arthritis, 62
abdominal ultrasound, 106 airway obstruction, 184 asbestos-induced lung damage, 33, 149
aetiological cause, 105 alcohol, 145 asbestosis, 36
annual abdominal ultrasound scans, alcoholic hepatitis, 143 ascites, 25, 143
106, 197 alcoholic liver disease, 143 aspirin, 13, 52, 134, 136, 144, 160
CT scan, 106 aldosterone-to-renin ratio, 68, 171 asthma diagnosis, 36, 151
features of, 198 alkaline phosphatase (ALP) levels, 61 atelectasis, 187
risk of rupture, 105 all-trans retinoic acid, 79, 178 atrial fibrillation (AF), 107, 198
abdominal CT scan, 88, 183 Alzheimer disease (AD), 53, 161 atrial flutter, 8, 131–132, 137
abdominal mass, 103 anaemia, 41, 83, 153, 180 atrophic gastritis, 82, 180
abdominal pain, 100 of chronic disease, 81, 179 auer rods, 79, 178
intravenous (IV) fluids, 95 iron-deficiency, 142 autoimmune adrenalitis, 67, 170
abdominal ultrasound, 23, 88, 184 macrocytic, 65, 169 autoimmune hepatitis, 24, 143
gallstones, 194 rheumatoid arthritis, 59, 165 autoimmune thyroid disease, 179
abdominal X-ray, 87, 183 anal carcinoma, 194–195 autosomal dominant polycystic kidney
of kidneys, ureters and bladder (KUB), anal fissure, 195 disease, 42, 153
43, 154 anal fistula. See fistula in ano avascular necrosis (AVN), 199
acamprosate, 144 analgesia, 82, 179 axillary nerve, 112, 163, 201
accessory nerve, 54, 162 anaplastic carcinoma of thyroid, 204–205
acetylcholine receptor antibodies, 47
achalasia, 21, 141, 144–145, 191, 193
anastomotic dehiscence, 187
aneurysm
B
acoustic neuroma, 116, 204 AAA (see abdominal aortic aneurysm B symptoms, 80, 179
acromegaly, 169 (AAA)) Bacillus cereus, 144
acromioclavicular osteoarthritis, 109, 199 false, 106, 194, 197 back pain, 63
activated partial thromboplastin time fusiform shape of, 105 MRI of lumbar region, 111
(APTT), 78, 83, 178, 180 mycotic, 106, 197 MRI spine, 110
acute abdomen, 90–91, 184 thoracic aortic, 105, 197 bacterial meningitis, 164
acute abdominal pain, 101 angina, glyceryl trinitrate, 13, 134 bamboo spine, 41, 153
acute appendicitis, 192 angina pectoris, 136 barium swallow perforation, 194
acute bronchitis, 151 angiodysplasia, 144, 192–193, 195 bendrofluazide, 10, 133
acute cholecystitis, 190–192 angiotensin converting enzyme (ACE) benign essential tremor, 49, 158
acute diverticulitis, 190, 192 inhibitors, 11, 15, 41, 133, 135–136, benign intracranial hypertension, 164
acute lymphoblastic leukaemia, 79, 178 153 benign oesophageal stricture, 144
acute myeloid leukaemia, 178 ankle brachial pressure index, 106, 197 benign prostatic hyperplasia (BPH), 119,
acute pancreatitis, 146, 185, 192 ankylosing spondylitis, 59, 165, 167 121, 207–209
CT abdomen, 20, 140 anorectal physiology studies, 97, 191 beta thalassaemia major, 82, 179
acute pericarditis, 12, 134 anterior cruciate ligament beta-blockers, 132
acute pyelonephritis, 156, 185 laxity of, 201 bilateral renal artery stenosis, 44, 155
Adamkiewicz damage, artery of, 105, 197 rupture, 111, 200–201 biliary atresia, 143
Addisonian crisis, 170 antibiotics prophylaxis, 14, 135 biliary colic, 146, 192
adenocarcinoma, 34, 149, 191 anticentromere antibodies, 60, 166 biliary disease, 30
adenolymphoma, 116, 203 antiemetics, 21, 141 biliary peritonitis, 145
adenosine antihypertensives, 10, 133 biopsy, 121, 208
IV, 7, 131 antinuclear antibodies (ANAs), 60, 165 gastric cancer, 191
supraventricular tachycardia, 7, 131 anti-Ro and anti-La antibodies, 60, 166 bisoprolol, 9
adioactive iodine treatment, 66, 170 aortic regurgitation, 10, 133, 136 bisphosphonates, 60, 166
adrenal glands, 75 aortic stenosis, 10, 15, 133, 135–137, 152 bitemporal hemianopia, 65, 169

213 
Index

bladder cancer, 122 carotid body tumour, 115, 203 congestive heart failure, 14, 135
initial investigation, 121, 208 carotid Doppler scan, 188 Conn syndrome, 172
painless haematuria, 208 carotid endarterectomy, cranial nerve connective tissue disease, 62
bladder stones, 121, 208 XII, 106 continuous positive airway pressure
bleeding time, prolonged, 44, 155 carotid sinus massage, 9, 132 (CPAP), 36, 150
blood cultures, 135 carpal tunnel syndrome, 44, 155 Coombs test, 82, 179
blood glucose measurement, 51, 159 cauda equina syndrome, 200 cor pulmonale, 137, 152
blood pressure cefotaxime, 21, 49, 141, 158 corkscrew appearance, 22
level, 11, 133 cerebral infarct, 164 coronary angiography, 8, 132
stab wound to chest, 13, 134 chemotherapy, 42, 154 corrosive oesophagitis, 191
Boas sign, 98, 191 surgery and adjuvant, 96 cortisol levels, 72
bone infections, 113 chest costochondritis, 136
bone mineral density (BMD), 70 drain management, 34, 149 cough, 136–137
bowel habit, 28 stab wound to, 13 cranial nerve lesions, 55
BPH. See benign prostatic hyperplasia chest pain, 16, 18 CREST syndrome, 167
(BPH) atypical (noncardiac), 12, 134 Creutzfeldt–Jakob disease (CJD), 53, 161
branchial cyst, 115, 117, 203–205 sick sinus syndrome, 7, 131 Crohn disease, 21, 23, 141–142, 144, 167,
breast cancer Wenckebach phenomenon, 7, 131 192, 194
chromosome 17, 125 Chlamydia psittaci, 32, 148 cryptogenic fibrosing alveolitis, 33,
ductal cancers, 126, 211 cholangiocarcinoma, 146, 192 148–149
management, 126–127, 212 cholangitis, 95, 189, 192 cryptoorchidism, 207
screening age, 125, 211 cholecystitis, 89, 184, 192 Cullen sign, 88, 183
stage 2, 126, 211 cholesterol embolism, 9, 132 Cushing disease, 172
stage 3, 126 chorea, 50, 158 Cushing syndrome, 149, 170
survival rate, 126, 211 chromosome 17, breast cancer, 125, 211 cyclizine, 21, 141
breast cysts, 125, 211–212 chronic lymphocytic leukemia (CLL), 78, cystic fibrosis, 31, 147, 151
breast lump 178 antibiotics, 31, 147
cysts, 125, 211 chronic myeloid leukaemia, 79, 178 cystitis, acute, 119, 207
excision, recommendations, 93, 187 chronic obstructive pulmonary disease cystoscopy, 121
fat necrosis, 125, 211 (COPD), 147
fibroadenoma, 125, 211
phyllodes tumour, 126, 212
acute exacerbations of, 151
chronic pancreatitis, 20, 25, 140, 143
D
stage 3, 211 chronic pyelonephritis, 156 dalteparin, 43, 155
stage 3 cancer, 126 Churg–Strauss syndrome, 44, 155 daytime sleepiness, 31
ultrasound, 126, 211 ciclosporin, 144 deep vein thrombosis, 177, 187
breast surgery, 125–127, 211–212 cilostazol, 105, 197 dehydration, IV fluids, 66, 170
breathlessness, intercostal space bore ciprofloxacin, tender prostate, 122 delirium, 55–58
insertion, 35 citrate, 119, 207 dermatomyositis, 60, 166–167
bronchial carcinoma, 35, 150, 152 clotting screen, 187–188 diabetes, 74
bronchiectasis, 35, 150 cluster headache, 48, 157, 162, 164 pancreas and, 74–75
Brown-Séquard syndrome, 87, 183 CN III nerve palsy, 49, 52, 158, 160 in pregnancy, 172
bubbling, 34, 149 CN VIII, 53 diabetes insipidus, 175
Buerger disease, 198 Codman triangle, 111 diabetescomplications, 73–74
Burkitt lymphoma, 81, 178–179 coeliac disease, 23, 81, 142, 179 diabetic ketoacidosis (DKA), 162, 171
bursitis, 201 diagnosis, 23, 142 dialysis, 41, 153
cold antibody haemolysis, 82, 179 peritoneal, 41, 153
C cold autoimmune haemolytic anaemia,
82, 180
diaphragmatic hernia, 191
diarrhoea, 27–28
caecal carcinoma, 193 colloid administration, 9, 132 diazepam, 51, 159
calcium oxalate stones, 120, 207 colonic carcinoma, 191 diet therapy, 69, 171
Campylobacter, 144 colonoscopy, 96, 190 diffuse oesophageal spasm, 142
Candida, 144 columnar cells, 19, 139–143 digitalis effect, 137
captopril, 136 compartment syndrome, 167 digoxin, 9, 132
carbimazole, 66, 170 complete heart block, 8, 131 with warfarin, 8, 131
carcinoma caecum, 192 complex partial seizure, 50, 53, 159, 161 dilated cardiomyopathy, 14, 135, 137
cardiac disease, clinical features, 16 compression stockings, 107, 198 direct inguinal hernia, 96, 189–190
cardiac failure, 12 computed tomography (CT) discoid lupus, 167
furosemide, 12, 134 abdomen, 120, 140 disease-modifying antirheumatic drug
cardiac murmurs, 11, 15–18, 133 pancreatic carcinoma, 194 (DMARD), 165
cardiac syndrome X, 12, 134 computed tomography pulmonary distal interphalangeal (DIP) joints, 165
carina, narrow, 11, 133 angiogram (CTPA), 34, 149 distal radius fracture, 201

214
Index 

diuretics extrinsic allergic alveolitis, 149 gallstone(s), 144–145


shortness of breath, 11–12 cause, 33 gallstone ileus, 192–193
sodium restriction and, 24, 143 gastric cancer, 20, 140, 191
diverticular disease, 145
diverticulitis, 89, 184
F gastroenteritis, 27
gastroenterology, 19–30, 139–146
dopamine, 50, 159 factor V, 78 gastrointestinal (GI) bleeding, 26, 99–104
Doppler ultrasound, 78, 177 factor VII, 78, 177 oesophageal varices, 19, 139
Duchenne muscular dystrophy, 47, 157, factor VIII, 78, 177 gastrointestinal distress, 9, 132
163 faecal incontinence, 97 gastrointestinal surgery, 95–99, 103,
duct ectasia, 212 familial adenomatous polyposis, 192 189–195
duct papilloma, 125, 211 familial clubbing, 167 gastrointestinal tract, 80, 179
ductal cancers, 126, 211 family history, 23, 142 gastro-oesophageal reflux disease
ductal carcinoma in-situ, 125, 211 Farmer’s lung, 31 (GORD), 22, 137, 141
Duke diagnostic classification, 15, 95, fat embolism, 53, 161, 201 gentamicin, 43, 154
135, 189 fat necrosis, 125, 212 giant-cell arteritis, 48, 157, 163–164
stage B cancer, 99, 191 femoral hernia, 96, 190, 194 cherry-red spot, 48, 158
stage C cancer, 190 femur giardiasis, 20, 140
duodenal ulcer, 21, 141, 144, 191, 193 fracture, 110, 199 Gilbert syndrome, 20, 140
Dupuytren contracture, 110 fractured neck of, 166 glasgow coma scale, 90, 184
dysphagia, 26–27, 29 fibrinous pericarditis, 137 Glasgow Coma Scale (GCS) Score, 52, 160
dystonia, 50, 159 fibroadenoma, 125, 127, 211–212 glossitis, 65, 169
fibroadenosis, localised, 125, 211 glue ear, 116, 204
E fibrosing alveolitis, 167
fine-needle aspiration, cytology, 66, 170
glyceryl trinitrate (GTN), 97, 190
angina, 13, 134
echocardiogram, 87, 183 finger clubbing, 61 gout, 166–168
murmurs, 11, 133 first-degree heart block, 7, 131 grade 3 encephalopathy, 24, 143
pericardial effusion, 135 fistula in ano, 98, 191 Graham Steell murmur, 136
preoperative investigations, 187 fistulogram, 191 Graves disease, 66, 72, 169, 174
shortness of breath, 11, 133 flail chest, 89, 184 Grey-Turner sign, 145
elbow pain, 110 flank pain groin lump, 103–104
electrical cardioversion, 8, 132 hypercalciuria, 120, 207 Guillain–Barré syndrome, 47, 157
electrocardiogram (ECG), 10, 17 uric acid stone, 119
perioperative care, 93, 187
tall T waves, 44, 155
flexible sigmoidoscopic decompression
with flatus tube, 97, 190
H
embolectomy, 89, 184 flexion deformity haemarthrosis, 111
emphysema, 151 hip, 201 haematemesis, upper GI endoscopy, 24, 143
empyema of gallbladder, 146 lumbar spine, 201 haematuria
enalapril, 11, 133, 136 flexor tendon, 200 bladder cancer, 121
encephalitis, 162 flucloxacillin and fusidic acid, 60 diagnosis, 120, 208
endocrine. See specific diseases focal motor seizure, 50, 159 painless (see bladder cancer)
endoscopic retrograde focal segmental glomerulosclerosis, 43, 155 squamous cell carcinoma, 120
cholangiopancreatography folic acid intake, 12, 134 haemochromatosis, 19, 139, 144
(ERCP), 98, 191 follicle-stimulating hormone (FSH), 65, haemodialysis, 43, 155
endoscopic ultrasonography, 194 169 haemoglobin, 44, 155
endoscopy, 98 foot, puncture wounds, 201 haemolytic anaemia, 82, 179
gastric cancer, 191 foregut investigations, 103 haemorrhoids, 144, 190–191
enoxaparin, 12, 134 fractures, 112–113 Handheld Doppler, 106, 198
epididymal cyst, 122, 208 complications, 112 Hashimoto thyroiditis, 65, 169, 174
epididymitis, 89, 184, 209 Frey syndrome, 116, 203 head CT scan, 51–52, 160
epididymo-orchitis, 120–121, 207–208 Friedreich ataxia, 47, 157 headache, 56–58
epigastric hernia, 97, 190 frontal lobe, 53, 161 heart disease, 17–18
epilepsy, 50–51, 56 frontal lobe tumour, 49, 158 heart failure, 133–134
Epstein–Barr virus, 78, 178, 180 furosemide, cardiac failure, 134 symptoms of, 17
ERCP. See endoscopic retrograde transudative pleural effusion, 34, 149
cholangiopancreatography
(ERCP)
G heartburn, 21
Helicobacter pylori infection, 22, 142
Escherichia coli, 24, 43, 143, 145, 154 galactorrhoea, 169 Heller myotomy, 98, 191
ethambutol, 151 gallbladder Henoch–Schönlein purpura, 60, 166
ethosuximide, 51, 159 diseases of, 192 hepatic encephalopathy, 19, 139, 143
extradural haematoma, 88, 164, 183 empyema of, 146 hepatitis A, 24, 142–144
extrapyramidal diseases, 56 mucocoele of, 194 diagnosis of, 143

215 
Index

hepatitis C, 24, 143 I knee pain


hepatobiliary disorders, 26, 100 in adults, 109
hepatocellular carcinoma, 20, 140 ibuprofen, 15, 135 meniscal tear, 111
hepatoma, 192 idiopathic intracranial hypertension, 164 knee stiffness, 59, 165
hepatomegaly, 137 IgA, 79, 178
IgG antibodies, 82, 180
haemochromatosis, 19, 139
iliac fossa pain, 96
L
herceptin, 126, 212
hernia, 99, 101 immune thrombocytopaenic purpura lactic acidosis, 171
diaphragmatic, 191 (ITP), 177 lacunar infarct, 164
direct inguinal, 96, 189–190 implantable cardioverter–defibrillator Lambert-Eaton myasthenic syndrome
epigastric, 97, 190 (ICD), 13, 134 (LEMS), 149
femoral, 96, 190, 194 incarcerated hernia, 192 laparoscopy, 194
incarcerated, 192 incisional hernia, 192 large bowel obstruction, 192
incisional, 192 infective endocarditis, 14, 135, 137, 167 leg swelling, 107. See also compression
inguinal, 191–192, 194 inferior myocardial infarction, 137 stockings
direct, 96, 190 inguinal hernia, 191–192, 194 Legionella pneumophila, 32, 35, 148, 150
indirect, 96, 190 direct, 96, 190 levodopa, 50, 159
obturator, 97, 190, 192 indirect, 96, 190, 209 limb pain
pantaloon, 191 repair, perioperative care, 93, 187 magnetic resonance imaging (MRI),
Richter, 191 injection sclerotherapy, 97, 190 109
Spigelian, 97, 190–191 insulin, 71, 172–173 Staphylococcus aureus, 109
strangulated femoral, 193 insulin-like growth factor 1 (IGF-1), 65, 169 limp, 61–64
umbilical, 96, 190, 192 insulinoma, 72, 173 liver
high-resolution CT, 31, 147 interstitial cystitis, 120, 207 cancer, 191
hip intestinal obstruction, 101–102 problems, 25–30
fractures, 201 intracranial pressure, 160 localised fibroadenosis, 125, 211
pain and X-ray, 111 intraosseous needle in the proximal tibia, loin pain
Hodgkin disease, 80, 82, 178–180 87, 183 calcium oxalate stones, 120, 207
horizontal interlobar fissure, 11, 133 intravenous (IV) ureteric stone, 121
Horner syndrome, 151 antibiotics, 119 urinalysis and culture, 119, 207
hydrocele, 121, 208–209 dextrose, 171 long thoracic nerve of Bell, 163
hydrocortisone IV, 66–67, 169 fluid replacement, 68 low-dose inhaled steroid, 32, 147
hydrogen breath test, 23, 142 fluids, 189 lower brachial plexus, 163
hydronephrosis, 44, 155 urogram, 88, 183 lower gastrointestinal disorders, 100
hyperaldosteronism, 175 intrinsic factor autoantibodies, 81, 179 lower urinary tract symptoms, 122–123
hypercalcaemia, 72, 162, 171, 173 intussusception, 95, 189, 191 lumbar puncture (LP), 158
aggressive rehydration, 72, 173–174 iodine deficiency, 169 lumbar spinal stenosis, 200
hypercalciuria, 207 ipsilateral hip abductor muscles, 201 lung cancer, 37
hypercapnia, 162 iron tablets, 80, 178–179 clubbing, 34, 149
hypercoagulability, 155 iron-deficiency anaemia, 81, 179 small cell, 34
hyperkalaemia, 12, 134 irritable bowel syndrome, 145 lung diseases, 38
hyperlipidaemia, 145 ischaemic acute tubular necrosis, 156 luteinizing hormone (LH), 65, 169
hyperosmolar nonketotic coma, 175 ischaemic bowel, 95, 189 lymph node biopsy, 33, 148
hyperparathyroidism, 171 ischaemic colitis, 97, 190, 192 and histology, 80, 178
hypersensitivity pneumonitis, 147 ischaemic heart disease, 7, 11, 133 lymph nodes, 204
hypertension ischaemic stroke, 160 lymphadenopathy, 83, 180
cause, 12, 134 lymphocytes circulation, 67
control, 106, 198 J
hyperthyroidism
medication side effects, 66, 170 Jacksonian seizure, 159
M
pregnancy, 66 jaundice, 28–29, 142, 191 macrocytosis, 81, 179
hypertonic saline, 68, 171 management, 24 magnetic resonance imaging (MRI), 199
hypertrophic cardiomyopathy (HCM), 7, joint malignant lymphadenopathy, 194
14, 131, 135 fluid microscopy, 60, 166 Mallory-Weiss tear, 20, 22, 140, 142, 144,
hypoadrenalism, 170 infections, 113 191, 193
hypocalcaemia, 35, 150 pain, 61–64 MALT lymphoma, 81, 179
hypoglossal nerve damage, 198 malunion, 201
hypoglycaemia, 171
hypokalaemia, 10, 133
K mammary duct ectasia, 125, 211
Marfan syndrome, 135
hyponatraemia, 79, 121, 178, 208 Kallmann syndrome, 174 massive haemothorax, 88, 183
hypopituitarism, 65, 169 knee dislocation, 201 mastitis, 212
causes of, 65, 169 knee injuries, 112–113 McBurney point, 87, 183

216
Index 

mean corpuscular volume (MCV), 77, 177 neck pain relief, 42, 154
Meckel diverticulum, 95, 189 lumps, 117 palliative therapy
medial femoral circumflex artery, 201 swellings, 117–118 pancreatic cancer, 191
median nerve, 163 necrotizing fasciitis, 201 pancreatic carcinoma, 25, 143
damage, 47, 157 Neisseria meningitidis, 49, 158 palmar fascia, flexion deformity, 200
medullary cell thyroid cancers, 170 neoadjuvant chemotherapy, 126, 212 palpitations, 10, 133
megaloblastic anaemia, 179 neonatal hyperglycaemia, 69, 172 pancreas and diabetes, 74–75
melaena, 142 nephropathy, radiological contrast, 93, 187 pancreatic cancer, 98
management plan, 22, 142 neurogenic diabetes insipidus, 70, 172 palliative therapy, 191
Ménière disease, 116, 204 nipple pancreatic carcinoma
meningitis, 163 discharge, 125, 211 palliative therapy, 25, 143
of bacterial aetiology, 49, 158 inverted, 125 pancreatic divisum, 145
lumbar puncture (LP), 49, 158 nonsteroidal antiinflammatory drugs pancreatic insulinoma, 172
meniscal injuries, 201 (NSAIDs), 60, 110, 199 pancreatitis, 27
meniscal tear, 111, 200 peptic ulcer, 142 Pantaloon hernia, 191
mesenteric ischaemia, 95, 189, 198 nonunion, 201 paracetamol and metoclopramide, 48
metanephrines, 69, 171 normal pressure hydrocephalus (NPH), Parkinson disease, 159
metastatic liver disease, 143 53, 161 Parkinson plus syndromes, 50, 159
metformin, 69, 145, 171 nose bleed, management plan, 116, 204 parotitis, 115, 203
side effects, 69 nystagmus, vertical, 53, 161 paroxysmal nocturnal dyspnoea, 136
methotrexate, 59, 165 patella fracture, 201
middle cerebral artery, 52, 160
middle meningeal rupture, 51, 160
O peptic ulcer, 22, 142
perforated appendix, 185
migraine, 48, 54, 158, 161–162 objective structured clinical examination perforated peptic ulcer, 185
aetiology, 48, 158 (OSCE), 201 perforated sigmoid diverticular disease,
management, 48, 158 obstructive sleep apnoea, 147 192
minimal change disease, 43, 154–155 obturator hernia, 97, 190, 192 perianal haematoma, 97, 190, 192
mitral regurgitation, 10, 132, 136 oesophageal adenocarcinoma, 193 pericardial effusion, 135
mitral stenosis, 9, 132–133, 136 oesophageal candidiasis, 19, 139 pericarditis, 13–14, 135, 137
mitral valve prolapse, 14–15, 135 oesophageal carcinoma, 145 perilymphatic fistula, 203
Mobitz type 1 block. See Wenckebach oesophageal perforation, 191 perioperative care, 93–94, 187–188
phenomenon oesophageal varices, 19, 139, 191, 193 peripheral and central (Hickman) line
mobitz type 2 block, 8 oesophageal web, 144–145 blood cultures, 23, 142
motor neurone disease, 157 oesophagogastric disorders, 102–103 peripheral blood film, 77
MRI brain scan, 49 ofloxacin, testicular pain, 119 peripheral nerves diseases, 55
mucocoele of gallbladder, 194 oligoclonal bands, 49, 158 peritoneal dialysis, 41
multiple myeloma, 78, 81, 177, 179 omeprazole, 144 pernicious anaemia, 80, 179
multiple sclerosis (MS), 158, 209 open pneumothorax, 87, 183–184 Perthes disease, 111, 200
MRI brain scan, 158 open/closed reduction and petit mal seizure, 50, 159
mural thrombus, 105, 197 immobilization, 111, 200 phaeochromocytoma, 171
murmurs, 13, 17, 134 ophthalmoplegia, 66, 169 pharyngeal pouch, 117, 144–145, 204
blood cultures, 14 oral antihistamines, 117, 204 phenytoin, 51, 159
muscle biopsy, 60 orthopaedic clinical examination, 112 Philadelphia chromosome, 178
muscle diseases, 55–56 orthopaedics, 109–113, 199–201 phyllodes tumour, 126, 212
myasthenia gravis (MG), 54, 157, 162 orthopnoea, 136 pituitary adenoma, 67, 70, 170, 172
mediastinum on chest X-ray, 54, 162 Osgood–Schlatter disease, 199 pituitary apoplexy, 172
Mycobacterium tuberculosis, 33, 148 osteoarthritis, 59, 165, 167 pituitary disease, 73–75
Mycoplasma pneumoniae, 32, 148 of hip, 167 pituitary MRI, 72, 174
mycotic aneurysm, 106, 197 of knee, 110, 199 pituitary tumours, 169
myelofibrosis, 77, 177 osteomalacia, 166 platelet count, 78, 177
myocardial infarct, 137 osteomyelitis, 109 pleomorphic adenoma, 115, 203
posterior, 13, 134 acute, 199 pleural effusion, 152
myoclonus, 50, 159 osteosarcoma, 109, 111, 199–200 pleural fluid, protein levels, 34, 149
myotonia, 47, 157 otitis externa, 115–116, 203 Plummer–Vinson syndrome, 191
otitis media, 116, 203 pneumoconiosis, 150–151
N antibiotics, 116, 203
oxygen inhalation, 54, 162
pneumonia, 36–39
community-acquired, 32, 148, 152
nasogastric decompression, 88, 183 CXR, 32, 148
National Institute for Health and Clinical
Excellence (NICE) guidelines,
P mild, 32, 148
right middle lobe, 152
132, 173 pacemaker insertion, 131 pneumothorax, 34–35, 149–150, 152
nebulized salbutamol/terbutaline, 32, 147 Paget disease, 126, 166, 212 spontaneous, 152

217 
Index

polycystic kidney disease (PKD), 41–42, 153 random blood glucose, 70, 172 shortness of breath, 38–39
polycystic ovary syndrome (PCOS), 71 reactive arthritis, 59, 165 diuretic, 11–12
Polymyalgia rheumatica, 167 rectal bleeding, 104 echocardiogram, 15
polymyositis, 166 Dukes stage B cancer, 99 upper GI endoscopy and colonoscopy,
posterior communicating artery, 52, 160 rectal villous adenoma, 194 22
posterior cruciate ligament rupture, 111, reflux oesophagitis, 144 shoulder dislocation, 112
200 Reiter syndrome, 167 shoulder pain diagnosis, 109
posterior duodenal ulcer, 193 renal cell carcinoma, 42, 153 sick sinus syndrome, 7, 131
posterior inferior cerebellar artery (PICA), management, 121 sickle cell disease (SCD), 80, 178
52, 160 nephrectomy, 121, 208 sigmoid carcinoma, 193–194
occlusion, 164 renal colic, 192, 209 sigmoid volvulus, 193
posterior myocardial infarction, 13, 134, renal disease, 45–46 sigmoidoscopy, 191
137 clinical features of, 45 silent chest, 36, 151
postoperative complications, 93–94 management of, 46 silicosis, 36, 150
potassium, 41, 153 renal failure, 41, 153 sinus infection, 115, 203
preoperative investigations, 94 complications of, 45 Sjögren syndrome, 166
preproliferative retinopathy, 69, 171–172 renal stones, 207 skin infections, 113
primary biliary cirrhosis (PBC), 144 renal tumour diagnosis, 208 slipped femoral epiphysis, 110, 199
pruritus, 19, 139 respiratory drugs, 37 slipped upper femoral epiphysis (SUFE),
primary hyperaldosteronism. See Conn respiratory infections, 31–39, 147–152 111, 167, 200
syndrome respiratory tract diseases, 37 small bowel obstruction, 185
primary sclerosing cholangitis, 144 resuscitation, 89, 184 small cell carcinoma, 149
proctoscopy, 191 retinal disease, grade 3, 15, 135 small cell lung cancer (SCLC), 35, 150
proinflammatory cytokines, 77, 177 retinal inflammation, 148 smear cells, 79, 178
prolactin level, 65, 169 retrosternal thyroid, 145 smoking, 105
prolapsed intervertebral disc, 209 rhabdomyolysis, 155 peripheral arterial disease (PAD), 197
prostate-specific antigen (PSA), 42, 120, rheumatoid arthritis, 59, 165, 168 and squamous cell carcinoma, 204
154, 207 rheumatoid lung, 152 sodium docusate, 21, 141
prostatic carcinoma, 209 rib fracture, 137 sodium valproate, 51, 159
prostatitis, 120, 207 Richter hernia, 191 solitary thyroid nodule, 205
ciprofloxacin, 208 rotator cuff tear, 110, 199 Spigelian hernia, 97, 190–191
prosthetic joint infections (PJIs), 201 Rovsing sign, 87, 183 spinal cord compression, 48, 157
protein electrophoresis, 81, 179 spinal magnetic resonance imaging (MRI)
prothrombin time, 24, 143
proton-pump inhibitors, 21, 141
S scan, 48
spirometry, preoperative investigations, 187
proximal interphalangeal (PIP) joints sapheno-femoral and sapheno-popliteal spironolactone, 7, 71, 131, 173
management, 59, 165 incompetence, 198 splenic abscess, 14, 135
pruritus, 19, 139 scaphoid fractures, 201 splenomegaly, 83, 180
pseudobulbar palsy, 53, 160 scotoma, 52, 160 sputum smear, 33, 148
pseudogout, 168 scrotal swellings, 122–123 squamous cell carcinoma (SCC), 96, 190
pseudo-obstruction, 187, 193 scrotum lump, epididymal cyst, 122 of lung, 33–34, 149
psoriatic arthritis, 59, 165, 168 scrotum pain, CT of abdomen, 120, 208 and smoking, 117, 204
pulmonary abscess, 167 scurvy, 71, 173 of urinary bladder, 120, 208
pulmonary embolus, 137 secondary hyperparathyroidism, 68, 171 ST segment elevation, 13, 134
pulmonary fibrosis, 36, 136, 150 second-degree block, 131 Staphylococcus aureus, 14, 32, 144, 148
pulmonary oedema, 134 seizure, 50 statins, 8, 131
pulmonary stenosis, 71, 172 complex partial, 50, 53, 159, 161 Sternberg–Reed cells, 179
pyloric stenosis, 95, 189, 193 focal motor, 50, 159 steroids, 48, 157
Jacksonian, 159 strangulated femoral hernia, 193
Streptococcus mutans, 14, 135
Q petit mal, 50, 159
temporal lobe, 159 Streptococcus pneumoniae, 32, 147
Q waves, 13, 134 tonic-clonic, 50, 159, 163 Streptococcus viridans, 15, 135
septic arthritis, 165 streptomycin, 33, 148
R septic joint, 168
septic shock, 93, 187
stress incontinence, 42, 154, 209
strokes, 57–58
radial nerve, 54, 110, 162–163, 199 serotonin, 48, 158 subacromial impingement, 109, 199
injury, 201 serum alpha-fetoprotein, 98, 191 subarachnoid haemorrhage, 160, 163
radiation pneumonitis, 36, 150 serum amylase, 21, 141 subdural haematoma, 51, 88, 160, 162,
radical cystectomy, 121, 208 serum antimitochondrial antibodies 164, 183
radiological contrast nephropathy, 93, 187 (AMAs), 25, 143 submandibular calculus, 204
ramipril, 44, 155 serum ferritin, 25, 143 sulphonylurea, 173, 175
Ramsay Hunt syndrome, 52, 160 shock, 189 sunray spicules, 111

218
Index 

superficial thrombophlebitis, 198 transurethral resection of the prostate urological investigations, 123
supraventricular tachycardia (TURP) urology, 119–123, 207–209
adenosine, 7, 131 symptoms, 121, 208 uterine cyst, 89, 184
surgical excision of tumour, 68, 171 trauma, 90–91 uterine fibroids, 194
sweat test, 31, 147 traumatic aortic rupture, 184 uveitis, 148
symmetrical sensory neuropathy, 69, 172 tricuspid insufficiency, 71, 172 uveoparotitis, 148
syncopal episode, 132 tricuspid regurgitation, 10, 133, 136
syndrome of inappropriate antidiuretic trigger finger, 110
hormone (SIADH), 68, 70, 171–172 tropical sprue, 21, 141 V
syringomyelia, 157 troponin, 13, 134 varicocele, 208–209
systemic lupus erythematosus (SLE), 109, TSH receptor antibodies, 66, 170 vascular dementia, 161
167, 179, 199 tuberculosis (TB) vascular disorders, 107
and macrophages, 33, 148 vasculitis, 62
rifampicin, 33, 148
T streptomycin, 33, 148
venesection, 77, 177
ventricular fibrillation, 8, 132
take patient to theatre, 88 treatment, 33 ventricular septal defect, 136
tar-coloured stool, 142 tubules and interstitium diseases, 46 vertical nystagmus, 53, 161
target cells, 79, 178 type 2 diabetes mellitus, 171 vertigo, 115, 160
temporal arteritis, 167 blood pressure for patients with, 71, 173 viral encephalitis, 164
temporal lobe seizure, 159 tyrosine kinase inhibitors, 79, 178 viral illness, 20, 140
tension headache, 163–164 viral labyrinthitis, 54, 162
tension pneumothorax, 89, 184
teratoma, 42, 154
U Virchow node, 205
vitamin B1, 19, 139
of testis, 209 ulcerative colitis, 23, 142, 145, 192 deficiency, 47, 157
testicular cancer, 207 ulnar nerve neuropathy, 110, 199 vitamin B12
testicular pain, ofloxacin, 119 ultrasound, breast lump, 126, 211 deficiency anaemia, 81, 179
testicular torsion, 89, 122, 184 ultrasound-guided diagnostic pleural tap, and folate deficiency, 82, 179
testicular tumour, 209 34, 149 vitamin C deficiency, 173
tetracosactide (synacthen) umbilical hernia, 96, 190, 192 vitamin D level, 60
long test, 67 upper gastrointestinal disorders, 99 vitamin E deficiency, 72, 173
short test, 67, 170 upper gastrointestinal endoscopy, 21–22, vitamin K, 177
thiamine deficiency, 139 141–142 vomiting, 98
thoracic aortic aneurysm, 105, 197 upper limb nerves, 57
thrombolytic therapy, 9, 132 upper quadrant pain, 96
thrombotic thrombocytopaenic purpura acute cholecystitis, 191 W
(TTP), 177 ERCP, 98
warfarin, 77, 79, 177–178
thyroglossal cyst, 115, 203–204 urate nephropathy, 156
Warthin tumour. See adenolymphoma
thyroid cancer, palliative care, 170 urea and electrolytes, 41, 153
water deprivation test, 68, 171
thyroid disease, 73 ureteric colic, 208–209
weight loss, pancreatic cancer, 19–25, 139
thyroid goitre, 204 ureteric stenting, 121, 208
Wenckebach phenomenon, 7, 131
thyroid hormone, 65, 169 ureteric stone management, 121
Wilson disease, 144
replacement, 73, 174 urethral stricture, 209
wound infection, 187
thyroid malignancy, 72–73, 174 uric acid stone formation, 207
thyroid status, 66, 170 urinary catheterization, 122, 208
tibial plateau fracture, 201 urinary protein and serum albumin Y
tobacco smoking, 147 measurement, 43
tonic-clonic seizure, 50, 159, 163 urinary tract Yersinia enterocolitica, 145
transient global amnesia, 52, 160 infection with associated pyrexia, 207
transient ischaemic attack (TIA), 51, 160 obstruction, 209
transitional cell cancer (TCC) of the urine dipstick, 209
Z
bladder, 122, 208 urine microscopy and culture, 43, 154 Zollinger–Ellison syndrome, 145, 191
transudative pleural effusion, 34, 149 urodynamic study, 42, 154

219 
This page intentionally left blank

You might also like

pFad - Phonifier reborn

Pfad - The Proxy pFad of © 2024 Garber Painting. All rights reserved.

Note: This service is not intended for secure transactions such as banking, social media, email, or purchasing. Use at your own risk. We assume no liability whatsoever for broken pages.


Alternative Proxies:

Alternative Proxy

pFad Proxy

pFad v3 Proxy

pFad v4 Proxy